Download as pdf or txt
Download as pdf or txt
You are on page 1of 151

Kinematics

centre. To be accurate, your instructions require an origin,

M
echanics, or the study of motion, consists of two
branches: kinema~ics and .dynamics. Kine~ati~s a distance, and a direction, which together make up a
is about describing motion, and dynamics 1s coordinate system (Fig. 2. I).
about forces and their effects on motion. In this chapter,
we will focus on the description of translational motion. y(m)
Translational motion, as opposed to rotational motion, is
movement of an object without a change in its orientation. 20
We will first introduce the idea of using a coordinate
system, which is absolutely necessary for any quantita-
tive description of motion. Then, we will introduce the

Classroom 1O
essential quantities in kinematics, including displacement, Student
velocity, and acceleration. In Sections 2.3 and 2.4 we will ~~------.-----...~ce~n~tr~e~------..---.--.... x(m)
discuss motion in two dimensions, where the concept of -200 -100 0 100 200 Bus
vectors plays a key role. Finally, we will briefly explain stop
the mechanisms of physiological detection of velocity and -10
acceleration.
Figure 2.1 A schematic diagram showing the student centre, the
bus stop, and the classroom in a two-dimensional Cartesian coordinate
system.
2.1: Setting Up the Coordinate
System Fig. 2.1 illustrates a two-dimensional Cartesian
coordinate system. It consists of two axes that are per·
If a student came up to you after class and asked where to pendicular to each other: the x-axis and the y-axis. Th.e
catch the bus, what instructions would you give her? To location of the bus stop can be described by its coord1•
start with, you need a reference point. Your instructions nates, x = 250 m and y = 0 m, or simply (250, 0) since the
have to begin somewhere. If the student is familiar with units on the axes are metres.
campus, you could suggest a central starting point, like On the other hand, if we shift the coordinate syste~
the student centre. If she is new to campus, it might be such that the origin is at the classroom, then the coordi·
best to give her directions that start at your current loca- nates (250, 0) would no longer give the position of the
tion. In either case, this starting reference point is called bus stop. Obviously, a quantitative description of the
the origin of your directions: the place that your direc- position of an object depends on the choice of coordinate
tions will be measured from . system.
If you said that the bus stop is 250 metres away from Also, if you want to determine the motion of a person
the student centre, the student would still have to hunt sitting in a moving train, the choice of coordinate syste~
around all sides of the student centre to find the bus stop. can make a big difference. In particular, if you atta~d
You could save the student a lot of time hy indicating your coordinate system to the ground, the person wou
that the bus stop is 250 metres to the east of the student be moving at the same speed as the train. On the other

22 PART ONE I THE MECHANICS OF BIOLOGICAL SYSTEMS

Scanned with CamScanner


hand, if you choose to attach your coordinate system to
the train, then the person would not be moving at all.
In general, any numbers and equations that you use to
Fig. 2.3 shows the x-position of an object as a function
describe motion would be meaningless unless you have
of time. If the object moves to the right, the x-position
set up a coordinate system.
increases. If the object moves to the left, the x-position
decreases. Which of the following best describes the
m U1!11I To describe motion quantitatively, set up your
motion of the object?
coordinate system first.
x(m)

2.2: Motion in One Dimension


If we want to describe the motion of a train car on a
straight railroad, or the motion of a molecular motor
along a microtubule in a cell, we only need one axis since
the motion is along a straight line. We often call this the
x-axis. A motion that is along a straight line is called one-
dimensional motion.
When an object moves, its position changes. In other
words, motion is defined by the change in position as a
function of time. Therefore, to analyze motion, we often Figure 2.3 Graph of position versus time. The x-axis is positive to
need an axis to represent time. When studying kinematics, the right.
it is useful to study a graph of the relationship between (a) The object moves to the left, stops, then moves to
position and time. the right, ending up further to the right than where
it started.
EXAMPLE 2.1 (b) The object moves up, stops, and then falls down
In the question about finding the bus stop, if the student lower than where it started.
walks only along the line between the student centre and (c) The object moves to the right, stops, and then
the bus stop, the motion would be one-dimensional and moves to the left, ending up further to the left than
we would only need the x-axis. Fig. 2.2 shows a graph of where it started.
the student's x-position as a function of time. Note that,
in the position-versus-time graph, the x-axis is the ver- (d) The object moves up and to the right, then straight
tical axis and time is the horizontal axis. If the bus stop to the right, then down and to the right.
is 250 m from the student centre, does the student get
to the bus stop at some point in the time interval shown
in Fig. 2.2? Where does the student end up at time
t = 10.0 minutes? 2.2.1: Distance and Displacement
x(m) When an object moves from an initial position at time t
to a final position at time t 2, we can use two quantities t~
200
describe how far the object moves: distance and displace-
ment. Distance is the length the object travels, measured
along the path of motion. It is always a positive number.
On the other hand, displacement is the change in posi-
tion, which has two aspects: magnitude and direction. In
physics, a quantity that has both a magnitude and a direc-
tion is called a \•ector. The magnitude of displacement is
the length measured along a straight line between the ini-
tial and final positions. The direction of displacement is
Figure 2.2 A graph showing the position of a student as a function from the initial position to the final position. Since we are
Of time. generally interested in both the amount and the direction of
motion, displacement is important for describing motion.

l:!iUolil•i
A vector is a quantity that has a magnitude and
Solution a direction.
The student goes to a maximum x-position of 150 m before
turning around. She never gets to the bus stop, and at In one-dimensional motion, there are only two pos-
1 == 10.0 minutes, she is right back where she started. sible directions: the positive and the negative x-directions.

CHAPTER 2 I KINEMATICS 23

Scanned with CamScanner


The direction of a vector in this case can be characterized at time 1 = 2.0 minutes). We have to look at the distance
by the sign of a number. That is. a positive number means travelled in the positive direction, and then add that to
that the vector points in the pos itive .r-direction, while a the distance travelled in the negative direction. If, as
negative number corresponds to the negative .r-direction. in this example, there is a change in direction travelled, the
distance travelled will be more than the magnitude of the
Therefore, w e will use a signed number to represent a
displacement. In general, the distance travelled can never
vector when we study one-dimensional motion. We will
be less than the magnitude of the displacement. In this
see later that such a signed number is a scalar component
case, the student travelled a distance of 75 m over the first
of the vector. A scalar is a quantity that can be represented
time interval, and then travelled a distance of 150 m over
by a real number, which can be positive, negative, or zero. the second time interval. In total, the distance the student
We will discuss vectors in more deta il when we analyze = =
moved between time 11 1.0 minute and 12 10.0 minutes
two-dimensional motion in Section 2.3. is 225 m.
If an object at initial time t 1 is at position x 1, and at
some later time t 2 it is at position x2, the displacement, /).x,
is defined by the equation:

/)..x = x2 - x1• (2. J)

Note that the displacement tells you how far the object Over a 5 minute time interval, an object moves from an ini-
is from where it started, but it does not tell you how tial position x1 = - 150m to a final position x2 = Om. What
is the total distance travelled during that time interval?
far the object travelled between time t 1 and time t 2. For
example, Fig. 2.2 shows the position of a student as a (a) 150 m
function of time. At time I 1 = 1.0 minute, the student
is at position x 1 = 75 m. At time 12 = 6.0 minutes, the
(b) 0 m
student is also at position x 2 = 75 m. The displacement (c) -150 m
is /).x = x 2 - x 1 =O m, meaning that the object is zero
(d) There is not enough information.
metres from where it was at I 1, but the graph shows that the
student moved throughout the time interval. The distance
travelled by the student between time t 1 = 1.0 minute and
12 = 6.0 minutes is d = 150 m: 75 m in the positive direc-
tion, plus 75 m in the negative direction. Because distance 2.2.2: Speed and Velocity
does not depend on the direction, both of these numbers Speed and velocity are the quantities we use to describe
are positive and do not add to zero. how fast an object moves. Speed is the distance travelled
~er unit time, whereas velocity is displacement per unit
time. Following the discussion on distance and displace-
EXAMPLE 2.2 ment, speed is always a positive scalar whereas velocity
Using Fig. 2.2, determine both the distance and the is a .vector that has both a magnitud; and a direction.
displacement of the student between time 11 = 1.0 minute Agam, for one-dimensional motion, we can use a signed
and 12 = 10.0 minutes. ~u~b~r to describe velocity. The direction of the velocity
is md1cated by the sign.
Solution
At time 11• = 1.0 minute'. the student is at position x 1 = l:J§lij.Jfrii
. ·- Veloc't · a vector that describes how fast an
1 Y is
75m. At time 12 = 10.0 mmutes, the student is at position Object moves in a particular d irection.
x 2 = 0 m. The dis~lacement is Ax= x 2 - x , = o m _ 75 m =
-75 r;1· ~nee agam, remember that the sign represents In general, the average speed is defined as the dis·
the d1rect10~ that the student moved. In this case, the dis- tance travelled divided by the time elapsed (At):
placeme.nt is ~ .neg~ti~e number, which means that the
chang.e tn pos1t1on ts tn. the negative x-direction. Earlier, distance
s av = -At - -.
we satd that movement tn the direction of the b t .
't' I US S op IS
post
. tve a
. ong
. the x-axis. So if the student' d' I
s tsp acement For instance, if you walked I 000 m to get from your
ts negative, it means that she is further away from the house to your friend's house, and that walk took you sOO
b~s stop at 12 than she was at i,. The magnitude of the seco~ds, you could find your average speed by dividing
displacement
'f tells us the distance between the students '
post ion at t I and the student's position at 1 the distance you walked by the time taken :
The ~istance the student travelled ~~n be found distance I 000 m m
. breaking the path into two sect'ions.. the time
by . sav = At -- 500 = 2 - .
mterval
. . between 1.0 minute and . t
2·0 mmu es and the At f d s s t
t ime mterval between 2.0 minutes and 10 0 . tmes uring that walk, you likely walked sornewha
faster than 2 mls, an d at ot her times
. 11:11
(note that the student changes the direct'o . fmmu!es you walked some''
1 no motion
slower
. than 2. m; s. The average speed doesn't give; . voll
COtlt/llU«/
any mfonnat1on about your speed at a particular inst:int.

24 PART ONE I THE MECHANICS OF BIOLOGICAL SYSTEMS

Scanned with CamScanner


it simply tells you that you would have been able to walk
that distance in that amount of time if you had walked at
a constant rate of 2 mis.
Can average speed ever be lower than the magnitude of
In Example 2.2, the average speed of the student
average velocity?
between time I 1 = 1.0 minute and 12 = I 0.0 minutes is:
(a) yes
distance 225 m m
sav = 6.1 = 540s = 0.42~. (b) no

The average velocity, on the other hand, is defined as


the displacement divided by the time interval: We can develop the concept of motion along the
x-axis further by looking at two examples from track and
!ix x -x 1 field events: the 100 m sprint and the long jump. The box
v --- 2 (2.2)
av - 6.1 - I - I in Fig. 2.5(a) represents the sprinter at the starting line
2 I
For example, the average velocity of the student in (position x 1) and at the finish line (position x 2), or the long
Example 2.2 between time t 1 = 1.0 minute and 12 = I 0.0 jump competitor at the start of the approach (position x 1)
minutes is: and at the point of takeoff (position x 2 ). Fig. 2.5(b) is a
graph of x-position as a function of time. The solid curve
6.x 0 - 75m -75m m
vav = t:f = 9.0min = 540s = -O. t 4 -; in Fig. 2.5(b) is a possible description of the motion of
each of the athletes as they get from position x 1 (starting
Again, the negative sign indicates that the average position) to position x 2 (final position).
velocity is pointing in the negative x-direction. Notice During the early stage of the sprint or the approach,
that the average speed is greater than the magnitude of or near time t I' the athlete covers a comparably smaller
the average velocity. In fact, since the distance is always distance per time unit than during the late stage, or
greater than or equal to the magnitude of the displace- near time t2 . In the case of the sprinter, this is because
ment for the same given time interval, the average speed the velocity increases toward a maximum value. In the
is always greater than or equal to the magnitude of the case of the long jump competitor, this is done to achieve
average velocity. maximum takeoff velocity.

EXAMPLE 2.3
At 3:14 p .m ., a train is 3 km east of the city centre. At 3:56 p.m., the train is 26 km west of the city
centre. What is the average velocity of the train during this time interval?

Solution
Step 1: Set up a coordinate system, which is an x-axis with the city centre at the origin. Indicate the
initial and final positions of the train, as shown in Fig. 2.4.

-26 0 3

Figure 2.4 The x-axis showing the position of the train as measured from the city centre, which is taken to be the origin.
Distances are in kilometres, and we set east as the positive direction, which means that positions west of the city centre
are negative.

Step 2: Find ~ and At:


~ = x _ x = -26 km - 3km = - 29km
2 1
At= t
2
- t = 3:56p.m. - 3:14p.m. = 42minutes.
1
If we want to find the average velocity in units of kilometres per hour (km/h), '.""'hich is a typical unit for
describing the motion of trains and other vehicles, we need to convert 42 minutes to hours:

At = 42min(~)
60mm
= 0.70h.

Now, using Eq. {2.2], we can find the average velocity:


_ A.x =~=-4lkm.
At 0.70h
v.v - h
The average velocity during that time interval is 41 km/h in the west direction.

CHAPTER 2 I KINEMATI CS

Scanned with CamScanner


the average velocity of the long jump competitor during
(a) t = ti
the approach run is useless for the coaches. They arc inter-
ested in the instantaneous velocity at takeoff.
The velocity at a particular instant called the instan-
• x
taneous velocity. The magnitude of its instantaneous
Xi
velocity is an object's instantaneous speed. This is why

l speed is a positive scalar. When we talk about velocity


and speed at a given time, we typically leave off the word
instantaneous. However, since instantaneous speeds and
velocities can change over a period of time, it is impor-
t= t2
tant to recognize that average speed and average velocity
can differ from instantaneous speed and instantaneous
• x
X1 X2
velocity.
We can determine the instantaneous velocity from
the velocity definition in Eq. [2.2]. We start with the end
(b) x(m)
points, shown as open circles in Fig. 2.5(b), and assume
they provide a rough estimate of the instantaneous
velocity at time t2. Then we improve that estimate of the
velocity by determining the average velocity for shorter
, and shorter time intervals, each ending at time t2. This
, ,,
,, is illustrated in Fig. 2.6: the slopes of the dash-dotted,
,
, ,, dashed, and solid blue lines are successively better esti-
, ,, mates of the slope at time 12.
,,
,/
, ,, x
, ,,
, ,,
, ,,
,,
,,

'----1--------------i---. t (s)

Figure 2.5 (a) The difference between the average velocity and the
instantaneous velocity is illustrated for an object that moves from posi-
tion x1 at time t 1 to position x2 at time t 2. (b) A graph illustrating how
the position of the object changes with time (solid line). If we measure
position and time only twice, at instant 1 and at instant 2 as indicated by
two open circles, then we obtain the average velocity as the displacement:
6.x = x 2 - x1 divided by the elapsed time M = t2 - t 1 (dashed line).

Eq. [2.2] corresponds graphically to the slope of the


Figure 2.6 Repeat of Fig. 2.5 with the initial instants (open and solid
dashed line in Fig. 2.S(b). We call the slope of the dashed
dots) chosen successively closer to the final instant As the time interval
line the average velocity of the object between time t 1 and approaches zero, the line through the two respective points on the curve
time 12. The dashed line in Fig. 2.S(b) doesn't follow the approaches the tangent at the final instant.
solid line, meaning that the actual velocity of the athlete
varies with position and is, at most points, different from The instantaneous velocity at time 12 is, when we
the average velocity. extrapolate to 6.t = 0, the slope of the green line in Fig. 2.~·
To discuss the usefulness of Eq. [2.2], we must com- In the limit that 6.t = 0, this line is the tangent to the sohd
pare the sprinter and the long jump competitor more curve at position x 2 , and the value of the instantaneous
carefully. The definition of average velocity in Eq. [2.2] velocity is the slope of the tangent:
is useful for the sprinter's coaching team. For example,
they could plot the average velocity of a I 00 m sprint, v = lim 6.x. (2.3)
a 200 m dash, and a 400 m race for the same sprinter to .l.r-+o!lt
get information about the endurance performance of the We use the mathematical limit notation in Eq. [2.3]; that
leg muscles. However, more often, we want to know the is, v is the limit of the function !lxl!lt as !lt approaches
velocity of an object at a particular instant. For example, zero.

26 PART ONE ( THE MECHANICS OF BIOLOGICAL SYSTEMS

Scanned with CamScanner


This is shown in tlu: velocity-versus-time graph in Fig. 2.7.
Velocity is 150 m/s for all instants between 0 s and 4 s.
It is gern:rally true that a region of constant slope on a
For the motion of a particular object, we find that the position-versus-time graph will produce a llat region on the
maximum instantaneous velocity is equal to the average vclocity-vcrsus-timc graph.
velocity. What can we conclude from this observation? The velocity for the interval from 4 s to I0 scan also
(a) The time interval must be very long. be found by calculating the slope of the curve in that time
(b) The object is moving at a constant velocity.
interval using Eq. 12.2]:

(c) The object is decelerating. v = Ax = x2 - x1 = 0 m - 600 m = _ 100 m.


av At 1 -1
2 1
10s -4s s
(d) The object is accelerating.
The slope for that interval is also constant, which means the
velocity is -100 mis for all instants between 4 s and IO s.
The slope of the tangent, or simply the slope, of the
position-versus-time curve at time t is the instantaneous
velocity at that point. That means that we can use infor-
EXAMPLE 2.4
mation about the slope of a position-versus-time graph to
The position of a particle as a function of time is given by
produce a corresponding velocity-versus-time graph.
the curve shown in Fig. 2.8.
t:iu4@1I The slope of an object's position-versus-time
8
graph gives the velocity on the object's velocity-versus-
6
time graph.
4
Figure 2.7 shows a position-versus-time graph for an .§. 2
k
object moving along the x-axis over a time interval from 0
0 s to IO s. The average velocity for the interval from 0 s 4 5 6 7 8 9 10
-2 t (s)
to 4 s can be found by calculating the slope of the curve
-4
in that time interval using Eq. [2.2]:
Figure 2.8 A position-versus-time graph for a non-constant velocity.
Ax x2 - x 1 = 600m - Om= ISOm .
v =-= (a) When does the velocity have its most positive value?
av At t2 - tI 4S - 0S S
(b) Is the velocity ever negative?
(c) When is the speed the greatest?
(a) 700 (d) When is the speed zero?
600 (e) What is the instantaneous velocity at time / = 2.0 s?
500 (f) What is the average velocity between / = 3.0 s and
g 400 I= 6.0 s?
300
" 200 Solution
100 (a) The velocity has its most positive value when the tan-
gent to the curve is most positive. This happens at
4 5 6 7 8 9 10
2 3 around 0 s.
t (s)
(b) The velocity is negative when the tangent to the
(b) 200 curve is negative. This happens in the time interval
150 between 5 s and 7 s.
100 (c) The speed is greatest when the slope has its largest
1 50 . magnitude. This happens at some time between
5 s and 6 s. Even though the velocity is negative in
0 10
" -50 p 2 6 8
that time Interval, the speed is the magnitude of the
- too velocity, and is always positive.
- I SO (d) The speed is zero when the slope of the curve Is
t (s)
zero. That happens twice: at around 4.8 s and again
Figure 2.7 A position-versus-time graph can be used to infer veloeity. at around 7 s.
(e) The instantaneous velocity at / = 2.0 s can be found
Th · stant which by drawing a tangent to the curve at / = 2.0 s, as
. e slope for that interval is a con ' ·n,
unrnediately tells us velocity must be constant dun_g shown in Fig. 2.9. The slope of that tangent can be
~hat tirne interval. That also means instantaneous. veloc1tf ,.......,
•s equal to average velocity for that time mterva .

CHAPTER Z I KINEMATICS 27

Scanned with CamScanner


2.2.3: Acceleration
found by calculating the rise over the run, which in
this case is: The du:ctah and the gazelle arc two of the fastest land anj.
ma ls 0 11 Earth. The gazelle is also a staple in the cheetah's
.ix X2 - XI 7.0m - O.Om
= 1.8 -m. did. The d1ectah 's maximum speed is around 110 km/h.
\{I = 2 s) = - = - - -
!:::.1 I 2 - 1, 4.0s - O.Os s The gazelle's maxim um speed is around 75 km/h. ft
(t) The average velocity during the interval between would seem as if the cheetah could have an easy meal
t = 3.0 sand 1 = 6.0 scan be found using Eq. [2.2]:
1 2

\I = !:::.x = X2 - Xt = 1.0 Ill - 4.8 Ill = _ 1.3 Ill.


av 6.1 '2 - ,, 6.0s - 3.0s s

8
6
4
.§_ 2
){
0
5 6 7 8 9 10
-2 t (s)
-4

Figure 2.9 The slope of the tangent to the curve at t = 2.0 s


represents the velocity at / =2.0 s.
Figure 2.11 The cheetah is the world's fastest land animal, but its
acceleration is what makes it such an accomplished predator.

CASE STUDY 2.1


Kinesins are motor proteins found in eukaryotic cells. They velocity of the kinesin in the time interval between t 1= 0.0 sand
can carry large molecules and cell organelles, and move t2 = 2.5 s? (b) Ignore the high-frequency noise and estimate the
along microtubules in cells. In a study of the motion of kine- average speed in the same time interval. (c) What can you say
sins, the position of a bead-bound kinesin motor protein about how the kinesin moves along the microtubule?
along a microtubule as a function of time has been measured
Answer to part (a) We can find the initial and final positions
and recorded. The result is shown in Fig. 2.10. Note that the
of the kinesin: x 1 = O nm and x 2 = 84 nm. The average velocity
high-frequency noise (the fine fluctuations in the graph) is due
is then:
to uncertainties in the measurements. (a) What is the average
6.x X2 - x, 84nm - Onm nm
v = - = - -- = = 34-.
av !!it t2 - t 1 2.Ss-Os s
Answer to part (b) If we ignore the noise, the position-versus·
time graph basically consists of a few flat steps. The distance
in the entire time interval is approximately the same as the
displacement. Therefore, the average speed is also 34 nnv's.
e.::. Answer to part (c) Since the average speed is equal to
.g
·v;
the magnitude of the average velocity, the kinesin does
not change its direction of motion which can also ~e
0 . , f t·..,e in
Q., seen directly from the position as a function o 111 '
Fig. 2.10. Therefore, a m icrotubule is a one-way route tor
. . moves
a k mesin. In fact, it is known that a kinesin always ·
81 11
toward the positive end of a microtubule, meaning th. •
11
moves from the central area of a cell toward the penP
0 ---------------------------------------- eries of the cell. .,,
. .
Cl ose examination . kines1
0.0 0.5 1.0 1.5 2.0 2.5 of Fig. 2.10 reveals that the . iS
. I .
moves 1n a ternating quick and slow steps. The s e.
t P size
teP·
Time (s)
about 8 nm. The dwell time is less than 0.1 sin a quicl<t~at 8
Figure 2.10 The position of a bead-bound kinesin motor along a but can be as long as 1 s in a slow step. It is known i·1<e
microtubule as a function of time measured by a research team led k .inesin
. 1·usl i
uses two heads to walk on a microtubule. 1·on
by Steven Block at Stanford University. we use two legs to walk on a narrow road. The aIterna' 8 15
Source: Block lab at Stanford University, http://web.stanford.edu/group/blocklab/ in dwell time between the quick and slow steps sugges
kinesinlkinesin.html. Copyright 2015 Steven Block. Used by permission.
hand-over-hand limping-like motion.

28 PART ONE I THE MECHANICS OF BIOLOGICAL SYSTEMS

Scanned with CamScanner


any time it found a gazelle. However, running at such high If we can rearrange Eq. [2.4] to solve for final speed,
speeds quickly exhausts the cheetah. If it docs not catch we can find a formula that easily relates final speed to
the gazelle within the first 15 seconds, it has to give up initial speed, acceleration, and time interval:
or risk brain damage due to overheating. In reality, the
cheetah starts from rest, accelerates quickly, and then runs Vz - VI

at its maximum speed. It turns out its acceleration is what


aav = Al =} Vz = v, + aavAt. (2.5)
makes it such an accomplished predator. To find out how far the cheetah travels during its 15 s
of exertion, we need to find a way to relate acceleration
m U @I Acceleration describes how much an object's
to position. As always, we will start with a simple phys-
velocity changes in a given amount of time.
ical model: when the cheetah is accelerating, its accel-
Much like velocity describes how much an object's eration is constant. This is obviously not true in that the
position changes in a given amount of time, acceleration cheetah's acceleration must gradually reduce to zero as
describes how much an object's velocity changes in a it approaches its maximum speed. However, it is a rea-
given amount of time. We can write average acceleration sonable first approximation. Fig. 2.12 shows a graph of
in a mathematical formula that is very similar to the one velocity versus time for an object moving with constant
used to define average velocity: acceleration. Notice that the velocity is increasing linearly
with time. This linearity allows for an easy definition of
V2 - VI average velocity:
(2.4)
'2 - '1 (2.6)
Again, similar to velocity and displacement, acceleration
is a vector, but can be represented by a signed number for But we already have seen in Eq. [2.2] that:
one-dimensional motion. So, if a cheetah can accelerate
Xz - x 1
from 0 km/h to I I 0 km/h in 4.0 s, we can use this equation vav =
to find its average acceleration. We need to first convert At
the velocity into units of metres per second: That means the right-hand side of Eq. [2.2] must be equal
to the right-hand side of Eq. [2.6]:
km ( lOOOm)(-l h-) =30.6-=31-.
v2 =1JO- m m
h Ikm 3600s s s (2.7)
m m
30.6- - O- If we substitute Eq. [2.5] in for the v2 in Eq. [2. 7], we have
s s m
a - =7.7-. an equation that relates position to acceleration:
av- 4.0s - Os s2
The average acceleration of a cheetah is around 7.7 m/s2. v1 + (v1 + a
= ____ 3
vAt)
__::..c..__

That means that, on average, it increases its speed by 2


7.7 mis every second.
On the other side of this dining arrangement, the
gazelle is able to accelerate at around 5.0 m/s2. This pales v
in comparison to the cheetah, but it is still more accelera-
tion than any human sprinter can produce. If the gazelle
is starting from rest, we can rewrite Eq. [2.4] to find how
much time it takes for the gazelle to reach its top speed of
75 km/h. Again, the first step is to convert velocity into
units of metres per second:

v2 =75km(IOOOm)(-lh_)=20.8m = 21 m.
h I km 3600s s s

Solving Eq. [2.4] for D.t yields:

m m
20.8- - 0-
At = v2 - vi = s s 4.2 s.
aav m
5.02
s Figure 2.12 An object moves with constant acceleration. Acceleration
!hat means that the gazelle takes 4.2 s to go from rest to is defined as rate of change of velocity with time. Constant acceleration
ns rn aximum
· speed. results in a linear relationship between velocity and time.

CHAPTER 2 I KINEMATICS 29

Scanned with CamScanner


I 2
· · t111s
Reorganmng · an d solv"1ng for x2· we find :
· equation x = xo +vol + 2at

x2 =x1 + v1dt + 2aAt


1 2
,
(2.8) = 62 m + (31 :)(11 s) + Ko~)(l1s)2
where acceleration is written without the "av" subscript = 403m.
because acceleration has been assumed to be constant. This means that, at the end of the 15 s chase, the cheetah
That means average acceleration is equal to instanta- has run a total distance of 403 m. . . .
neous acceleration at every instant. Eq. [2. 1OJ and Eq. [2.11] can be com~med to elimi-
Just like what we did for velocity, the instantaneous nate the variable time. The resulting equ~tion ~an ~om~­
acceleration is defined as: times be used to quickly solve problems m which time is
neither given nor of interest:
dv (2.9)
a=lim-.
61-+0 di
v2 - v6 = 2a(x - xi). (2.12)

Instantaneous acceleration is represented by the slope of Much like Eq. [2. IO] and Eq. [2.11 ], this eq~ati~n
the tangent to the curve on the graph of ~elocity ve~sus is useful only for situations in which acceleration 1s
time. This means we can find the veloc1ty-versus-t1me constant.
graph from the position-versus-time graph, and fur-
thermore the acceleration-versus-time graph from the
velocity-versus-time graph.
EXAMPLE 2.5
MU!iltl The slope of an object's velocity-versus-time How much distance is required for the gazelle to reach its
graph gives the object's instantaneous acceleration. maximum speed?

We can rewrite Eq. [2.8] to express position as a


function of time. Typically, we can choose t 1 = 0, x 1 = Xo,
Solution
To find where the gazelle is at the end of its accelerating
and v = v ; meaning, initially the object is at position x 0
1 0 phase, rearrange Eq. [2.12] to solve for the final position .r:
with velocity v0 . Furthennore, we choose t2 = t, ~2 =~· and
v = v; meaning, at an arbitrary time t, t~e object 1s at x
2
with velocity v. Then, Eq. (2.8] can be written as: v2-vij (
21 m)
s
2
-(o m)s 2

-
x = x 0 + ~ = Om + - 44m.
(2)(s.o :Z)
l 2
(2. l 0)
x=x0 + v0 t + 2at .

Then, the velocity equation Eq. [2.5] becomes:


At the end of the accelerating phase, the gazelle is 44 m
v= v0 +at. (2. l l) from where it started. Notice that we don't know how long
it takes a gazelle to get to its maximum speed. So long
We can use Eq. (2.1O] to find how far the cheetah travels as we are not interested in that number, Eq. [2.12] is the
if it starts from rest at position x 0 = 0 m and accelerates at fastest way to solve for distance.
7.7 mJs2 for the 4.0 sit takes to accelerate to its maximum
speed:
l Fig. 2.13 shows the position of an object as a function
x = x0 + v0 t + 2at2 of time. Between points A and B, the slope of the position-
versus-time curve is positive and constant, which means
7
=Om+ ( 0 }4 s) + ~(7.7 :i}4s)2 = 62m. the velocity is positive and constant and the acceleration
is zero. Between points Band C, the slope of the position-
Jn the first 4 s, the cheetah travels 62 m. To find how versus-time curve is positive and decreasing, which means
far the cheetah travels in the 15 s it has to catch the the velocity is positive and decreasing and the accelera-
gazelle, we will have to break the problem into two tion is negative. Between points C and D , the slope. of
parts: the accelerating phase and the constant velocity the position-versus-time curve is negative and getung
phase. During the accelerating phase, it travels 62 m. We more negative, which means the velocity is negative ~nd
can use the same equation to find how far the cheetah is getting more negative and the acceleration is negauve.
from where it started after it travels the remaining 11 s of Between points D and£, the slope of the position-versus-
its chase. The acceleration during that 11 s time interval time curve is negative and constant, which means 1 ~~
is zero, the initial position is 62 m (where the cheetah velocity is negative and constant and the acceleratio~ 1 ~
is at the end of the accelerating phase), and the initial zero. Without carefully measuring the slope in the region
velocity is 31 mis (the velocity of the cheetah at the end between points B and D, it is hard to know if the slo~~
of the accelerating phase). This gives us the following: of the velocity-versus-time graph is constant, but in th•~

30 PART ONE I THE MECHANICS OF BIOLOGICAL SYSTEMS

Scanned with CamScanner


C D The position of the cheetah at time / can be deter-
B
mined by:
x

.!_a ,2 when I s 4 s
2 c
xc = { 62 + vc(t - 4) when/ > 4s,

A
where ac = 7.7 m/s2 and vc = 31 mis. Meanwhile, the posi-
v tion of the gazelle at time t is:

t I
150+zai when t $ 4.2s

xg= { 150+44+ vgCt-4.2) when t > 4.2 s,

where ag = 5.0 m/s2 and vg = 21 mis. ..


To visualize the relationship between the pos1t1ons
of the animals, we can plot their x-versus-1 (position-
versus-time) curves on the same graph, as shown in
Fig. 2.15. The intercept between the two curves tells
us the time at which the cheetah catches the gazelle.
From Fig. 2.15 we can see that this occurs at 1 > 4.2 s.
Therefore, the time at which the two animals meet
Figure 2.13 The velocity-versus-time graph can be inferred from the can be determined by solving the equation xc = x g for
slope of the position-versus-time graph. The acceleration-versus-time 1 > 4.2s; that is,
graph can be inferred from the slope of the velocity-versus-time graph.

62 + vc(t-4) = 150+44 + vg(t-4.2),


chapter we will consider only constant acceleration, 62+31(1 - 4) = 150+44+21(1-4.2),
whether positive, negative, or zero. Constant acceleration t = 16.8 (s) ""' 17 (s).
produces straight lines on the velocity-versus-time graph.
Since the cheetah can't run at its full power for longer than
15 seconds, the hunt would likely fail.

EXAMPLE 2.6 600


When a gazelle realizes a hungry cheetah is quietly 500
approaching from 150 m away, a desperate chase starts.
400
How long would it take for the cheetah to catch the
gazelle? 300 - x Cheetah (m)
200 - x Gazelle (m)
Solution 100
First, we need to set up a coordinate system. Assuming 0
the chase occurs in a straight line, the animals are in one- 0 10 20 30
dimensional motion and we only need an x-axis ..We can Tune (s)
choose the initial position of the cheetah as the origin
and the positive direction toward the gazelle, as shown in 40
Fig. 2.14. That is, the initial positions of the cheetah and
the gazelle are x Oc = o and x Og = 150 m respectively. 30

20 - v Cheetah (mis)
- v Gazelle (mis)
JO

0
100 400 x (m} 10 20 30
200 300 Time (s)
~~ 2-14 A coordinate system for the description of a cheetah
Rgure 2.15 Position-versus-time and velocity-versus-time graphs
ing a gazelle.
continued
for the cheetah and the gazelle.

CHAPTER 2 I KINEMATICS

Scanned with CamScanner


12
n
10 A
8
E 6
lot
4

0.5 1.5 2 2.5 3 3.5 4 4.5 5


t (s)

Figure 2.16 Position as a function of time for object A and object B.

At time t = 1 s, which of the following correctly describes a comparison of the velocity and
the acceleration of objects A and B?

(a) The velocity of A is larger than B. The acceleration of A is larger than B.

(b) The velocity of B is larger than A. The acceleration of A is larger than B.

(c) The velocity of A is larger than B. The acceleration of B is larger than A.

(d) The velocity of B is larger than A. The acceleration of B is larger than A.

In reality, most biological systems do not produce effects of air resistance for now, and talk about motion
constant acceleration, although it might be a reasonable that occurs when only gravity is allowed to accelerate an
approximation. However, there is one type of accelera- object at 9.8 m/s2 . When that occurs, the object is said to
tion that we are all familiar with and that is well approxi- be in freefall. We can use the equations for motion in one
mated as being constant. The acceleration due to gravity dimension to predict the motion of objects experiencing
at Earth's surface is something you notice any time you freefall acceleration. We simply use acceleration due to
drop a pencil or throw a ball. gravity in place of acceleration.
Acceleration due to gravity points down toward
Earth, and the magnitude of the acceleration is the
same for all matter, regardless of size, shape, or mass.
The magnitude of the acceleration due to gravity, com-
monly represented by g, changes slightly depending on EXAMPLE 2.7
where you are. On Earth's surface, it is approximately A freezing rainstorm on March 2, 2007, resulted in a thick
9.8 m/s 2 . layer of ice coating the top of the CN Tower in Toronto
You have likely noticed that if you drop a sheet of (see Fig. 2.17). When this ice began to melt, it fell to the
paper and a heavy textbook from the same height, they ground below. If ice fell from a height of 550 m, how fast
do not typically reach the floor at the same time. This would it be travelling when it reached the ground, if you
ignore air resistance?
means that the accelerations of the sheet of paper and the
textbook are different. However, if you place the sheet of
Solution
paper directly on top of the textbook and then drop both,
you will notice they fall at the same rate. As you might Assume initial velocity is zero and that the x-axis is vertical
and positive in the upward direction. Also assume the ice
expect, the difference between the two experiments has 2
is experiencing freefall acceleration a = -g = -9.8 rn/S ·
to do with air resistance. When you place the textbook Rewriting Eq. [2.12) to solve for final velocity 1·2• we have:
in front of the sheet of paper, the textbook pushes the air
out of the path of the sheet of paper and allows it to fall
with considerably less air resistance. Objects that have
a smaller surface for the air to interact with experience
less air resistance, and air resistance has a harder time
resisting objects with a lot of mass. We will ignore the

32 PART ONE I THE MECHANICS OF BIOLOGICAL SYSTEMS NEL

Scanned with CamScanner


y

x
Ax = Acos(B)

Figure 2.18 A vector can be projected onto perpendicular axes to pro-


duce x- and y-components.

points in the positive x-direction. Projecting that vector


onto the y -axis gives a vector component that has length
Asin(O) and points in the positive y-direction.
In general, a vector Ain a two-dimensional space can
be written as
(2. 13)
where
Figure 2.17 People often worry that a penny dropped from the
Ax =AcosO,
top of Toronto's CN Tower would reach a speed high enough to kill (2. 14)
someone on the ground. It is a good thing air resistance prevents Ay =AsinO.
this from being a threat. Here f and } are unit vectors along the x - and y -axes
respectiv:Jy. A uni~ vector is a vector whose length is one.
Taking the square root, we find that the speed is a little
!hen, Axi and AyJ are the vector components of vector
over 100 mis. This is more than 300 km/h! It is a good
A; whereas Ax and AY are the corresponding scalar com-
thing air resistance kept it to a small fraction of that
ponents. A scalar component is just a real number that can
speed, otherwise more damage would have occurred. As
be positive, negative, or zero. Since the x - and y-axes are
it was, police evacuated the area and only windows were
perpendicular to each other, a vector along the x-axis has
damaged.
Notice that Eq. [2.12] takes the square of the velocity. no component along the y-axis, and vice versa. That is
That means the solution can be positive or negative. This why an object's motion in the x-direction is independent
equation can be used only to determine the magnitude of of its motion in the y -direction.
the velocity, in other words, the speed. Acceleration due To find the total magnitude of a vector from its
to gravity points toward Earth though, so the velocity must c~mponents, we can use the Pythagorean theorem (see
also have been pointing in that direction. Fig. 2.18):
A= YAx2 + A2y · (2.15)
We can also use the components to find the angle between
f_.3: Motion in Two Dimensions the vector and the x-axis:
I~ the real world, not everything travels along a one- Ay
~tmensional axis. The good news is that the sai:ie eq~a­ tan(} =A. (2.16)
x
ttons we developed to describe motion in one _dime~sion
For more details about vectors, please read Vectors and
can also be used to describe motion in two d1menswns. Basic Vector Algebra in the Math Review of this book.
All we have to do is break the two-dimensional problem In general, the location of an object whose coordi-
into two one-dimensional problems. The reaso~ we ~an nates are (x. y ) can be described by a position vector 1:
do t~~t is that all the key quantities are vectors, tn~luding
POSttton, displacement, velocity, and acceleratwn. A r =xi+ y}. (2. 17)
vector can be broken into components that are indepen- Clearly, the position vector can be well characterized by
dent of each other. its x- and y-coordinates, which are the scalar components
Fig. 2.18 shows a vector of length A that makes ~n of 1. If an object moves from position 1 1 at time t 1 to
a~gle 0 to the x-axis. Projecting that vector onto the x-axis position 1 2 at time t2 , the displacement vector is
gives a vector component that has length Acos(O) and

CHAPTER 2 I KINEMATICS 33
..

Scanned with CamScanner


(2.18) of kngth A in Fig. 2.18 represents velocity or accelera-
tion. Urc:iking motion into x- and y-components is almost
As shown in Fig. 2.19, the displacement vector starts from always the first step for solving motion in two dimensions.
the initial position and ends at the final position. Note that Note that this method can be generalized to the study of
the distance /, which is the length measured along the three-dimension:il motion.
actual path, is always greater than or equal to the magni-
tude of the displacement !Ari. mu@• 13reaking vectors into components allows a two-
dimcnsional motion to be: analyzed as a combination of
y simple one-dimensional motions.

EXAMPLE 2.8
A person jogs in a straight line across a park at a speed of
2.0 mis in the direction of 60° south of east. After 5.0 min-
utes of jogging, how much further south is the person?

Solution
The first step of most problems in physics is to draw a
diagram and set up a coordinate system. Fig. 2.20 is a
diagram that shows the velocity projected onto a two-
dimensional coordinate system. The choice of axes is
arbitrary; however, they must be perpendicular. Because
the question uses cardinal directions (north, east, south,
and west) that are perpendicular to each other, it makes
sense to align our axes such that they are parallel to
these directions. Fig. 2.20 is drawn such that the posi-
tive y-direction is north and the positive x-direction
is east.

Figure 2.19 The initial and final position vectors, the displacement
vector, and the distance. Note that the displacement Ar, which is the y
final position vector minus the initial position vector, is a vector that starts
from the initial position and ends at the final position. Also note that the
distance is greater than the magnitude of the displacement.

2cos (60°)
The average velocity vector is the displacement
divided by the time interval:
-+ -+ -+
.... Ar r2 - rl
v =-= (2.19)
av At f2 - fl .

Then, just as in one-dimensional motion, the instanta-


neous velocity is the limit of the average velocity as the
size of the time interval approaches zero:
_, Ar Figure 2.20 The velocity vector of the jogger is shown in red. The
v = Jim - . (2.20) components of the velocity in the x- and y-directions are shown in
111-+o At
orange.
Similarly, we can define the average acceleration:
_, Av We want to know how far the person jogs in the south
aav = -;;:; (2.21) direction in 5 minutes, so we can ignore the component of
the velocity that points in the east direction. The velocity in
and the instantaneous acceleration: the south direction is v1 = -2.0sin(60°) mi s = - 1.73m/s.
Notice that the answer is negative because the i-~axis is
_,a =
lim - .
Av
(2.22) positive in the north direction, and the y-compon~nt of the
41-o At jogger's velocity is in the south direction. To find how far
Jn general, breaking vectors into components does the person travelled in the south direction over 5 minutes
not change the result. It simply allows a more complex (300 seconds), use Eq. [2.10) and set the initial position
and acceleration to zero:
motion to be analyzed as a combination of simple one-
dimensional motions. This remains true even if the vector

34 PART ONE I THE MECHANICS OF BIOLOGICAL SYSTEMS NEL

Scanned with CamScanner


I ,
y = Yo+vo/+2.a/..

=Om+ (-1.73 7
)(3.ox J02s) +K OJ}3.0x J02s)2
= -520m.
This tells us the jogger moved 520 m to the south during figure 2.21 The horizontal and vertical components of velocity for
that time interval. a projectile. Notice that the horizontal component of the velocity is
Using the same technique, we can also find how far constant, whereas the vertical component is accelerated downward
to the east the jogger moved. In this case, the velocity is by gravity.
vx = 2.0cos(60°) mi s = 1.0 mi s, or 1.0 mis in the east direc-
tion. We can find the x-component of the position vector
using Eq. [2.1 OJ, with the initial position and acceleration set ·
to zero: These eqw 1tions, however, can already be simpli-
fied. For projec ;ie motion, we know ax= 0 and ay = -g.
I
x=xo + Voi + 2ai· So we can 1ewrite the equations that describe projectile

=Om+ ( 1.0 7}3.0 x I02s) + Ko~}3.0 x I02s)2 motion as:

= 3.0 X 102m.
x-direction:x = x0 + v0/
Notice that the distance as the sum of magnitudes of the vx = v0x {constant); (2.23)
components of the position vectors does not add up to
1 2
the distance that the jogger actually travelled. Because y-direction:y =Yo + v0/ - 2gt
the components are perpendicular, they must be summed
using the Pythagorean theorem: Vy= VOy - gt.
d=V(-s20)2+(3.0x102)2 = 6.0 x !02 m.
This is the distance that a person would travel if their
speed was 2.0 mis and they jogged for 5.0 minutes. EXAMPLE 2.9
Fleas are great jumpers. When a flea jumps vertically, it
can reach a maximum height of 18 cm. (a) What is the ini-
One example of two-dimensional motion is worth tial velocity at its takeoff? (b) If the flea jumps at an angle
looking at simply because it is somethi~g _we all h~~e of 60 degrees with respect to the ground, how far can it
experienced. If you take a ball and throw 1t, 1t ha~ an m1- jump?
tial velocity and a constant downward accelerat10n due
to gravity. Motion of this type is generally referred to as Solution
projectile motion. If we set up our axes such that the We first set up a coordinate system, as shown in Fig. 2.22.
y-axis is positive in the upward direction, then the accel- (a) When the flea jumps vertically, e = 90°, its one-
eration due to gravity is negative and acts only to change dimensional motion is along the y-axis with ay = - g.
they-component of the velocity.
Figure 2.21 shows the x- and y-components of
the velocity at different points along the path of a ~all y
thrown at around a 45° angle to the horizontal. Notice
how the ball starts out with a positive y-component of
its velocity· at the peak of the path, the y-component of
the velocity' is zero and after that the y-component fthe
. 1s
. mcreasmgly
. . .
°
Ve!oc1ty negative. 1n marked contrast, the
x-component of the velocity stays the same over the.whole
path. The equations used to describe projectile motwn are
the same as our standard kinematic equations (Eq. [ 2·.1O]
an~ Eq. [2.11 ]), except we now have to include subscnpts x
to identify which axis we are describing:
1
x-direction: x = x 0 + Vo/+ 2.a/2
v =v0 +a/;
x x I 2 figure 2.22 The coordinate system for a jumping flea.
y-direction: y= Yo+ vo/ + 2.a/
vy= v0Y +a/·
CHAPTER 2 I KINEMATICS 35

'It
Scanned with CamScanner
When the flea lands on the ground, Y = O; that is,
Initially, Yo= 0 and vo1•=''o: and at the maximum height,
y = 0.18 m and vy ; O. Then, we can use Eq. (2.~2] 1 2
0= llo/ - 2gt .
to find "o (now we have a y-axis instead of an x-axis):
We can solve for the time it takes to fall back to the ground:
v2 - 1~ = 2a(11 y~ 0)
= -2110..·
-

1 (reject the solution t =


-1i = -2g(v - 0) g
Then, the horizontal position at time t is:
"o= v'2W
~~~~~~~-
2 110.•"ov
"o= v'2 x 9.8 m/s2 x 0.18 m = 1.9 mis. x = 11Oxt = --
g
· = 0.31 m.

(b) When the flea jumps at an angle of o=60°, its motion That means the horizontal distance over which the flea
is two-dimensional projectile motion. The compo- jumps is 31 cm. .
nents of the initial velocity are: Furthermore, the last equation can be written as
v
Ox
= v0cosO = (l.9m/s)cos 60° = 0.95 m/s• 2v0.. v0 ., 2v02 cos0sin0 = _::__
vij sin(28)
__
vOy = v0sin0 = (l.9m/s)sin60° = 1.6 mis. x=-- · =
g g g
The acceleration is the gravitational acceleration; that Here we have used the trigonometric identity
is, ax = O, a.1. = -g. In other wor~s. the motion i_
n t~e sin(20) = 2cos0sin0. Then, the maximum distance occurs
x-direction has a constant velocity, and the motion in o
when 20 = 90°, or = 45°. Therefore, 45° is the optimal
the y-direction has a constant acceleration. Then, the angle of jumping that results in the maximum horizontal
position of the flea is given by Eq. [2.23]:
jumping distance:
x = "oi ,
1 VO
x ma.x = -g = 0.37 m = 37 cm.
Y= "oi - 2g12.

CASE STUDY 2.2


The current theory of the evolution of fligh.t proposes that
intermediate species approximated flight by gliding off tree
branches. We can observe such behaviour in animals such
as the Malayan Wallace's tree frog or the flying dragon, a
Southeast Asian lizard (Fig. 2.23). These animals have mem-
branous extensions in various places along their bodies,
which they can unfold to sustain a gliding fall after jumping
off a tree. As a reference, (a) what formula describes how
an animal that jumps off a tree but cannot glide will fall (for
example, a human), and (b) how must this formula vary for
a Wallace's tree frog or a flying dragon for us to accept that
the animal can successfully glide?

Answer to part (a) The first step we take in solving this


problem is a simplification that is almost always justified in
mechanical problems: we identify a two-dimensional plane
in space in which the relevant physical parameters vary and
the resulting motion occurs. This plane is defined as the
xy-plane, allowing us to use Eqs. [2.23].
We can draw an initial sketch of what happens based
on our everyday experience. This sketch is shown in
Fig. 2.24(a). The object starts at an initial position with
a given initial velocity vector. If distance matters to the
jumping animal it will choose to make this velocity vector
point above the horizontal. We now calculate the actual
path without gliding. If a person jumps off a tree and you
observe the event looking at the plane of motion from the
Figure 2.23 The flying dragon has a membrane that is stretched
side, you would see the path sketched in Fig. 2.24(a).
between elongated ribs and acts as a strut.
We define the horizontal direction as the x-axis and
the vertical direction as the y-axis. Once the animal

36 PART ONE I THE MECHANICS OF BIOLOGICAL SYSTEMS

Scanned with CamScanner


(::t) y has jumped from the tree, the object no longer acceler-
l 'u ates horizontally (a, = 0) and accelerates vertically with
a, =-~. as discussed. They-component of the accelera-
tion is negative because the gravitational acceleration is
pointing downward; that is, in the negative y -direction.
For convenience, the initial position is chosen as the
Yo
origin. This means that we modify the sketch we made
in the schematic approach as shown in Fig. 2.24(b)
such that (x 0 ,yo) = (0, 0). We can now substitute the two
acceleration components in Eqs. (2 .23]:
X:-
o------.....1....-~ X
x
x-direction: x = v0_
,t
(b) y . .
y- d1rcct10n: y = v0_..t - I '
2gt-.
These two formulas are now combined to eliminate the time
variable. This produces a relationship that expresses the
y-position as a function of the x-position:

y = Vo -.::__ - _!..g(-.::_)2
!Y Vox 2 Vox
VOy g
=-x- --x2
Vox 2v6x '
Figure 2.24 (a) Sketch of the path of an object released at position which is mathematically the form of a parabola. Thus, an
(x0, y ~ with an initial velocity vector v 0. The object does not accelerate animal or a person that cannot glide will travel along a
horizontally and moves with a constant gravitational acceleration down- parabola from the branch to the ground. This path is shown
ward. The mathematical treatment of the motion yields a parabolic path in Fig. 2.24 and is an example of projectile motion: any
called a projectile trajectory. (b) The calculations are greatly simplified object under the exclusive control of gravity follows it.
by choosing the position (x0, y~ as the origin (0, 0) in this case. Answer to part (b) Any path with y(x) > y . . (x) means
. . PIOJeclile
t he ob1ect 1s at a greater height than a freely falling object.
Thus, that object is gliding.

2.4: Uniform Circular Motion


Uniform circular motion refers to the motion of an ,,. ,,. ... ....
;

object that moves along a circular path at a constant /


;
''
speed. Although the speed is constant, the velocity is not, /
/
''
I
/ ''
because its direction changes. As illustrated in Figure I '\
I
2.25, the direction of the velocity changes continuously, I
\
\
but is always in the tangential direction of the circle. I
I \

Furthermore, since the velocity keeps changing its di~ec­


\
I I
I I
tion, the acceleration is not zero, even though the object I I
I
neither speeds up nor slows down. As we will see in the I I
I
following discussion the acceleration is always perpen- I
di~ular to the velocit;, and therefore changes the direction I
I
without changing the magnitude of the velocity. \

v . We want to quantify the relationshi~s betwe.en the


\
\

anous parameters that describe a uniform circul~r


\

'
~.otion. To do so, we specify several parameters in '
''
•g. 2.26. Part (a) shows two points, P1 and P2. along ''
the circular path with centre C. The two points are char- ' .... .......
acterized by position vectors 1 and r r2•
which subtend ------------
:n ~ngle 0. The two points on t he path are separat~d ~
displacement Ar. An object at point P 1 has v~locity v, Figure 2.~5 . An object in a uni_for~ circular motion, at times t1 and ~­
2
v
;:nd at point P it has velocity 2. Fig. 2.26(b) illustra~es The velocity 1s along the tangentlal direction with a constant magnitude.
c corresponding relationship between the two velocity

CHAPTER 2 I KINEMATICS 37

Scanned with CamScanner


vectors: they also subtend an angle fJ due to the gcomet· in which the limit of AVI At is and the limit of Ar/At is a,
r, v
rical fact that l. I and 1\1. V2· The difference between v. As shown in Fig. 2.26(b), the direction of which is a,
v1 and \i2 is defined as Sv. the same as that of Av in the limit At--+ 0, points directly
We relate the acceleration to the velocity and the to the centre of the circle. Therefore, this acceleration is ·
radius of the path using the two sketches in Fig. 2.26. called centripetal acceleration. Centripetal is translated
Assume only a very short time At has elapsed as the from Latin meaning centre seeking.
object moved from point P 1 to point P2• That means
W U@i Centripetal acceleration is the acceleration that
both Ar and Av are very short vectors and () is a small
points toward the centre of curvature of the path of motion.
angle. Using trigonometry we find from the two parts of
Fig. 2.26 respectively: It is important to note that the direction of the cen-
tripetal acceleration continuously changes. Therefore, the
.
sm() = - -
\Sr\ centripetal acceleration is not constant even though its
r
(2.24) magnitude is constant.
sinfJ = \Av\
v ,
In a non-uniform circular motion, the speed varies. In
this case, acceleration has a non-zero tangential compo-
nent as well as a centripetal component. We often use a1
where \Ar\ represents the magnitude of Ar, and \Av\ rep- to represent the centripetal component and a to represent
resents the magnitude of Av. 11
the tangential component. In general, the subscript 11 is
used to refer to the tangential component of the motion,
and the subscript l. is used to refer to the perpendicular
(a)
component of the motion.
The period T is the time required to complete a full
cycle. Since the circumference of a circle has length
27Tr, this definition allows us to rewrite the magnitude of
velocity as 27Tr/T. Substituting this in Eq. [2.26] leads to
another formula for centripetal acceleration:
v2 47T2r
a
.L
= -r = -
T2 .
(2.27)

Centripetal acceleration is larger for an object moving


around the centre with a shorter period, and it is larger for
an object at a greater distance from the centre of the path.

CONCEPT QUESTION 2.6


~gure 2.26 Position and velocity vectors for an object in a uniform
circular motion. at times ~ and fi· The rotation axis of planet Earth is a line that joins the North
and South Poles. Your distance to that axis detennines the
acceleration that you must experience to continue rotating
In Eq. [2.24), we simplified the notation to
P\\ = \12\ = r and \v 1\ = \v2\ =
v since both position
with the planet. Where on planet Earth is a .L equal to zero?

vectors and both velocity vectors have the same magni- (a) the equator
tude for an object in uniform circular motion. Next we (b) the South Pole
combine both formulas in Eq. [2.24):
(c) the North Pole

\Av\= ~\Ar\.
r
(2.25) (d) both the North and the South Pole

We divide Eq. [2.25) on both sides by the elapsed time


interval l:.t: ; EXAMPLE 2~10 . . . . . .
\L\v\
--=--
v\Ar\ : Calculate the .centripetal acceleration du.e to the rotatio~ '.
At r At of Earth of a person in New York. New York is located a .
· 40.s• northern geographical latitude. Note that this accel· ·
Taking the limit At --+ 0 on both sides, we get the magni- '. · eration is independent from the gravitational accelera· ·
tude of the acceleration: · tion caused by the attraction between the Earth and ttie
v2 ~ person. Use REanti 6370 km for the radius of Eart~·
=
a=-;• (2.26) : . . . ' .. lfll/Jlill'!
-
~i
- •
.• . ~---
. ' ' -
. ~ .. . •
""-
~ ...
~
~,., . ...

38 P.ART ONE I THE MECHANICS OF BIOLOGICAL SYSTEMS

Scanned with CamScanner


Solution
The period of Earth is 1 day, which corresponds to
86400 s. As noted in Concept Question 2.6, a point at
o=40.8° above the equator moves on a circle with radius
r = Rr.anhcosO, as illustrated in Fig. 2.27. This allows us to
calculate the centripetal acceleration required to keep a
person in New York on the ground:

4rr 2REanhcosO
a=

which, with the specific values, leads to:


47T 2(6.37 X I06 m)cos(40.8°)
a=
(8.64 x I 04s)2
= 0.0255 m/s2•
Note that this result is much smaller than gravitational
acceleration.

Figure 2.28 A laboratory centrifuge.

Figure 2.27 Illustration of the position of New York, at latitude The test tubes must be carefully supported in the
O = 40.8°. Note that the distance to the axis, r, is different from centrifuge to withstand this tremendous acceleration!
Earth's radius, R.

EXAMPLE 2.11
A centrifuge spins with an angular frequency of 5.0 X
104 rpm (revolutions per minute). The bottom of the test 2.5: Physiological Detection of
tube in this centrifuge is 13 cm from the axis of rotation.
Find the effective value of the centripetal acceleration as
Velocity
a multiple of the gravitational acceleration g at the bottom Given the importance of motion to our everyday life ·t
of the test tube.
is int~resti~g to look at one of the ways our body dete~t~
velocity. Fig. 2.29 shows a light micrograph of a hum
Solution
fingertip, including Meissner's corpuscles just below t~
The period is epidermis. The functional role ofMeissner's corpuscles e
. d as
T= 1 = 2 ox 10-smin=0.0012s.
ve Ioc1ty etectors relies on the fact that they are very near
Th 5.0 x 104 • the surface of the palm. Note also that they are located
e centripetal acceleration is: near the steepest slope of the ridge-valley pattern in th
= 47T2r = 4rr2 X 0.13 = 3.56 X 106m.
fin~ertips. Meissn~r·s corpuscles measure the speed ~ 0
a J. T2 0.0012 2 52 objects that. come into contact with the skin • Eacl1 cor-
a J. = 3.56 X t06rnJs2 = 3 6 X 10s.
puscle consists of a stack of cells in the shape of a 11· _
"d . h h . . n e •p
That. g 9.8 mis
2 • s?' , wit t e more rigid cell ~ucle i l~cated on alternating
IS, a J. =360QQQg. sides from cell to cell. Dendrites are intertwined between
these cells.

CHAPTER 2 I KINEMATICS
39

Scanned with CamScanner


The mechanism by which a Meissner's corpuscle
detects velocity is illustrated in Fig. 2.30. Shown is a
piston just bdore and while pressing the skin oftht: finger.
!he corpus~le gets squeezed, which leads to a change
m the stack mg structun: of the cells. As a consequence,
the intertwined dcndrit1: is bent, causing nerve impulses
to the brain. Th1: impulse rate, which is the number of
impulses sent per second, is higher wh1.:n the dendrite is
bent faster.

2.6: Physiological Detection of


Acceleration
Since acceleration plays a key role in kinematics, par-
ticularly in predicting the motion of an object, it is cru-
cial for organisms to detect and measure accelerations.
Interestingly, vision is not particularly effective for this
purpose. You know this from your everyday experience:
u
.!:
if a small insect moves somewhere in your field of vision,
.."..... ....._... ] you immediately notice it. Compare this to your visual
~ sensitivity for accelerations. When you sit in an airplane
~ you feel the thrust of the aircraft immediately when the
~ pilot receives clearance for takeoff (as the acceleration
~ pushes you into your seat). But, if you watch the initial
:.0
-~ acceleration from an observation deck, you have the
~ impression that the airplane takes forever to get off the
= end of the tarmac. Thus, our vision is not sensitive to
~
cs acceleration, only to velocities.
The human body is capable of detecting accelerations
Figure 2.29 Light micrograph of a Meissner's corpuscle in the skin of
in two ways: accelerations of objects in contact with our
a human fingertip. Shown is an oval-shaped corpuscle of approximately
I oo µ.m in length, with cells stacked in an alternating arrangement of the
skin and accelerations of our own body, particularly the
cell nuclei. The dendrite is intertwined between the cells. The corpuscle is head. Pacinian corpuscles, which are acceleration sen-
surrounded by numerous densely stained epidermal cells. sors located in the skin, allow us to detect acceleration in

Nerve

Figure 2.30 Mechanism of Meissner's corpuscles. At left, a piston is shown at time zero approaching the skin surface with velocity v.At right, the same
area is shown after the piston has pushed the skin a certain distance down. The piston is assumed to move with constant speed. Meissner's corpuscles
get deformed in the process.

40 PART ONE I THE MECHANICS OF BIOLOGICAL SYSTEMS

Scanned with CamScanner


CASE STUDY 2.3
Fig. 2.31 shows both a linear plot and a double-logarithmic Answer to part (a) The linear plot shows variations in the
representation of the relationship between velocity and steepness of the curve. The regions of greatest steepness
nerve impulse rate for Meissner's corpuscles. What can represent the regions of greatest velocity sensitivity. Thus, the
you conclude from (a) the linear plot and (b) the double- linear graph shows that the greatest sensitivity of Meissner's
logarithmic plot in Fig. 2.31? corpuscles occurs at very small speeds-less than v = 1 cm/s.

(a)

(b)

-I
10 10
~
~
-I
~
.... ~

.,,
~ ~ 5
:; ..."'
:;"'
0..
.§ 5
"'
0.
..§
2

5 10 0.1 10
v (mm/s) v (mm/s)

Figure 2.31 The nerve impulse rate for Meissner's corpuscles as a function of the speed of an approaching object. The impulse rate is given in unit
impulses per second, and the speed is given in the units millimetres per second. (a) A linear plot of the data showing the greatest sensitivity to speeds
less than 1 emfs. (b) In the double-logarithmic plot, the same data fall on a straight line.

They are suitable for distinguishing fonns of gentle touching, in which v is the speed of the object making contact
not for measuring the speed of an incoming baseball! with the skin. If the pre-factor a or the exponent b in this
power law is not constant, deviations from a straight line
Answer to part (b) The double-logarithmic plot al!ows us ~o in the double-logarithmic plot occur. Any deviation from
distinguish whether one or more mechanisms are involved rn a straight line is, therefore, an indication that additional
a process, because it shows straight-line segments for each physical explanations are needed. In this case, how-
power law. In the specific case of Fig. 2.31(b), the power law ever, no such deviations occur, suggesting the single
for the nerve impulse rate, P, is written in the fonn mechanism illustrated in Fig. 2.30 is sufficient to explain
P= avb,
Meissner's corpuscles.

Accelerations of our head are measured by the semi-


objects that we touch. The sense that tells us if our body is
circular canals of the vestibular organ. ·The semicircular
being accelerated is located in our head, as part of t~e ear.
. 2.32 presents an overview . canals consist of three orthogonal, crescent-shaped tubes
Fig. o f t h e ear. The vestibular
filled with a fluid called the endolymph. The orthogonal
organ consists of two components: orientation of the three tubes allows the brain to detect
. hlighted in yel- acceleration along the three Cartesian coordinates: the
• the orthogonal semicircular cana Is, h ig
low, which measure accelerations, and acceleration components right and left, back and forth,
and up and down.
• th e maculae (plural of macuIa ), w h 1·ch are 0.3 mm The mechanism of the semicircular canals is illus-
. h ·circular cana1s.
Wide spots located just below t e semi d trated in Fig. 2.33. Resting on the crista is the swivel-
The maculae are indicated in Fig. 2.3 2 as elongat~ mounted cupula. Dendrites reach from the neuron
red bars. The upper one is called the utricular ma~u a embedded in the crista into the cupula. While the head
and the lower one the saccular macula, b~cause ~ ~y
1

1 is motionless, the endolymph surrounding the cupula


are located in small chambers called the utncle an · t e is at rest. When the head accelerates parallel to the
asure the onen-
saccule respectively. Both macu Iae me. 't orientation of a semicircular canal, the inertia of the
tation of the head relative to the direction of gravi y.
CHAPTER 2 I KINEMATICS 41

Scanned with CamScanner


Its inertia c<1uses the water to stay behind. The flow 01
the erH.Jolymph pushes the cupula to the side so that the
tilting is sensed by the nerve endings in the cupula and
is communicated to the brain. Note that the semicircular
canals cannot sense speed because motion with constant
speed does not cause an acceleration of the .cndolymph
against the cupula. If you move your head with co~stant
speed, the endolymph is at rest and the cupula retarns an
upright position, in the same fashion as if the head were
motionless.
Acceleration detectors developed early in animals
and are widespread among vertebrates. The lateral line
Figure 2.32 Overview of the human ear. There are three main sections system in fish is illustrated in Fig. 2.34. You see this canal
of the ear: the outer ear, with the auditory canal (1) ending at the eardrum; system as lines running the full length of the fish, from
the middle ear, with the three ossicles: the hammer, anvil, and stirrup
the gills to the tail along both sides (trunk canals); the pat-
(from left; 2); and the inner ear, with the vestibular organ. The vestibular
organ includes the semicircular canals (3), which we discuss in the con-
tern at the head is highlighted in Fig. 2.34(a). The canals
text of acceleration detection in the head, and the maculae (4), which we lie below the scales of the fish. When the fish acceler-
discuss in the context of gravity detection. ates, water pushes against neuromasts (see Fig. 2.34(b)),
which are the equivalent component to the cupula of the
semicircular canal system in the human ear. Water flowing
endolymph results in flow of the endolymph in the direc- past the bendable neuromast causes dendrites that reach
tion opposite to the direction of the acceleration. This into the neuromast to bend and trigger a nerve signal to
phenomenon can be simulated by holding a half-full
glass of water in your hand and suddenly accelerating it.

(a)

HEAD AT REST
(b)

Figure 2.34 The mechanism of the lateral line system of fish. Fi~~
!
can detect the acceleration of water as it moves past their body. (a).T
ACCELERATION lateral line system consists of a system of canals with external openin~
OF HEAD that run below the fish's epidermis between the scales. The line systers
in the head region is highlighted by thick Jines. Acceleration recepto ~
Agure 2.33 Sketch of the mechanism of the semicircular canals in the
are located at various points along these canals, indicated by small ti~
vestibular organ of the Inner ear. The top part shows the head motionless.
marks along the highlighted lines. (b) The receptor consists of a veof
When the head accelerates toward the right, as shown in the bottom pare,
the endolymph (2) flows because of its Inertia toward the left. This pushes
similar arrangement to that of the cupula in the semicircular canals rt
the vestibular organ: the cupula rests on hair cells with hair-like sen~
I
I
the cupula (1) resting on the crista (6). The cupula tilts, bending the den-
extensions reaching into the base of the cupula. When these sen terTI
drites (3) that belong to a neuron (4) embedded in the crista, triggering a
hairs are bent, a nerve signal ls sent to the centre of the nervous sys
signal In a nerve (5) to the brain.
of the fish.

42 PART ONE I THE MECHANICS OF BIOLOGICAL SYSTEMS

Scanned with CamScanner


Freefall: motion where the only acceleration of an object is
the animal's brain. The lateral line system allows fish to • the acceleration due to gravity:
monitor not only their own accelerations, but also water
pressure changes due to other moving objects (such as
g = 9.8 m/s2
predators or prey) and low-frequency sounds carried
through the water. Resultant vector: The vector that results from the addition
• or subtraction of multiple vectors
• Projectile motion: freefall motion where a component of the
SUMMARY initial velocity is perpendicular to the acceleration:

DEFINITIONS x-direction: x = x0 + v0/ and vx = v0x = constant, and


• One-dimensional motion: motion that takes place along a
straight line I
+ vOy t - -gt2 and v
Y-direction-· y = y 0 2 y = vOy - gt
• Vector: a measurement that has both a magnitude and a
direction • Uniform circular motion: motion with zero tangential accel-
eration along a circular path of constant radius
• Position vector: a measurement showing the position of an
object relative to the origin • Centripetal acceleration: acceleration that points toward the
centre of curvature of the path of motion:
• Displacement: the magnitude and direction of the straight-
line path between two position vectors: v2
a =-
1- r

UNITS
• Distance: the length between two locations measured along
the actual path of travel
• Displacement x: m
• Velocity v: mis
• Velocity: the vector that describes how fast an object is
moving, and the direction of motion • Acceleration a: m/s2

• Speed: the magnitude of the velocity vector


• Average velocity: the constant velocity that would produce
a given displacement:
MULTIPLE-CHOICE QUESTIONS
....
-+
Ar
-+
r2 - r,
-+ MC-2.1. A student walks from home to school and back again.
v =-= Which of the following is most correct?
av At t2 - ti
(a) The student has zero displacement and zero aver-
• Average speed: the constant speed that would produce a age speed .
given distance of travel (b) The student has zero distance travelled and posi-
tive average velocity.
I Instantaneous velocity: the velocity at a particular instant:
(c) The student has zero average velocity and zero

v=lim-
Ar distance travelled.
'11-+0 At (d) The student has zero displacement and positive
I Tangent: the best straight-line approximation to a curve at average speed.
MC-2.2. An object has zero acceleration in the x-direction. In
a given point
they-direction, its acceleration must be which of th
I
Acceleration: the vector that describes the change in veloc- following? e
ity over a time interval (a) positive
I (b) negative
Average acceleration: the constant acceleration that would
(c) zero
produce a given change in velocity:
(d) Any of the previous answers is possible.
MC-2.3. A father ~ra~s his daughter by the hands and spins her
-+
v
A ....
u
V2 - Vi
a =-= around him m a nearly horizontal circle. The daugh-
av At t2 - ti ter's motion is clockwise around the father wh
I . d m
~nstantaneous acceleration: the acceleration at a particular ~'1ewe. fr~m abov~. Th: dau~hter's acceleration points
instant: 1~ a direct10n that 1s pomted m approximately a direc-
t10n that goes from the centre of her body to her
....a= hm
. -Av (a) right.
.11-+o At (b) left .
• ~.cel~ration due to gravity: the acceleration ca~sed by an
(c) father.
(d) feet.
~cct s mass. On Earth's surface, this acceleration can be
approx·1
mated as constant.

CHAPTER 2 I KINEMATICS

Scanned with CamScanner


Thermal
Physics

E
nergy in a mechanical system is contained in the exchange of heat between the system and the environ-
sy~te1~ 's position (potenti~I energy) or its movement ment). All practical processes of interest can be modelled
(kmet1c energy). Mechanical energy is transferred using these four, including the important cyclic processes
as work. In a non-mechanical system, energy comes in in respiration and blood circulation.
a different form. In a thermal system, energy can also be Building on our exploration of the four basic pro-
transferred in the form of heat. If heat is transferred to cesses, and their application to cyclic processes, we will
the system, we would say that the system has received be able to formulate the second law of thermodynamics.
heat. The system can also transfer heat to its environment, The first law of thermodynamics is not enough by itself
in which case we say that the system has released heat. to fully characterize systems that undergo dynamic pro-
Think about how you feel when you exercise. Your mus- cesses such as chemical reactions. The first law distin-
cles do mechanical work (let's say by lifting weights), but guishes possible and impossible processes based on the
thennal energy is also released as heat. conservation of energy. But it does not allow us to iden-
. To explore this, we begin by looking at the relation- tify the spontaneous direction of a possible process. The
s_hip between work and energy in gases. We will con- second law can also do this for physiological systems. It
hn~e .to use the human respiratory system as an example, identifies entropy as a variable of the state of a system
building on what we learned in Chapter 8 to look at how that remains constant in an isolated system with revers-
Work and energy interact when we are actively breathing, ible processes, but increases for spontaneous irreversible
~dhow they depend on pressure, volume, an d tem- processes.
P~rature. This will lead us to formulate the first law of
t errnodynamics, which is a generalization of the law of
conserv .
ation of energy
9.1: Quantitative Representation
h
Th .
errnal systems are defined by the transfer of work, of Dynamic Breathing
eat,idand energy, and by changes in
the · the parame te rs of
So far, we have looked at the physics of the respi:atory
Pe eal gas law that contribute to these: pressure, tem-
rature . I . system at rest. Now we will move on _to the physics ~f
Ch ' and volume. We first saw the ideal gas aw 111 d mic breathing to illustrate an important phys1-
apter 8 ·
cess
es th
· There are four basic thermodynamic pro-
. ofo"~cal example of the relationship betwe~n pressure,
or Volulllat vary in whether or not pressure, temperatur~, g k d energy in a non-mechamcal system.
volume. wor ' an d J
done e change, and therefore whether or not work ts . ' used to describe pressure an vo ume
lllodynor en. ergy or heat are transferred. The baste · ti1er- The p- V d1~gram . le breath (fig. 9. I) is different from
patterns dunng a sing nd volume when we hold
Provid arn1c processes we will focus on in this chapter the one that represents pressure a
leadin: useful insight into thermal systems, without
our breath (fig..8.8)._ f' 9 1 represent the respiratory
~re is0 ~o _complex mathematical formulations. They The solid Imes m tg. : t tic) breathing, meaning
150ther: one Processes (processes at constant volume),
system with very. s~ow ([;;;~:~s:esistance in the air pas-
t~re}, is0 ~I ~rocesses (processes at constant tempera- that there is neghg1ble a I looking at the volume
and ad1ab
. ar1c. pr· 0 cesses (processes at constan t pressure), Note that we are on y
atic processes (.processes that do no t allow for sageways.
THERMAL PHYSI C
s z33
~lt CHAPTER 9 I
~

Scanned with CamScanner


(a) (b)
9.4: Heat and the First Law
of Thermodynamics
Now that we have established the relationship between
work and energy and the influence of pressure and volume
on them in a gas system, let us look at the thermal energy
of a non-mechanical system. Thennal energy is stored in a
system's temperature. In this case, we will devise an exper-
iment in which only the temperature of a system changes.
We will not identify the energy that flows into or out of
the system as work. We instead call this energy flow heat.
We could a rgue that we should redefine the term
1
work to include heat. This would be possible if heat and
work were just two forms of the same thing. However,
heat is distinct from work. The distinction between heat
and work will become clear when we discuss the second
law of thermodynamics later in the chapter. Figure 9.12 Conceptual sketches of Joule's two experiments.
When we talk about how heat moves in a system, (a) In the first experiment, heat is quantified by measuring the amoont
we usually say it is either received or released, as these of gas burned. (b) The second experiment conceptually illustrates that
terms help describe both the transfer and the direction of mechanical and thermal energy are equivalent. The mechanical energy,
energy flow. which is released by the falling object at the left, is used to heat thewater
by operating a stirrer.

9.4.1: Joule's Experiments This means that a material-specific constant is needed.


These observations allowed Joule to write the following
Anders Celsius's invention of the thermometer made
definition of heat:
it possible to properly define thermal energy. In 1798,
Benjamin Thompson (Count Rumford) concluded from Q = cmAT, (9.3)
cannon drilling experiments that mechanical work leads
in which the material constant c is called the specific beat
to heating, which in turn represents an increase in the
capacity (see list in Table 9.1 ). The heat flowing into
thermal energy of a system. In 1842, Julius Robert von
the beaker is then identified as the change in the thermal
Mayer studied human blood in a tropical environment,
energy of the system:
and found a correlation between the thermal energy and
the work obtained from chemical energy. ~errmann von 9.4l
Q = AEthcrmal = Ethermal.f - Ethennal.i" (
Helmholtz formulated the first law of thermodynamics
and credited it to Mayer's work.
In 1843, James Prescott Joule defined both thermal ~TA~B~L=E~9~.1=---~~~~~~~~~~----­
energy and heat. He connected them quantitatively to the Speci f i c heat capacity for different materials at
concepts of work and mechanical energy. His arguments atmospheric pressure
were based on two experiments (Fig. 9.12). In one
[Fig. 9. l 2(a)], just the temperature in the system of Specific heat capacitY
interest changed-in this case the water in a beaker. The Material (J/kg K)
other [Fig. 9.12(b)] then relates the transfer of energy in Water 4186
the form of heat or work.
Ice 2090
Joule used the first experiment to define heat, Q. The
Steam 2010
heat source in this experiment is a combustible gas burner.
Joule varied the amount of heat added to a system, and Aluminium 900
measured the resulting temperature of the water. Note that Iron 448
burning twice the amount of the same gas will release
Glass 837
twice the amount of heat.
Joule showed that the system temperature is directly Copper 387
proportional to the heat added to the system, Q oc AT. Mercury 138
Joule further noted that the heat that is needed to achieve a Gold 129
particular tempe rature increase is directly proportional to
the amount of water in the beaker, Q :x: m(H 0). Finally,
2 H1.stoncally,
· · troduce0
the unit calorie (cal) was in ,o•''1.;
liquids differed in the amount of heat needed to generate 01
heat, and the unit Calorie (Cal) for the energY co 001et1
a given temperature increase of a fixed amount of liquid.
food, with 1 Cal = 1000 cal. You should always c
ii' .
240 PART TWO I ENERGY. THERMODYNAMICS, AND TRANSPORT PHENOMENA

Scanned with CamScanner


aloric'·

the SI 1111it
to
. joule
. (I cal = 4 · 184 J) · Tl ic lllllt. for
.
~pc:citk hc:it capac ity. is J/(kg K). We call it he·c11 capac11v
• .
To quantify the conversion of potential to thermal
• . energy, we need to ensure it is not converted into the
bcc:iusc a. materia l \\'Ith a larg1: sp1.:cific heat capacity h;s
kinetic energy of the falling object, meaning that the
the capa~·1t~· to absorb more heat bdore the temperature
object must fall with a constant speed. Experiments such
rises by a g1n:n an~mmt than does a material with a small
as the one in Fig. 9. I 2(b) show that thermal energy and
spc:cific heat capacity.
mt:chanical energy arc equivalent. In Joule's case, the lost
Eq. [9.3] is otkn rewritten to replace the mass of the
potential energy 6.Er(•I is quantitatively converted into
liquid by the amount of material in unit mol. I !ere we thermal energy 6.£11crma •
define the number of moles 11 as: 1 1·

m
-mbluckg!ly = ell20 111112oAT, (9.7)
II = M' (9.5) which is generalized to:
in which /11 is the mass of the material and Mis its molar £pot + £thermal = const. (9.8)
mass. This leads to:
Q = C11!lT, (9.6) CASE STUDY 9.1
with C the molar heat capacity, which is a material con-
stant with unit J/(mol K). You may see either the specific In using the experiment in Fig. 9.12(b) to prove the equiva-
heat capacity c or the molar heat capacity C used for heat lence of thermal and mechanical energy, what should not
happen or should not be done?
capacity-always confirm the units used and then use
either Eq. (9.3) or Eq. (9.6). (a) The liquid will become warmer.
Eqs. (9.3) and (9.6) allow for an exchange of energy
(b) The object suspended from the string will
in two directions: heat can flow into or out of a system. approach the ground.
This is consistent with our observation for work: a system
can do work on a piston, or the piston can do work on the (c) A liquid other than water should be used.
system. Thus, we need to adhere to a strict sign convention (d) The stirrer axis will spin faster and faster.
for both work and heat.
(e) The ambient pressure will rise above the normal air
pressure.
mJE Any amount of heat flowing into the system and
any work done on the system are positive, as they increase Answer (d). In (d), potential energy is converted into kinetic
the total energy of the system. The opposite processes are energy rather than mechanical converted into thermal
negative, as they lead to a reduction of the total energy of energy.
the system. Choices (a) and (b) must happen for the experiment
to work. Choices (c) and (e) can apply in Eq. [9. 7] if we
When checking a calculation, imagine yourself as the properly exchange the specific heat of water for that of
system: whatever you receive is positive, whatever you the liquid used, c 1;quid'
give away is negative.
Now let us review the difference between heat and
thermal energy in a physiological context. The tem- EXAMPLE9.5
perature detectors in the bodies of endotherms that we (a) Calculate the work that an object of m = 400 g can do
discussed in Chapter 8 obviously are not needed in ecto- as a result of falling a distance of 3 m. (b) Assume that
thenns, as their body temperature adjusts to the environ- the object falls into 10 L of water in an isolated beaker.
mental temperature automatically. Thus, ectotherms are If the entire kinetic energy of the object is converted to
usually not able to measure the thermal energy content in thermal energy, by how much does the water temperature
th . rise? Hint: For the specific heat capacity of water, use the
eir body. However some ectotherms are able to measure
value from Table 9.1, and neglect the heat the object can
heat arriving from the environment. They do this in a dif-
absorb.
ferent manner and for a different purpose: vipers are noc-
turnal hunters that possess heat detectors between their
Solution
eyes and nostrils to allow them to find their endothermic
Prey eVen when they cannot see them. Solution to part (a): By dropping the object 3 m, its potential
energy has been reduced by:

9·4.2: General Form of the Conservation Mpo1 = mg(hr - h)

of Energy = (0.4kg)(9.s$}-3.0m) =-II.SJ.

~ow, let's look at Joule's second experiment [Fig. 9.12(b)]. Solution to part (b): When the object is ~eleas~ e~d
there, when the object on the left falls, it moves the stirrer
s/ough the water. The water resists the motion of the
,.,,...
falling toward Earth, it is converting potential to k1net1c

irrer er . . . that heats the water.


' eating friction
CHAPTER g I THERMAL PHYSICS 241

Scanned with CamScanner


. . d · tllcir days. such as Enrico Fermi's 194,
1mag1nc 111 •
. · 11 experiment that led to the development
energy. As it comes to rest in the isolated beaker, ~t:~~n:t~ nuclear I1ss10 •
. d. llticlcar power technology. The first law of
energy must convert to thermal energy, ll.£mrm1at - ·. 0 I mo cm · . . .
. th eleased mechantca1 . 1.11·cs is not hm1tcd to mechanical systems
This value is positive smce e r thermo dyn.11 . . .. ·
. dded as thermal energy to the system (water). and therefore surpasses the range ol apphcab1hty of
energy 1s a
We use Eq. l9.3l in the form: Nl!wton 's laws.
""
;.>.Lthermal = c H 0 m II :0~f.
2
Energy In a Closed System
Next we determine the associated change in the Now let us consider only the water as t~e system. Th~
water temperature, ll.T'. nd side in Eq. (9.7] 1s a work term
means ttlC left-!la .
~Ethcnnal 't presents energ•y that flows into the. system.
ll.T= V c ' because 1 re _. .
P11p H20 llzO We combine this observation with Fig. 9.12(a), m which
heat is flowing into a system. I~ b?th cases, the ener~
=
in which PHp is the density of water: P11:0 1.0 g/cm3. The
flow leads to the same change m internal energy. ~1s
value for c11p in the denominator is taken from Table 9.1 .
a\lows us to formulate the first law of thermodynam1n
This leads to:
l l.8J for a closed system:
(i.o k~)(loL) 4.186 Kkg
ll.T = ( kJ )
l3uij@1I The sum o f a 11 energy fio rms in a closed system
changes by the amounts of heat and work that floll'
= 2.8 x 10- 4 K.
between the system and its environment.
So, the temperature change is negligible.

This is written as:


(9.10)
9.4.3: The First law of Thermodynamics t::.Uclosed system = Q+ W.
We can look at Joule's experiment [Fig. 9.12(b)] in two In this equation, the change of the internal energy as~
ways: system goes from an initial to a final state does no~ de 1
fer in or ou
• the falling object, the stirrer, the beaker, and the water on what form (heat or work) the energy trans 11
,.th
together can be considered an isolated system, or of the system takes or in what order it is exchanged ' 00
' d ds only
the environment. A system property that epe~ histon',
• the water in the beaker can be considered the sys- the system's current state and not on its detailed . ·•
. tern, and the stirrer and the falling object as the ' rg)' JS•
is called a variable of the state. Thus, internal ene ~ res·
environment. variable of the state of the system, like temperature. pteis
We will now look at how each of these scenarios differs sure, and volume. The change of a variable of the sta tetll
in how we view the total energy ofthe isolated superstructure, zero for any sequence of processes that returns the sysdis·
the conversion of energy between different forms, and the to its original state. Cyclic processes like these are
flow of energy between system and environment. cussed later in this chapter.
Energy in an Isolated System
When we consider all components in Fig. 9.12(b) as an EXAMPLE9.6
etriC tesl
isolated system, Eq. [9.8) states that energy conservation A standard man performs the standard ergorn nzontal
also applies to non-mechanical energies, such as thermal illustrated in Fig. 9.13. The test consists of a 6 rn hO t niflt
energy. In \ 847, Hermann von Helmholtz generalized this run followed by running up a staircase for at leas~
finding to formulate the first law of thermodynamics for steps. For our calculations, we focus only on the;ard bY
an isolated system: part of the test, in which the person moves upthis ~
h = 1.05 m from the third to the ninth step. foreel (f~
WU 1!11i Conservation of energy: the sum of all energy of the test the person requires o 8 kcal of stor pers°"
forms in an isolated system is constant. energy. How much heat must the body of the ·c test?
dissipate as the result of this part of the erg~rn~~e steP
The sum of a\\ energy forms in the system is the Supplementary physiological information. tact,,,#
internal energy of the system. As we introduced in test in Fig. 9.13 is usually supplemented bY con tirt1e ~
on every third step to allow a measure of th~dered 111
8
Chapter 8 for the ideal gas, the internal energy is labelled
~· T~us, the energy conservation for an isolated system patient needs to complete the test. It was cons'.ces s~
ts wntten as: standard ergometric test until stationary de~\0 use· iS
as ~icycle e~gometers and treadmills c~rne i ~ t,5· II~
1

~U.1soIated system = 0· (9.9) stationary bicycle ergometer is shown 1n F ~t ,1


used for exercise ECGs in which several vital t ~o te•
This is. a law because it predicts the outcome of futu re teS ·
are measured on the patient during the of 11-1: ;
expenments.
. It even correctly predicted the outcomeof
athletes for fitness, coaches use measurements "1"
expenments that Joule and Helmholtz could not have

242 PART TWO I ENERGY, THERMODYNAMICS ANO TRANSPORT p


' HENOM~~A

Scanned with CamScanner


9.5: Basic Thermodynamic
Processes
i
h Now that we have looked at the first law of thermody-
namics, let us examine some basic, fundamental ther-

----------- ------
l modynamic 1>rocesscs. To do this, we will use a model
system that includes a closed container with an ideal gas
(Fig. 9.14). The system includes a mobile ideal piston that
6m allows us to exchange work, W, with the gas. Work can be
done on or by the gas. There is also a heat source (called
figure 9.13 Standard ergometric test. The person must run 6 m a heat reservoir) that allows us to exchange heat, Q, with
toward a staircase and then move upward at least 9 stairs. Contact the gas. Heat can be released or taken up by the gas.
mats on the third, sixth, and ninth stairs record the motion. The height
difference between the third and the ninth step is h = 1.05 m.

consumption, heart rate, and lactose levels in the blood


of the athlete to predict performance in track and field
events.

Solution
Here we use conservation of energy for the person Reservoir
moving between the initial position on the third step and
Figure 9.14 Model system for the discussion of the basic processes in
the final position on the ninth step. Treating the person as
thermodynamics. An ideal gas is sealed in a container (closed system with
an isolated system, the formula for the conservation of
internal energy U). Work Wis exchanged with the environment via motion
energy (Eq. [9.9]) can be written in the form:
of a frictionless piston. Heat ais exchanged with a heat reservoir in the
environment. The heat reservoir is ideal, meaning that its temperature
does not change during heat transfer.
= £pot + Ekin + £thermal + £chemical·
Food energy is E h . 1• The kinetic energy in the
. c==
equation can be neglected if we assume that the person s
' When we refer to a heat reservoir, we think of it as
one of two practica l devices you likely know from your
Speed between the third and ninth steps does not change.
chemistry and physics laboratories:
For the other three forms of energy, we use the last
equation in the form: • a simple Bunsen burner, when we a llow for heating of
£1'«.i + £thermal.i + Echemical,i = Epot,; + Ethennal,f + EchemicaJ,f) the system with a source that is at a higher tempera-
Which is written as: ture than that of the system;

A£pot + AEthermal + AEchcmical = O. • an oil bath (or heating j acket), w hen we a llow heat
to flow into or out of the system when brought into
Each term represents the difference between the final
and ·.. w
initial amounts of energy in the system. e wan
t thennal contact with the gas under equi librium con-
ditions. This means that the system and the bath are
~Calculate the thermal energy difference, AE1hennat· The at the same temperature. The oil bath is an ideal heat
er two terms are:
reservoir when we assume that it ma intains a given
AEl'OI == mg(hr - hi) temperature even during heat exchange.

anQ·
== (70kg)(9.8~){l.05m) = +7201 In ca
lculations, we can idealize the heat reservoir. An
. . h' h h
ideal heat reservoir is a hea~ r~servo'.r m w 1c t e htem-
ot change while 1t receives or re 1eases eat.
Th AEchemical = - 800cal = - 33501. perature d oes n . b
A fundamenta l thermodynamic procdes~ sh?ubld .e
lhefina~ Poten:ial energy difference is posi~i~e becau~e that we benefit from stu ymg 1t, ut 1t
energy Potential energy is higher than the initial potential · portant enough · ·1
tm ffi · tly simple that its properties can eas1 y
~ bcoof the body. The food energy is negative because should be su cie~d range of practical cases. Certainly,
SutistitJt. stores this amount less at the end of the test. b e app lied to a WI e · ffi · J
rocess would be one that 1s su c1ent y
ing th 1
ese values yields: a fundamenta z~ mathematically. Remember from the
AE easy to analy d mies for a closed system that the
"the chemical = 3350J - 720 J = +2630J .
<.s ~ • therrn • about fi rst la""'. of th~n::~;:nergy of a gas, AU, is the sum of
~ ""· It rn a1 energy released in the process rs f change 10 the 10 [ OJ). This suggests that the mat~- '
0 9 1
.,__<1t; 0 th"".U.st be dissipated by the body in the form
·'l;f~. -·wvise it
-~. ' Would cause a d angerous tempe
rature heat and work (Eq. f a fundamental thermodynamic I,
. l treatment o
emauca
.I
2 43
rl-IAPTER 9 I THERMAL PHYSICS

Scanned with CamScanner


11 ,, 1 in l·quilibri11111 .will und~rgo spontaneous irrevers-
i~k' pl\'l't'~St'S th:ll rncrcase its entropy. Such processes
UNITS
i.'l·:i~e "ht'n tht' sys~l'lll re:whl'S t·~uilibrium. At that point • Work W, heat Q, energy£: J = Nm = {kgm2)/s2
tht' t'fllT\'PY l':11111ot 111ai:ase any further. • Specific heat capacity c: J/(kg K)
aICl:i:IJ Entropy is highest in n system in equ il ibrium: • Molar heat capacity C: J/(mol K)
5 = maximum. • Entropy S: J/K
N

LAWS
• First law of thermodynamics for an isolated system (con-
servation of energy):
I we did not identify a heat transfer to the gas in the AUisolatcd = Ur - Ui = 0,
) irreversible isothermal expansion in Fig. 9.27(b). Does the
[ entropy of the gas change? in which U is the internal energy (the total energy of a
system)

• First law of thermodynamics for a closed system:


AUcloscd = Ur - Ui = W + Q
SUMMARY • Heat Q for:
• isochoric process for an ideal gas:
DEANITIONS
1 Work Ir: Q = CVJAT
• for a gas in a container, with constant pressure and Cv in unit J/(mol K) is the molar heat capacity at con-
changing \'Olume: stant volume, Cv = 3R/2 for the (monoatomie) ideal
gas.
IV= -p:l V
• isobaric process for an ideal gas:
• sign convention for pressure-related work (gas pro-
cesses): when the gas undergoes an expansion, i-j < Vt' Q = CpnAT
we use JV < 0: when the gas undergoes a compression,
I'; > l't' we use IV> 0. CP in unit J/(mol K) is the molar heat capacity at con-
stant pressure: CP = 5R/2 for the (monoatomic) ideal
• Heat Q
gas.
• for a liquid or solid system of mass m given in unit kg:
• Work: for an ideal gas with frictionless piston:
Q = cmt1T, • isoehoric process (V= const):
in which c is the specific heat capacity
W=O
• for a liquid or solid system of amount /1 in unit mol:
• isothermal process (T = const):
Q = Cnt1T,
in which C is the molar heat capacity W= -nRTln(~~)
'~ign con\'ention for heat: heat flowing into the system
isobaric process (p = const):
• is Positive; heat leaving the system is negative •
Energy £ 0 f
" a system in equilibrium: W = -p(Vr - . V)
• for any tyPe of energy except therrnaI energy.. Change of the internal energy for a cyclic process:

AUcycle = O
AE =Er- E; =W
· energy this is called the work- k'111 e t'c
for kin etrc Secon d Iaw Of thermodynamics: In a .cyclic process,. .it is
,
energy h
I eorern
1
• . .bl t take heat from a reservoir and change 1t mto
rmposs1 e o . h h d
for the work wit. 1iout re leasing
' a fraction of t e eat to a secon
rtnal energy:
resef\'Ol·r at lower temperature.

r therni1::::£ -E =Q Entropy change for
cntrop
Y: chan
a thennal,f thcnnal.i
. • • reversible process: l'lS == 0 .
ge of entropy with the state of a system.
. , rsible process (system and environ-
• spontaneous irre\ e
As : : sf -si -
- 2: Qi ment): l'lS > 0
I
. r.
I

THERMAL PHYSICS 257


CH APT ER 9 I

Scanned with CamScanner


TABLE 9.3
ANALYTICAL PROBLEMS
Metabolic rate for given activities
of I 0 L to a volume of
P-9.1. A gas expands from a voluVmd~ n~ of Fig 9 38 How
5 0 L as shown in the p- iagra . . ·., . Activity I Metabolic rate [cal/(s kg))
~uch' work does the gas perform on the piston . Sleeping 0.263
Sitting 0.358
p (arm)
Standing 0.621
Walking 1.0
Biking 1.81
I
I
I
Swimming 2.63
I
I Running 4.3
I
I
I
I TABLE 9.4
I
1.0 ---~---------I Food energy content
I I

~d
I I
I I Energy content (cal/g) ,
I I
I I Carbohydrate
I I 4100
I I
I I Protein 4200
I I
Fat 9300
1.0 3.0 5.0
V(L)

Figure 9.38 P-9.6. A standard man climbs 10 m up a verttca · I ~ope ..How


much energy in calories is dissipated as heat m a sm~e
climb if 20% of the total energy required is used to o
the work? . · the
P-9.2. The density ofsolid aluminium at 20°C is p = 2. 70 g/cm3• P-9 7 Joule used brewery horses to spin the st1rre~ I 0
When it is liquid at 660°C, the density of the liquid state . . experiment
. . which
with . he d emonstrated the equivalence
is decreased: 2.38 g/cm3 . How much work does a block of of heat and work. Assume that Joule ,s brewery horses d
aluminium of mass 100 kg do ifit is heated atp= 1.0 atm
from 20°C to 660°C? each did p = 750 J/s of work per second (this correspo~ ~
roughly to the definition of horsepower). If Joule aa
P-9.3. Calculate the work done by 1.0 mot of water when (a) it . m . a circ
. Ie fior one hou r to operate r
four horses movmg
freezes at I atm and 0°C, and (b) when it boils at 1 atm 3 d'd
. filled wit'hJmwae
stirrer in a well-isolated contamer 1
and I 00°C. (c) The latent heat is defined as the amount of
at an initial temperature of 25°C, to what final value ·~c
energy required for a given amount of material to undergo
the water temperature rise? Use Table 9.1 for the speci
a phase transition at the phase transition temperature.
For example, the latent heat of vaporization of water heat capacity of water. h'ts the
is the energy required to boil 1 mo! of liquid water at P-9.8. When a raindrop of mass 30 milligrams (mg) 1 e if
100°C to form water vapour. Compare the work values ground, by how much does the temperature increasrted
calculated in this problem with the respective latent we assume its kinetic energy is completely conv~ific
heats of fusion and vaporization, with the values for into thermal energy? Hint: Use Table 9.1 for the sp
water £,0 5 1.0 " = 3.33 x 105 J/kg at 0°C and E . . = heat capacity of water. ture of
2.26 x I"06 J/kg at I 00°C. vaponza1ion P-9.9. Water at the top of Niagara Falls has a tem~erathat all
P-9.4. When people run, they dissipate about 0.6 J of mechanical + I0.0°C. It falls a distance of 50 m. Assummf nergY·
energy per step per kilogram of body mass. If a standard its potential energy is converted into therma ~orn of
man dissipates 80 J of energy per second while running, calculate the temperature of the water at the bo
how fast is the person running? Assume the steps taken the falls. . . ·allYat
P-9.10. A piece of iron has a mass of0.4 kg and is mit~ , arer
1
are 1.6 m long.
5000
P-9.5. Table 9.3 shows the metabolic rate for given activities C. It is lowered into a beaker wit• h 20 L oratur'·9
of the adult human body, and Table 9.4 gives the energy at 22°C. What is the final equilibrium temP:fic h(a'
content of the three most important components of Neglect heat loss to the environment. The 5Pec1 1.
food. (a) How much energy is expended by a standard · · of iron
capacities · . 9
and water are foun d ·in Table · 1Iv'
·fic311
man who walks for one hour every morning? (b) If P-9.11. The air temperature above coastal areas is sig~~ate'~e
the body of the person consumes body fat reserves to affected by the large specific heat of water. Estlr· e on'.
5
produce this energy, how much mass will be lost per
hour? amount of air for which the temperature canh 1,,....-i·fi'.
3 5r ·t'·
Te
degree if I m of water cools by one degree. . den51•
heat of air is 1.0 kJ/(kg K); use 1.3 kg/m3 for its

262 PART TWO I ENERGY, THERMODYNAMICS, AND TRANSPORT PHENOMi:Mft

Scanned with CamScanner


CHAPTER 11

Static F.-lui"ds

. definition, the term fluid refers to both liquids and gases,

L
fe on Earth began in water, and remained there
exclusively for more than three billion years. Even but distinguishes them both from solids. Solids remain
today, most life on dry land maintains close ties to unchanged when placed in containers of different shapes
water. The importance of water is evident from its abun- and sizes.
dance throughout the human body. The fraction of the There are differences between liquids and gases that
human body mass made up by water is about 75% for a we have to take into account here. The molecules in a
baby, about 60% for young adults, and 50% for seniors. liquid are in a condensed state; that is, they maintain a
Two-thirds of the water in an adult's body is located in the fixed intermolecular distance. If a liquid is put into a con-
cells, with the remaining one-third in extracellular fluids, tainer with a volume larger than the volume the liquid
including blood plasma. occupies, the liquid forms a surface. In contrast, gas~s
This chapter is one of four that focus on water from adjust their intermolecular distance and fill any space uni-
different perspectives: formly. Thus, gases have no natural surface.
Under certain conditions, a fluid can change between
• the macroscopic properties of stationary water as an
liquid and gaseous states. In general, when intermolec-
equilibrium system (this chapter),
ular forces dominate, the fluid is found in the liquid state.
• the macroscopic phenomena of flowing water as a When the thermal energy of the system dominates, the
dynamic non-equilibrium system (Chapter 12), fluid is found in the gaseous state. .
When we deal with fluids that are in mechanical equi~
a the microscopic structure of the water molecule as an
librium, such as water in a tank, we often use a mode
electric dipole (Chapter 16), and
known as the ideal stationary fluid. The following are
a water as a solvent and main constituent of mixed the essential properties of an ideal stationary fluid.
phases such as blood. We discussed this in Chapter 10.
• The ideal stationaryfluid is incompressible Physically,
We don't focus exclusively on water here, however. this means that the density of the fluid is constan.t.
15
We will study a more general model system that includes Usually this is a good approximation for liquids. It
t nee·
many of the properties of liquid water. This model system not a~wa~s appropriate for gases. Gases are no be
is called fluid. essanly ideal stationary fluids because they can. n
compressed relatively easily. However, if the vari.atJO _
. d . . of1nter
11.1: Model System: The Ideal m ens1ty 1s negligible in the physical process
est, a gas can be considered effectively incompressi
"bfC,

Stationary fluid and the ideal stationary fluid model may apply.
. det the
A ftuid is a system that yields to any force that attempts to • The ideal stationary fluid is deformahle un ui·
alter its shape. That force causes the system to flow until influence of forces and seeks a mechanical. eq is
it reaches a mechanical equilibrium, at which point the librium. Only when the mechanical eq.uilibrtU~fliS
fluid conforms to the shape of its container. Based on this established does the fluid become stauonaTY·

290 PART TWO 1 ENERGY, THERMODYNAMICS. AND_TRANSPORT PHENOMENA

Scanned with CamScanner


applies equally to liquids and gases. This condition
y
is obviously very useful because we already know a
great deal about the mechanical equilibrium.

Note that we did not include a condition for the type

~·-
of interactions between fluid molecules or between fluids
and container walls. In particular, we do not need to limit
interactions to elastic collisions as we did for the ideal gas, Yo + Ay
as that would exclude concepts such as surface tension and Yo
capillarity. We introduce these concepts later in the chapter.
___ _ f•., ----;
11.2: Pressure in an Ideal
Stationary Fluid x

We are all well aware of pressure variations in the atmo-


Figure 11.1 A small rectangular prism of fluid is identified in a beaker.
sphere. You may have noticed changes in air pressure
Three vertical forces act on this element: two contact forces due to the
while flying, when your ears pop while taking off or
remaining fluid, and the weight of the fluid element. Note that the element
landing. Pressure variations in liquids are less familiar, is at rest in an ideal stationary fluid. The corresponding free body diagram
but much more profound. In fact, many diving-related for the fluid element is shown at the right.
accidents are caused by the incredibly strong effects of
pressure variation in water. How can we express this
• the contact force due to the fluid above the element,
strong effect quantitatively? The idea goes back to Blaise
Pascal in 1653, 34 years before Sir Isaac Newton pub-
FdOV.'TI'
which pushes the fluid element downward.
lished his foundation of mechanics. The fluid element neither rises nor sinks because it is
in mechanical equilibrium. Newton's first Jaw applies in
11.2.1: Pascal's Law the vertical direction:

We now know that pressure increases with depth in water. Fnet,y = 0 = Fup - W - Fdown·
The deeper you immerse yourself, the more pressure you We use this equation to find a relationship between pres-
are exposed to. Recreational divers cannot safely descend sure and depth. We can rewrite all three forces as follows:
more than 30 m, and even the most experienced profes- the weight to show density and volume, and the two con-
sional divers with the most advanced equipment are lim- tact forces to show the related pressure terms. Because
ited in how deep they can dive. Here, we will explore density is mass divided by volume, the mass of the fluid
foundations of the relationship between fluid depth and element is given as:
pressure.
Instead of following Pascal's original reasoning, we m=pV=pAAy,
will use the mechanical equilibrium introduced in Chapter 4 where pis the density of the fluid. Thus, the weight of the
to study an ideal stationary fluid. We can establish that fluid element in Fig. 11.1 is:
Pressure varies with depth in a fluid by selecting a small
fluid element at a certain depth, represented in Fig. 11 . I w = mg = pgAAy.
as a small rectangular prism. This fluid element must ~e The two contact forces contributing to the net force in
small compared to the size of the container, but must still the vertical direction are replaced by the respective pressure
contain a macroscopic amount of the fluid. The rec~an­ terms because the rectangular prism is an extended object.
gular prism has a horizontal surface of area A and a height for this, we first note that the fluid element stretches verti-
bAy == Yup - y down• wh1"ch we choose to be a small length . cally from y 0 to y 0 + Ay. At the position Yo the p~ssure is
ecause the pressure in the fluid is expected to vary verti- labelled p, and at y 0 + Ay it is labelled p + Ap. This allows
cally. The volume of the fluid element is then V = At.y. us to express the magnitudes of the forces acting on the two
Ve _The sketch at the right side of Fig. 11 .1 sh~ws t~e horizontal surfaces of the fluid element:
rticat forces of the free body diagram for this fluid
~~ent. Hori~ontal forces act on it, but _cancel eacl~ other. IFupl = pA
three vertical forces acting on the fluid element mclude
IFdownl = (p + Ap)A.
• the Weight of the fluid element, w, which is directed Using these equations and the formula for the weight, we
downward·
' rewrite Newton ·s law in the form:
• ~e contact force due to the fluid below the element,
Fup• Which pushes the fluid element upward; and
pA -(p + Ap)A-pgAAy = 0.

CHAPTER 11 I STATIC FLUIDS 291

Scanned with CamScanner


There are a few important notes about these two for~
After combining the first two tenns and then dividing by
mulations of Pascal's law:
A, we obtain:
• Pascal's law, or Eq. [ t 1.1 ], does not apply to the
tlp =-pgtly.
pressure variations in the atmosph~re. This is illus.
Note that this equation applies regardless of whether tratcd by trying to calculate the height of the upper
the density is constant or varies with depth [p =fly)]. end (surface) of the atmosphere by substituting for
We restrict further discussion to cases with constant the heighty2 == Ymax• which is the maximum height of ·
density for two reasons: (i) the ideal stationary fluid is the atmosphere:
incompressible, and (ii) the more general case requires
calculus methods to derive how pressure depends on (pYmax - Pground) = -pg(Ymax - Yground).
depth.
When p = canst, the previous equation applies for At the maximum height, the pressure drops to a·
any depth difference fly. Thus, choosing two arbitrary value of p = 0 atm; that is, Ymax is the height at
depths y 1 and y 2 with respective pressures p 1 and p 2, the which the ~acuum of outer space would begin. The
previous equation is written in the form of Pascal's law: ground-level values are Yground = 0 m, and the.pressui:e
3
(11.1) Pground == p aun' with p = 1.2 kg/m for. the density
.
of atr
at sea level. We find from the previous equation:
13JUl!lii Pascal's law states that the difference between
the pressures at two different positions in a fluid of
constant density is proportional to the vertical distance Ymax =
between these two positions. The proportionality factor
is the product of the density of the fluid and gravitational
acceleration.
The assumption that density is constant throughout the
Eq. [l l.11 is the first of two formulations we intro- atmosphere is clearly inadequate, as the atmosphere
duce for Pascal's law. It is used in this general form when would terminate at 8600 m, which is 200 m below
the surface of a fluid cannot be identified and thus cannot the peak of Mount Everest! Thus, Pascal's law does
be used as a reference point. An important example is not apply to gases because gases are compressible
the blood in the human cardiovascular system because it and their density depends on pressure. Example .~ .2
is a closed system with no identifiable surface of blood provided a detailed discussion of how gas den.sit)'
toward air. depends on pressure.

• Pascal's law does not contain any information about


the shape of the container. Thus, regardless of th~
11.2.2: Pressure in Liquids with a Visible
Surface shape of the container, the pressure increases belo"
the surface and results in a fixed value at a given depth·
In systems with an identifiable surface of the fluid, This is illustrated in Fig. 11.2, in which the fluid 5~
. such as water in a glass, index 1 in Eq. ( 11.11 refers to face is located at the same height above the connect
the surface of the liquid. Therefore, we set y 1 = 0 and
p 1 = Patrn· The atmospheric pressure is the proper value for
the pressure of the fluid surface because it is in mechanical
equilibrium. The force pushing the surface upward equals
the force caused by the air pressure pushing downward.
Note that we used this argument before when we studied
Boyle's experiment in Chapter 8.
It is more convenient in this case to define the
position axis downward, meaning that we define the depth
below the surface as a positive distance. This changes the
negative sign on the right-hand side of Eq. [ 11. l] into a
positive sign. Writing y 2 = d, with d the depth below the
surface of the fluid in unit m, we get:
P = Patrn + pgd. ( 11.2)
This equation is a second, frequently used fonnulation of
a
Pascal's law. lt expresses the pressure in fluid as a func-
. tion of the pressure at the surface arid the depth d below
the surface.

Scanned with CamScanner


bottom tube in each column. Note that there are devia-
tions from this observation for fluid containers with
tiny diameters. These are called capillarity effects, and
we will discuss them later in this chapter. The nautilus in Fig. 11.4 rises to a depth of 60 metres
below the surface of the Pacific Ocean at night to feed on
I Pressure data arc often given in non-standard units.
plankton. What is the water pressure at that depth, using
Blood pressure, for example, is usually recorded
the rule from Example 11 .1 that the pressure in water
in unit mmHg. These units should be converted to increases by roughly 1 atm per 10 metres depth?
standard units before doing any calculations. The
(a) 3 atm
standard unit of pressure is Pa (Pascal). Some pres-
sure data, such as blood pressure, are reported relative (b) 4 atm
to atmospheric pressure. A pressure value relative to (c) 5 atm
atmospheric pressure is called a gauge pressure:
(d) 6 atm

P gauge = Pabso\ute - Patrn' ( 11.3) (e) 7 atm

which may have either a positive or a negative value.


In particular, the term pgd in Eq. [ 11 .2] represents a
gauge pressure. Gauge pressures in the human respi-
ratory system are discussed in Chapter 8.

EXAMPLE 11.1
What is the pressure 10.0 m below the surface of a lake?

Solution
The density of fresh water is p = 1.00 g/cm3 = 1.00 kg/L =
1.00 x 103 kg!m3. We convert Patm = 1.00 atm to ~atm .=
1.013 x 1os Pa, because we want to do the calculation in
standard units. From Eq. [11.2}, at 10.0 m depth we get:

Pi0m = 1.013 x 10sra + (1000~)(9.8~)(10.om) Figure 11.4 The nautilus is a relative of the long-extinct spiral-
shelled ammonites. Its shell (16-27 cm in diameter)_ not only protects
= 1.99 x 105 Pa. the soft-bodied mollusk, but also provides the animal with perfect
control over its buoyancy. The soft body takes up only the outermost
chamber inside the shell, while the other chambers (up to 30 for_an
We use the .inverse pressure convers1·an from unit Pa
dult anima~ are filled with a mixture of air and seawater to ad1ust
to unit atm to find that this result is equivalent to P10m =
1.97 atm-the pressure below the water surface rises fast,
~e overall density. The nautilus's shell is coiled, calcareous, and lined
with strong mother-of-pearl to allow it to resist the effects of pressure
doubling at just 1 o m in depth. In Fig. 11 .3, the result of
th.is example is used to .illustrate the pressure in water (preventing implosion to a depth of 800 m).
. . as
a function of depth. The fast pressure increase is a cntica1
issue for diving, as we noted in Case Study 8.5.

P (atm)

4 .
The nautilus .in F"1g. 11 ·4 is a temperature-sensitive
.
. I diving to greater dept hs during daytime to
3 anima ' rface temperature increases due to solar
esca_pe near-s~ been found as deep as 420 m below
heating. Theyw: :tepressure variation occurs during this
the surfa~e. . f ? Use the same rule as in Concept
2
daily vertical m1gra ion
Question 11 .1.
(a) less than 30 atm
O +--- "-T"----.---... d (m) {b) 32 atm
0 10 20
(c) 34 atm
Figure 11.3 Water pressure as a function of depth.

293
CHAPTER 11 I STATIC FLUIDS

Scanned with CamScanner


Solution to part (b): For a water barornet
(d) 36 atm
P == 1.0 x 103 kg/m 3 : er,
(e) 38 atm
p im 1.013 X 10s Pa
(f) 40 atm h = ;g == (1.0 X 103 kg/m3)(9.8 mJs2) = I0.3 m.
(g) more than 40 atm
A water barometer would be taller than 10 metres! In other
words, the pressure that the atmosphere exerts on us is
as strong as the pressure due to a 10 m tall water column.
Solution to part (c): The mass of a standard man is
EXAMPLE 11.2 70 kg. The total area of his feet is estimated to be 0.04 irf.
A barometer is a device to measure atmospheric pressure. Therefore, the pressure he exerts on the floor is
Figure 11.5 illustrates a mercury barometer. It consists of
2
a glass tube with a closed top end and an open mercury _mg= (70 kg)(9.8 m/s ) = X n4 p ""'
p _ 1•7 1v· a 02 . p._,..
reservoir at the base. When the system reaches equilib- A 0.04m 2
rium, the pressure due to the mercury column in the tube
should be equal to the external pressure on the open sur- It is interesting to note that the atmospheric pressure is
face of the reservoir, which is the atmospheric pressure. In about five times as strong as the pressure you apply on
other words, the atmospheric pressure can be determined the floor.
by the height of the mercury column, h.
(a) Use Pascal's law to determine the height h for
a mercury barometer when the atmospheric pressure is
1.013 x 105 Pa. (b) In principle, the fluid in the barometer 11.2.3: Blood Pressure
can be another liquid, such as water. Determine the height
h of a water barometer when the atmospheric pressure is Human blood pressure measured by a doctor is the gauge
1.013 x 105 Pa. (c) Estimate the pressure a standard man pressure of blood in the arteries at the height of the heart,
exerts on the floor when he stands up, and compare it which varies between the diastolic and systolic pressures.
with the atmospheric pressure. However, the pressure of blood in the entire cardiovas·
cular system varies in a much wider range, as illustrated
in Fig. 11.6.
The cardiovascular system can be divided into two
parts: the high- and low-pressure sections.
.
The high-pressure part includes the aorta, the arten
~

and arteno· 1es, and the capillaries of the systemic


· Cl·rcult.
tion. The blood pressure in the arteries typically van~
h between l 0. 7 kPa (diastolic pressure, equal to 80 nunHg
and 16.0 kPa (systolic pressure equal to 120 mmHg).
' . d tht
The low-pressure part includes the veins an
P = 1 atm pul~onary circulation; in this circulation the pressu~
vanes between only 1.3 kPa and 3 .3 kPa ( l 0-25 rornlt

P(mmHg)

1 2 4
5
3
Figure 11.5 A barometer
120

100
Solution
Solution to part (a): At equilibrium, 80
60
pgh = Paun
40
h = PIUI\ = 1.013 X 105 Pa
pg (13.6 X 101 kg/ml)(9.8 m/s2) 20

= 0.76 m = 760 mm. o-&.__._.i_.ll...J..~~-1.~~-1.--J


A
gure 11 •6 Blood pressure variations along the cardiovascu ~
~~
While s~pine (lying down): (1) left ventricle of heart. (2) lar9~ ~
Note that mmHg is a unit for pressure. 1 atm Is equal to
760mmHg.
(~) artenoles, (4) capillaries, and (5) venules and veins. llle ~iues -'
line represents mean blood pressure. Note that the preSSU~.
gauge pressures, or pressures relative to the atmospheriC prii....-

294 PART TWO I ENERGY. THERMODYNAMICS • AND TRANSPORT PHENnMs:t.ia

Scanned with CamScanner


Since all blood pressure values in Fig. J 1.6 arc posi-
tive ~auge press.ure values, we conclude that the pres-
sure m our cardiovascular system exceeds the ambient
air .press~rc everywhere ..This is correct, however, only
while lymg down; negati ve gauge pressures can occur
while standing (see Example 11.3 ). Dlood pressure also
changes with age, particularly the higher systolic pres-
sure (see Table 11.1 ).

TABLE 11.1
(b)
Blood pressure as a function of age

Normal blood pressure


Age (mmHg) (kPa)
Newborn t I 60-80 I 8.0-10.7
Babyt I 80-90 10.7-12.0
Up to 10 yearsf I 90/60 12.0/8.0
10-30 yearsf
I 110/75 14.7/10.0
30-40 years* I 125/85 16.7/11 .3
40-60 yearsf I 140/90 18.7/12.0
Vein A

88
>60 yearsf I 150/90 20.0/ 12.0
tsystohc blood pressure only.
*Systolic/diastolic blood pressure.

Figure 11.7 Blood pressures and blood vessel sizes in the feet of a
person (a) in supine position and (b) standing upright The vessel sizes
EXAMPLE 11.3 are indicated by the areas of the circles shown. The numbers in the
Relative to blood pressure when in the supine position, vessels are the respective blood pressures in unit mmHg.
calculate the additional blood pressure difference between
the brain and the feet in a standing standard man. Use Substituting the given values yields:
=
P 1.06 g/cm3 for the density of blood.
Supplementary physiological information: The term
supine position specifies that the person is lying down, !lp = (1.06 x J03 ~~)(9.8;)(1.73m)
as shown in Fig. 11. 7(a). The blood pressure in supine = 1.80 X 104 Pa = 18.0 kPa.
~ition is quantified in Fig. 11.6, with a maximum varia-
;~on of about 15% of the atmospheric pressure value This difference is about 20% of the atmospheric pressure;
F 60 mmHg). When the person stands, as shown in . it is of the same order of magnitude as the pressure varia-
ig. 11.7(b), an additional difference between the blood tions within the cardiovascular system in supine position.
Pl'essure in the feet and in the brain is due to the extra For physiological applications, it is more useful to
;:o1
umn of blood that rests on the blood in the feet. Recall refer to pressures that are measured relative to the pres-
rom Table 4.1 that a standard man is 173 cm tall. sure at the height of the heart. For a standing person who
is 1.73 m tall, the heart is at a height of 1.22 m, and the
Solution arterial and venous pressures in the feet are increased
!~quantify the additional difference for the standing
relative to the pressures at the height of the heart by:

~d man,
as give we use Pascal's law in the general form Ap = Pbloodg!lh
·
n in Eq. (11.1) (because no blood surface exists):
= (1.06 x J03 ~~)(9.8~)(1.22 m}
Pbrain - Prcri = -pbloodg(ybrain - Yrccr). = 12.7kPa = 95 mmHg.
This equ r1
Pressur a .on Is rewritten with !lp =Prcri - P brain for the The pressure is increased to an average arterial value
the Pe e difference, and !lh = Ybrain - Yrcri for the height of of 190 mmHg and an average venous value of 100 mmHg.
the ex;:""· !he choice to write !lp in this form eliminates This is illustrated in Fig. 11. 7. The figure shows two pairs of
eqllation~ minus sign on the right-hand side of the last circles that indicate the relative sizes of veins and arteries
in the feet. The numbers in the circles refer to blood pres-
sures In the respective vessel in unit mmHg. In the scalp,
the pressures decrease for a standing person by:

CHAPTER 11 I STATIC FLUIDS 295

Scanned with CamScanner


Seismosaurus. a 40-m-long herbivore that lived in Nortti

Ap = (1.06 x 103 ~)(9.8~}-o.51 m) America during the late Jurassic period. With their Upright
gait and long necks (up to 10 m long), cardiovascular
= - 5.3 kPa = -40 mm! lg. adaptations were needed to compensate blood pressure
variations as a function of body posture.
The average arterial pressure drops to a value of
55 mmHg, and the average venous pressure becomes
-35 mmHg. This low venous value does not cause the
veins in the skull to close, though, since the blood ves-
sels in the brain are surrounded by cerebrospinal fluid.
The pressure in that fluid also drops by a corresponding I
An oceanographer chooses to report water pressure
amount relative to the extracellular fluid in the chest when
the person is standing upright.
values below the ocean surface as gauge pressures. I
(a) What value would they report when referring to surface
1
water? (b) When would they report negative values?

CASE STUDY 11.1 11.2.4: The Principle of Transmission of


Fluid Pressure
For which animals does the effect discussed in Example 11 .3

I
pose the greatest challenge?
Consider a confined ideal stationary fluid system.
Eq. [ 11.1] states that for any two points in the fluid, the
Answer In large land animals, this effect can be much more pressure difference is determined by pg!ly, where ~y
profound than in humans. The additional pressure required is the difference in height between the two points. In
when blood must be pushed above the level of the heart
other words, if the pressure at one point is changed d~e
can be generated only with the four-chambered heart of
mammals. Of these, the pumping challenge is greatest for
to an external force, the pressure at the other point \\1U
animals with long necks. A standing giraffe needs to pump change by the same amount so that the pressure diffe~n~
blood as much as 2.5 m above the heart to the brain. That between the two points remains the same (pg!ly). This 15
requires significantly more of an additional blood pressure known as the principle of transmission of fluid pres-
in the left ventricle; the normal systolic pressure at the heart sure, or simply Pascal's principle.
of a giraffe is therefore more than 250 mmHg. Such a sys-
tolic pressure would be extremely dangerous for humans. l:!i\Ui!ljl The principle of transmission of fluid pres~ure
Special valves and a feedback mechanism reduce cardiac states that a pressure change applied to anyw_he~e. 1;~
output when the giraffe bends its neck down to drink (see confined ideal stationary fluid is transmitted und1mim5
Fig. 11 .8). In this position, the brain is suddenly almost throughout the fluid and to the walls of the container.
2 m below the heart and would otherwise be exposed to
a tremendous blood pressure due to the changed height The hydraulic lift shown in Fig. 11.9 is a cornlll~:
difference. application of Pascal's principle. It has two cylinde:ftJ~ed
Another group of animals that had similar issues to
much larger than the other, connected by a t_ub~ an niined
deal with were the large quadrupedal dinosaurs, such as
with a fluid such as petroleum oil. The fluid 15 co mall
by two pistons. When a force J.m is exerted on the
5

piston, it results in an increase in pressure fin•


Fin
ljn = - .
Ain

Figure 11.8 Agiraffe bending down to drink water.


Figure 11.9 A hydraulic lift

296 PART TWO 1 ENERGY, THERMODYNAMICS. AND TRANSPORT PHENOMENA

Scanned with CamScanner


AccOrdinc~ to Pascal's principle,
. this increase in prcs-
water even though the roe k sin· ks.. In 1;r.act • when a rock is
ure is transmittl'd to thl.! large piston. So, !in = /~ut' which ' . • • d 1·l eld below the surface
5 suspended from a spring sea 1e an
rcsu1ts ·I0 a much grl'ater output force at the large piston: of water, the rocks · i t d is
• apparent we1g11 · played . on the scale
Fin Fout is smaller than its actual weight. Buoyancy is the up~ard
- = --
force exerted by a fluid on an object that is either partially
or completely immersed in the fluid. To ~e exact, buoy-
ancy is the resultant force exerted by a fluid.

In a typical hydraulic lift, Aout >> Ain· Therefore,


11.3.1: Archimedes' Principle
F »F. Let us look at buoyancy more closely. Jn Fig. I I. JO, a
out Anoth~r common application of Pascal's principle is beaker with a fluid and a block 8 suspended above t.he
scuba diving. When a diver reaches a depth of 30 m below fluid are shown in the left sketch. A fluid element F with
the surface of water, the pressure due to water is about the same shape and volume as the block B is identified
3 atm, which could crush the diver's chest (remember, an below the fluid surface. For the fluid element F, mechan-
atm is about five times the pressure that a standard man ical equilibrium conditions apply in the _s~me ma~ner as
exerts on his feet). To prevent the chest from being crushed discussed in Fig. 11.1. Choosing the pos1t1ve y-axis ~o. be
by the water pressure, the diver needs to maintain an equally directed upward, the mechanical equilibrium cond1t1on
strong pressure inside his lungs. We know that human lungs for the fluid element F is based on the vertical force com-
are composed of millions of tiny air sacs called alveoli. ponents as:
Fortunately, these sacs are connected by a network of ducts F up -Fdown -wF = 0,
and tubes called bronchi and bronchioles. According to
Pascal's principle, the pressure in each of these millions where F is the contact force due to the fluid below the
of alveoli must be the same since pressure is transmitted
up
chosen fluid ....
element, Fdown ·
1s the contact force due to the
uniformly throughout an enclosed fluid. Therefore, what fluid above the chosen fluid element, and wF is the weight
the diver needs to do is to breathe in compressed air with of the fluid element itself. In the first step, we assume
the same pressure as the water pressure at the present depth. that the fluid element F is removed from the beaker and
In fact, the air cylinder for scuba diving is connected to a placed in a small container, leaving an empty bubble
regulator that automatically maintains the air pressure at of equal size in the beaker. We can think of this bubble
the proper level. This is particularly' useful when the diver simply as though it were filled with a negligible mass of
moves across a significant range of water depths. Note that, air. This bubble is not in mechanical equilibrium because
to avoid breathing in too much oxygen or nitrogen, deep- its weight is significantly lower than that of the removed
see divers use a compressed mixture of gases other than air. fluid. The two contact forces remain unaltered as the fluid
around the bubble, which exerts these forces, has not
changed. The vertical component of the net force acting
!1_3: Buoyancy on the bubble is:
Fup - Fdown = WF > 0.
When a piece of wood is released into water, it floats.
This must be because an upward force is exerted by the As a result, the bubble accelerates to the surface.
Wat T
er. he same upward force is also exerted on a roek 111
·
Before this happens, the bubble is replaced by block B

@~ \
\

--
\

---\
....- _, \
~

(jJ -
-
- -
1
se~n~
Int'llure
the 11 . element with
Illustration of Archimedes' principle. We consider two steps: In the first step, a fluid . the same shape as the object is removed.
step, the object is placed in the void created In step 1.

Scanned with CamScanner


n role in the transport of water ·111·' s .
plays " . . ' u ap in tall
tree S·
To trnnsport
,
\\atn
••
hy capillarity

into th··
~
ca .
nop1es SUMMARY
of even the t.nllcst tr1:1:~ (for t:~ample, eucalyptus trees
of 150 111 height). cap~ llary diameters of smaller than DEFINITIONS
O.I µm would be n:qu1red. Howev~r, the xylem fibres
in which water has been shown to nse in trees, typicall;
• Atmospheric pressure: p 3101 = 1.013 x I Q5 Pa

have diame.ter.s of 20 µ,.m to 300 µ,m. Xylem tissue of • Gauge pressure: a pressure value relative to the atmospheric
pressure:
a leaf midrib is. shown Ill a coloured scanning electron
micrograph in Fig. 11.24. In the cross-section shown, the Pgaugc = Pahsolutc - Patm
rnidvein runs through the centre. The layer surrounding
it is large mesophyll cells, f~llo~ed by a thin outer layer • The specific gravity of a substance is defined as the ratio of
of epidermal cells. The m1dvem contains the larger, its density to the density of water at 4°C.
water-carrying xylem tubes bundled at the centre and
surrounded by a ring of smaller phloem tubes that carry SG = P objcct

nutrients. Pwatcr

Clearly, other effects play a role in the transport of


• Surface energy er is the energy required to fonn an area of
water from the ground to the leaves. Let's look at the
1 m2 of new surface:
wood anemone as an example. The wood anemone is
a flower that stands erect and opens only when the sun t:.E
shines. When a cloud covers the sun or when the flower er=-
t:.A
is artificially shaded, it closes and the stem bends down.
This is a sign of reduced water pressure in the stem. When • Surface tension (equivalent to surface energy):
the shade is removed, the flower resumes its erect pos-
ture as the water pressure in the stem increases again. In F
er= -
the anemone, the water pressure is regulated by chem- L
ical reactions that operate only under sunny and warm
conditions. L is the length along which the force acts tangentially to the
surface.

UNITS
• Surface tension er: Jlm2 =Nim

LAWS
Pascal's law (for fluid without identifiable surface):

P2 - P1 = -pg(y2 - Y1) ,
in which index l and index 2 refer to two arbitrarily chosen
vertical positions in the fluid
Pascal's law (depth d measured from surface downward):

P = Patm + pgd
Archimedes' principle:

F buoyant = Pt1uid Vimmersedg
, ~ rmula for pressure difference across the surface
Laplace s 10
for (r is radius):
• hollow bubble: tlp = 4afr
droplet or hollow cylinder: t:.p = 2alr
• homogeneou S
. d r· flp = air
• homogeneous cyIm e ·
Jurin 's law for capillarity:
2aliquid ~
I11iquid - -
- Pliquidg r
,
f:igllte 11
~asr... '24 Leaf · . · electron
"'· ~Y (S~i. midrib as seen with coloured scanning . d (J is contact angle of fluid with
'llaliln c'YI) Th'15 . . 'b the con- in which r is radms, an
" IJJi Of aleai" is a cross-section through the midn ' . 11 d
0-1ein, runs ~stem along the centre of the leaf. The main vein, ca e capillary wall.
rough the centre of the midrib. 309
~l CHAPTER 11 I STATIC FLUIDS

Scanned with CamScanner


(c) It is curwd downward because the foot .
· · · ketch Fie 1I .21J(c) • increased pri.:ssurc from ahovc the watesirnu1ates
(c) The fn:e body d1agra111 111 s ~· . ' 111 r SUrf;1r•
. -· 11 '>8(a) and the trl'e (d) It splits and the lower end of the foot d ~
belongs to the case 111 1·1g. ·- ' · ' .
. skl'lcI1 r·
body diagra111 111 , 11 "9(d) belongs to
rig. ·- bi.:low the surface. angles
the case in rig. 11.2!\(b). .. 21 In Fig 11 13 a forc1.: is shown acting on a mo!
Mc- 11. . I I
'd) The fr~·e body diagra111 in sketch h g. I l.29_(c)
. . . k ccule in
the water surface. Our ·now ct gc of Newton'
' bdonl.!s to the case in fig. 11. 28() s1a11s
• a · an dth·trei.:
1..:
tells us that
body ;liagram in sketch fig. I I .29(a) belongs to
( ''l) th1.: water molecule accelerates downward Until. it
the case in fig. 1I.28(b). hits the bottom of the beaker.
(e) The free body diagram in sketch Fig. I l.2~(b)
( b) the watt:r molecule accelerates downward unt1.1.
belongs to the case in Fig. I l.28(b). and the lri.:c has li.:11 the surface.
11

body diagram in sketch Fig. 1I .29(c) belongs to


(c) at least one more force must act on the molecule
the case in Fig. I I.28(a). to keep it at the surface.
MC-11.15. Which is the standard unit of surface tension?
(d) the water molecule is ejected into the gas space
(a) J
above due to the reaction force to force /:.
(b) Pa
(c) (kg m)/s2
(d) kg/s2
(e) kg/(ms 2) CONCEPTUAL QUESTIONS
MC-11.16. Laplace's law describes the pressure in an alveolus in
the lungs in the form Pinside - flo utside = 2cr/r, in which Q-11.1. Vapour bubbles in a beakerofboiling water get larger
u is the surface tension and r is the radius of curvature as they approach the surface. Why?
of the alveolus. In healthy alveoli, a surfactant is used 0-11.2. Fig. I l.30(a) shows a beaker on a scale. The beaker
to reduce the surface tension by coating parts of the is filled to the rim with water. It is then taken from
surface. Which of the following statements is false? the scale and a piece of wood is lowered into the bea·
(a) The surfactant particularly must coat areas with ker. The beaker with wood is then placed backon lht
a large radius of curvature. scale, as shown in Fig. I I .30(b). How has the reading
(b) The surfactant particularly must coat areas with of the scale changed from part (a) to part (b)?
a small radius of curvature.
(c) A surfactant does not change the pressure in the
bubble.
(a) (b)
MC-11.17. Fig. 11.18 shows in three steps how a lung collapses and
then is re-inflated under medical observation. We call the
collapse "step I ," the step from alveolar radius r tor
\\'ood i
--
"step 2," and the final step from alveolar radius ,.t to ,.2
"step 3." In which step must a health practitioner exe~
2

the greatest pressure from outside through the mouth? Water Water
(a) during step I
(b) during step 2
(c) during step 3
--
MC-111 (d). after ste.p 3 to keep the lung from collapsing again
. 8. Which law is used to quantify the pressure in a soap Scale ~
bubble?
. . . ~ di'
(a) Jurin's law Figure 11.30 (a) A beaker filled to the rim with water 15 pl ~f
(b) Pascal's law scale. (b) A piece of wood is lowered into the beaker and the
(c) Newton's third law placed again on the scale.
(d) Laplace's law
MC-11.19. You study a large and a sm II ~
of the two is the · a soap bubble. In which 0-11.3. · tized feel ir
. . , air pressure higher? A few lizards have developed specia · has'
~
(a) Neither; it s the same a 11 ow them to climb smooth suriaceS' 5tJC5 iO(·l'J'.
.
(b) in the larger bubble · · mP1e .~
dows, without the aid of claws. Exa 11,31k,i1,
. tilt~ ~·I
(c) in the smaller bubble
several gecko species, which can even
(d) depends on the common . ti ·1· ) Llo11 ca11 . ,,
(e) impossible to predict outside pressure 1e ce1 mg in tropical homes. (a ri nthe) '
MC-11.20. A water strid · . · on to such a smooth surface? (b) H0"' ca 5urfll'<1
. er Is an msect that ca .. ' ol:iSS (I
Lookmg at one of th . I n walk on water at a reasonable pace up the w111d011 " ·n~qs. l
. e six egs of th . 0-11.4. "' . fi JlllS I ,..,~
restmg on the water surface h' h e water strider vve introduced Pascal's law in two 0 cnil 1•"
nces .
the case for the surface dw ic of the following is an d [11.2]. Under what circumsta
Refer to Fig. I I.Ii. un emeath the insect's foot? form in Eq. [ 11.1] be used?
(a) It i.s perfectly flat.
(b) It 1s bent upward d
the foot. ue to the attractive force of

Scanned with CamScanner


ANALYTICAL PROBLEMS
The. sphl·re is tht' shape with the small"st . " Iior
J
P-11 .1• . ~ suraace
3 1!1\
~ .
en nilumc.
. . lo prove. this statc ,nt
111 c proper1y
n:•1uires
, v:mat1onal
. analysis. l lert' we want toron1m fi
this result only to~ a selection or highly symmetric
sh:ipes by c:ilcul:iung
. Ii
the ratio of surface to vo1ume.
Find these ratios or each of these shapes.
(a) sphere
(b) cylinder
(c) cube
(d) pyramid
(e) tetrahedron
(f) cone
Does the statement hold for these six shapes?
P-11.2. A diver accustomed to standard snorkel tubing of
length 25 cm tries a self-made tube of length 7.0 m.
During the attempt. what is the pressure difference
between the external pressure on the diver's chest
and the air pressure in the diver's lungs? For those
interested: What happens to the diver as a result of
the attempt?
P-11.3. A scuba diver takes a deep breath from an air-filled
tank at depth d and then abandons the tank. During
Figure 11.31
the subsequent ascent to the surface, the diver fails
to exhale. When reaching the surface, the pressure
difference between the external pressure and the pres-
sure in the lungs is 76.0 torr ( I torr = 133.32 Pa =
I mmHg). At what depth did the diver abandon the
tank? For those interested: What potentially lethal
danger does the diver face?
P-11.4. What is the pressure increase in the fluid in a syringe
when a force of 50.0 N is applied to its circular
plunger, which has a radius of 1.25 cm?
P-11.5. What minimum gauge pressure is needed to suck Unknown
liquid
water up a straw to a height of 0.10 m? Recall that
the gauge pressure is defined as the pressure relative
p to atmospheric pressure: Pgaugc =.p - Pa1.m· .
-1t.6. Collapsible plastic bags are used m hospitals for mfu-
sions. We want to use such a bag to infuse an elec-
trolyte solution into the artery of a patient. For this
we mount the bag at a height h above the arm of the
patient, as shown in Fig. 11.31. Assuming the aver-
age gauge pressure in the artery is 13.3 kPa, and the
3
density of the electrolyte solution is 1.03 g/cm , what
· the m1111mum
is · · height hat which· the miuslO
· "- · n would
P~11.1. Work?
Refer to the hydraulic lift shown in Fig. 11.9..The
d. d the diam-
iameter of the larger piston is 0.30 m, an
eter of the small piston is 0.030 m. Determine the Figure 11.32
force needed to apply to the small piston so t~at a car
P~11.a. of mass 1200 kg can be raised by the large piston.
The U-shaped glass tube in Fig. 11 .32 contains t':"o . I of mass 250 kc (basket
liquids in mechanical equilibrium: water of d~nsity P-11.10. We consider a hot-air bal .o o~ th;
m·elopt' of balloon
Pw == 1.0 kg/Land an unknown liquid of density P1· and envelope). The sphencha e"- II)' inffated. To what
. fJ6m wen1u •
The unknown liquid is in the left tube, floating on has a diameter o I d i·r bt' heated for the
t the enc ose a
top of the water with a clearly visible interf~ce. Use temperature mus
fi standar men·
d ·> Assume the sur-
h, == ISO mm and h = 15 mm with the heights as balloon to carry our d . treated as an idl.'al gas.
. ir is at 2ooc an is .
P..11.9, labelled in Fig. 11.31. What is the density p,? roundmg a I lar mass of air.
The density of ice is P = 920 kg/m3, and t~e av~r­ Use 29 g/mol for t ic mo
~ge density of seawat~~e is P = 1.025 g/cm . W d~t,
1ra · w · pose 313
ClJon of the total volume of an iceberg is ex ·
... rHAPTER 11 I STA! IC FLUIDS

Scanned with CamScanner


Fluid Flow

A
fluid that is not in mechanical equilibrium will and forth into the lungs, and the cardiovascular system as
flow. Different aspects of flow are described in this blood flows through the pulmonary and systemic circula-
chapter, with two models introduced for dynamic tions in sequence. We were able to discuss key properties
fluids: the ideal dynamic fluid and the Newtonian fluid. of the respiratory system when developing the gas laws;
Both are idealized, as we assume the fluid is incompress- for the cardiovascular system we need the fluid model
ible and turbulence free. The flow under these conditions introduced in the previous chapter.
is called laminar flow.
In the ideal dynamic fluid, molecular interactions are
limited to elastic collisions. This yields frictionless motion 12.1: Basic Issues in Blood Flow
of the fluid at stationary walls. Two laws detennine the Our survey of the most important features of the cardio-
?roperties of the resulting flow: the equation of continuity vascular system starts with Fig. 12.1, in which we com-
is an expression of the conservation of fluid mass, and bined an anatomical overview with quantifying its prime
Bernoulli's law represents the conservation of energy. physiological function:
These laws predict that the flow through a tapering tube
ac.celerates, and that the pressure in the fluid decreases • In the systemic circulation, blood is pumped out of
Wah increasing speed. the left ventricle of the heart to the capillaries in the
. In a Newtonian fluid, the fluid molecules interact organs throughout the body, where it delivers oxygen
inelastically with each other and with the container walls. and nutrients. Loaded with carbon dioxide, a by-
Thee
. quat1on · of continuity still applies, but Bemou11·· 1 s Iaw product of cellular respiration, the blood then returns
is no 1onger . sufficient to describe the flow of the fl u1"d. We to the heart. From the right atrium it proceeds into the
develop th · · to take into
· account fl ow right ventricle, entering
. e concept of v1scos1ty
resist ·
th ~nee. In a Newtonian fluid, two forces are present tn • the pulmonary circulation. From the right ventricle
e direction of the flow: a forward-acting force based on blood is pumped to the capillary bed in the lungs,
athat d re d'·ffierence along the tube, and a resistance
Pressu · fiorce where carbon dioxide is exchanged for oxygen. From
a epends on the viscosity of the fluid. Jn steady state, the lungs, oxygenated blood then returns to the left
tubParaboric velocity · distribution results m · a cy1·mdnca · I atrium to complete a full cycle.
Po:· and the volume flow rate is proportional to the fourth
Jn the systemic circulation, organs are arranged in
~of the radius of the tube (Poiseuille's law). parallel to allow the body to prioritize oxygen supply
assu h~n the properties of a flowing fluid violate the
based on the vital relevance of the organ a~1d the current
beh tnpttons
. rnade for a Newtoman· fluid,· non-Ncwtonia ·n
av1our . . . ~ metabolic demand. For the major organs. Fig. I 2.1 shows
Pois . is observed, most notably as dev1at1ons 1rom
the fraction of the received blood flow, /!lo· I = ~VIM
nuidcuille·s Iaw. Examples include turbulent flow at h"1gh !o
is the volume flow rate through an orga1~, and is the
thcnspeed
. s and. • velocity-dependent interactions · Wit· Iltn
·
total volume flow rate of the systemic c1rculat1on. The
llld Wh
1\v0 en it 1s in a mixed phase. term volume flow rate refers to a fluid volume ~hatdfl~ows
lure ft . Physiological systems in the human body f ea- . . I can be determme irom
u1d flo . · streams bac k through a vessel per unit ttme. o
w. the respiratory system as air
~El
CHAPTER 12 1 FLUID FLOW
315

Scanned with CamScanner


:c
·;::

-- "'
.~ "'
"'t:0 "'E t:......
-1:: ..,, "'
5 X JOH 2
l 1.6 x 108

GJ
DIAMETER (cm) 3.2
2.6

0.3 0.002 0.0009 0.0025


CD
3500

CROSS-SECTION (cm2)
CJ 5.3 20
20 30 18

VOLUME (cm 3 )
180 250 250 125
:.::.-----~~----..:.=.::...._---
CJ

Figure 12.3 Anatomical data for various types of vessels of the system.1c · c1rcu
· 1at1on..
· The top row 1'dent·f·
1 res the type of
. blood vessel.
area, ~;
Thevo1urne
and the
gives numbers of blood vessels for some types. For each type, the outer diameter of a single vessel, the outer cross-sectional
vessels of this type are illustrated. The numerical values are given in the indicated units.

,t
weO·
.. . . . fl w Later, uo~I
12.2: Flow of an Ideal non-equ11tbnum of the fluid dunng 0 ·~furthers ~
develop the Newtonian fluid model, whi onfinioS
Dynamic Fluid an interaction between the fluid and the c
The description of flow cannot be based on the stationary tainer walls.
fluid model we introduced in the previous chapter because
it is a dynamic process that requires us to drop the require- 12.2.1: Ideal Dynamic Fluid Model d defl~
ment of a mechanical equilibrium. To develop a suitable 'b)e an /flS(e
The ideal stationary fluid is incompr:ss~ in the d'. ~i,-(1
model, in this chapter we ultimately modify two proper-
able. Both of these properties are retain.ed nils ll~)i·b~~~
ties of the stationary fluid model. Also in this section, we . ry flUl jcal eQtJI Ii6,f<11
case. Further, the ideal stat1ona
introduce the ideal dynamic fluid model, which requires container such that the fluid is in mecha!l curs sr&
the fewest modifications to address the mechanical This condition has to be replaced: flow oc
/

318 PART TWO I ENERGY, THERMODYNAMICS, ANO TRANSPORT PHENOMENA

Scanned with CamScanner


in this textbook we touch the issue only hrictly later in this
chapter when we introduce an empirical threshold speed
above which flow becomes turbulent.
The second ussumption is automatically applicable
if the fluid is incompressible, since sound waves arc the
result of loculizl.'d compressions in the fluid. However,
sound propagation through a fluid is extremely important
in physiology as it leads to acoustics. Thus, we will even-
tually drop this assumption (in Chapter 15).
Fluid flow that satisfies the first two assumptions is
called laminar flow and is illustrated in Fig. 12.5. The
(c) figure shows an amount oftluid entering the field of view
through the green area at the left side. If we imagine
the fluid divided into small fluid segments, then we can
follow each segment as a function of time. Its path is
called a flow line and can be envisaged to lie fully within
an envelope we call a flow tube, as indicated for one flow
line in the lower part of Fig. 12.5.

Flow line

Figure 12.4 Comparison of the circulatory systems of various verte-


brates. (a) Fish have a two-chambered heart (1) with a single circula-
tion. Gills (2) and capillary beds in the rest of the body (3) are shaded.
(b) Amphibians have a three-chambered heart with two circulatory
systems. These are called the pulmocutaneous system and the systemic
system. The systemic system is a high pressure system. The lungs (4) are
shaded. (c) Mammals have a four-chambered heart with two circula-
Flow tube
tions: the systemic and the pulmonary circulations. The heart completely
separates oxygen-rich blood from the lungs and oxygen-depleted blood
emerging from the systemic circulation.
Figure 12.5 Sketch of laminar flow ~rough an arbitrari~y ch~se~ area.
The path taken by each fluid segment 1s drawn as a flow lme (highlighted
· ed). one of the lower flow lines in the sketch is enclosed in a flow tube
mr d · Iammar· flow.
bec~~se the fluid has not yet achieved this mechanical (grey cylinder). Flow lines cannot cross· each· otherd Nurmg
Laminar flow characterizes ideal dynamic fluids an e oman fl U1·ds.
wt ·
equ1hbrium .
. However, this alone does not define the ideal dynam~c
fluid be .. hen a ftutd
fl cause add1t10nal effects can occur w
ows. Thus, we have to make further assumptions that ar•w111liJ Lami·nar flow is established when
.. (i) flow lines in
·d ver cross each other, and (11) flow tubes never
address th · 1 . . h phenomena · The the fl UJ ne
id emc us1onorexclus1onofsuc penetrate each other.
1eat dynamic fluid is called ideal because it excludes t~e
argest nu b . bserve in minar flow and the ideal dynamic fluid are
real ft . rn er possible of phenomena that we o Note that Ianonymous · I dynam1c
concepts. For an idea ·
. uids. Specifically, we require that therefore not sy t always be laminar because it shares
i. not oath
ft urbulences (which are a departure from sm ft ui·d' flow mus t' ns with the laminar .
flow concept.
·· ow as specified below) occur during flow, me assump 10 . · h
the sa . fl v can occur for fluids that violate t e
II n . ) r laminar O\ . . I
· o sou d · ft ctuattons Howeve • . fi the ideal dynamic fluid. For examp e,
n waves (which require density u
.. deveto . . . third assumpti~n or minar flow after we include flow
11i P m the flowmg fluid and . t'll discuss 1a
· no fr· · ' b' t adja- we w1 11 s 1 d half of this chapter.
cent t~Chon oc~urs with walls or other 0 ~e~i~erent resistance t·n the secon . fior the ideal dynamic · fl ui'd has
than ththe flowing fluid that move at a spee~ f the The thir
. d assumption
. ti rther as it can easily lead to mis-
.
d a bit u
fluid fl. ui·d· This requires that the ·interaction o
e . h
to be d1scusse . it differs notably from the ot er t~o
'W Particles is limited to elastic collisions. conceptions. First, . ddresses objects beyond the fluid.
a~s0 hat do 1h . I ? The first nditions in that it a b formulated somewhat vaguely
or n·lptj 0 ese three assumptions imp Y ftoW
1I t ~~is condition has tdo elop widely applicable laws that
air th n
Inc1 . rough h rules out consideration of the actua we want to eve
. II turbu en . because
Uding t t e trachea, which 1s usua Y . and
urbutence is mathematically challenging, CHAPTER 12 I FLUID FLOW 319
\.

Scanned with CamScanner


arc not confined to particular containers. For exa111ple. we current section regardless. but will be forced to aband
00
want to apply these laws to the fl ow of air past the wing this assumption when we introduce Newtonian fl uids.
of a bird. or to a stream of water falling fro111 a water
tap toward the kitchen sink. Deriving laws in this chapter 12.2.2: Equation of Continuity
is easier when referring to a flu id-confining container or We now establish the laws governing fl ow of an idea(
tube: however. the existence of the container or its shape dynamic fluid. We start with Fig. 12.7. which shows a
may not matter to the laws· applicability. fluid fl owing from Jell to right through a tube of varyino
In the easiest case. the ideal dynamic fluid is an ideal .
cross-sectional area. The cross-scct1ona I area of the tube"
gas. In this case. the thi rd assumption is equivalent to
changes from A 1 to A 2• with A 1 > A2 • From experience.\\ e
the respective assumption in the kinetic gas theory we know that fluid is neither created nor lost nlong the tube
introduced in Chapter 8. Elastic collisions were studied
Thus, during. any given ti me interval. the same amount of
in Chapter 7. including results for a collision with a
mass of fluid that enters th rough cross-section A 1 (grey
wall. This interaction is shown in Fig. 12.6: the com-
area in upper sketch) must leave through cross-section A,
ponent of the velocity of the fluid particle parallel to
(grey area in lower sketch): ·
the wall (II-direction in Fig. 12.6) remains unchanged.
while the velocity component perpendicular to the
wall (.L-direction in Fig. 12.6) changes its sign. In the Ll.m I 6.1112

kinetic gas theory. a non-zero component parallel lo Tt= Tt· (12. I J


the wall allows for collisions with variable angles: for
a flowing ideal gas. the component parallel to the wall Since the fluid is incompressible, the conscrYation of
also contains the velocity of the collective fl ow. Since fluid mass is equivalent lo a conservation offlu id volume.
this velocity component does not change as a result of Using p for the density of the fluid, Eq. [ 12. 1] is written
an elastic collision. no additional effect occurs due to in the fom1:
the particle- wall interaction that is not already cov-
ered in the discussion of the ideal gas. This means wall Ll. V1 Ll. V2
inte ractions are fri ctionless and can be neglected in
p/f(=P/f(·
the discussion of flow of an ideal gas. Unfortunately,
though. we saw in the previous chapter that the ideal The tcm1 6.111/ tit in Eq. [ 12. 1] is called the mass flow rate.
gas is not always a good example for a flu id because it and Ii VILl.t is called the volume flow rate.
is compressible. The volume of a fluid segment in Fig. 12.7 can he
If the ideal dynamic fluid is a liquid, neglecting written as the product of the cross-sectional area of th~
inelastic collisions is problematic: in a dense liquid, the tube and the segment's length:
fluid particles interact extensively with each other and
with the walls. We exclude inelastic collisions in the

.1

L11 Fluid flow


- - - - -II>

l l
Figure 12.7 Model of a tapering tube that we use to derive the equat10~
figure 12.6 Elastic collisions of a fluid particle with a stationary wall. of continuity. Fluid flowoccurs from left to right. (Top) The fluid segment is
Its velocity component parallel to the wall (ii) remains unchanged; its initially represented by the grey area at the left with cross-sectional areJ
velocity component perpendicular to the wall (1.) changes its sign but not A, and fluid speed v,. (Bottom) Later, the fluid segment is positioned in
its magnitude. the grey area at the right with cross-sectional area A2 and flowspeed V:·

320 PART TWO J ENERGY, THERMODYNAMICS. AND TRANSPORT PHENOMENA

Scanned with CamScanner


CASE STUDY 12.1
Wo study tho stoady flow of wator from a water tap, for
oxamplo, in your kitchen sink. Tho jot of water (a) broadens
( 12.2) os it lolls, (b) narrows os it falls, (c) d005 not change its cross-
:;octional shapo, or (d) slows before hitting the bottom of tho
sink. Hint: Nogloct effects that could lead to the break-up of
Il~-c.ausc: Eq. 11 2.2 I opplies.lx·tw~·l'll
. . a11y I\\ o. points I und 2 a continuous flow Into droplet formation.
nlulll:\ the tubl'. we t·un "rite 11 111 a generali:tcd form:
Answer (b). Even though tho flow docs not occur In a tube,
tho equation of continuity applies as long as the flow is
A \' = COllSI. (12.3) continuous. From faucet to sink, the water accelerates due to
gravity. With tho spood of the fluid increasing and the volume
This is the equutlon of continuity. Note that the rea- flow rate constant, the cross-section of the fluid must diminish.
soning that leJ to Eq. I 12.31 also establishes a second
usd'ul equation for the volume flow rate:
EXAMPLE 12.1
av = A1·
-
The heart of a standard man (Table 4.1) pumps 5.0 litres
( 12.4)
at ' of blood per minute into the aorta. (a) What is the volume
flow rate in the cardiovascular system? (b) What is the
speed of blood in the aorta? (c) If we assume that the
which applies anywhere along the tube. including at the blood passes through all systemic capillaries in our body
cross-sectional areas with indexes I and 2 in Fig. 12.7. in series, how fast would it have to flow through each cap-
illary? Use dat a from Fig. 12.3. Would this result make
sense? (d) What is the speed of blood in a capillary if we
C!I1%lI13 The equat ion of continuity is an expression of instead assume that the blood flows in parallel through the
the conservation of mass or the conservation of volume systemic capillaries in the human body?
of an incompressible fluid. It states that the volume fl ow
rate is constant along a tube. The fluid flows faster when Solution
it passes through a section of the tube with a smaller Solution to part (a): The amount of blood flowing through
cross-section. the aorta per minute corresponds to a volume flow rate of:

Let's check this law carefully. We still need to estab- (dV)


di aorta
= 5.0 L = 8.3
60 s
X 10-~ m3.
s
lish whether it applies to laminar flow and/or to an ideal
dynamic fluid. The two differ in that the former does Solution to part (b): The diameter of the aorta is given
not include an assumption about the interactions within in Fig. 12.3 as d = 2.6 cm, which leads to an outer cross-
sectional area A = rr(d/2)2 = 5.3 cm 2 (also shown in the
the fluid and with the container wall. If you read once
figure). The inner diameter defines the lumen, which is the
more through the derivation of the equation of conti-
open volume inside a blood vessel. To calculate the inner
nuity, you note that the type of interaction plays no role: diameter of the blood vessel, the wall thickness has to be
the mass of the fluid is conserved regardless. Thus, the taken into account. The fraction of the total diameter attrib·
equation of continuity applies to laminar flow whether uted to the blood vessel wall lies between 15% and 20%;
the fluid is an ideal dynamic fluid or not. thus, a typical inner d iameter of the aorta is d•O<U = 2.2 cm.
This leads to the cross-sectional area of the lumen:

CONCEPT QUESTION 12.1


A tube widens from a cross-sectional area A 1 to a
cross-sectional area A = 3A 1• As a result, the speed The speed of the blood in the aorta is obtained from
2
of an ideal dynamic fluid in the tube changes from v 1 its inner cross-sectional area and the volume passing per
to v2, where second. Using Eq. [12.4], we obtain:

(a) V2 = v,.
(b) V2 =V,13. - (~~;na)
Jv.orul = A
··~~
(c) v2 = 3v . aorta
1
ml
(d) V2 = V,19. 8.3 x 10- s_
5
= 0.22~.
....
(e) v2 =9v,. - - - --
3.8 x 10- 4 m2 s ..
.:,ti4

CHAPTER 12 I FLUID FLOW 321

Scanned with CamScanner


12.2.3: Bernoulli's Law
This is a frequently used result: blood flows through the
aorta at an average speed of about 20 crn/s. The changes we observed in the speed of a flow·
. f h 1ng n.
Solution to part (c): Let's assume blood passes due to changes in the cross-section o t e tube also • u1d
1
through each single systemic capillary at the rate found in changes in the pressure in the fluid. This can be ill eild l(J
part (a). For the outer diameter of a capillary we use 9 µm . II Ustrat
experimentally with an instrument ca cd a Ventur·1 td
from Fig. 12.3. This value leads to an inner diameter of · F. I 2 8 Th . t
which is shown I~ . ig. . . . c ms rument consists lllttcr
of.
=
dcapillary 7 µm (capillary wall thickness is about 1 µm). tube with a constnct1on zone at its centre. A W-~hap 'd a
The cross-sectional area of the capillary is: · ht tothe w1·de sections · of thec lubt
connects at the left and ng .. ·
th ·
tube· in the middle 1t 1s open o e constnct1on zon .main
· · t ·
_
Acapillary -
(dcapill•ry)
1T - --
2
_
-
(7 I0- m)
X
7T - - - -
6 2
W-shaped tube is partially filled with a liquid to indicC. IJie
. h . ate the
2 2
Pressure in the section .
of the onzontal tube abo\e
fl .d . h . each
= 3.8 X 10- 11 m2 = 4 x 10-11m2. column. Initially, while the u1 m t e m~m horizontal tube
is at rest the liquid in the W-shaped tube 1s at the same 1
We use the equation of continuity to derive the speed ' . . · . C\el
in all three columns, md1catmg that the pressure 1s the sa
for blood in the capillary, lvcapillaryl: When the fluid flows thro~gh. th~ main t~be, a pressure ~~:
ference is observed: the liquid m the middle column ri
. h
highest, indicating that the pres~ure m t e constriction zone
ses
is lower than in the wider sections. The reduction in fluid
ml Pressure when the fluid flows through . a constricted zone
8.3 x 10- 5 - is known as the Venturi effect. It 1s named after Giovanni
s km m
---- - = 2200-
11 2
3.8 x 10- m s
=2 x 106 -.
s
Battista Venturi ( 1746-1822), an Italian physicist.
Daniel Bernoulli (1700- 1782) quantified this obser-
Even if the capillaries could sustain blood rushing through vation starting from Fig. 12.9. Shown is a horizontal tube
at such speed, it would no longer be possible to exchange that tapers from cross-section A 1 to Ai· We assign pressure
oxygen and nutrients with the surrounding tissue; that is, p 1 to the wider section of the tube and Pi to the narrower
the physiological purpose of the cardiovascular system section. With these definitions, the grey fluid se~ment
would be lost.
Solution to part (d): A slow flow of blood in the sys-
temic capillaries is achieved by arranging them parallel to
each other with a combined cross-section that is larger
than the cross-section of the aorta. Fig. 12.3 suggests
that this is the case with 3500 cm2 for the capillaries, com-
pared to 5.3 cm 2 for the aorta. The equation of continuity
allows us to determine the actual speed of blood in the
capillaries once we have corrected the cross-sectional
areas from Fig. 12.3 to represent the lumen. A correction
factor k is defined as the ratio of the lumen cross-sectional
area to the outer cross-sectional area, and is quantified
with the inner and outer diameter for a typical capillary:

(7µm) 2

l
Alumcn
k = - - = -- 2 =06
Aouter (9µm) . .

The two diameters are squared because the area is pro-


portional to the square of the radius. With this factor, we
obtain for the cross-sectional area of the lumen of the
capillaries A capo.11ary = (0.6)(3500 cm2) = 2100 cm2 • Thus:

(¥)aorta
lvcapillaryl = A .
capillary

ml
8.3 x 10-s_
10- 4 ~.
5
- -- -- = 4 X
0.21 m2 s Figure 12.8 The Venturi meter is an instrument to measure the speed
of a fluid in a horizontal tube (liop) It indicates the same pressure in e~
Again, this is a frequently used value: blood flows very · . 'dflO\I>
section of the tube while the fluid is at rest. (Bottom) When the 1101 ..
as in 1cated by the arrows In the tube, pressure variations bee eil
slowly through the capillaries, at less than 1 mrn/s. · d" ome
dent: the pressure is higher where the speed of the fluid is slower.

322 PART TWO I ENERGY. THERMODYNAMICS, AND TRANSPORT PHENOMENA

Scanned with CamScanner


FluiJ flow
- - - - _.,. in the part of the tube with pressure p 1• and adding the
fluid segment to the part of the tube with pressure !'2·
Quantitatively. this means that the volume of the segment
is changed from a V to O in the top part of Fig. 12.9, and.
coneurn:ntly. its volume is changed from 0 to av in the
bottom part. The work is:
{J l

Pt _.I r- IV= -p2 AV- p 1(-AV)


P,M.v.
l
.h'1 = -(p2 -
The conservation of energy for the fluid segment
requires that au = W, because no heat exchange takes
place. The only form of energy that changes in Fig. 12.9

c P1

figure 12.9 A sketch defin_ing the parameters needed to derive


is the kinetic energy, thus liU = /i£kin:

We separate all the terms related to positions 1 and 2 in


(12.5)

Bernoulli's law. We study a fluid segment (grey) that initially occupies the equation and divide by AV:
tile volume Ai .lx1 (top) and later the volume Az.l-'2 (bottom). Fluid flow
in tile sketch occurs from left to right through a tapering tube. The fluid
(12.6)
pressure varies from p1 to p2 at the constriction. The change in the speed
of tile fluid causes a change in the kinetic energy that is accounted for
by awork term associated with the transfer of fluid into the constriction. Eq. [12.6] applies at any position along the tube:

I
p + -p v 2 = const. (12.7)
is studied. We want to determine its kinetic energy at 2
an initial and a final instant, and then relate the change
This is Bernoulli's law.
in the kinetic energy to the work needed to move it into
the constricted section. We specifically choose the initial
mU!ilii Bernoulli's law is an expression of the conserva-
time (index I) when the fluid segment occupies the grey
tion of energy. It states that an increase in the speed of
volume shown in the top part of Fig. I 2. 9. At that instant an ideal dynamic fl uid is accompanied by a drop in its
the volume of the fluid segment is AV= A 1Ax1. The final pressure.
instant is shown in the bottom part of Fig. 12.9 (index 2).
Now its volume is AV= A 2 Ax2 . In the above derivation, we assumed that the fluid
The cross-sectional area and the speed of the fluid are flows in a horizontal tube where there is no change in
related by the equation of continuity. A force F1 has to be height. A more general form of Bernoulli's law, which
applied to accelerate the fluid segment through the tube. can accommodate changes in height, can be obtained by
The change in the kinetic energy of the fluid segment is: modifying the above derivation. Specifically, when the
height varies from z 1 to z2 , the change in energy should
I 2 I 2 include the change in potential energy. That is, a term
AEk. = -tlmv2 - -Amv 1,
m 2 2
pg(z2 - z 1)AV should be added to the right-hand side
of Eq. [12.5]:
in which 6m is the mass of the fluid segment. With P the
density of the fluid, we rewrite the mass as tlm = pt!.V: 1
-(p2 - p 1)AV = 2p(v~ - vi) AV + pg(z2 - z 1).
(l 2.8)
Then, Eq. [12.6) becomes
·
s· ince the tube becomes narrower, the speed must
incrcas 1 1
. e, and thus the kinetic energy of the flu1'd seg-
2 - 2
rnent Iner . .
P1 + 2PV1 + pgzl - P2 + 2PV2 + pgz2. (12.9)
" eases. To achieve this increase in kinetic energy,
~~ . .
r . rnust be done on the fluid segment. This work ts Finally, Eq. [12.7] becomes
fiequrred to t rans1er
r the fluid segment from its · · · I t o t'ts
· 1mt1a
na1Pos·f1 · h I
wori. . ion. In a gedanken experiment, we can split t e p + 2pv 2 + p gz = canst. (12.10)
Into two contributions: removing the fluid segment

CHAPTER 12 I FLUID FLOW 323

Scanned with CamScanner


. 1s
T h1s . the complete form of Bernau 11·· 1 s Ia w· However,
. ht
as a common approximation when the variation · · in he1g fl
. wit
1s . h.111 a relatively
. small ,range, the tem1 pg'?- is o tien A blood ves
sel of radius r splits into two srnaue
ed f r Ves
'
ignored. Such an approximation is particula~ly use u1 with radius r/4. If the spe o the bloOd in tn Seis,
when the details of the variation in height are irrelevani each . what is the speed of the blo<>d in ea e large
unknown, or complicated, as illustrated in Case Study l 2. vessel is vsias~i~
(v ,J? Treat blood as an ideal dynarncn Of the
smaller ve sma re fluid
and Example 12.2. .
Bernoulli 's law applies only to the ideal dynamic
(a) Vsmall =8 V1arge
fluid. The third condition limiting the interactions ?~the (b) Vsmall = 4 V1arge
fluid particles with the container wall to elastic collisions
is necessary to apply the conservation of energy in t~e
(c) Vsmall = Vrarge
fom1 used to derive Bernoulli's Jaw. When we revise this (d) Vsmall =V1arg/4
condition in the next section, we must revisit the calcula-
(e) Vsmall =V1arg/B
tion of the pressure in the flowing fluid.

CASE STUDY 12.2


(a) In a person with advanced arteriosclerosis (artery con-
striction due to accumulated plaque on the inner walls, as
shown in Fig. 12. 10), Bernoulli's effect produces a symptom
called vascular flutter. To maintain a constant volume flow
rate in this situation, the blood must travel taster than normal
through the constriction. At a sufficiently high blood speed,
the artery collapses and immediately reopens, leading to
a repeated temporary interruption of the blood flow that
can be heard with a stethoscope. Why does vascular flutter
occur? (b) An aneurysm is a weakened spot of an artery
where the artery walls balloon outward; Fig. 12.11 shows
an aneurysm of the aorta. Blood flows more slowly through
this region, resulting in an increase in pressure at the aneu-
rysm relative to the pressure in adjacent sections of the
artery. This condition is dangerous because the increased
Figure 12.11 An artist's rendering of an aneurysm of !hf rta. The
inner layer of the aorta wall (pink) has ruptured. Blood (da· ·;ct) has
pooled in the fissure of the wall, producing the visible bulr· )~rgical
repair of the aorta is required.
I
I
I
pressure can cause the artery to rupture (see Chap•• ; 13 for 1

a detailed discussion of aneurysms). What slows b'u;.~ d fiow 1

in an aneurysm? /
I
Solution to part (a) The artery collapses since tfie high
speed of the blood inside the vessel lowers the pressure rn
the bloodstream relative to the pressure in the stationar)'
extracellular fluid. This is due to Bernoulli's law: a hrgh 1
value for the speed, v, leads to a low value of the pres· '
sure, P· Once the pressure difference is large enough 10 '
cl~se the artery, the blood flow stops momentarily. Whe;
this happens, the blood upstream from the clogged vess
causes a pressure increase that is sufficient to reopen the
artery Th I · con· 1
. · e c os1ng and reopening of the artery then
t1nues in a cyclic manner. '

Solution to p rt (b f a bioad .
Figure 12.1O Advanced arteriosclerosis is an artery constriction due
b
vessel and th
a
e speed of blood in the vessel are oad
.~
) The cross-sectional area 0 rela t:<>
1
to accumulated plaque on the inner vessel walls. Shown is a coronary
artery cross-section with atherosclerotic plaque (yellow) in the lumen.
fl~~h: equation. of continuity. Thus, the speed of 01 ion
. ecreases in a blood vessel when its cross-sec1
increases in an aneurysm. I

Scanned with CamScanner


eXAMPLE 12.2 from h:l1 to right (direction of arrow). Three smaller, ver-
tical columns arc placed at different positions along the
Blood nows smooth!y thr~ug.h the aorta as Its cross-
tion tapers to 75 !{, of its m1t1al value, similar to the tube. The height to which the liquid rises in each of these
:Cse illustrated in Fig . .12.9 . What is the pressure differ- columns dq1~·nd~ on the pre~!>ure in the fl owing liquid
below. If that liquid can he modelled a~ an ideal dynamic
ence lfl between the wide and. the narrow sections? Hint:
oata we used before and required for this calculation are fluid. all three columns have tu be equally high. as shown
tne volume flow rate m the aorta .l. 1'l.l.1 ::: 83 cm3/s, the in Fig. 12. 12(a). 'I his prediction results from the laws
density of blood 1> = 1.06 g/cm3, and the lumen cross- of the previous section: the equation of continuity states
section of the aorta A...:..u = 3.8 cm2 • that the fluid speed docs not change for a constant cross-
scction of the tube. then Bernoulli's law states that the
Solution pressure in the fluid docs not change either.
We start with Bernoulli's law. Let index 1 in Eq. [12.6) refer
to the wide section and index 2 to the narrow section of (a)
the aorta. From the equation of continuity we know that
the flow is faster in the narrow section: that is, ,., > 1• •
tnserting this inequality in Eq. (12.6) leads to l'i > p,~ Thu~.
we predict that the pressure drops from section 1 "to sec-
tiOO 2. This must be taken into account when writing the
pressure difference in the form .l.p = p 1 - P~· In this form,
lp > O. Note that we could have chosen to define llp as
Pi - PP in which case .l.p would be a negative value.
Eq. (12.4) allows us to quantify the two speeds, 1·
1
and v,, since we know how the two cross-sectional areas (b)
are reiated: A 2 = O.75 A 1. Thus:

AFl .l.t
,. = - -
1 A, '

and:
.l. VI .l.t 4.l. VI At
vi=--=---
A2 3A 1 •

Substituting the last two equations into Eq. (12.6], we find: Figure 12.12 Comparison of (a) an ideal dynamic fluid and (b) a
Newtonian fluid flowing through a horizontal tube. Flow resistance leads
= _!. (16(.l.Vl.l.1)2 _ (AVl.11)2) to a pressure drop along the tube, as indicated by the lower column height
~ 2p 9A 2 A2 • of the fluid above the tube at the right in part (b).
I I

which leads to: The actual experimental result for a real liquid is
1p(AVl.l.1)2 shown in Fig. I 2. I 2(b ). The farther the Iiquid progresses
~= 18A 2 • along the tube, the shorter the vertical columns. Thus, the
I
experimental result <lifTers fundamentally from the predic-
The given values are substituted next: tion in Fig. 12.12(a): the liquid speed and/or the pressure
along the tube must change; that is. the equation of conti-

1( 1060~)(8.3 x 10-s~y nuity and/or Bernoulli's law must be modified. The equa-
tion of continuity applies as long as fluid flow is laminar
~= 18(3.8 x 10- 4 m2)2 because fluid cannot accumulate in or vanish from the
= 20Pa. tube. Therefore, an approach beyond Bernoulli's law is
needed to quantify the fluid pressure.

12.3.1: Newtonian Fluid Model


!b_3: Flow of a Newtonian Fluid The Newtonian fluid model is developed to correctly
describe the observation in Fig. l 2. l 2(b). It is derived from
Up to this point, fluid flow was discussed for the ideal
the ideal dynamic fluid model by removing the assump-
dynamic fluid. We established two laws: the equation of
tion that was identified as too restricti ve: a dense fluid
continuity and Bernoulli's law. We now test how closely
~~ · . .
•ctions based on these laws correlate with expen-
. such as a liquid cannot travel past a solid wall without
extensive interactions. We noted these interactions already
:~~I observations. The experiment we use ~s illustrated for the ideal stationary fluid in the last chapter, when we
•g. 12.12: a liquid flows through a horizontal tube

CHAPTER 12 I FLUID FLOW 325

Scanned with CamScanner


S1·nce the required external force varic .
11 fiorces typicalsI funher t
attributed the capillarity effect to a significant interface •
energy tenn. fl Ul"d to fluid, with sma• er 1· "d y n t!Jrt
and larger forces m 1qu1 s, a matcri I ccdcQ . 1
How do we formulate a condition to replace the elastic gases . . . a s co 1r
. t dltced called the n~la•1 .'
collision restriction we used in the previous section?. In tn ro . v1scos1ty
. coefficient
2 lj. n ., ~
Eq. [ 12.11 ]. the unit of 1J is N s~m . Table 12. ascQ !Ji
the natural sciences we usually proceed by conducti~g 1 lists\' 1
experiments. In this particular case, the macroscopic cost·ty coefficients for several fluids. 1~

observation of viscosity provides a promising approach


even though it is not primarily a phenomenon describing TABLE 12.l
the interaction between a fluid and a container wall. Viscosity coefficients of various fluids ~
Viscosity coefficient
11 (N s/m2>
TemPe'
12.3.2: Viscosity Fluid l C)
0

----.....;.- -
Viscosity is an interaction between neighbouring layers Gases
of a moving fluid. This is illustrated in Fig. 12.13, in 1.78 x 10-5
Nz 25
which two parallel fluid layers of area A are highlighted. 2.08 x 10-5
Oz 25
Let's assume the lower layer is at rest. This could be due, 1.71 x 10-5
for example, to close proximity to the resting walls of Air 0
the tube. The upper layer, a distance Ay away from the 9.0 x 10-6
Hz 25
lower layer, moves with velocity Av toward the _!"ight. 8.4 x 10-6
Hz 0-----
The moving layer encounters a resistance force R that 9.8 x 10-6
tries to slow it down. To maintain a constant velocity, HzO 25
Newto_!:!'s first law requires the presence of a second Liquids
1.79 x 10-3 --..
force, F ext' with which the upper layer of fluid is pushed
forward. Thus, if a fluid encounters flow resistance, an
HzO
HzO 1.01 x 10-3
0
;~o
-.
external force must be applied to push the fluid through
the tube. The magnitude of this external force is found
empirically by submerging two parallel test plates into a
HzO
(Whole) blood
2.8 x 10-4
2.3 - 2.7 x 10-3
1 (;~

17
---
---
resting fluid and moving one plate relative to the other, Blood serum 1.6 - 2.2 x 10-3 ·'·)
as Fig. 12.13 illustrates. From such experiments, the
Ethanol 1.19 x 10-3
force Fext is found to be proportional to (i) the area A
of the fluid layers that face each other and (ii) the dif- Glycerine 1.5 ~J
ference in speed of these layers, IA"vl. The force is also
inversely proportional to (iii) the distance Ay between
the two layers: Note that the third column in Table 12.1 .:1ves the
temperature at which the reported values appl:.. :mplying
IA"vl that these values change with temperature. Vi. (Osity is
Fext = 11A Ay . (12.11)
one of the transport phenomena like diffusion Jnd heat
conduction, which we discussed in Chapter I0. Amicro-
scopic model is needed in addition to the phenomeno-
logical law of Eq. [ 12.11] to describe the temperature
dependence of the viscosity coefficient. .
Eq. [ 12. 11) establishes that viscosity is a dynamic
effect that requires a velocity gradient perpendicu~artothe
- - -1>_-:.7-,,.-------~-~t:.-V - direction of the flow lines in the fluid, f:iVI l:iy. Thts mealb
_-;~-2=---_-_ A -~ - in turn that viscosity does not play a role in stationar)
fluids (v = 0) or in ideal dynamic fluids (6V/ay==0).e'en
· · \\VW1
1-- though the viscosity coefficient for all phys1ologica 1 .
. . h I there IS

t-Z--
--
A
evant flmds ts 1J > O. This means, further. t a
no fl u1d, not even the ideal gas, that doesn t behaveedas
. '
Newtonian fluid once a velocity gradient is introduc ·
1

·n
. · · 1erac110
13!Ull!il In _a Newtonian fluid, the rnel:ist1~ 111!!flldient~
Figure 12.13 Experimental setup to measure viscosity coefficients. Two with the fluid-confining walls causes velocity e · .jon~
parallel plates are immersed in a fluid at a distance Ay from each other. V1.scos1ty
. I . co11 I~
replaces the assumption of e ast1c . C('o·
The lower plate is held at rest and the upper plate is pushed by an external . .d nd t!S J
required between the ideal dynamic flu 1 3 . lr"11
force F .The resistance force of the fluid balances the external force 101111
fining walls. In Newtonian fluids, the flow is !:
leading mto a constant velocity Av- of the upper plate toward the right '
viscous.

326 PART TWO I ENERGY, THERMODYNAMICS, AND TRAN<::PnoT ,. .. ~ .. - .

Scanned with CamScanner


·.,> ..'1~

In steady-state flow, two forces act on the fluid in the


EXAMPLE 12.3 tube. In the direction of the motion of the fluid. there is
-mm-thick coating of glycerine is placed betw
a
A 1·..,icroscope sI'd
1 es o f w1'd t h 2.0 cm and length 7 0 een
a force due to a pressure difference along the tube. Th is
twO•" . . cm force pushes the fluid through the tube. The viscosity of
Find the force required to move the micros
each. cope the fluid causes a resistance force acting in the direction
'd s at a constant speed of 0. 10 mi s relative to h
sh e . . ff' . eac o pposite to the direction o f motion o f the fluid. This force
other. The v1scos1ty coe 1c1ent of glycerine is found in
tries to s low the moving fluid. The mecha nical equ ilib-
Table 12.1 .
rium between these two forces varies w ith the position
in the tube: near the stationary wall, viscosity dominates
solution
a nd the fluid fl ows s lowly (with the speed van ishing
This problem is solv~d with Eq. [12.11 ]. Each of the terms
directly at the wall), whereas toward the c entre of the
o0 the right-hand side. of the equation is given in the tube, the force pushing the fluid dominates and the fluid
example text. The area 1s:
A = 0.020m X 0.070m = 1.4 x 10- Jm2. moves comparably fast. The velocity profile is written
quantitatively in the form:
Note that A isn't twice this value for the two faces of a slide
beC<luse only t~e cross-~ectional area enters Eq. [12.11 ]. r~ube - r2 llp
v = (12.12)
The difference in speed 1s ~v = 0.1 mis, and the coating 471 I '
thickness is the distance ~Y = 1.0 x 10-3 m. Using 11 = •I
I
2
1.5 N stm , we find: in which r tube is the radius of the tube, llp is the pressure
difference along a segment of the tube of length /, and 1J
F= (l.SS)(l.4 X 10- 3 m2)(0.10~) is the viscosity coefficient of the fluid. The term j,p // is
a constant pressure gradient along the tube. Eq. [12.12]
1.0 X 10- 3 m provides a parabolic velocity distribution. as shown in
= 0.21 N. Fig. 12.14. This velocity profile can be demonstrated w ith
a slow-flowing fluid such as honey. If you open a j ar of
This is a notable force, given the rather wide 1 mm separa- honey at room temperature and tum it upside down, the
tion between the slides. Imagine you reduce their separa-
honey travels fastest at the centre of the j ar, while honey
tion to 1 µm. Now a force of F = 21 O N is required; that is, a
near the glass surface won' t flow out.
mass of more than 20 kg has to be suspended from one of
the slides to achieve the stated motion. This phenomenon
was used in the development of adhesive tape. .
rtu~t -+:
.

12.3.3: Fluid Velocity Profile


in a Cylindrical Vessel
In a Newtonian fluid the inelastic interaction of fluid
molecules with a stationary wall causes velocity gradi-
ents perpendicular to the flow lines. This leads to a non-
unifonn velocity profile across the fluid in the direction
perpendicular to the stationary wall. The actua l velocity
profile depends on the shape of the stationary wall ; we
co~fine the discussion in this section to cylindrical tubes,
which are physiologically important because they include
blood vessels.
. Eq. [ 12.11] is used to quantify the inclusion of
inelastic co 11 1·s1ons
· of the fluid particles with the stationary
·
wall. Eq. (12. I I] can be used only when the fluid flow has
.
...
v
reached a steady state· thus steady state is an additional
assu · ' ' .
11 .dmption for quantitative predictions for the Newtonian Figure 12.14 The steady-state velocity profile of a Newtonian fluid
11u1 . Th'is assumption
. . .1s reasona ble for blood flow m .
flowing through a cylindrical tube. The highest speed is reached at the
the cad'
w r iovascular syste m. A more general approach centre of the tube Oongest red arrows), While the fluid layer directly in
that.
1
~dreq·
uire us to replace Eq. [ 12. 11] with a formu Ia contact with the wall does not move. Note that the figure shows a physical
ex inctudes transient fluid accelerations. Recall that we s~etch of the syste~, whi.ch.includes the walls of the tube. and a diagram
1
beriC Uded t rans1e
. · nt b eha viour in transpo rt phenome na with the velocity axis pointing downward and the position axis pointing
ore, for exampIe, when using Fick's law for d1·o·us10n. . to the right.

CHAPTER 12 I FLUID FLOW 327

Scanned with CamScanner


the viscosity Tl, and tho Inner radius r ol tho t
CASE STUDY 12.3 method of dimensional onalysla intrOduc()(j in lJ~ ~ uno.
· ·11es
derive Po1scu1 ' Iaw. Cha"' '
v L•lf 11o

In hospitals and in the food industry, workers are required


to wash their hands frequently and extensively. Why are Solution
elaborate hand-washing procedures required in these We express the volumo flow rato in terms Of
. al eq 1· . tho rel(,.
environments? variables using a gener · ua ion with et-r>on - .,_.q
Answer Eq. [12.12) shows that fluid flow near a stationary determined: t-n~ to~
surface vanishes (v __. o for r __. 'iuool· Thus, allowing water
to flow past your hands does not wash away pathogens and
toxins that are attached to the skin, where the water flows
-AV = k -
At I
(A'')" '' -1J r y
'
slowly if at all. Only extensive rubbing with soap may loosen
these. This always applies; however, hospitals are more
where k is a dimenslonle~s constant. It i:; straigtiuor.,. •
directly concerned because of the larger number of patho-
to determine the d1menS1on of each of the quam a:,
gens and toxins their employees come in contact with. . ~~
the equation:

( ~~] = [L P[ T]-
1
12.3.4: Poiseuille's Law
Jean Leonard Poiseuille used Eq. [ 12.12) to detenninc the
volume flow rate through a cylindrical tube. We motivate (A;]=(~] = (r~~2 ] =[M}[LJ- 2(r J-2
his result with a simplified argument. Eq. [ 12.12] shows
that an average velocity of the fluid is proportional to the F.. Ay] [MLT -2L]
[ Al.~vl = L2Lr-1 =
1
square of the radius of the tube, r~ubc' and the pressure gra- (11) = (M][LJ-l[r]-1
dient along the tube, .lpl/. It is also inversely proportional
to the viscosity coefficient of the fluid, T/· We can substi- [r) = (L ).
tute this average velocity in the equation of continuity,
which states that the volume flow rate .lVl.it is equal to Then, the dimensional equation is:
the (average) speed of the fluid and the cross-sectional
area of the tube, A. with A :< r~ubc· Thus, the volume flow [LP[TJ- 1 = {[M) 1 (l)- 2 [ T) - 2 }"{[M) 1 (L ]- 1 [T]- 1 ~[L~
rate must be proportional to the pressure gradient and
= [M)" +l3 [LJ- 2a - fl • y[TJ- 2a- /l.
inversely proportional to the viscosity coefficient. It also
is proportional to the fourth power of the radius of the
Equate the exponents on both sides of the equation:
tube, r~be:
O=a+{3
£lV 7T £lp 3 = -2a - {3 + -y
-=-r4 - (12.13)
£lt 8 T/ tube I . - I = - 2a - {3.

Solve for the unknown exponents:


This is Poiseuille's law, with the proportionality factor
7T/8 that applies specifically to a cylindrical tube.
" = I
{3 = -I
(IJU l!.ti Poiseuille's law states that the volume flow rate
of a N ewtonian fluid through a cylindrical tube is propor- 'Y = 4.
tional to the fourth power of the radius of the tube. Therefore,

Thus, a narrower tube reduces the flow severely. For


example, when the diameter of a tube is reduced by a ~V = ~(lp)r~.
lt T/ I
factor of 2, the flow through the tube is diminished by a
1¢ ,
factor of 16!
Th~
value of the dimensionless co_nst~t tutlt· c
determined experimentally. For a cylindncal
turns out

EXAMPLE 12.4
consider the flow of a viscous fluid through a cylindrical
tube: the variables that may affect the volume flow rate . 'sit'· ,s
AVl.lt have been identified as the pressure gradient ~,7, !hen, we have the exact equation of Poiseuine
IS, Eq. (12.13).
CllllflnuN

328
PART TWO I ENERGY. THERMODYNAMICS, ANO TRANSPORT PHENOMENA
'

Scanned with CamScanner


£q. (12.13] c?n be £.eneralized for arbitrarily shaped
• ·ners
in the lorm ol Ohm's law: EXAMPLE 12.5
con ta 1
AV
Ap=R- What is the pressure gradient (the drop of pressure per
At' (12.14) length unit) in the aorta? Assume that blood flows as a
Newtonian fluid. The viscosity coefficient of blood is from
I re
~~
R is the flow resistance, with unit Pa s/m3. o-
Table 12.1 as 17bi.-.i = 2.5 x 1 3 N s/m2• Note this example
differs from Example 12.2 because no change in the aortic
~ Ohm :s I.a w states _that the volume flow rate of a cross-section is assumed.
Newtonian flu id rs proportional to the pressure difTerence
along the tube, and that the proportionality constant is the Solution
floW resistance. The volume flow rate in the aorta is AV/At= 8.3 x 1 o-s
m3/s
(see Example 12.1 ). The aorta is cylinder-shaped with
This relates to the everyday use of the word resis-
an inner diameter of 2.2 cm. We apply Poiseuille's law to
tance since, if the resistance is high, a large pressure dif- obtain the pressure gradient Apll:
frrence leads to only a small volume flow rate. In our
discussion of electric currents in Chapter f 8 we will com- Ap = AV 811 _ 1_
pare Eq. [ 12.14] to one of the laws of electricity, which I tJ.1 7T r~t.,'

is also called Ohm's law. We will see then that both laws This yields:
are conceptually the same, except that we study viscous
flow of fluids in the current chapter and then the flow of
charges in a conductor.
Ap = s(s.3x 10-s~)(2.s x 10-J ~)
for a cylindrical tube, the flow resistance is defined I TT(l.I x 10-2 m)4
by Eq. [ 12.13]. It is directly proportional to the viscosity = 36 Pa.
coefficient 1/ of the fluid: m

81
R = -4-1/· (12.15)
1Trtube
It is important to note that Poiseuille's law cannot be
extrapolated to the case 1/ = 0. In particular, Eqs. [ 12.12]
12.3.5: Newtonian Fluid flow with Variable
and [12.13] do not predict an infinite velocity or an infinite Tube Size
volume flow rate in this case. This interpretation would be When ideal dynamic fluids flow through a tube of vari-
inconsistent with many other laws of physics. Why is that able size, we see that the combination of the equation
so? Essentially, the answer is that T/ > 0 is an assumption of continuity and Bernoulli's law is sufficient to deter-
in the derivation ofEq. (12.12]: we used mechanical equi- mine all fluid parameters at any point along the tube. The
librium between a force pushing the fluid forward and a fluid pressure in particular varies because of variations in
force holding it back. If 1/ = 0, this equilibrium requires the speed of the fluid. The associated changes in kinetic
that the pressure gradient along the tube is also zero. Thus, energy are accounted for with work done against the pres-
substituting the conditions for an ideal dynamic fluid into sure in the fluid.
Eqs. [12.I2J and [12.I3] leads to a division of zero by Once we move from the ideal dynamic fluid to the
zero, which is mathematically undefined. Newtonian fluid , that is, once we include interactions of
the fluid molecules with each other and the stationary tube
walls, an additional efTect on the pressure in the fluid has
to be included: to overcome viscous flow resistance to
sustain steady-state flow, a pressure gradient is required
ANewtonian fluid is forced through a tube to obtain a cer- along the tube that leads to a decreasing pressure toward
tain volume flow rate (experiment 1). If the same fluid is downstream.
then forced through a tube of the same cross-sectional area In the next step we return with the Newtonian fluid
with double the length (experiment 2), how has the pressure to the case of variable tube size. More specifically, we
difference ~p2 along the tube changed from the previous identify which role the equation of continuity, Bernoulli's
value Ap 1 if we observe the same volume flow rate?
law, and Poiseuille's Jaw play in this case.
(a) .lp2 = Ap1
• Fig. 12.15(b) illustrates why the equation of continu-
(b) Ap2 = 2 Ap 1 ity must also apply to Newtonian fluids in tubes with
(c) Ap2 = 4 Ap1
variable tube size: with or without flow resistance, no
place exists for either excess fluid to collect or fluid
(d) Ap2 = 8Ap1 to disappear along the tube. Thus, all equations from
(e) Ap2 = 16Ap the beginning of this chapter up to Eq. [ 12.4] apply
1
to Newtonian fluids.

CHAPTER 12 I FLUID FLOW 329

Scanned with CamScanner


.....
s4
li
radius along the studied tube length 1
• Fig. 12.15 illustrates the effect of 13ernoulli's law in a . . . n~tca·•
10 segment the tu be 11110 sections of f1 u, we
Newtonian fluid for a variable: tube (while: neglecting . . . 'd IXc(j r . 111«.
then combine t11e tnt11v1 ual flow r •.· ad1u~ ·~
at this point changes to Poiseuillc:'s law due to th.e . . csi ~tu •iii
overall flow resi stance lor the var'i · hi nccs 1 (
constriction). Part (a) of the figure shows the lami- a c t o4·
approach was taken hy Robert Gust· . Uhc. 'lh·1
nar flow through a tube of fixed diameter. A constant av K1r h 11
pressure gradient applies as predicted by Poiseuille's 1845 and kd to.
two new laws one r
• • 1o r a ·
c hor1
1~
of varying radius and one Ior branchin} ~' 11 glc tu~
law. Fig. 12.15(b) highlights the clwnges that occur g tubes. '~
if the tube diameter varies locally: 13ernoulli's law
allows us to determine. at any given point along the 12.3.6: Kirchhoff's Laws
tube, the pressure relative to the value at the same
Kirchhoff's laws describe how flow resista
location if the tubc diameter had not changed (dashed . . h' h nccs ha
be combined m cases m w 1c a Newtonian ti . ve ~;
line). Thus, Poiscuillc's law governs the actual flow . or para IIeI, for exarni1u1d fl 0~1
through tubes .m series
through the tube, while 13ernoulli's law allows us 1
blood passes through the various sections oft~ c, 11 heri
to correct the pressure locally due to tube diameter e card
variations. vascular system. Two Iaws are to be formulated I<>
vessels in series (for example, the aorta and 'one f1.t
leading to the liver) and one for parallel ve:n arie~
1
(a)

- -- ·11 . b
example, a bed o f capt anes etween an arterial
se s ff0r

--- -
venule in the liver).

· h
e arid a
Kirchhoff derived these laws originally not ~o fl .
.
but for flow mg electric c arges. In the physics liter
. .
r Uldi
ature
therefore, you find them applied primarily in elecir· . ·
• • !Cit\'.
However, m physiology they are more important:
10
fluid flow.

Blood Vessels in Series


(b)
- --- Let's assume that a given amount of blood passesa

--- -. vessel I with a given fl.ow resistance R1, and then passes
a vessel 2 with a flow resistance R 2. We assume funhc
that the blood vessel does not branch between vessel I
and vessel 2.
The volume flow rate of each of the two vessels
obeys Ohm's law, as given in Eq. [12.14]. In other
words, the respective drop in blood pressure along the
Figure 12.15 Role of Bernoulli's law for Newtonian fluids. (a) Laminar vessel is equal to the product of the flow resistance ii
flow through a tube of fixed diameter. (b) The tube diameter varies. the vessel and the volume flow rate of blood throu~
Bernoulli's law predicts the pressure relative to the value at the same the vessel:
location if the tube diameter had not changed (dashed line).
AV
Apt = RI "";ft
Why is that so? The physics we discussed when
deriving Bernoulli's law was correct. We neglected only AV
the wall interaction, which we later found to be substan- Ap2 = R2 "";ft·
tial enough that it cannot be neglected in practical cases.
However, we can argue that the pressure gradient in a Note that the volume flow rate is the same in both vei·
blood vessel is small enough that its effect over short dis- sels because the volume of blood is conserved for a.i
tances can be neglected. Thus, if a blood vessel changes incompressible fluid. And it does not change be~a:
10
its diameter along a distance of a few centimetres or mil- no branching points exist between the two vessels
limetres (for example, at a vasoconstriction), the pressure combined system. ~
variation predicted by Bernoulli's law will be the domi- In addition to studying each vessel separately, "'.e
4
nant effect we will observe. also describe the combined system with Eq. [12.l l·
• Further to the effects that we established already
for the ideal dynamic fluid, the formula for the A = AV
P Requivalcnt At '
flow resistance (Eq. [12.15]) also suggests that the
flow resistance changes as the tube diameter varies. . . . esist3II'"'
Thus, we cannot use Poiseuille's law in the form of
111 which we mtroduce an equivalent flow r ~e 1~ 11
R h. · · n of 11' 4
Eq. [12.13] as it does not allow for a variable tube . equivalent• w tch must be a combmatto . th' ~ '
individual flow resistances. The term AV/~ 1111

330 PART TWO I ENERGY. THERMODYNAMICS, AND TRANSPORT PHENOMENA

Scanned with CamScanner


f". •.

of 1 BO L th
li0 handle the daily filtration
2 14 • e two L ·
A detailed view of the glomerulus in Bowman's cap- ave more than N = x 10 pores Th· ~'dn...
must h . h · is nu -n
sule is shown in Fig. 12.18, which is a sketch that shows. obtained from the result m t e last equation: rn~ "
from left to right, two different magnifications. In part (a}.
the supplying arteriole is visible at the top left. It l~ads
to tangled loops of capillaries-resembling a skein of
(~~)kidnt)S ~~t,/•
= (

wool-that are embedded in a capsule and finally leave


as a blood vessel at the top right. The renal tubule, wh"ICh which leads to:
collects the filtrate is shown at the bottom where it leaves L
180-
Bowman's capsul~. which therefore serves as the primary day - 2.2
fluid collection container. N = ___ ..,.;..__
3 - x 1014 "" 2 x 1014
9.
5 x I0 - 21~ .
s
(a) (bl
The number of pores per nephron, the u .
· d b d. 'd' nit Sho
in Fig. 12.17, is o?ta1ne Y. 1v~ mg N by the nurntier'fin
nephrons in our kidneys, which 1s about 2.4 million Of
rons. Thus, we need roughly 9 x 1?~ pores Per ne~~­
_.. --- which is a number close to 100 million. We see th OI\,
at the
Physiological. performanceh of hthe. kidneys on a macro.
scopic scale 1s based on t e p ys1cal properties of
. . f . atre-
mendous number o f m1croscop1c unct1onal units.

Figure 12.18 Detailed sketch of Bowman's capsule. (a) Blood


supply to the capsule is shown at the top and the renal tube for fil-
trate removal at the bottom. (b) A cross-section of the wall inside
Bowman's capsule is highlighted (green dot) with the capillary mem·
brane (1), the basement membrane (2), and the podocytes (3). In this
sketch, blood is at the top and urine is at the bottom.
12.4: Special Topics in Fluid Flow
12.4.1: Beyond Laminar Flow: Turbulence
-
and Convection
Fig. 12.18(b) shows a cross-section through the capil- When local velocity gradients in the fluid become 100 '
lary inside Bowman's capsule. The capillary membrane (1) big, turbulences occur. Turbulent flow is a superposition
is porous, but enclosed by the basement membrane (2).
of laminar flow (discussed above) and vortex fonnation
The basement membrane is embraced by podocytes (3)-
cells with arm-like extensions-leaving slits open for fluid
vortex motion. The different flow patterns for laminar
flow. The pores in the capillary membrane on the blood flow and turbulent flow around a solid cylinder are sho11n
side are typically 20 nm in diameter. The basement mem- in Fig. 12.19. In the turbulent case, flow lines are not COll-
brane at the centre is 50 nm to 80 nm thick and contains tinuous but terminate or start in vortices.
pores of 12 nm diameter. Thus, the pores in the basement
membrane determine the volume flow rate.
The actual value for the pressure drop across the Fluid flow Fluid flow
basement membrane varies between zero and 1.3 kPa to ----+ ----+
allow the body to regulate the flow using variations in the
blood plasma pressure.

Solution
We use r 1ubc = 6 nm for the radius of the pores in the base-
ment membrane; Ll.p = 1.3 kPa is the maximum pressure
difference across the membrane; 50 nm for the length
of the pore, which is equivalent to the thickness of the
basement membrane; and Tl = 1 .4 x 1 o-3
N s/m2 for
Rgure 12.19 Two flow patterns: laminar flow (left) and turbUle!lt fll'I
the viscosity coefficient. With these values we find from
(right) around a solid cylinder immersed in fluid. Note the vortei ¢
Poiseuille's law the volume flow rate of a single pore:
tion for turbulent flow.
AV= 11'(6 X 10- 9 m) 4 (1.3 x 103Pa)
At 8( 1.4 x I0- 3 N s)
m2 (5 x 10-sm)
Turbulent How has a major effect on th.e
flow rate. This is illustrated in Fig 12.20, which ·"I
5
,-o~ I
m3 h · d·ff eren•
= 9.5 X 10- 21-
3
... I x 10-20~ t e volume flow rate as a function of pressure 1 •5~9
s s. along. a given
· tube. At low pressure differences.Qhll11101bl
appltes; that is, the volume flow rate is proporuo~3 • ~
pressure difference. The vertical dashed line indicat~

334 PART TWO I ENERGY. THERMODYNAMICS. ANO TRAN


SPORT PHENOMENA

Scanned with CamScanner


~--- ..... ~

rcssure difference at which the flow undergoes the tran- density or the fluid. /fr is a dimensionless parameter. In a
~itionfrom laminar to turbulent flow. At larger pressure cylindrical tube, laminar flow is predicted for Reynolds
differences Ohm's law no longer applies. and increasing numbers Re :5 2000 and turhuli:nt flow for /fr 2: 2000.
the pressure difference to obtain a volume flow rate The Reynolds nurnhcr is useful when we discuss
increase beyond the transition is incffrctive. Thus, once systems in which turbulence control is required. For
the flow has become turbulent, no significant increase of example, turbulence suppression is a design criterion
the volume flow rate can be achieved. for birds' wings in addition to providing lifi and thrust
to overcome air drag. Wings arc shaped primarily such
that air above the wing travels faster than that below. Due
to Bernoulli's law, a lower air pressure results above the
wing. The net ellcct is a lift force sullicient to compensate
the efTect of gravity on the bird. The faster the bird flies
AV through the air the stronger the lift force. and flapping of
At
the wings is no longer necessary. Energy conservation is,
however, only one reason why large birds, such as most
birds of prey, use a flap-and-glide flight pattern. Avoiding
flapping or flapping slowly further addresses turbulence:
air is a Newtonian fluid in which a transition from lam-
Ap inar to turbulent flow occurs at high relative speeds of air
and wing surface, that is, when large velocity gradients
figure 12.20 Transition of the volume flow rate as a function of pres- are involved. Ceasing wing motion therefore minimizes
sure difference along a tube (red curve) from laminar to turbulent flow the occurrence of turbulences. Smaller birds, particularly
(dashed threshold line). Turbulent flow is characterized by minimum or finches and woodpeckers, use another approach to mini-
negligible volume flow increments for increasing pressure differences. mize flow resistance related to slowing of their flight: they
rise on one or two wing beats, then fold their wings to the
A semi-empirical threshold number is introduced body and dart through the air, eliminating turbulent air
to determine whether flow is laminar or turbulent. It is motion past their bodies at high speed. These birds can be
called the Reynolds number, Re, in honour of Osborne identified by their undulating flight pattern because they
Reynolds, who proposed it in 1883. The present discus- need to re-establish lifl through another few wing beats
sion is based on flow in a cylindrical tube. For that tube after several metres to avoid crashing to thl! ground.
geometry, the Reynolds number is given as: IJirds show a good sense for turbulence in many
ways, even when they an: not airborne. When fri gid winds
p(v)d blow along the seashore in winter. you can sec seagulls on
Re=--, (12. 19)
T/ the beach all facing in the same direction. As illustrated in
in which (v) is the average speed of the fluid, dis the diam- Fig. 12.21, the birds align their streamlined bodies such
eter of the tube, T/ is the viscosity coefficient, and f' is the that they offer the least resistance to the oncoming breeze.

· bod' wr·th the w'rnd to allow a laminar airflow. Minimizing turbulences is


Fig h us align therr res
ure 12.21 When cold winds blow across the s ore, seagu f th sand allow the body to lose heat excessively.
es~ntlal for these animals as the air vortices would ruffle some of their ea er

CHAPTER 12 I FLUID FLOW B S

Scanned with CamScanner


. t the aorta Is pulsatile, with peak veto . .
· IealIs to a lam mar
· flow 1n o f c1t1es 1
Tllls tlow ol· atr
· amun d their bodies. s range, turbulent low seems to b 11 ti...
1 - 2 mi . . I e fav ·~
avoiding the rnnling of their feathers due to turbulences r turbulent flow IS partlCU arly Undesirabl . Our~
Howeve.. d" . . h e1nb ·
that would expose their body to the low temperatures and I ·ince it greatly . 1m1nis es the votu..... 'Ooa
~~SS · ·~~
possibly cause hypothermia. . n pressure gradient. Nature again rna . rat.
for a give . x1rn1z '
. . ncy of the aortic blood flow based on F'ig ~the
eff1c1e . 12 2
pushes the volume flow rate to the greatest POssib\e.v O;~
for laminar flow, then de~elops a way to cornpensat a1\Je
EXAMPLE 12.9 velocities at which blood flow would h efer
the peak . ave lo
Determine the Reynolds number of the following three turbulent: immediately beyond the heart the aorta arc bi:
systems: (a) water flow in a creek, (b} airflow through the 0 which allows it to buffer the rushing blood . hr:s
180 • d. th. wnh u..
trachea, and (c) blood flow in the aorta. Windkessel effect. We iscuss is effect in Chapter "II!
because the elastic response of the aortic wan Pl 13
. . ays a
Solution key role tn it. . .
Solution to part (a): In most creeks of 1.0 m width, water Pathological vasoconstrtct1on (vessels becom·
flows at speeds of 1.0 mis to 10.0 mis. The density of narrower due to illn~ss) may c~u~e turbulent blOQd ~
water is 1.0 x 1o3 kg/m3, and its viscosity coefficient Based on the equation of continuity, the blood SP0e<J .·
is 1.8 x 10- 3 N s/m2 (see Table 12 .1 ). This leads to a increases in this case because the heart still pumps ~
Reynolds number of 1.0 x 1os s Re s 1.0 x 107 ; that is, same amount of blood through the aorta. This leads t
the flow in a creek is always turbulent. potentially dangerous ·increase ·in the Reynolds number oa
Solution to part (b): We assume 15 inhalations per even though the diameter of the b lood vessel d .IS I I

minute of 0.50 L each. For the volume flow rate of air reduced. To illustrate the net effect, we use the equation
through the trachea, this yields: of continuity to determine the dependence of the blOQd
speed on the vessel diameter:
.lV .
Ti= 2(15 mm- 1)(0.5 L)

m3
lvlA = lv17T(~)2 = const =? lvl oc ~r
= 2.5 x 10- 4-.
s
Thus, even though the diameter of the blood vessel is
An additional factor of 2 is introduced since each reduced in the case of pathological vasoconstriction, the
inhalation is followed by an exhalation, doubling the overall effect on the Reynolds number is an increase:
volume flow through the trachea per breath. Using the
diameter of the trachea as d = 1.0 cm, the average speed p(v)d I
Re= - - c c -
of air is determined from the equation of continuity: T/ d"
m3
2.5 x io- 4 - Let's focus specifically on the aorta. Peak blood veloci-
1 AV s m ties in the aorta can also increase due to a defective aati:
v = - - = = 3.2 -.
A At 7T(5 X 10-3 m)2 s valve. Fig. 12.22 shows the peak velocity of blood ejectoo
from the heart as a function of time for a particular patient.
Using the density of air as p = 1.2 kg/m3 , and the vis-
cosity coefficient as T/ = 2 x 10-5 N s/m2 (Table12.1), we
find Re = 2000, that is, a value near the threshold to tur- v (mis)
bulent flow. The actual flow is turbulent because the inner
trachea surface is not smooth. Turbulent flow is desired
because the inhaled air must be moistened in the trachea;
5 ------------------
moistening occurs when dry air is brought into contact
with the moist trachea wall. This contact is more efficient 4
for turbulent flow. Fig. 12.20 then shows why airflow in the
trachea has a Reynolds number close to the laminar-to- 3
turbulent transition. Once flow is turbulent, little gain in the
volume flow rate is achieved by increasing pressure gradi-
2
ents along the tube. Thus, operating far into the turbulent
regime would unnecessarily increase the physical work
required for breathing. 0 3 4
2
Solution to part (c): The average speed of blood in the Time (years)
aorta is 0.20 m/s (see Example 12.1). Using d 2.2 cm for =
the inner diameter of the aorta, T/ = 2.5 x 1o-3 N s!m2 for Figure 12.22 Peak velocity of blood ejected from the 11eart ~
the viscosity coefficient of blood, and p = 1.06 x 1o3 kg!m3 function of time for a particular patient, as measured bY ~
for the density of blood, we find the Reynolds number ultr~und. The patient's peak blood flow velocity increased sin~
is Re = 1900, that is, again a value close to the transi- ~~~~g a_3-year observation period, approaching athreshOld 8~ 11
tion laminar to turbulent. If you keep in mind that blood add this threshold Is reached, open-heart surgery is reQ
_,...,.., ress a defective heart valve. _,J

336 PART TWO I ENERGY. THERMODYNAMICS, ANO TRANSP


ORT PHENOMENA

Scanned with CamScanner


The speed of blood is measured by Doppler ultras d
diagnostic tool we discuss in Chapter 23. This pat~unt' •
a . . 1en s
1oc1ty increased steadily during a _
peak blood flow ve. d . 3 year
observation perio • ap.proaching a threshold at 5 mis,
which is considered clinically the maximum tolerable value.
This value exceeds the. value we used to calculate the
Reynolds number for this system by a factor of 2S, illus-
trating the added tolerance due to the elasticity of the blOOd
vessel walls. Data such as those shown in Fig. 12.22 allow
the medical team to plan a heart operation and prepare the
l
patient for the subsequent treatment over a considerable
PIJ mJ
timespan.
\X'.1trr

12.4.2: Beyond Newtonian Fluids:


0 10 20 30 40t i 50 60
41 4 1'
HrmJt<),:nt
The Viscosity of Blood
We have so far treated blood as a Newtonian fluid. Figure 12.23 The viscosity coefficient for whole blood relative to the
viscosity coefficient of water as a function of the hematocnt value. which
However, its heterogeneous composition leads to no\'el
is a parameter measuring the volume fraction of blood cells in \\tlole
properties that we cannot explain with the model devel- blood. The average hematocrit value for males is 46 and for fem:iles 41 .
oped for the Newtonian fluid. \Ve illustrate this point for Thus, males have. on average. blood of a higher viscosity, as indicated by
the viscosity coefficient of blood TJ. as listed in Table 12.1. the dashed lines.
and the flow resistance R, which we defined in Eq. [ 12.15].
As that equation showed. both parameters are closely
related in the form R :x: TJ. Earlier in this chapter we
defined the viscosity coefficient as a material constant.
which depends only on the temperature. As no depen-
dence on other macroscopic parameters was identified,
blood viscosity in the cardiovascular system of an endo-
thennic species should be constant. The flow resistance R
in tum depends on geometric factors. such as tube length
and tube radius, and the viscosity coefficient TJ. Thus, for
blood flow in a particular blood vessel of an endotherm,
flow resistance should be constant as well. Physiological
observations illustrate, however, that the viscosity of
blood and its flow resistance in a given blood vessel
depend strongly on two additional factors: Figure 12.24 Pumping is required to force Newtooian and non-Newtonian
fluids (for example. water and blood) at various rates of flow through a
• The hematocrit value is the volume fraction of blood straight tube. As the intended volume flow rate increases. the pressure the
cells in blood. A higher hematocrit value leads to pump must produce increases as well. Note that for Ne>o'v1onian fluids (1)
a higher viscosity. This is illustrated in Fig. 12.23. the slope of the line relating flow rate and pressure is constant However. for
which shows the viscosity coefficient of blood (rela- blood (2), the flow resistance is very high at low flow rates but apprroches
tive to the viscosity coefficient of water) as a functio~ the value foc Newtonian fluids at higher flow rates.
of the hematocrit value. With an average hematocnt
value of 46 for males and 41 for females, we note that
the blood of males is more viscous. As a consequence of Fig. I 2.2~. hlood flow cannot he
1 The flow velocity of the blood: The viscosity co.ef- allowed to fall below a minimum speed. as a d:lngenms
ficient is inversely proportional to the flow velocity. feedback-loop effect may occur. \\'e illustr.He this fredhack
This is illustrated in Fig. 12.24, which shows the vol- loop in the context of an an:lphylactic sh<X·k. An 1111,1phy-
ume flow rate of a Newtonian fluid (I) and the volume /actic shock is an allergic reaction of the b...xi~ ·s immune
flow rate of blood (2) as a functi on of the pressure system in response to a second contaet \\ ith an anti£en
difference along a vessel. The viscosity coefficient to which the body has become sensitized. :\n example is
a se\'ere peanut allergy. The initial reaction l)f the body
affects the flow resistance; it is constant in the case
of a Newtonian fluid (blue curve in Fig. 12.24), but is a histamine release that leads tl) a peripher.il 'asodila-
tion. that is. blood \'essels \\ iMning. The increas1.-d cross-
varies as a function of pressure difference for blood
(red curve). sectional area of the blood wssels causes the blood !low

CHAPTER 12 I FLUID FLOW 337

Scanned with CamScanner


2
Viscosity coctlicient 17: N s/m
to slow down, as predicted by the equation of continuity. • 3
Flow n:sistance R: Pa s/m
Due to the non-Newtonian behaviour of blood, the slower
flow leads to an increast• in the viscosity. Thi s causes

further slowinl.! of the blood flow, which again results in LAWS
a further incre~se in the viscosity. In the end, this feed- Eciuation of continuity (fluid mass conservation) for larn·1nar
back loop lc.:ads to the cessation of the volume flow rate. • flow:
:1VIJ.t = 0, a state called stasis. AV
But why does the viscosity change with the flow - =Av= const,
At
velocity? The answer lies in the fact that blood is hetero-
geneous. Normally, blood cells are well-immersed in the where AV/At is the volume flow rate, A is the cross-sectional
area of the tube, and vis the speed of the fluid.
blood plasma due to the fact that they are nucleus free,
contain a low viscosity cytoplasm. and have a highly flex- • Bernoulli's law for an ideal dynamic fluid:
ible cell membrane. This allows blood to behave like a
low viscosity emulsion (mixture of two liquids). When I
p + -pv2 + pgz = const,
blood flows slowly. however. aggregation of the red blood 2
cells (erythrocytes) occurs in a process called 1111mm11/a- where p is the pressure, z is the height, and v is the speed
tion. The red blood cells form a structure resembling a of the fluid.
stack of coins. Nummulation creates a highly viscous sus-
pension (mixture of solid in liquid). • Poiseuille's law for a Newtonian fluid in a cylindrical tube:

AV 1T Ap
- = - r4 -
SUMMARY At 8 T/ tube I '

where rtube is the radius of the tube, and Ap is the pressure


DEFINITIONS difference along the length I of the tube.
• Laminar flow: flow tubes (around flow lines) are not cre-
ated in the flow; they do not intersect or vanish in the flow. • Ohm's Jaw for a Newtonian fluid:

• Ideal dynamic fluid: a fluid whose flow is laminar. The fluid AV


molecules interact only through elastic collisions with con- Ap =Rt:(,
fining walls (frictionless motion).
where R is the flow resistance. The flow resistance in a
• Newtonian fluid: a fluid whose flow is laminar. The fluid cylindrical tube is:
molecules interact inelastically with each other and with
the confining walls, causing a velocity gradient in the fluid. 8/
R = -4-T/·
1Trtube
• Viscosity is a property of resistance to flow in a fluid. It is
quantified by the force needed to move a plate at constant
velocity Av parallel to a plate at rest at distance Ay, both
immersed in a fluid: MULTIPLE-CHOICE QUESTIONS
MC-12.1. The diameter of a tapering tube decreases from d to
1
d2 = d 112. As a result, the volume flow rate oflaminar
flow changes to
(a) AV/At= AV/At.
where A is the area of the plates, T/ is the viscosity coef-
(b) AV/At= (l /2)AV/At.
ficient, and /Av/I Ay is a velocity gradient.
(c) AV/At= 2AV1/At.
• Reynolds number for the transition from laminar to turbu- (d) AV/At = (l /4)AV/~t.
lent flow in a cylindrical tube (Re < 2000 is laminar· Re > (~) AV/At= 4AV/At.
2000 is turbulent): ' MC-12.2. F.rg. 12.25 shows a cylindrical tube of changing
diameter with an ideal dynamic fluid (blue) fl owing
p(v)d toward the right with initial speed r. The vertical col·
Re = --
T/ ' umns are connected to the main tube. Which of the
where (v) is the average speed of the fluid, d is the diam- '.our choices shows the proper elevations of th~ fluid
eter of the tube, 11 is the viscosity coefficient and is th m each of the three vertical columns?
· o f t he fl· urd.
· ' P e MC-12.3. The volume flow rate and the mass flow rate in fanii·
dcnslly
nar flow are
(a) the same.
!NITS
(b) proportional to each other.
Volume flow rate AV/At: m3/s
(c) inversely proportional to each other.
Mass flow rate Am/At: kgls (d) unrelated.
(e) related in a non-linear manner.

PART TWO l ENERGY, THERMODYNAMICS ' AND TRANSPORT PHE NOME NA

Scanned with CamScanner


·ru
..
3
.!
I
4
J
}
~
4
3
2
4
J
2
4
3
2
MC-12.a. Bernoull i's law is an expression of
(a) the conservation of mass.
(b) the con~ervation of kinetic energy.
(e) the conservation of total energy.
(<l) the rnnservation of vclm:ity.
(e) the conservation of momentum.
MC-12.9. The conservation of mass leads 10 the following law
that we use tu <lc~cribc laminar flow in a fluid .
(a) Bernoull i's law
(D) (b) Pascal's law
(Cl
4 (c) equation of continuity
.. 4 3 4 (<l) Poiseui lk"s law
J 4 2 3 4
J (c) Ohm's law
2 3 2 3 MC-12.10. Do the following experiment as shown in Fig. 12.26:
2
2 l
Push a pin through the centre of a thin sheet of card-
board. Locate the tip of the pin in the central hole
of a thread spool from below. Hold the cardboard
from below and start to blow through the hole. The
cardboard will not drop to the floor when you release
it. Which law explains this effect?
figure 12.25 An ideal dynamic fluid (blue) flows through a cylindrical
(a) Poiseuille's law
tl.ibe with initial speed v. Vertical columns are connected to the main tube
(b) Ohm's law
to measure the fluid pressure. (c) equation of continuity
(d) Bernoulli's law
(e) Pascal's law
MC-12.4. An artery has ballooned at one location outward (to
a larger cross-section) due to an aneurysm. Which of I
the following statements is correct for blood flowing I
I
through this broadened section? Treat blood as an .-----.•----
ideal dynamic fluid.
(a) Blood will rush faster through the broadened
section due to the equation of continuity.
(b) The equation of continuity predicts a decrease
of the blood pressure in the broadened section.
(c) Bernoulli 's law predicts that the blood pressure
in the broadened section is lower than in an
adjacent section of the blood vessel, causing the
blood vessel to temporarily collapse.
(d) Bernoulli ' s law predicts that the blood pressure
in the broadened section is higher than in an
adjacent section of the blood vessel. figure 12.26 A pin is placed at the centre of a thin cardboard sheet.
MC-12.5. The equation of continuity is an expression of The pin's tip is then placed in the central hole of a thread spool. Hold the
(a) the conservation of mass. cardboard then start blowing through the hole. Release the cardboard.
(b) the conservation of total energy.
(c) the conservatio n of kinetic energy.
(d) the conservation of velocity. MC-12.11. Do the following experiment: Hold two sheets of
MC-12.6. Bernoulli's law contains a tenn that was derived from paper parallel to each other at a distance of about
the kinetic energy of the ideal dynamic fluid. What 1 cm to 3 cm. Dlow gently between the two sheets
unit does this tenn carry? from their edges. The two sheets will be pulled
(a) J together. Which law explains this observation?
(b) N (a) Poiseuille's law
(c} mis (b) Ohm's law
(d) mJs2 (c) equation of continuity
(d) Bernoulli's Jaw
(e) Pa . d mic
Mc-12.1. Which law connects the speed of an idea 1 yna (e) Pascal's Jaw
fluid to its pressure?
Ca} Pascal's law
(b) Newton's second law
(c) Bernoulli's law
(d) Laplace's law

CHAPTER 12 I FLUID FLOW 339

Scanned with CamScanner


l)'P 0 <lcnnic n~·cdlc is 4.0 cm lorw ·lnd Ii . .
216. '\ l , "' • as an inner
pA · d"ini-:t~·r
1. of 0.- 5 mm. What cxccss pres"ur . · .
., e ls required . P-12.19. We study How of a Newtonian fluid through two
,, the necdk so that tl11: fl ow rate of wat , th di fferent tubes (index I and index 2). The pres-
a10 "'
11
.., .. • , .. . . . . er rough
't ·s· o 10 ~c./s. Use th1;; \ 1srns1ty codlret"'nt
1 1 • "
of water at sure differences hetween the two ends of the tubes,
10oc from Tabk 12.1. Ap, arc the sa me fo r both tubes, ilp 1 = :ip2 . The
217. Ahorizonta.I tub_c of radius r "" 5.00 nun and length 1"" tubes differ in radius and length : length of tube I
p.-1 . 50.0 m carnes ml (_l7 "" 0.1 20 N s/m2). At the end of the is 11 = 2 m, length of tu be 2 is /2 = I m, radius of
rube. the tlow ra~e 1s 85.0 cm 3/s and the pressure is I' "" tube I is r 1 =2 cm. and radius of tube 2 is r 2 = I cm.
I·00 atm. What 1s the gauge pressure at the be,- · Calculate the ratio of volume flow rates through the
ginning
of the tube'? two tubes.
A patient is to be injected with 0.50 L of an electrolyt P-12.20. Fig. 12.30 shows a tube A with radius R that splits
8.
P-121
. solution over ha It' an hour. Assuming that the solu-c into two equal tubes Band C with radii r = R/3. (a) If
tion is elevated by 1.0 m above the arm. and that the we substitute the ideal dynamic fluid for a Newtonian
needle is 2.5 cm long, what inner radius should the fluid with given viscosity coeffici ent, how do the
needle have? Use water parameters for the solution and results in P- 12.6 change? What additional parameter
must be measured for tubes B and C? (b) Sketch the
assume the pressure in the patient's vein is atmospheric
pressure in the fluid from left to right along tubes A,
pressure.
B, and D.

ANSWERS TO CONCEPT QUESTIONS area of the two small blood vessels. The equation of con-
tinuity leads to vsma 11 = 8 v1arge •
Concept Question 12.1: (b). Eq. [ 12.2] reads in this case
1• /v2 =AzlA 1 = 3A 1/A 1 = 3. Thus, v 1 = 3 v2, or v2 = v/3.
Concept Question 12.3: (b). We use Poiseuille's law twice to
1
write the ratio for the volume flow rates in both experiments
Concept Question 12.2: (a). The cross-sectional area of (note that the radius of the tube is the same in both cases):
the large blood vessel is 16 times as large as that of a small (~V/~t) 1 /(~V/~t)2 = (i1p 1/ 2)/(i1p2 / 1). Now we substitute (~ VI
one, and thus 8 times as large as the total cross-sectional ~t) 1 = (~V/~t)2 and /2 = 2/1• This yields tlp2 = 2tlp 1•

Scanned with CamScanner


CHAPTER 14

Waves

e are most familiar with waves in th: _cont~xt

W of water. You have certainly been fam1har with


waves since you were a child-perhaps from
splashing water in a bathtub, or tossing a pebble into
a calm pond. The disturbance produced by a splash or
pebble generates a ripple of water, a circular wave that
spreads outward. By repeatedly splashing, we can pro-
duce a series of ripples, each a circular wave that moves
outward. In reality, it is the disturbance that is moving
outward, not the water. If you watch an object resting on
the surface of the water, such as a toy, a paper boat, or a
leaf, you will see that the object moves up and down at its
resting position as the wave passes by. The object does not
move along with the wave. The upward and downward
motion of the object shows that it gains kinetic energy. Figure 14.1 When wind moves through a grass field it produces ms.
Thus, waves transfer energy. but the grass does not move in the direction of the wave's motion.
Humans can make waves during a football or soccer
game in a stadium. People raise their anns or swing them and treatment (internal organs and tissue imaging), and
right to left or left to right. Even though the wave moves ultrasound is used to image internal organs.
around the stadium, the fans remain at their seat. The fans In this chapter, we will study wave motion. We \\ill
make the wave without changing their location, in the examine different types of waves with an emphasis on
same way that waves pass through water without moving mechanical waves. We will describe their physical pro?'
the water itself. Wind can also generate waves in a wheat erties, their mathematical description, and what occurs
or grass field. As the waves propagate through the field when waves encounter each other (superposition).
the grass stalks swing back and forth without changin~
their location (Fig. 14. I).
We are surrounded by waves: mechanical waves _14_._1:--..:.W. :. .:h..:.:::a:..:. .t~'s~a~W~a~v~e'~
· _ ____
such as sound, water, or seismic (earthquake) waves· and A · ~ its
electromagnetic waves, such as light, radio waves: and wave is a disturbance that travels outward irolll
source ·m a med'mm. A wave disturbance is produced.b·1
X-rays. Waves are central to our daily life since we rece·
. Ii .
most o f our 111 onnat1on from or by them (radio TV II
1ve a~ oscillator that creates vibrations. Thus, a wave 15'
. F" • , ce vibration that is travelling through a medium. ..
phones, I nternet~ W1- 1). In medicine, radiofrequency
waves are used 111 MRI (magnetic resonance ima · ) % ves are placed into two broad cateo' 00 1
· fi d d ·
111 rare an microwaves are used in heating and heat
gmg' M.echanical waves include water waves, waves er
Slinky so d . the wa1 •
treatment, X-rays and gamma rays are used 1·n d"1agnoses· ' un waves; they cannot exist without .! #
the Slinky, or the air that they travel through. The subs '

376 PART THREE I ELASTICITY ANO VIBRATIONS WAVE


' S, AND SOUND

Scanned with CamScanner


-~roug...I I which these. waves travel is call c·d t 1lC "'"'' •
v• All irn:chamcul WU\'cs n:4uirc· (i) as
aw~y again. In this case, by oscillating the end of the Slinky
t:
fdiU11•· . . . · · ourcc of dis-
111 • (ii) a medium to be d1sturb1.·d. and("') . .
(b.1ncc. · 111 ·' physic I horl/ontally. you make a series of alternating compression
Ill h nisnl through wh1d1 clements of the n1 . 1· a and. exp·111s·
ic• :i Tl . Cl n11n can · • · 1un (rare1·actmn)
· ·
regions. \VI11c
· h traveI aIong tI1c
11 c, each otl11:r. 1c other k111d of waves . . Slinky. In this cast'. the o~cillation (motion) of' the particles of
111
• 11u~·n ~ · • - 1111.·1ud1ng
licht. r
adio waves. microwaves, X-rays- an: k
• . .. . •, 1·:1. . . . nown as t~ic disturbnl mcdiurn is in the same dirt·ction as the propag:i-
- omu"ndlC \\,IHS. . cctro111.1gnctic w·iv . . d tion of the wave. We call sud1 a wave a lonj.!itudin:il WU\'l'.
tfcctr " . ' cs o not
requ1·r·~ :1 im:J1um to propugatc, but . . thcy do rcqtii'r.e u source
. ~ound is another example of a longitudinal \Vave; a propaga-
roduccs thc wa\c (lluctuat1on). In this cli t tion of alternately compressed and cxpandt•d air travelling
t11at P . ap cr, we
o~1tward from the soun.:c, as illustrated in Fig. 14.3. We will
w1.11 .rocus
•• on mechanical
. wa\'es. Electromagn··...·ti' c. wavi:s
. b' discussed 111 Chapter 20.
\\'I 11 t
discuss sound waves in Chapter 15 .

~ A wave is the propagation of di sturbanc~ · .


~ d ' . e111 ,1
Jl.0111 in spacc an time. 1 hus. a wave is ., ,.111
rnc .
· . • 1• cm t 11at
010, cs along the ~11cd 1 um .
but there 1s no net motion of the
rncJium along w11h the wave .
..., _.,.. • '"'"-'llr"r~~ ,,,.,

CONCEPT QU.~SJl~~l4·1·
Figure 14.3 A continuous sound wave from a speaker.
Why is it important to keep quiet and not make an impul-
sive sound such as a gunshot in avalanche country? Cliiµo@I The direction of oscillation of a particle and the
direction of the propngating wave arc perpendicular for
A great way to demonstrate waves is with a metal transvcrsc waves and parallel for lonJ.!ltudinul waves.
spring toy, commonly called a Slinky. Take a long Slinky,
fix one end of it to a wall. then pull the other end and keep
it stretched. Move your hand that is holding the end of the
Slinky up and down continuously. as illustrated in Fig. 14.2.
Which one of the following statements Is false?
This movement will cause a series of alternating upward
and downward regions to propagate along the Slinky. Even (a) A wave can have both transverse and longitudinal
though your hand moves up and down vertically in the components.
same spot. the disturbance you produce moves horizon- (b) A wave carries energy from one place to another.
tally. disturbing each coil (point, position) along the Slinky
(c) A wave does not result in the flow of the material
l'ertically. The wave travels horizontally along the Slinky. of its medium.
This is an example of a trans\'Crsc wave. The transverse
wa\'e tm·els perpendicular to the motion (oscillation) of (d) A wave is a travelling disturbance.
theparticles (Slinky coils) in the medium (Slinky). (e) A transverse wave is one in which the disturbance
Another way to make a wave on a Slinky is to move your is parallel to the direction of travel.
hand back and forth horizontally toward the wall and then

l
1 Oireciion of prop:1i:;:i1ion of wave
(:i I Transverse wave

Direction of prop:i~ation of wave

.___ -o+ (b) Lon~itudinal wave


~ e end of a Slinky perpendicular to Its length. (b) Alongitudinal wave,
· ivure 14 2 oduced by mov1ng on
Pr •Awave on a Slinky. (a) Atransverse wave, pr
Oduced by moving one end of a Slinky along (parallel to) Its length.
CHAPTER 14 I WAVES 377

Scanned with CamScanner


T that it takes for one. period
- Q
of the wave t
Obe
ated (fig. 14.5). At time t ~ , y~u start to mo gentr.
CASE STUDY 14.1
han d up and down so .
that 1t oscillates v,.. . Ve Yu
. -.rt1ca11 ~,
amplitude A, a max1mu~ d1s~a~ce your hand Y'1. itij
Identify examples of {a) transverse waves and {b) longitu-
d of the Slinky from its ori ginal positio . lakestn
d inal waves. en A . _ n in v . t
direction up or down. t time I - T/4, You h t111ca1
Answer to part (a) Electromagnetic waves, including vi~­ mpleted a quarter period and has reached n r . and h~ 1
ible, ultraviolet, and infrared light, microwaves. and radio cO . . . h ,,os1t10
waves are transverse waves. S1·nce the Slmky 1s continuous, t e parts adJ'ac ent ton·~·
hand must have been pulled up as well, as .11 Yoo1
Answer to part (b) Sound waves, including audible sound . . 14 .5. Later, at tirnI Ustrated
in the second frame o f Fig.
and ultrasound for medical applications, are longitudinal your hand has returned to its origin~! position. ~~: 1.2.
waves. since the Slinky has been following your ha d ~1 er,
Seismic waves caused by an earthquake contain both . JJ fl . n It .
not stretched honzonta y at as 1t was initial! · 11
a longitudinal and a transverse component. They travel . i~ ~
with different speeds in Earth's crust and are recorded in
sequence by a seismograph. The relative delay allows Earth
scientists to determine the epicentre of the earthquake. t= 0
Note that we don't list surface waves of water because
the medium {small water elements) moves in a circular
fashion.

When you move your hand that is holding the end of


the Slinky continuously, you produce a series of repeating
disturbances, which move along the Slinky. Each repeat
l t = T/4

of these disturbances occurs with a definite time interval, A


or period of repetition. These periodic waves are waves
whose pattern repeats indefinitely along the direction of
propagation with a fixed interval of repetition. Periodic
waves are produced by sources that vibrate in a periodic
way. Some examples of non-periodic and periodic waves
are illustrated in Fig. 14.4. l
__ .,.
t = T/2

y (x,t ) y (x,t)

A. ~6
r • x A
~;;;:::; /".. •x

l
"""-..)

y (x ,t) t = 3T/4
y (x,t )

r n • x
8'-n LJ
•x
__ .,.

y(~
y (x,t)
-A
~
I (a) Not periodic
•X
(b) Periodic
•x

l t=T
__ .,.
Figure 14.4 Non-periodic and periodic waves.

14.2: Physical Properties of Waves


We can again use the Slinky as an easy way to illus-
trate the connection between a local vibration and the Figure 14·5 Sketch of the various stages of the motion of 3 snn~
resulting wave. As before, take a long Slinky with one the free end at the origin oscillates up and down with peri()(1 T. b·~ ~
full period f th . . th osc111a .
end attached to a rigid wall and the other end stretched . o e v1brat1on is completed (bottom panel). e Slin~ 6
out by your hand [Fig. 14.2(a)]. Let us look at what hap- displaced a distance A, called the wavelength. Note ~at t11e1eft er.1'
pens in the wave that form s along the Slinky in the time assumed to be elastic and therefore stretches to maintain t11e
the same horizontal position.

378 PART THREE I ELASTICITY AND VIBRATIONS, WAVES, AND SOUND

Scanned with CamScanner


d as shown in the third frame of Fig
rcI·t1etwo
11 • d · 14·5 . 1n the
frames , your 1lan completes another h If
ne" oving down to the pos1llon · · - A and back A a . cycle .CONCEPT_ QUESTION 14.3
bY n1 f h SI' k r . gain the
. cent part o t e m Y mllows the hand and d '
ndJ3 Th · . . a own-
, d arch resu1ts. e m1t1al, upward-directed h . The wavelength of a wave in a medium increases as a
war arc did result of which of the following changes in parameters?
isappear, but moved farther to the right as sh .
110t d • • own in
the fifth frame. Thus, at lime t = T, the local oscillation (a) an increase in frequency
our hand has been transformed into a full sin 'd
of Y . k h' h . uso1 a1 (b) a decrease in frequency
ape in the S1m y-w 1c 1s the wave-reaching t
sh . \fi h oa (c) an increase in speed of the wave
point at distance I\ rom your and .. If you continue to
oscillate your hand, .after anot.her lime T-the second (d) a decrease in speed of the wave
period-the wave will repeat itself and travel another
distance ,\. You have produced a periodic wave.
EXAMPLE 14.1
CGJEllII The maximu~. di.stance of a point on a wave A buoy is bobbing up and down in the water as waves
measured from the equ1hbnum position is the amplitude. from a passing boat move past it. If these waves, having
wavelength of 7.9 m, travel at 2.7 mis; what is the fre-
Thus, A is the distance between two successive quency of the buoy bobbing up and down.
paints that .are in the sam: state of oscillation, as illus-
trated in Fig. 14.5. The distance ,\ is called the wave- Solution
length. Tis the period, or the time interval between two The wavelength and the speed of water waves are known.
successive points that are in the same state of oscillation. Using Eq. [14.1], the frequency at which the buoy bobs,
The inverse of the period is the frequency, f, which is which is the same as the frequency of the wave, can be
the number of waves that pass a given point per second. found as:
The frequency of a wave is determined by the source. A
wave travels at speed v a distance of one wavelength, ,\, 2.7~
vwavc S
during a period r. v
wave
= ..\f => f= -,\- = - - = 0.34Hz.
7.9m

C!mm The period T is the amount of time it takes for


a point on the wave to go through one complete cycle of
oscillation. EXAMPLE 14.2
What is the wavelength of a wave whose speed is 75.0 mis
In our exercise with the Slinky, we have now and frequency is 200 Hz?
described a wave with a wavelength ,\ and a period T.
We can use these to calculate the speed at which the Solution
disturbance caused by your hand propagates along the The frequency and the speed of the wave are known.
Slinky: Using Eq. [14.1]:

,\ m
V wave -- T --,\f,' ( 14.l)
v
VW&\'C
75.0-
= ..\f => ..\ = -f - = -
s
- = 0.375m.
wave 200Hz
where f is the frequency. The frequency, f, is related to
period as/= l/T. .
As defined in Chapter 13, the unit of frequency is EXAMPLE 14.3
hertz, labelled as Hz, with l Hz = 1 s-1, or 1 cycle/s.
Bats use ultrasound echolocation to detect small insects
in flight. For this to work, the wavelength used by bats
Ciilm The frequency, J, is the number of ~o~plete
must be smaller than or equal to the size of their prey. Bats
cycles of a wave that pass a given point per unit time. It therefore use frequencies of about 80 kHz. Dolphins and
is the number of oscillations per unit time in the wave. porpoises also use ultrasound echolocation for hunting.
(a) If a dolphin's prey were as small as the insects eaten
by bats, what frequency would dolphins have to use?
Gmill!iJ The speed of a wave is equal to the product of its (b) Dolphins actually use frequencies up to 225 kH~. How
wavelength and frequency. much bigger is their smallest prey compared to the insects
that bats hunt? Hint: Use v1 ;, = 340 mis for the speed of
Note that the frequency of a wave is the same as the sound in air, and use "sc• = 1530 mis for the speed of
frequency of the source, assuming that neither the source sound in seawater.
nor the medium moves. In contrast, however, the w~ve­
length is determined by the properties of the medium
through which the wave propagates.

CHAPTER 14 I WAVES 379


NEL

Scanned with CamScanner


I
Solution f= T= I Hz
Solution to part (8): The length of prey and the frequency L1x1a1 5.0 m m
used by bats are known. Knowing that A should be equal l\vol'c ~ - - - :::: - 2- = 2.5 -
lpussbo>l s S
to '·rrc)' and using Eq. {14.1]:
Using Eq. (14.1):
' 'in •ir = lb>l prcyfbal
,...... = .\f ~ - l !. - "wave
{ ''in water - dolphin prey dolphin \lw~vc = A/ =} A= J
m
2.5-
s
). = - - = 2.5m
\'innir fb:i.t I Hz
~--=--
10inw3ttr /dolphin
or A= Tvw.,·c =(I s)(2.5~) = 2.5m.
\"inw3t«
./dolphin = -,-..- _-/b>l
m:ur

1530~
s
--(SOkHz) = 360kHz.
111 14.2.1: Wave Function
340-
s Let us go back again and consider the long horizontal
Solution to part (b): The result in part (a) shows that
Slinky shown in Fig. 14.2. Move the free end of the Slinky
dolphins cannot hunt prey as small as the insects hunted up then back down to its initi~I position at rest, causing a
by bats. So: single pulse that will move with constant speed alonl!~ the
Slinky as illustrated in Fig. 14.6. The vertical displacement
·: lb>tprc)' * ldolphinprc)' of the Slinky (the medium) is in the direction perpendicular
=}
1'inoir = lbatprcy X _J;_b_
>t_ to the Slinky. Fig. J4.6(a) illustrates the profile (shape) and
L dolphin prey /dolphin
position of the pulse at I = 0, and Fig. 14.6(b) illustrates the
profile (shape) and position of the same pulse at t seconds
ldolphinprcy 11inwatcr fb>t later. Although the position of the pulse along the Slinky
- - - = -10 - X - -
l b>tprcy inoir /dolphin has changed, its shape has not changed.
1530~
= __ s x 80 kHz = 1.6.
!E. 225 kHz
340
s

Remark: Thus, dolphins are more versatile hunters,


able to detect prey that is much smaller in relation to their
body size than can bats. The typical size of an insect
hunted by bats is just over 4 mm. The much larger dolphin
can detect objects as small as 7 mm under water. The
smallest adult squid are 2 cm to 3 cm long. In addition, the
dolphin can detect objects of that size over a much longer
distance because sound absorption in water is much less
than in air, so sound can travel further in water.
Figure 14.6 A pulse moving along a Slinky. (a) The pulse on the Slinky
initially at time t = 0. (b) The pulse moving along the Slinky at time t
seconds later.
Source: Physics, 7th ed., Cutnell & Johnson, C 2006 John Wiley & Sons. Reproduced wi1!l
EXAMPLE 14.4 permission of John Wiley & Sons Inc.
A person in 5.0-m-long boat on a lake watches as con-
secutive waves reach their boat separated by 1 s time
intervals. If each wave takes 2 s to pass the boat, find the Let's take the horizontal axis of the Slinky as the
period, frequency, and wavelength of the waves. x-axis, and the vertical displacement axis as the y-axis.
'!le ~an show the pulse with a simple graph as shown
Solution 1~ Fig. 14.7, assuming that the pulse moves to the right.
The time between two consecutive waves is the period. Fig. 14.?(a) shows the pulse at time t = O. The profile of
The length of the boat and the time it takes for a wave to the pulse at this time can be represented by y =/(x). This
sweep the boat Is known. Therefore: states .that the displacement in y (the vertical position) _of
T = Is the Slinky at any point, x, is a function of x. So the dis·
placement depends on both the position x and time t, and
may be written as y(x, t). The displacement y(x. t) is called

380 PART THREE l ELASTICITY AND VIBRATIONS, WAVES, AND SOUND ~EL

Scanned with CamScanner


"'a"e f
unction. Since
.
the pulse moves w'tl
1 l speed
wet}le n'ght• it is a distance at time I flro . . . v
\'I the ri~ht). The positive sign in (x + vi) shows that the
. m tts mttial . 1s moving
wave · m . the negative x-direction (toward the
10 . . (at / -::: 0), assuming that the pulse fil
s1uon d· · pro e does 1eft). The wave function y(x, I) as a function of x defines
pO hnnge as illustrate m Fig. 14.7(b). At any t'
t c .,,. ' · d' 1me I, the 3. curve representing the shape of the wave at a given
no fii.JJlCtion (the vert1ca1 1splacement of the SI· k .
ivave . m y) is time t. When a wave moves through a medium. it dis-
....,ented as. ~laces particles (perpendicular or parallel to its motion)
rePr"~
y(x, 1) = f(x - vi).
m the medium from their undisturbed position, and then
(14.2) returns them to their initial position. Thus, the function
~x, I) represents the displacement of each particle from
'[bis equa~ion shows that the pulse is moving to the right its undisturbed position.
..direcuon), and has the same profile as it had t .
(~
r- . 0Jfthe pulse moves to the left (-x-direction) a .t11ime
. . .
irated in fig. !4.7(c), the wave function is:
, as 1 us-

. EXAMPLE 14.5
y(x, 1) = f(x + v1) . (14.3) Which of the follo.;...ing equations represents a wave
function?
The most general mathematical form for the dis- (a) y(t, t) = x - 3.0t
placement pro~uced by a wave disturbance is given by
(b) y (I, t) = (x - 3.01)2
the wave function:
(c) y(T,t) = (2.4x - 3.71)3 + 4.5
y(x,1) = f(x + vt). (14.4) (d) y(T,t) = 6.8sin('C - 3.0r) + 1.2

The term (x + vt) is called the phase of the wave. This Solution
equation is a general equation that can be applied to many Using Eq. [14.2], you see that all these equations are in
types of waves. The negative sign in (x - vi) shows that the form of y(t, t) = /(x - ,.,), so each of them represents
the wave is moving in the positive x-direction (toward a wave function moving to the right; the +x-direction.

t =0
EXAMPLE 14.6
Which of the following equations represents a wave
function? ·
(a) y(I,t) = x + 3.0t
(b) y (x, t) = (4.0x - 3.0r)2 + 3.0

(c) y(x, t) = 3.4cos(3. lx + 1.7t) + 2.4


t=t
t =0
y (d) y(x, t) = 4.1 e(l.lx+2.St) + 2.4
(bl

Solution
Using Eq. (14.3], you see that all of these equations are in
the form of y(x, t) = J{x + vt), so each of them represents a
L..:::=---L--::::::::::---~:::...--J..-_.....;::::........-__. x wave function moving to the left, in the -x-direction.
v,

t=t
t 0 CONCEPT QUESTION 14.4
Which of the following functions is not the general form
of a wave?
2
(a) y(x, t) = 6(x2 + 2t'C + 1 + 3)
2
(b) y (x,t) = 6(x2 + 1 + 3)
(c) y(t, t) = 6(x + 2t) 2 + 3
v, (d) y(x, t) = 6sin(x2 + 3t2 + 9)
~ 14·7 Awave pulse with a simple graph. (a) A pulse at initial ti~e (e) y(x, t) = - 2 sin(31 - x) + 9
2

~ ~ (b) The Pulse moving to the right with speed v. (c) The pulse movmg
left With speed V.
CHAPTER 14 I WAVES 381

Scanned with CamScanner


EXAMPLE 14.7
Fig. 14.o 6 hows tho wavo puloo that 1:i rnovi
ith speod v. What dooo tho d1:iptac() ng ilk>r.
ropo W . , rncnt ""'4 t
Which of the following functions rcprosont(s) o wnvo? ropo os 0 function of time look hke ot Point 11 ()f ~~
(~ .1(t . 1) - t-(1: - :1: + ·h )

(b) ·' (.1. 1) - t>(I : + ~)


(C) 1{ I. I) .. ~(11· - :1): T J
p
(d) all of the above

(e) none of the above


- - - -·- ------'
Figure 14.9 Snapshot of a wave pulse on a rope moving to ll'it
As discussed. the waw function l'(x. t) defines a with speed v. ~
graph representing a wave. Thus. we ar~ dealing with the
waw functions of two inMpendent \'ariabh:s. position x. Solution
and time t . We plN the waw graph only with one inde- If you stand at point P and look at the pulse c
pcncknt 'ariable. t:ikinl! the other one as const:int. So if toward you, first you s~e the front point, and at : : :
t is held constant. the graph of _1'(x. 1) plotted as a func- time you see the end point of the pulse. The displace
tion ofx shows an instantaneous pil:ture of the wave. This with respect to time is the history graph of the Pulse~
snapshot graph is shown in Fig. 14.R(a). If a point on a illustrated in Fig. 14.10. ·
wa,·e is singled out and its position _,. is held constant, the
graph plotted shows the motion of that particular point as
a function of time t. This graph is called a history graph
and is illustrated in Fig. 14.8(b).

(a) y (x. t )
figure 14.10 The history graph of the pulse in Fig. 14.9.

rL....---L-~-t
=const.~~ x

+
End point
+
Front point The wave pulse in Fig. 14.11 is moving with uniform speed
of v in a medium. Which of the graphs (a-d) shown Ii
Fig . 14.12 correctly shows the relationship between d"&
(b) y (x, t)
placement y of point P and time 1?
x = const.

+
Front point End point Figure 14.11 A pulse displacement as a function of posilioO al 31
instant of time.
Figure 14.8 Two graphs of a wave function. (a) A graph of displace-
ment as a function of x at an instant of time Is a "snapshot." (b) A
graph of displacement as a function of t at a particular point is a
"history graph.·
l•I' .l~li
I hUl!lil A graph that shows the displacement of a wa\'e
~ L3~~
as a function of position at a particular instant of time is (c) y
(d) t

l
a snapshot graph.

l '~ lUl!:il
A graph that shows the displacement of a wave
as a function of time al a specific position is a history
~ ..i ~~
Figure 14.12 The pulse displacement as a function of tirne.
graph.

382 PART THREE I ELASTICITY AND VIBRATIONS, WAVES, AND SOUND

cr1
Scanned with CamScanner
constant <p provides adjustment for the displacement, Now we have to find phase constant. At t = O and
.I'. " 0
position of the wave, at t = 0 and x = 0 as discussed in y s::2.6m: •
simple harmonic motion in Chapter 13. Fig. 14.23 illus- = 3.6 sin(O -
trates a sinusoidal wave function at y = 0 and y 0 * 2.6
sincp
0 + cp)
= 2.6/3.6 = 0.722
at t = 0 and x = 0.
cp = 46.2° = 0.81 rad = 0.257 7T.
Solution to part (c):
l•I y lhl y
y(x,t) = 3.6sin(52x - 16711 + 0.81)
y(2.7,5) = 3.6sin[52(2.7) - 1671(1) + 0.81 ]
I
I
= 3.6(-0.165) = -0.593 m.
I

14.4: Superposition
.=......:..:....:..:~~._____.~~~~~~~~~~-------

Figure 14.23 Displacement function for a given wave for two cases.
1f two or more waves with small amplitude pass one
(a) The displacement y =Oat x =Oand t =O; and (b) the displacement another in a medium in which the particles obey
y ~ 0 at x = Oand t = 0. Hooke's law, they will pass each other unchanged.
When the waves overlap, they produce a resultant
wave that is the sum of all the waves at the region of
overlap. The displacement of the resultant wave in the
EXAMPLE 14.12 region and during the overlap is the sum of the displace-
ment of the individual waves. Thus, waves can be com.
A long string with linear mass density of 0.15 kg/m is
stretched horizontally under a tension of 155 N. The left
bined in the same location in space without losing their
end of the string is tied to a simple harmonic oscillator character. This property is known as the principle of
of frequency of 266 Hz that vibrates perpendicular to the superposition.
=
string with amplitude 3.6 m. If at 1 0 the oscillator is at Generally, when the wave amplitude is not too large.
an upward displacement of 2.6 m and moving down, find the principle of superposition is valid and is applicable to
(a) the wavelength of the wave produced on the string, all waves we consider here. Waves that obey the super-
and (b) the equation of the travelling wave. (c) What is the position principle are known as linear waves. Thu '. if
string's displacement at 1 = 1'.0 s at a point 2.7 m from two or more travelling waves that are moving tb,. , ~h
the oscillator? a medium overlap, the resultant value of the wave f : 1-:-
tion at any point is the algebraic sum of the vah.il of
Solution the wave functions of the individual waves. Assum,· 10
Solution to part (a): Using Eq. [14.6], find the wave speed: individual waves with the wave functions y 1(.u; . nd
y 2 (x,t) are travelling simultaneously along a strfo ·~d
v=Jf =
155 N =
0.15-
kg
32 ~.
s
string and then overlap. The wave function of the rv 11-
tant wave on the string is the algebraic sum of their \', a1e
m
functions:
Using Eq. [14.1 ]. find the wavelength:
y(x,t) = y 1(x,t) + y2(x,t) . (I-U S)
32~
5 The resultant wave may have a complicated pattern.
,\ --.!'.
/ -- -
266-I -
lz--0 ·I 2 m. but each individual wave retains its pattern (shape or
form), and this pattern re-emerges when the waves mo\'e
Solution to part (b): Calculate k and w, and use away and separate from each other.
Eq. (14.14), the general wave equation:
IUIQill!il Principle of superposition: If two or more tra\'el·
k = 21T = 21T = 52 rad
A 0.12 m ling waves overlap, the resultant disturbance (wave) is the
21T rad sum of the disturbances of the individual waves.
{JJ = - . = 27T/= 27T(266) = 1671-
7 s
Consider two pulses produced at the two ends of a
y(X,1) = 3.6sin(52x - 16711 + <p) . stretched string, producing waves that move toward each
other. After they meet, they combine to fonn a result:int
wave, and then pass by each other unchanged. as illus·
trated in Fig. 14.24. The resultant wave is obtained b)'

aa PART THREE I ELASTICITY AND VIBRATIONS. WAVES, ANO SOUND

Scanned with CamScanner


~+
---
(a) Pulses movr toward each other

----....---
+ -~

Pul~cs rnn~1 ruc1ivcly interfere

-
- Pulses move away from each other

+--

+--
~+ (b)
Pulses move toward each other

~-+
+-\\// ~-+
Pulses destructively interfere

+-\;/
Figure 14.24 Sketch of two pulses produced at two ends of a
stretched string. As they pass each other, the vertical displacement
,' Pulses move away from each other
of Ille string is obtained by adding the displacement of the two pulses
at every point

f
! 4- - -
add' .
lb mg the displacement of each wave at each point where

l
•'·
l!'f overlap.
. Combining individual waves in the same location
~~space to yield a resultant wave is called interference.
Figure 14.25 identify the sketch of Interference of two pulses that
(a) constructiVely interfere and (b) destructively interfere.
igu~e 14.25 shows the interference of two wave pulses
rnov1
ma ng. toward each other. The two superposed waves
t
t. in Yhrcinforce each other, so the displacement of waves
C:UV!llli If the displacements of two overlapping waves
t co t c comb'rnrng· region adds. In this case, there 1s · a
add with each other to produce a bigger displacement, the
twnstroctive interference [Fig. t4.25(a)]. Conversely, interference is coiled constructive. If the displacements of
f ca: superposed waves may completely or partially

l"
t two overlapping waves subtract from each other to pro-
• is cbtacl~ach other out. In this case, the net displacement duce a smaller displacement than that of at least one of the
0
" tailed tned bY subtracting one from the other. Th'1s is ·
waves, the interference is called destructive interference.
destrucHve Interference [Fig. 14.25(b)).
CHAPTER .14 I WAVES 3gg

Scanned with CamScanner


and
EXAMPLE 14.13 y 2(r, t) = A sin(lr - wt + 1,'),
Two pulses are travelling at 7 emfs toward each other on A·. the 1,hase constant. as di~l·ussl·d in
a long string Initially their leading edges are separated by when:.,, is .. . \. I I... • • et"t1l)n I
.I . one " a\·e 1s shi t ~ l 1•) :1 pha~c • , . h ~ l
10 cm, as shown in° Fig. 14.26. Sketch the shape of the There or~· .,. 'it r
ther one :ind thus. they :m: 11u1 \lf "h s~ ,
string at 1 = 1 s. to t 11e o . .. . • ,. 3. c h
11t:int wa\'e f und1on. 1).IS(d \lll the " . ) ~
The n:st • • 1 I h r ·nn~1~1
t cm superposition. is givc1_ 1 by tie ll ge raie sun1 ilf th ea'
interfering wa\'e functi ons: ~ l\a

y(x. t) = Yt (r. t) + Y2(x, t)


= A sin(kx - cut) + A sin(b· -
WI + l )

Using the trigonometric relation:


Agure 14.26 Two pulses on a string 10 cm from each other moving
at 7 emfs.
sina + sin b a-
= 2 cos(~ h)sin. (a +b).
~
Solution
Fig. 14.27 shows the sketch of the shape of string at
I= 1 S. The resultant wave function is given by:

= 2Acos(~)sin(h cut +~). (l~.lbl


T8 cm
y(x, t) -

The resultant wave is a travelling sinusoidal " ave \lith the


same wavelength and frequency, but shit\ed by r;/~. ~O'.r
j that the amplitude of the resultant wa\ e is ~A eos(.r :l
Fig. 14.28 illustrates the intc:rfi:n:ncc: of the t\\\l id~nti,'Jl
waves given abo\'e with phase difference rp.

Agure 14.27 Shape of the string at t = 1s as two pulses reach each


other and Interfere. (a)yp
/\ I\ /\ (\
:VVV V '
''
(b)yl : / \ / \ / \ / \ (\
Since waves add to one another, does the resultant wave 7~~\J\ '

"''r
I
always have greater amplitude than either of the individual I

waves y 1 and y 2 ?

: \ JI\
\ J/\
\ ) "\ J/'__'
(a) yes

(b) no
(c) It depends on their frequency.
Agure 14.28 Interference of two harmonic waves hJving pl\35t !
(d) It depends on their wavelength. ference ip: (a) the sinusoidal wave y1: (b) the sinusoidal wave 1~:~
phase difference If! with respect to y : (c) the sinusoidal resullnnl ~
(e) It depends on their relative speed. 1
Interference of y1 and y •
2

~ 0.
14.4.1: Interference of Sinusoidal Waves When there is no shift between tWl' w:in:s. ir • ot
• . ~( P
smcc cos 0 = I the amplitude of the rcsult:int " h31t
Consider two similar sinusoidal waves (having the same · · · I til
its maximum value, 2 A. The two waves are s:Jll ,r"n!
wavelength and the same amplitude) moving on a Slinky com1>lctc ronstructln~ lntcrfcrcnCl'. Thi~ h~I · I.ii'
in the same direction. The equations of the two waves arc .. ' ,n (f
w hen t11e crests or the troughs of the tW(l " ·'' c. . 1t'o'
given by: complctl!ly and combine witt1 "ach other. pn)\hu.:i~~....t
Y t (x, t) = A sin(kx - wt) resultant wrive. Gencmlly whcncwr'"
the pha~c
• 1\itli:!IY
lflt-:I
between two harmonic waves is 'P = O. 21T. ~ lf, ....

390 PART THREE 1 ELASTICITY AND VIBRATIONS, WAVES, ANO SOUND

Scanned with CamScanner


_~) :::: "!: I a~~ con 1rucli\'C interfrrcncc oc,·urs.
,,~{,. ~t! f'h3SC Jifkn:llCC l"-'l\H'l'll (\\II h:tnnonic \\ :l\ l' EXAMPLE 14.14
\\l'~:::: :r.):r.5rr..... then cos(<{/:!) = Cl1S( 7TI :::!) = o. If the ~uperposition o f two identical travelling waves of the
" "die :iniptiru<lc l'f thl:'.' r:sultant " J \ e has it · minimurn samo frequency, wavelength, and amplitude is a resultant
J.., J • ('\c."f"\\\h~·n:. 1111.: t\\ o \\J\ l" .ire sa id l o h:l\e travelling wave of the same amplitude as each wave, what
~.r~ · ,,f'\'· . · I · f · ·
1J• d•~struCflH' Iller Crl'lll'C. rh1 ~ OCl' Ur ' \\ ti ~n ls the phase constant between these two waves?
ph•tl' • . ~
ct1tl1 ... l'f 3 ,, 3, c O\ erl.1ps "rth the troui;h of the uthl·r
"" ,-rt.I ·, ·c n•rsa. I f' ll lC pIiase ull
(;t"
·1 · l 'crelll'C is 0 < "' < .... Solution
( l'' \ l . ... ...
\IJ' ~I.' t\\ 0 ,, 3 ,es h:ne panral t·ornructi\e anJ partial According to Eq. (14.16], the amplitude of the resultant
tt•'ll .:tr'c." intertC:rcnct'. anJ the amplitude of the resul- wave is 2A co~(ip/2) . Since the amplitude of the resultant
~l(SU'I .. is Sl'rne ' alue lxt\\ een 0 and 2 A. In this case wave Is equal to the amplitude of each incident wave, then:
-1 " 3'" . '
iJ:· . ~rfrn:n.:-e ll f the t\\ O \\3\es 1s called intrrmcdiate 211 cos(ip/:?) =A
t. t. int . r· I ' ") ' .
-'erence. as sho''" in tg. -+.- 8. Fig. 14.29 shows cos(ip/2) = 112
'°ttr•· . .d . I h . .
•...-frrence ot t\\ o 1 enttca am10111e wa\'cs wi1hout
t e.111·"' . ip = 2cos- 1(1 /2) = 2( := 60°) = =120°.
t:J " ith ph.35<! sh1 ns.

Y1 EXAMPLE 14.15
/ Two identical sinusoidal waves. having the same ampli-
tude of magnitude 12.8 cm and moving in the same direc-
tion along a stretched rope, interfere with each other. If the
phase difference between two waves is 128° what is the
amplitude of the resultant wave?

Solution
According to Eq. (14.16], the amplitude of the resultant
wave is Arcsuriani = 2Acos(<p/2). So

Arcsultani = 2Acos(1p/2) = 2(12.8 cm)cos(128°/2)


Arosul1.1ni = 11.2 cm.
Remark: As the result shows, the interference is not
_, totally constructive or totally destructive; it is intermediate.
The magnitude of the resultant amplitude is between the
' Y1
maximum and the minimum:

{Arcsullanl,Min = 0 < Arcsuhan1


= 11.2 cm < A,....uhant,Mu = 25.6 cm}.

EXAMPLE 14.16
Two identical sinusoidal waves, each of amplitude
12.8 cm, interfere with each other. If the resultant wave
has an amplitude of 6.8 cm, what is the phase difference,
in radian and in wavelengths, between two waves?
f9re 14.29 krterterence of two identical harmOnic waves (a) with
;~ i!.~in:e If = o. resulting in complete constructive interference, Solution
~ ~ ~ d:fference If = Tr. resulting in complete destructive According to Eq. (14.16), the amplitude of the resultant
~~.
wave is Arcsuhani = 2A cos(<p/2). So

6.8 cm = 2(12.8 cm)cos(ip/2)


~ When the pha~e difTcrcnce bct,,ecn t \ \ O har- cos(fl'/2) = 6.8 cm/25.6 cm
' ~ Y.a~ e\ is an e\ en i nteger (multip lec;) of rr. the inter-
.c-e--.cc . = 2.65 x 10- 1
'YJ . • L~ tun<-trut ti\c. W hen the pha c difTerence is an
••1 rr~tt of 77. the interference is di: tructi' e. \\'hen the <p = 2 cos- 1(0.265)
" · < c!1ff
e· ' ererxe ·1<; Oc:l\i. cen the<.<: t" 0 ca-.e'>. the ·inte11t.'f-
...r
ip = := 149° = :=2.60 rad.
~ " r.cr:'--- r · d
' ~ ~.d ..i q •Ull) conc,tructi\e nor full) d1..-structl\ e. an
to be intermcdratc.

CHAPTER 14 I WAVES 391

Scanned with CamScanner


reflected pulse and the incident pulse int .
The phase difference may be reported as wavelength, . ely making an 11ntinode. We will d' crfcrc ci
t IV ' • I Scu~ •n\ti
which represents the path difference between two antinodes in Section 14.7. ~ 111'<lti.'<\.
waves, so: ~
t!HHD T he reflected wave on a fi xed c:nu i, i
'I' = 'I' rad X ( wav;~ngth) do es not change ~h a pc . T he rc:fl cctc:d wav
.
""en••
c on I 'II ~,
is not inverted nor docs 11 cha nge !>hapc. a Uc>;et>'
'I' = :t 2.60 rad(
wavelength)
2
7T = :!: 0.41 wavelength. Let's consider another bo undary ch
. I . ange ~·
"
Let's repeat our s1mp e experiment, but th· . itu~ 11 ,
. .h . IS lJJn r.,
Two answers, in rad and wavelength, indicate that replace the Slinky wit two strings of diffierent dc "c.1
the same resultant waves can be obtained by letting one attached to each ot her at one end (a light st . tn1111
. I rin,, a t
wave either travel ahead or travel behind the other one by
2.60 rad, or by 0.41 wavelengths.
to a heavy strmg). A pu se produced in one 0 1liitfltl /6
move along wit · h constan t speed until· it rea h thtrn ~ii·
. Cedthe .
nection point of the two strings, or the hound ei~,
Part of the wave will be transmitted and marl P~io 1
ove Iv 1
second string, and part o f t he wave will be fl u(
14.5: Reflection . . h b
the connection point (t e oundary) and ret . ~
re ected
. . . f h . .d urn in tr
When a travelling wave reaches a point where the opposite direction. o .td e mc1 ent ':ave. Jn an ideal ca-'c
medium changes, part of the wave will reflect and part the energy of the mc1 ent wave will be divided be it
. d 111~
of it will trans mit or be absorbed. How much is reflected the reflected and t he transm1tte waves. Thus th ··
. d • e ampl·
or transmitted depends on the nature of the boundary tude of reflected and transm1tte waves will be 1 ··
. .d es~ tri<:,
between the two media. Let 's repeat our simple experi- the amplitude of the inc1 ent wave. If the pulse m .
. h h . 01 111;
ment with the long, stretched Slinky (Fig. 14.2), and along the lighter string reac est e intersection of th •
. . b . ehLt
make a pulse on it by moving the hand holding one end strings where the heav1er strmg egms, the reflected a:-
of the Slinky up then back down to its initial position, will invert and move back on the lighter string, as iii~
as presented in Fig. 14.6 . The pulse moves along the trated in Fig. 14.31 . If the pulse moving along the heaii
Slinky with constant speed and reaches the other end, string reaches the intersection of the two strings .,. h~
which is attached to a support, and reflects back from the lighter string begins, the reflected part will move ~
the support due to the change of medium . How does this on the heavy string w ithout inverting, as i ll ustrated ~
reflection happen? As Fig. 14.30 illustrates, it depends Fig. 14.32.
on whether the Slinky is tightly attached to the support
(a fixed end) or loosely attached to the support (a loose (a) Incident pulses
end). The reflected pulse from the fixed end will be
inverted and will move in the direction opposite to the
motion of incident pulse [Fig. I 4.30(a)]. The reflected
wave and the incident wave interfere destructively and
~~~~~~~-~-~~~~-------------
cancel each other at the support, making a node at the
(b) Transmitted pu!\~
support. Conversely, a reflected pulse at a loose end will - - --+
not be inverted, but will move in the direction opposite
to the direction of the incident pulse [Fig. 14.30(b)]. The

·----
Reflected pulses

.A.-_--+------!/
(a) .. /\-_-...__ (b)
Figure 14.31 (a) A pulse moving along a light string attached ~
1

__
heavier string. (b) The reflected wave on the light string is inverted.
---+A ---+ j\
-----~
/ (a) Incident pulses
----·
\) .A.--- .---!\ --------....J,A..~--------------
.--- (b) Reflected pulses Transmitted pulstS
I ·---- ----·
~ Il l
Figure 14.30 The reflection of a pul~e due to a change of medium: (a) ~gure 14.32 (a) A pulse moves along a heavy strin~ a ~
reflection at a fixed end, and (b) reflection at a loose end. fighter string. (b) The reflected wave on the heavy string 15 not

PART THREE I ELASTICITY ANO VIBRATIONS, WAVES, ANO SOUND


392

Scanned with CamScanner


- When a wave •.ravels fr~m a light (low density)
iul11 to a heavy (111gh density) medium. the wave
rried b d . .
o~-cted at the oun :iry JS lll~'e rted. Conversely, if the
rt ·e travd s from a heavy to a light medium the reflected
11'3'
. at the bound:iry does not mvert.
. •
1111' e

Since the speed o~ the wave on the string decreases


as t linear masshdensity
he . of the
. string increases , thew ave
irarels faster on t e 11g1lier string.

~ When the speed of a wave go in g from one


medium to another decreases, the reflected wave will
be inverted. When ~he speed of a wave going from one
medium to another mcrcascs, the reflected wave will not
be inverted.
Figure 14.33 The standing waves on an ocean shore.

14.6: Standing Waves


Astanding wave is a wave pattern that is fixed in space y(x, t) = y 1(x, t) + y 2(x, t)
and does not move. Standing waves are produced in a
medium either because the medium is moving in the = A sin(kx + wt) + A sin(kx - wt) .
opposite direction with respect to the wave, or because Using the trigonometric relationship:
two identical waves, with the same wavelength and the
same amplitude, moving in opposite directions are inter- sin(a :± b) = sinacosb :± cosasinb.
fering with one another in a stationary medium. The best
way to have identical waves is to have reflected waves of The resultant wave function is given by:
incident waves from one end of a bounded medium. You
can easily generate a standing wave by tying a string to y (x, t) = 2A sin(kx)cos(wt). (14.17)
a post, stretching it out, and then shaking the loose end.
The incident waves that you make by shaking the loose Eq. (14.17] represents the wave function of a
end interferes with the waves reflected at fixed end and standing wave. The interference of two similar waves
make a standing wave. It is a wave pattern that has some moving in opposite directions is not a travelling wave,
specific points along the string that is standing still. The but a pattern that oscillates by varying its amplitude, as
standing wave pattern is only produced in a medium at illustrated in Fig. 14.34. The absolute value of the quan-
tity 2A sin(kx) represents the amplitude of the oscillation
specific frequencies.
of a point in the string positioned at the location x. Thus,
Standing waves on a rope or string occur when the
the amplitude of oscillation varies with position. Each
frequency of the driving force-shaking one end of the
point located at x oscillates with angular frequency w,
rope- is equal to one of the harmonic frequencies (the
and the amplitude varies with position in a sinusoidal
~atural patterns of oscillation of the string, to be discussed
1 ~ Section 14.6.1). This is an example of resonance, as
manner.
d~scussed in Chapter 13. Standing waves can occur in mU@I When two sinusoidal waves, having the same
vibrating systems, such as buildings, bridges, and ocean wavelength and amplitude, move in opposite directions
shore sand (see Fig. 14.33 ). along a Slinky (or stretched spring), they interfere with
Consider two similar sinusoidal waves (having the each other and make a standing wave.
~e wavelength and the same amplitude) with no phase
~fference, moving in opposite directions along a Slinky. Note that the amplitude of the individual wave is A
e wave functions of these two travelling waves can be and the amplitude of the simple harmonic motion of the
represented by: particle is 2A sin(kx), which indicates that the particle
is oscillating within the envelope described by function
y 1(x, t) = A sin(kx - wt) 2A sin(kx) (see Fig. 14.35).
The amplitude of a standing wave is zero at the points
and
where it is given by:

y 2(x, t) = A sin(kx + wt). sin(kx) =0 => sin(kx) = sin(nrr)


lheth resulta nt wave function of these two waves ts
as . given
.
e algebraic sum:
kx = nrr forn = 0, 1, 2, ....

CHAPTER 14 I WAVES 393

Scanned with CamScanner


.~A.- >
___..____, _. ~~·

,,. t
r
, ,. -12 '[ 1
' .. l l I· Yi I

fl~~m• ·~~ h1t1;1lt,.'fllr~ {If two C'li)(X.ltvtv ouvcte(f klent1ca1harrr1rN11r. wavefi movino in me ~amr. mr.rlium (ii) t/JfJ ll(itlli"!lre "~'~ ;sr Ti4 ::.~r. r.rr::-
it>l n ~1,.'l lt:it..~l\'V 1,11w-'vC!t w1tll 1~ ~>c t to ttn1i: a t T/4 t1111e il11t:rval~

By 1>ub<itiruring J. = 2 rrl A in the aho-, e equa!mr~ tt<! ~.r-


• rangi ng it, we find ;
f
./.

l"
t.
! Eq. (14 .20) gives the ~ition of m.u.i!::::r~-:: a.-:;f~-
~ tude, the antlnode:

.........., ti
Flgurt 14.~ SUnd:ng waves on a string
Fig. 14.36 shows a standing .,.,a~e on a ~.::r _...;.
Sit 1.'I.~ k :c ::: rr 1 A. "ha c A is the wa\'ckngth of the travel- nodes and antinodes. TY. o adjacent antino<.!e-; a:·:· ,._~­
tin~ " l.\H '. sub~tituting it in the above l'quation and rcar- rated by half a Y.a\elength. A/2. Each :i:uir:oO: 15 r..:-.~r: ·:
ran~in~ it ~i ' l'S: halfway bet\\een mo nodes. and e:ich noee li t.:...~
between two antinodes. Thus, the nod-'"5 ~~ a-":""C1!.e
.\" = n-2A fior11 "" 0. I ' ....., ( 14. 18) are periodically repeated along the string..

Eq, [ 1-l. l SJ ~in· th1.· position of 1cro amplitude, the node: rnfoi 11 T'\ o adjacent antino<l!"5 a:e se;.ap:!.T-..0 ~ 1-:<• 1
wavelength . T\\O adjacent node-s :ire ~p:l!:l:..."'- ~ 1 1
' :

wavelength. An antinode and an adjacet:t r:o.J.~ .!...~ ~~


(14.19) rated by a quarter of a \l.a\elength.

The J i. tJnc1.· bet" ccn two adjacent nodes, Ax, is given by:
L lf2
.ix :: (11 + I)~ - n~ == ~ Y r -{ !·.u.; ••
2 2 2· Anrinode Anrinodc ~..o.!.e
This is equ:il to half a wan:kngth.
____ l _____ 1
The: amplitude of a st:mding wave is the maximum
\'aluc. :!y. at the position "here:
-A
sin(kx) c :!: I node node

k.:c = (11 + ~ )rr forn == 0, I. 2, . . ..


r A.f2

Agure 14.36 A stanomg wave on a stmg wi:ll r.cCes ~ ~¢

394 PART THREE I ELASTICITY ANO VIBRATIONS. WAVES. ANO SOUND

Scanned with CamScanner


.
'
hnd tho speed of the wave on the string. Using Eq. [14 .6),
gives:

Q.ON 111
"" 40. 0 -- .
k·g s
0.0056 -
111

Using Eq. [14.1], gives the wavelength:

m
40.0 -
A= ~
5
=- - = 0.36 m.
.l = : (:'cm) = IO cm. f 11 0 llz

Space between nodes:

~ = 0.36 m = 0.18 m.
fXAMPLE 14.18 2 2
~ ~ a' a \liave on a stre!ched string is 2.0 mis. Solution to part (b):
s.,.~-g \\-:l'.es a-e se~ l.'? on the string y,,ith nodes 4.0 cm
~- ·.~.a! is the frequency of the waves? How many Y~1a., = 2A = 2(0.003 m) = 0.006 m = 6.0 mm.
:~ pet s...-"COnd coes the string have its unstretched
~.:rd~ a stra"'ghHne fonn?

So.ution
r:-: noO:s ~ is ec;i.Jaj to haif a wave!ength, thus:
" = :! x (-rn cr::i) = .ocm. A standing wave on a long string has antinodes at points
!lsr.g 5:;. (1 ~. 1 }: 4 cm. 8 cm, 12 cm, and 16 cm. What are the wavelengths
of the travelling waves that produce this standing wave?
\" \"
>. = - =>- I = - (a) 0.78 cm
I A
?:l (b) 2 cm
2-
s (c) 4 cm
/ =- ---
.0 x io-: r:l
= !5Hz.
(d) 8 cm
cr.e ~.e c fde from one antinode to the same anti- (e) 12.6 cm
~ t;e string •tfil be stra.'gtrt twice; therefore, with a
r~J' a 25 Hz, tr..e s!ring vti.:l pass through a straight-
1-e ::r.n 2 x 25 =SO times per second.
14.6.1: Resonances
Consider a string of length l, where both ends are
fixed, such as a guitar string stretched between two
clamps. By plucking the string at some point. you
EXAMPLE 14.19 produce a standing wave on the string. The wave is
:.. "'?J kr.g ~Ji."19 has a Enear density of 5.6 gfm and_is reflected at both ends and the reflected waves inter-
~....:.ed •1.-:::i a t~ af 9.0 N. Waves with frequencies fere with each other. At each end. the wave will have
i t) P.z a.--ct a:r~'!Udes of 3.0 mm are generated at the zero amplitude; thus, the two fixed ends arc always
e-;~cf ~.e s-Jir.g. (a) \'mat es the node spacing along the nodes. Since there is an antinodc between two nodes,
r?:C-...:IJ s-.a-l(fr .g wa-1e? (b) Y.'ha t is the maximum d is- the standing wave on the two fixed ends may have I,
~~ cl &.e st'ir.g? 2, 3, ... antinodes (see Fig. 14.37). Note that since
St!ution the string is fi xed at both ends, the wave must have
a frequency that permits nodes at each fixed end;
~~to ;:ia-r (a): The nodes' spacing is equal to half thus. only certain frequencies can make interferences.
a ""?.YEle--~ thus, to ca!cu'.ate wave!ength we have to Since the two nodes are separated by A/2, due to the
boundary condition, only those waves with an exact
integer of half-wavelengths can fit on a string of

CHAPTER 14 I WAVES 39 ~

Scanned with CamScanner


UNITS
Solution • Frequency f' s- 1 .. llz
Since the string resonates at certain frequencies deter-
• Wave number k: rad/m
mined by the length L of the string and the wave speed v,
using Eq. [14.24]: • Angular frequency w: radls

f. = -11 = -II
" A,. 2l
Jf- µ.
forn = I, 2, 3. LAWS
• Speed of wave on a string: l'wavc = (T/µ.) 112• where T ·
. · I~ the
tension andµ, 1s the lmcar mass d enslly
· (
mass per length)
As Fig. 14.38 shows, T= mg, so:

,,~rg
f.n = -2l -µ. forn = 1,2,3, ....
MULTIPLE-CHOICE QUESTIONS
For the fourth harmonic 11 = 4: MC-14.1. Waves are typically characterized by frequency' angu.
lar frequency, period, wavelength, and amplitude.
Which of these pairs of parameters are related to each
J; = ~ jmg = ~ r;;;g other in a linear fashion? Note: More than one ans11 er
4 2l\jµ_- l\jµ_-
may apply.
4L2f~µ. (a) period and frequency
m=---
42g (b) period and angular frequency
(c) frequency and angular frequency
(d) wavelength and period
(e) amplitude and frequency
MC-14.2. The shorter the frequency of the wave
Substituting l = 1.6 m, h =!vibrator = 180 Hz, and µ. =2.4 g/m (a) the smaller its amplitude.
in above equation for m yields: (b) the shorter its period.
(c) the smaller its speed.
(d) the longer its wavelength
(e) the greater its amplitude.
MC-14.3. The particles of the medium in a transverse 11a1·e
move
m = 5.08kg. (a) with waves.
(b) in ellipses.
(c) in circles.
(d) parallel to the direction of the wave tm •' •
SUMMARY (e) perpendicular to the direction of the wai f " m l.
MC-14.4. An object hit by a wave makes 20 complete l'i;-,. ;ons
DEFINITIONS in I 0 seconds. What is its period?
(a) 2 Hz
• Speed of wave: "wave= A/, where A is wavelength and/is
(b) I 0 s
frequency
(c) 0.50 Hz
• Wave function: y(x, t) = f(x ± vt) (d) 2 s
(e) 0.50 s
• Hannonic wave function: y(x, t) = Asin(h ±wt), where k =
MC-14.5. The distance between a crest of a sinusoidal water
2rr/A is the wave number and w = 2rr/T = 2rr /is the angular
frequency wave and the next trough is 2 m. If the frequency of
the water wave is 2 Hz, what is its speed?
• Superposition principle: For two overlapping waves, the (a) 8 m/s
resultant displacement at any point is the algebraic sum (b) 4 mis
of the displacement of the individual waves. yR(x, t) = (c) 2 mis
y 1(x, t) ± y 2(x, t), where YR is the resultant displacement (d) I mis
(wave function). (e) Not enough information is given to determine the
• Standing waves for two similar waves moving in opposite wave speed. h
MC-14.6. If the time interval between two successive troug 5
directions: y(x, t) = 2A sin(h)cos(wt) . wave past an observer .ts o·4 sec.
o f a trave Ihng
• Position of nodes: xnodes = n A/2 for n = 0, I, 2, 3, .. . then
(a) its wavelength is 2.5 m.
• Position of antinodes: xantinodes = (n + Yl)A/2 for n = 0, I,
(b) its frequency is 5 Hz.
2, 3, ...
(c) its wavelength is 5 m.
• Harmonics for a string of length L fixed at ends: /,. = (d) its frequency is 2.5 Hz
n(v!U) = nJ; (e) its velocity of propagation is 2.5 m/s.

398 PART THREE I ELASTICITY AND VIBRATIONS, WAVES, AND SOUND

Scanned with CamScanner


The fundamental fre~uency of a string with fixed
Q-14.6• The graph in Fig. 14.48 shows displacement as a
...
,..Ac.2'-en ds is 120 llz.. Which of these frequencies is its function of position for a particular wave at I = 0. ~f
higher harmontc7 . Ie a t x -- 0 IS·
the instantaneous velocity of the part1c
(a) 120 Hz positive at this time, in which direction does the wave
(b) t60 Hz
propagate?
(c) 240 Hz
(d) 320 Hz
(e) 440 H~
A string with fixed ends has a fundamental frequency
.-r1• 25· of J35 Hz. What is the frequency of the harmonic
shown in Fig. 14.46? 4
(a) 270 llz
{b) -162 Hz
(c) 240 Hz
(d) 540 llz
(e) 405 Hz

Position

Figure 14.48 Displacement as a function of position for a wave.

ft9!re 1U6

Q-14.7. What type of wave do people make when they are


CONCEPTUAL QUESTIONS doing the wave in a stadium, and how? Can they
change the type of wave they produce by changing
~14.1. We frequently model tendons as massless strings.
their body's motion? How?
Why is this not a useful model when describing waves
Q-14.8. How do you change a string frequency in your violin
on strings? when you tune it?
~14.2. What is the difference between a transverse wave and Q-14.9. As Fig. 14.34 shows that there are times when a string
a longitudinal wave? Show it by using an example of in its fundamental mode becomes completely flat as
each. it oscillates. What happens to its energy of oscillation
~14.l What kind of information can one deduce from the at these times? Does the energy of oscillation become
amplitude of a wave? zero?
~1U. A runner lost his control and tripped behind the last Q-14.10. Imagine a long nail that is embedded halfway into a
person of a long row of people lined up single file. board. Where should you hit on the nail with a ham-
What kind of wave would the push of the runner gen- mer to make (a) a transverse wave and (b) a longitu-
erate in the people lined up? dinal wave on the board?
G-14.5. Fig. 14.47 shows a wave on a long stretched cord, Q-14.11. Assume the tension in a string remains the same.
moving to the right. When it moves a little to the but you double the frequency. What effect does this
right, which points- a, b, c, d, ore- move downward have on the wavelength of a wave moving along it?
and which points move upward? Explain.

y ANALYTICAL PROBLEMS
P-14.1. A wave with frequency 5.0 Hz and amplitude 40 mm
moves in the positive x-direction with speed 6.5 mis.
~r----~~-----il{-----,.,. x Whal are (a) the wavelength, (b) the period, and (c)
the angular frequency? (d) Write a formula for the
wave.

~14A7 Aharmonic wave on a long stretched cord.

CHAPTER 14 I WAVES 401

Scanned with CamScanner


, lses on a string travel toward each Oth
P-14.9. Two waved puf. I mis as illustrated in Fig. 14.51 Whc:r
25 llz wave travelling in the with spec s o • · at
P-14.2. Fig. 14.49 sIiows a . b . wave- will be the shape of the string at I = I and at t "" 1.5 s
. t' Calculate (a) its amplitude, ( ) its
x-d1rcc 1011. • d u cI =
length, (c) its period. and (d) its wave spec . s 'I afler the instant shown?
18 cm and L2 = I 0 cm.

y (tn)
1

0.5 + - -

3 4 5 x (rn)
0 0 2

-0.5

Figure 14.51

P-14.10. lfthe tension of a guitar string with 0.005 kg/m linear


mass density is 150 N, what is the speed of waves on
the string?
P-14.11. Two children stretch a 3 m jump rope of mass 0.5 kg
Figure 14.49 by exerting 40 N force. If the children make wave
pulses moving along the rope back and forth, what is
the speed of the wave on the rope?
P-14.3. What is the frequency of a wave with a wavelength
of 0.6 m moving with speed 210 mis? P-14.12. A transverse wave on a string is given by y = 4.5 sin
An FM radio station broadcasts at 88 MHz. Detennine
(28x + 321), where x and y are in centimetres and 1
P-14.4.
(a) the period and (b) the wavelength of the radio is in seconds. What are the amplitude, wavelenr.nh
waves. The speed of wave is 3 x 108 mis. frequency, period, and speed of the wave? ~ '
P-14.5. What is the frequency of a 2 m wave that a TV station P-14.13. Does the expression y = 5.6 sin{21T[(x/8) + (t/5)Jl,
broadcasts? The speed of the wave is 3 x 108 mis. where x and y are in centimetres and t is in sec0ds,
P-14.6. The tsunami of December 26, 2004, initiated by an represent a wave? Sketchy as a function ofx for , -, o,
earthquake of magnitude 9. I , travelled from the west- I, and 2 s. What are the amplitude, wavelength f~e­
ern coast of the island Sumatra to the coast of Africa quency, period, and speed of this wave?
in just over 7 hours. If its speed in deep water with P-14.14. A transverse sinusoidal wave of wavelength 3 .•. m
a 7 km depth is 945 km/h, and its wavelength at that and amplitude of 1.4 m is travelling to the left \\ .:!a
depth is 300 km, what is the frequency of the wave? speed of 16.4 mis. At t = 0 andx = 0, its displacL·;'-:nt
What is its frequency at a depth of 200 m if its veloc- is 0.54 m. Write an expression for its wave fun'. "'' n,
ity is 160 km/h and its wavelength is 50 km? y(x, t).
P-14.7. The sp~ed of a wave in shallow water of depth d is P-14.15. A transverse harmonic wave with a wavelen;;;', of
a~prox1mated by vshallow = '\/id, and in deep water 65.0 cm, frequency of 8.00 Hz, and amplitud: 0f
with a wavelength A is approximated by v =
34.0 c~ is travelling to the left (in the -x-direct•• 11).
~-(~)What are the speed and the freq~;~cy If the displacement of the wave at t = 0 and x -- :) is
of a wave with A = 1.5 cm at a depth of d = 3.5 cm in
y~-().OO cm and a particle of the medium at the ~-~ me
water? ~b) What are the speed and the frequency of
wave with A = 5.4 m in deep water? poi~t has positive velocity, (a) find the wave sp.-:cd.
P-14.8. !wo waves travel toward each other on a string as period, angular wave number, and angular frequencv.
illustrated in Fig. 14.50, each with a speed of2 ml (b) Write an expression for its wave function, r(.ui.
P-14.16. A transverse wav . ts. given
. · 1) :=
Sketch the string at t = 1 1.25 and 1 5 ft s. . e on a string by y(x,
instant shown. ' ' · s a er the < ~m)sm[(1T/l2 cm)x - (1T/18 s)t], where x andy
25
~re m c~ntimetres and t is in seconds. (a) What is
y(m) its amplitude? (b) Wh . .
. . · . at ts its wavelength? (c) What
2 1s its frequ ?( · .
.t ency· d) What is its period? (e) What is
1 s speed? (f) I h" .
· n w tch direction does it move? (!!.}
Sketch this "' . -
t =0 a _wave irom x = 0 to x = 15 cm at mnes
/
. nd - 4.0 s, separately.
P-14.17. Wnte an expre . fi . .
· ssion or a smuso1dal wave propagat-
mg a1ong a ro . h
pe m t e - x-direction The wave has a
waveIength f 90 · d
amplitud f~ .O cm, frequency of 5.00 Hz. an
t = o de _ -~cm. The displacement of the wa,·e at
0
Figure 14.50 an x - 0 1sy- o w · · r. the
above wa "f - · nte the expression 1or
1 the d· I
and x = ve . tsp acement of the wave at 1-- 0
o~ 10.O cm isy = o.
PART THREE I ELASTICITY ANO VIBRATION
· S, WAV~"- /\l.IP"t. -~ .

Scanned with CamScanner


Concept Question 14.7: (c). Using Eq. j 14.5] gives:
ANSWERS TO CONCEPT QUESTIONS
6.x 7m - 2.5m m
Concept Question• 14.1: An 1mpu· ts1ve
· · sound • such .as \' = - = = 2.3 - .
6.t 3s - Is s
a gunshot or cannon fire, can cause an air· Pressure. .d1s-.
turbance (or an air density disturbance) that propa.gates Concept Question 14.8: (d). Since the string is .
through air. When it impacts a loose ledge of snow, it can . d . un1fonn
and has a unique 1inear cns1ty µ = mil, its line .
cause the snow to break free and produce an avalanche. " . . . ar den
·ity remains the same a11er cutting 11 in half So th ··
It happened during WWI in the Alps when a cannon fire s .r ' h . · esPeell
remains the same as be1ore, since t e tension also r .
caused an avalanche. erna1ns
the same.
Concept Question 14.2: (e) The disturbance is not par-
Concept Question 14.9: (a)
allel to the direction of propagation in the transverse wave.
Concept Question 14. l 0: ( d)
Concept Question 14.3: (c). The frequency of the wave
is specified by the source, not by the medium, so fre- Concept Question 14.ll: (d)
quency stays unchanged. The wavelength will change by
the speed of the wave. As Eq. [ 14. I] shows, the wave- Concept Question 14.12: (e). Look at Fig. 14.17 an~
length increases as the speed increases. similar to it, sketch two more waves beside the gi\'en waie
(Fig. 14.22), illustrating the wave as it moves tO\\ lfd the
Concept Question 14.4: (b) and (d). Based on Eq. [14.4],
right, and identify how the assigned points are morinu in
they are not in the fonn of y(r,t) =Ar::;:: vt), so they do
the vertical direction (oscillating) upward or do\\ nw~d.
not represent wave functions.
Concept Question 14.13: (b)
Concept Question 14.5: (c). It is in the form of
y(x, t) = J(x - vt). Concept Question 14.14: (d)
Concept Question 14.6: (c). First the shallow side of Concept Question 14.15: (e)
the pulse moves through P. then the horizontal side, and
finally the steep side. So, with respect to time, the dis-
placement increases more slowly than it decreases. Note
that (b) and (d) represent physically impossible situations:
in both cases, there is more than one possible value of
displacement at a given instant.

404 PART THREE I ELASTICITY ANO VIBRATIONS W


• AVes. At.ii\~- JI

Scanned with CamScanner


·CHA·PTER 15·

Sound

ound, like light, is part of our everyday sensory

S
soft tissue in humans and animals. We can now use sound
experience. Indeed, sound is the basis for commu- wave pulses to break up the stones inside kidneys and
nication and for gathering vital information. Sound gallbladders, removing them safely without the need for
does not exist in a vacuum; it requires a medium. Humans more dangerous and expensive surgery.
typically experience sound that has travelled through air, Animals use sound waves to detect and locate
but sound also travels in solid and liquid media. Sound objects, and to exchange information. Bats and porpoises
travels through its medium in the same way that waves use sound to navigate and to locate food. Dolphins and
move on the surface of water (see Chapter 14). whales use sound to navigate, communicate, and hunt.
Sound is a longitudinal mechanical wave that is the In this chapter, we will focus on sound waves: how
propagation of a disturbance of pressure and density, or they are produced and how they travel in a medium, and
position of small elements, in a medium such as air. The the processes and limitations of perceiving sound.
disturbance can be vibrations or harmonic oscillations
produced by a vibrating source, such as a person's vocal
cord, the vibrating diaphragm of a stereo speaker, and the 15.1: Sound Waves
vibrating string of a violin. The oscillatory motion of the
element of the medium, such as air, represents the motion Sound is mechanical waves propagating through any
of the total energy in the sound waves. This energy travels medium: solid, liquid, or gas. The wave is a disturbance or
through the medium at the speed of sound from one vibration in the pressure or density of a medium. You can
element to the next in all directions. The product of energy produce a disturbance by clapping your hands, knocking
and speed defines sound intensity, which attenuates with on wood with your pen or fingers, clicking two stones
the inverse square of the distance from the sound source. together, vibrating speaker cones, or suddenly stopping
In confined media, such as tubes, harmonic sound your car. These disturbances propagate through air and
waves reflected off the closed or open end of the tube are reach your ear. Sound is a longitudinal wave in all media,
superimposed on themselves, forming standing waves. but it may also have a transverse component in solids. Let
Such standing waves are called harmonics. Harmonics us concentrate here on sound propagating in air.
result from an external excitation that couples into the A simple way to demonstrate sound is with a vibrating
c.onfined system via resonance. An example is the vibra- tuning fork (Fig. 15. 1). A tuning fork is a U-shaped metal
~ton of the vocal cords to generate the human voice. The object with two tines that vibrate when struck. The fork
inverse process causes the vibrations of the eardrum when vibrates with an antinode at the end of each tine, so
sound enters the outer ear. it vibrates at its fundamental frequency. The back and
In addition to producing sound with our vocal cords forth vibrations of the tines disturb the surrounding air
and pe . . . molecules, and the vibrations are then passed to adjacent
. rceivmg it with our ears we use sound waves m
d1fferent ' sound to exp1ore and molecules. These disturbances produce alternating high-
ways every day. We use
Probe Earth's crust for oil to detect underwater o bstac1es pressure regions, or compressions, and low-pressure
(sonar) t · ' h regions, or rarefactions, which together are called sound
' 0 navigate or detect submarines, and explore t e

llEL
CHAPTER 15 I SOUND 405

Scanned with CamScanner


v
Vibrating
tuning fork

o o
.. I o
.
'
I
I
I

: Raref~ction!
0

..
I
o
: Rarefaction : 0 I

pt : : Compression: o
I
I
Pressure J______ _ 0 I
I
o
I

~
I I

.
I
I

1 arm - - - - - - - - - - -

Figure 15.1 A sound wave representation produced by a tuning fork. Density J______ _
The red curve shows the air pressure in the direction of the moving
disturbance: the sound wave.
Figure 15.3 A sound wave produces the same._ in-phase, wavelike lis-
turbance in the pressure and the density of the air.
waves. Note that since the air molecules are compressed
together in the compression regions, the pressure there is
; : .: ; . .
higher than normal air pressure. Pressure is lower than : Compression : Rarefacrio~
normal air pressure in rarefaction regions. Thus, a sound
wave can be shown by a pressure-like wave- a series
Pressuret-__
of pressure hills and dales. Compressed and expanded
regions (compression and rarefaction regions) in air
produced by a tuning fork and reaching an ear (or loud-
speaker) are similar to the longitudinal wave produced on
a Slinky (Fig. 15.2).

Figure 15.4 A sound wave produces similar but out-of-phase wavelle


disturbance in the pressure and in density of the air.

Ear drum
15.2: Vibration of a Gas Confined
by a Piston
As discussed in Chapter 14, a mechanical wave can be
produced by harmonic vibration of a mechanical system.
.,_._ --+ like a Slinky. These vibrations are produced by the
figure 15.2 Both the sound wave and the wave on the Slinky are lon- system (Slinky), which obeys Hooke's law (Chapter m
gitudinal waves. with a linear restoring force. Now, we will use the ~m:
model for a gas in a closed container confined ~e
piston. We demonstrate that a restoring force acts 0~ .
0
Sound waves can be represented by variation in piston if it is moved away from its equilibrium pos~o~
air pressure, air density, or by displacement of an air by an external force, and that the restoring force l 0
element. The variation in air density and air pressure Hooke's law. din'
are similar, so they produce similar wave-like distur- ~ig. 15 ..5 illustrates an ideal ga~ th~t is confi~eglecl
bances when they are plotted versus position. Both dis- container with a movable piston at its nght ~nd. th the
1
turbances are in-phase: crests line up with crests, and gravitational effects on gas or piston. Constd~r ~al 1o
troughs line up with troughs (Fig. 15.3). Thus, as sound pressure of the ideal gas inside the container 15 eq jston
propagates through air, the region. in air. with high pres- the atmospheric pressure outside. Therefore, the(fop of
sure has high density, and the region with low pressure
is at its mechanical equilibrium positioned at Xeq
has low density. Fig. 15.5). A( tO
As sound waves pass through air, those regions with . t nee ;> ,
We displace the piston by a small d ts a f'' 15.5)
the highest change in pressure (regions of compression
a new position at x = x - A.x (bottom of igA 0e1r
or rarefaction) have no displacement change, and those · an external force · de F r
bY exerting eq
of magmtu ,esfjrst la11'•
regions with no change in pressure have the highest dis-
mechanical equilibrium governed by Newton. s ositioD·
placement change (Fig. 15.4). Thus, these two waves (the
is established when we' hold the piston at thISd~tertlline
pressure wave and the displacement wave) are out of phase.
The mechanical equilibrium then allows us to

406 PART THREE I ELASTICITY ANO VIBRATIONS, WAVES, ANO SOUND

Scanned with CamScanner


the piston is set into motion toward the right. At
. 'JI . I ::> 0
nRT (I 5.4) piston will undergo a harmonic ~sci at1on, as dtSc . lt-t
Bidc:ilgas =V =Peq· in Section 14.2. Our focus here 1s on the air in l'. U\~
eq 1r0nt
the piston. or
Thus, Eq. (15.4) gives the bulk modulus for a~ ideal.~as The four lower frames in Fig. 15.6 are equi\ 1
under isothermal conditions. which is equal to its equilib- he last four frames in Fig. I4.5 for the forrnat·a ent It
t . IOn or
rium pressure. wave on a Slinky. When the piston reaches its arn . i
toward the right at I = T/4, a locally compressed Phtudt
I luJ•1!11i A small displacement of a piston sealing an ideal of air is created in front of the piston. The gas lllo!Poc~ei
isothermal gas in a container is a valid case of Hooke's · d ecu1
·n this pocket of increase d dens1ty an pressure ha ~
l .. . ie al
law. een given an additional velocity component
b . . to\\ ardthIQ
right. When the piston moves back to Its equilib _ '
. - .,.'/2) h nuni
position (reached at time I - ' ' ' t e gas lllolecu!
CASE STUDY 15.1 in front of the piston are pulled back toward the 1 ei
. d . et
causing a pocket of decrease d air ensity and pres·
. f h . _ ~Ure
Assume that the external force Fcxt is suddenly removed indicated by the low dens1ty o t e vertical lines_ ·
in the lower panel of Fig. 15.5. Describe qualitatively what By moving beyond the equilibriu~ position to the
happenS next. amplitude point at the left (reached at time t = 3Ti-l). the
Answer The system discussed in Fig. 15.5 is in mechan- piston increases the volume of the gas pocket directli -
ical equilibrium, since the external force is balanced by an front of it at the right. Thus. the volume with decreased Ill
equal but opposite force exerted by the gas on the piston. density (pressure) increases further. In the meantime. ;
If we remove the external force by releasing the piston, a initially created zone of increased air density (pressure~
mechanical non-equilibrium situation is created-a case
governed by Newton's second law. The unbalanced force
acting on the piston is the force exerted by the gas on the

~rn !~
piston. This is a restoring force since it points in the direction
of the equilibrium position of the piston. 0
The unbalanced restoring force initially accelerates the t. 111111111111111 1
piston toward the right in Fig. 15.5. As the piston moves, I
I
it reaches the equilibrium position. At that instant, no
force acts on the piston; that is, it no longer ~ccelerates.
However, the inertia of the piston prevents it from sud-
denly coming to rest. Thus, the piston moves further to the
right. Once the piston has moved beyond the equilibrium
position, the confined ideal gas has a pressure lower than
, • T/4 H
I
I
11111111111111111 w-1 -

~ 111 ~111111111111 1 !
the external atmospheric pressure. Therefore, a restoring
force acts on the piston, pulling it back toward the equi-
librium position; that is, a force acts toward the left. t • Tll + - l
This force slows the piston down to rest, which occurs I

~~ ~
at xcq + .:h- if the piston is moving without friction in the
cylindrical container. Thereafter, the piston continues to
move back and forth.
t • JT/
4

I
1111111ii11111111 1! j
15.3: Longitudinal Waves in a Gas
Sound waves arc longitudinal waves, as opposed to
transverse waves. We can demonstrate sound waves using
t- T ~ I IW 11111111111 11 ~
a tube containing a gas. like air, with a vibrating piston at
one end. Like we did for a transverse wave in the Slinky
in Section 14.1, let us look at the sound wave produced in Figure 15.6 Ave snapshots Illustrating the development of an 3~
the tube by vibrating the piston back and forth (Fig. I5.6). wave In a gas-filled container due to the vibrational motion of aP~
In the first frame of Fig. 15.6, at time t = 0, the gas shown about its equilibrium position x . The piston reaches the amplitude p:ti"ll
toward the right at time T/4, ~nd the amplitude point toward tfld k'~
to the right of the piston has a uniform density. The equi-
at _lime 3Tl4. When the piston moves toward the right, tile air ~;..
librium position of the piston is indicated by a vertical
ad1acent to the piston Is compressed (indicated by an Increased dt~
dashed line. at xc_~r w~ gra~hical.ly illustrat~ gas density of .vertical lines). When the piston moves toward the left. tile air p.'\~
as the density or vertical Imes m the gas 111 Fig. 15.6, ad1acent to the piston Is expanded (Indicated by a decreased dt'nsrtl'
which can illustrate gas pressure as well. At time / = o• vertical lines).

408 PART THREE I ELASTICITY AND VIBRATIONS, WAVES, ANO SOUND

Scanned with CamScanner


·ivellcd toward the right and r .
has tr· ema1ns · I
(st1 ·11 illustrated as. a pocket of denser . m1·t ie system
vertical The speed of I . .
tit is the density (pressure) variati . mes). Note given as: a ongitudmal wave in a solid rod is
th;i indiv1dua
. . I mo Iccu . Ics .111 the gas Thons that trave I, not
thc . . · c absol t <l
surc) of the gas 1s very high and ti . u e ensity
(prcs ' HIS llld' ·d
moleculcs encounter . frequent
. .
~c:eP them near their original position.
collis·ions that 1v1 ual. gas
essentially
v .
sound m solid rod -
-
\jJr
p' (15.7)

In tht: last frame of Fi. g•. 15 6


· • we s h
ee t at th
rning toward the right after / = 3 T/4 e piston, in which Yis th y •
tu . . , compresses th . densit of th e _oung s modulus of the solid andp is the
in front of 1t once more while completin a fi . e air wave Y. . e solid. In general, the speed of mechanical
..., ion. Thus, at the final time / = T g ult penod of s is given by:
11• 01 • • ' • a second p k
increased air density and pressure is generated t oc et of
quently trav:ls_tow~rd the right. hat subse- elastic property
V mechanical waves = inertial property ·
What d1st111gu1shes the case in F. 15
· F. 14 5 · h h · ig. .6 from the
case m ig. . is t at t e direction of th . .
the piston and the direction of the pro e vi_bration of TABLE 15.1
. h d. . pagahng sound
wave are m t e same 1rechon, produc·mg 1ong1tud111al . . Speed of sound in various materials
waves.

15.3.1: Speed of Sound


l Material
Gases
Speed of sound
(mis) I Temperature (K) I
The speed of Air
. sound as a mechanical wave dependson 331 273
the properties of . .the medium in which it propagates Air 343 293
(Table 15.1 ), similar to any mechanical wave (as in Air 386 373
Chapter 14). The speed o~ sound depends generally on Liquids
two parameters of the medium: the density and the co _ Water 1400 273
pressibility, an elastic property. Compressibility in sol:s
Water 1490 298
is directional. Liquids and gases are fluids in which com-
Seawater (3.5% 1530 298
pression must occur from all directions. Their compress-
salt)
ibility is governed by the bulk modulus B. The speed of
Solids and soft
sound then follows from the bulk modulus, B, and the
matter
density, p, of the medium, given as:
Steel 5940

vsoundinfluid = .)%. ( 15.5)


Granite
Human body
6000
1540 310
tissue
The speed of sound in a gas can also be derived from
Vulcanized rubber 55
Eq. [15.4]. Note that we calculated the bulk modulus for
an isothennal ideal gas in Eq. [15.4]. We cannot substi-
tute that result in Eq. [15.5} because Eq. [15.4] is derived
with the assumption of a constant temperature ( T = const). CASE STUDY 15.2
For gas expansion to occur isothermally, it must be done
slowly. This does not apply to the fast vibrations of air in a The speed of sound in a gas is limited by the speed of the
travelling sound wave. The sound vibrations of air have to gas particles in the gas. Why then can a solid cany sound
be described by an adiabatic compression (see Chapter 9), even though its atoms or molecules remain at the same
location within the lattice?
since the time during which the compression occurs is
too short to allow heat to exchange with the adjacent air. Answer Gas molecules are not chemically or physically
Using an adiabatic process to calculate the bulk modulus bound to each other, and interact only through collisions.
Thus, a physical change of the parameters for a given gas
of an ideal gas leads to Laplace's equation for the speed
particle can reach other gas particles only when the initial
of sound:
particle has travelled into their vicinity and has undergone
a collision. In a solid, all particles are chemically or physi-
/KP" (15.6) cally bound to each other. When a physical parameter in
vsound in gas = \J p ' an area is changed, neighbouring atoms or molecules are
affected very rapidly due to the strong interactions along
these bonds. Thus, the speed of sound in a solid is usually
in Which K = C/C is the adiabatic coefficient, with Cp
much higher than in a gas.
the rnolar heat cap~city of the gas at constant pressure,
and Cv at constant volume (see Chapter 9).
CHAPTER 15 I SOUND 409

Scanned with CamScanner


EXAMPLE 15.2
. he data for air at 0°C from Tablo 15
EXAMPLE 15.1 .. =
Using t and T 20°C (room temperature) (a) t.hi ' lil'l<l ,.
. · nder normal cond1t1ons
The adiabatic coefficient for air u . . 3 t P -- .1 atmd (b) the speed o f sound .1n air.
. e d ..
. - 1 4 A~sume the density of air is P = 1.293 kg/m a Hint: U E!risn,
ISK- • • ~ d " es of air an h 1. se ari . ,
ooc, and confirm the speed of sound at ooc an air pr - oximation for t e mear expansion ldeai
g as •appr Coeffic·
ient
sure 1.0 atm from Table 15.1. for air.

Solution solution
Substituting the given values into Eq. (15.6), we find: . n to part (a}: We calculate. the density of a·1r at r
Solu t10
erature from Eq. (15.8) first, using a tern Cofll
tern P Peratu
i ·in 1ir = J¥ =
(1.4) x (1.013 x 1osra) =
kg
1.2933
m
331 ~.
s
change of 20°C:

1.2933
kg
re

m _ kg
p = ( I ) - 1.205- .
I + 273.15°C
(2ooc) ml

~C~NCE~T.QUE.$TION is.i Solution to part (b}: Eq. [15.9] allows us to calc


oC. 1
Uate
What is the ratio of the speed of sound in aluminum to the speed of sound a t 2o ·
the speed of sound in water? Assume a solid bar of alu-

(331;m) (I + 273.150c
2
minum has a Young's modulus of Y = 7.0 x 1010 Ntm 20°c ) ,,,
v= = 342 -~
and density of p = 2. 70 x 103 kg/m3. Assume water has
a bulk modulus of B = 2.1 x 109Ntm2 and density of
r = 1.00 x 1Q3 kg/m3.
(a) 0.08

(b) 0.28

(c) 3.51
Humidity, or the moisture in air, varies, which effect
(d) 12.3
on the speed of sound in air. Does the spee· sound
(e) 14.0 increase in dry air or moist air? Consider botr. ; at the
(f) 24.6 same temperature.

As Table 15.I shows, the speed of sound in gases and


liquids depends strongly on the temperature. According CASE STUDY 15.3
to Charles's law (see Chapter 8), the ideal gas expands
linearly with increasing temperature as: In Chapter 8, we stated that the density of an ideal gas can
=
be written in the form p Mpl(RT). Is this formula consistent
V = V0(1 + a 7), with Eq. [15.8]?

in which V0 is the gas volume at 0°C, Tis the gas tempera- Answer The difference between the two equations is that
ture in degrees Celsius, and a = J/273. I5°C is the linear the. te.~perature in Eq. (15.8] is given in degrees Celsius.
expansion coefficient for an ideal gas. It can be shown that whrle it is given in unit kelvin in the formula from Chapter 8·
the density, p, of ideal gas changes with temperature as: To test consistency, we convert Celsius in Eq. (15.8] into
kelvin with:
Po
p = I + aT' (15.8) T(K) = T(°C) + 273.13.
~here Po is the ~as density at. 0°C. Substituting Eq. [ t 5.SJ Multiplying by the coefficient a yields:
mto Eq. [ 15.6) gives the relationship between sound
. ( .) wave
speed an d med rum air temperature:
aT(K) = aT(°C) + I.

(15.9) Subsrt
1 r ·
u rng thrs term in Eq. (15.8] then leads to:
where v0 is the speed of sound in gas (air) at ooc
. tem- Po
perature. Th us, the speed of sound m gas increa .
. . ses wnh P = ar"
mcreasmg temperature. For each degree above ooc
speed of sound in dry air increases about 0.6 mis. ' the

410 PART THREE I ELASTICITY AND VIBRATIONS W


, AVES, ANO SOUND

Scanned with CamScanner


formula from Chapter 8 and this equation are
I TM harmonic s1 1
I .....tent if: ti • . . lllnl waves. The displacement D(x, 1). as
IeQIJI•"' lC wave f unct·
ion o f' t I1c harmonic sound wave, similar
I -=-,,
/lo
nR '
At lo thl· lnr111 •
. ' on1c wave function r(x, 1) as discussed in
Section 14 3 .. b •
· • can e represented as:
. h shows that Po and the coefficient a are m t .
wh1C a enals D(x, 1) = Drnsin(cut - kx) (15.10)
dependent (due to the dependence on At) and have a dif- '
_,.,.,t pressure dependence. Both of these cone! .
ft:1--·· . us1ons in which D111 is the disphu:rmrnt amplitudl' which is
COf1Sistent with what we expect: the density at ooc .
are d th ff" . the max·11 d. •
proportional to p an
IS
e coe 1c1ent er is independent of . lll~m 1sp1acement of the small gas clement in
the x-d1rect1on to either side of its equilibrium position.
pressure.
----~~~~~~~~~~ For sound. as a longitudinal wa\·c. the wave number k,
the .angular frequency <u. wavekngth A. frequency f. and
period T arc dcfirll·d exactly the same as for a transverse
wave (sec Section 14.3). Note that the distance between
1s.3.2: Travelling Sound Waves two adjacent compressions or two adjacent rarefactions is
Let us again .cons.ider ~ long tube containing a gas, like wavelength, A (as shown in Fig. 15.8).
air, with a vibrating piston at one end (the left end in
fil!. \5.6). Moving the piston back and forth produces
a ~eries of compressions (high pressure) and rarefac-
tions (low pressure) in the gas in the tube as it moves
c:
0
·;;;
...,"'
a.
c:
·~
.,.."'
c:
.S2
t;
.,..
c:
0
·e
I
...,..
a. ~ ~
away from the piston (left end) to the other (right) end
u
E
0
E
0 ..
. . :1·.. . . . ... .·. .. .,::. . ..,I..·:
of the tube (Fig. 15.7). Let us define the displacement u ~ ~

of a small gas element of thickness ilx at position x in


Fig. 15.7 as D, and its pressure change as up. We choose ~,~·;.;
··:~-..:: •• :.,.... ~-...• -~~-~--.··;.:
·:. : ::;k. .::••• ·-e-: ·:.: •·:~- .....
D instead of y because the oscillation (displacement) . ..
: :·.; ~~ •• : ~: ::. : j.
• .: ·~· • •.":
. .-
• • • • .-. I • •.:
.
:.: •. ~ ::: ~- ·. : ~: ~:. : ...
·~: • •.-: • • • • ...
:.· ••
•I •
·.--
of the gas element is parallel to the x coordinate, but ••• •~ • •• .r, • ••
·.: • --
• • ••,,.. ••
.......:;;;,,.· . --.'·--
•••••• ' ·..
••••••,.,.
• ·•9'; •• • • 1.--
•• ·ii!.:-. • • 'II'- • : • :to~ ...... ,,.,..... :.r: •• : •:P:. •
displacement of string element is parallel toy. Let us
assume that the piston in the tube of Fig. 15. 7 vibrates
harmonically (sinusoidally), so the small element of the
gas, as well as its pressure difference, oscillates sinusoi-
dally. Thus. the small gas element oscillates right and
left in simple harmonic motion about its equilibrium
position in the x-direction (bottom of Fig. 15.7), trans- figure 15.B As a sound wave travels through a gas (air), it produces
ferring oscillation to the adjacent element and producing a series of compressions and rarefactions. The distance between two
adjacent compressions or two adjacent rarefactions is the wavelength, A.

As the harmonic sound wave moves in the tube,


6.< the pressure change at any point in the x-diri:ction
x also varies sinusoidally (Fig. 15. 7). Thus. the pressure
change in the gas element varies in the same mannc.·r as
::;;_~-~--i~---. x the displacement of the gas element. meaning that the
pressure difference oscillates sinusoidally as it moves
,, ,, in the x direction. The pressure difference tlp(x. t) as
,, ,,
- - - • R~udJ4..1ion
the pressure wave function of a harmonic sound wave
,, I
I'I
can be represented as:
I
6.p(x, t) = tlp 111
cos(wt - kx). ( 15. I I) 1./,
I•
D
· which 6.p is the pressure amplitude. which is the
I'
in m 1 . r. .
maximum pressure variati~m. Tie cosmc 1Lmct1on 1s use.
. d /:1
II
sure that maximum displacement and pressure max1- I
to cn . d .. W
f.lcrnenr tquilibrium po~irion s do not occur at the same tune an pos1t1on. e can ,11 ,
tig1re 15 7
alli..._ • A harmonic
· · filled tube by
sound wave produced in an air- 1 ft to 1
:i ~: that the pressure a~pli1udc 6.p01 is related linearly 11
ing from 1e 1/I
~,
·~...-i asc·11
"srnai1
1 • .
aling harmonically back and forth, is mov
. .
. .d lly
k d forth s1nuso1 a
t the displacement amplitude D111 •
° Consider the small gas e.lemcnt in r:ig. 15.7 l~avi1~g a ,.
I'

~its .~as. element is shown oscillating baC an s shown in . k ss A ,. and an area A, displaced D m the x-clircct1on. .i1'
tie e e(juihbnum with a maximum displacement of Dm• a t I11c ·nc u.. j'
Xilande11 view
. in the bottom part.
CHAPTER 15 I SOUND 411 "
1/1
11

Scanned with CamScanner


Solution
The frequency and the speed of the sound w cn.ergy)
. Eq. (14.1]:
known. Using ave are ( ttme
intensity
V, area
" sound = A/ => ,\ =~
(~~)
f
344~
A=~- I==
A -.
440 Hz - 0.782 m. (15.17)
Thus the unit of intensity is J/(m2s). Therefore, intensity is
the rate of energy, which is power (P; unit of Wis) trans-
ported per unit area perpendicular to its direction:
CONCEPT QUESTION 15.3
!==~==~ (15.18)
area A.
A child strikes an end of an iron pipe to send a sound
pulse to another child at the other end. Given that the Sound intensity is defined as the amount of sound
speed of sound at room temperature in air is 344 mi s and energy received per unit time per unit area perpendicular
in iron it is 5130 m/s, what is the ratio of time it takes the to its direction. Let us consider the small element of gas,
sound wave pulse to reach the other end in air and in the like air, of thickness Ax as shown in Fig. 15.7, having
iron pipe? area A and mass f>.m oscillating back and forth as the
(a) 0.07 sound wave passes through it. Since the gas element is
oscillating about its equilibrium, its total energy can be
(b) 0.70 expressed as the maximum kinetic energy when the ele-
(c) 1 ment passes through its equilibrium at maximum speed.
This is analogous to calculating the total energy of a
(d) 1.49 vibrating system in Chapter 13. Using Eq. [13.24] and
Eq. [13.27] yields:
(e) 14.9

I I
!l£tota1 = f>.EM ax, K.m = -f>.mv~
2 ax == -f>.mw
2 2D~, ( 15 .19)

CASE STUDY 15.4 where m is the mass of the vibrating gas element. By
dividing both sides of Eq. [ 15.19] by the arbitrary volu~e,
A bat hears sounds at frequencies of up to 120000 Hz. av, of the gas element, we introduce the energy density
Why does a bat need to use sound waves of such high etota 1 with unit J/m3:
frequencies?

Answer This frequency has a very short wavelength: i.\Etotal = .!_ i.\m v2 == .!_ w2D2 ,
(15.20)
AV 2 AV Max 2p m

344-
m where p is the density of the gas (air). Intr~du~ing the
s = 2.87 x 10-J m = 2.87 mm. energy density eliminates the problem of spec1fym~ what
,\ = I=
vsound
1200001s e mean by a small gas element: the right-hand side of
~q. [15.20] no longer contains arbitrary parameters such
Since a wave is disturbed only by objects as longI as ~~ as AV.
th 't hould be a short wave eng
longer than a waveleng • 1• s . . effect Bats emit
that will pass by smaller obiects wrth httle . d and IUlgo]!lll The energy density o~ a sound w~v;;iis propor-
hort-wavelength soun s,
a series of high frequency'. s ft r it has been reflected tional to the square of the amplttude, etotal m·
senses the sound return time a ef d food and to avoid
by an object. Bats use so~~df ~~e,:ound should be short The energy density travels with ~pe~d v in a medium
obstacles. So the waveleng ~ t
carrying sound. Thus, the intensity / 1s given as:
enough to reflect off a small obiec ·

A£total
I= vetotal = v~ (15.21)

15.3.3: Sound Intensity 1 2 2 ( 15.22)


mount of energy transported
J = -vpDmw,
Intensity is a me~s~re oftheua h unit area perpendicular to
2
by a wave per umt time t~ro ~ with the unit for the intensity J/(m2s).
the wave's propagation direction.

CHAPTER 15 I SOUND 413


NEL

Scanned with CamScanner


1:1i1;oa;.1 The intensity of a wa,·e is the amount of energy CASE STUDY 15.S
passing through a plane of unit area perpendicular_to the
propagation direction of the \\ a\·e in unit time. It 1s pro- How does the sound intensity chango with distance frorn a
portional to the square of the amplitude. point sound source?
Answer Fig. 15.9 shows a point sound sourco. Two conccn.
Eq. [15.16) allows us to relate intensity to pressure tric spherical surfaces with areas A 1 and A2 ore drown With
variation. From Eq. [15.16) we get: the point-like source at the centre. We know that tho sarne
amount of energy must flow through each ol these surfaces
per time unit to satisfy energy conservation, Al:/M :: con:;t.
Dm = (-APm)
vpw ·
Sound would have to pile up or disappear between the two
spheres if this were not the case. Thus, for Fig. 15.9 we
specifically write:
Substituting the above equation into Eq. [15. 19) yields:

t
I == -(vp) (Ap
~ w2 )2
2 vpw
In the above equation, we replace the time change ol
(ApnJ2 the total energy w ith intensity. Using Eq. (15.21) gives:
/ = = - -. (15.23)
2vp

So, the intensity of the sound wave is proportional to the
square of the maximum pressure variation (pressure wave
I = v A.£ =
V
(Ar) AE = ~
111 AAr A '
amplitude); that is, I :x (Ap.J 2. This result is useful when
we quantify sound intensity as perceived by the human ear.
in which the speed of sound is expressed as the chanll~
of the radial position with time, v = AriAt, and V = A..'i;· 5
the volume containing the sound energy. This volume .
EXAMPLE 15.6 the volume of a spherical shell of thickness Ar. Thus. :• :
A loudspeaker puts out 0.15 W of sound through a square
intensity is the rate of change of the total energy per 1• : I
area 2.0 m on each side. What is the intensity of this
area, as stated before at the beginning of this sectior. .• I
Eq. [15.17). Considering the two above equations. ,.
sound?
yields:
Solution
Using Eq. [15.18) and calculating the area of square gives:

I _ P _ 0.15 W _ _2 W
- A- (2.0m)(2.0m) - J .S X IO m2"

EXAMPLE 15.7
The pressure amplitude, Apm, of a sound acceptable to a
human ear is about 30 Pa. What is the intensity of sound
corresponding to this pressure amplitude in air at room
temperature?

Solution
We use the air density and the speed of sound calcu-
lated in Example 15.2: p = 1.205 kg/m3 , v = 3.43 mis and
"' = 21ff. Using Eq. [15.23) gives: '

~gure 15•9 . A point sound source (blue dot) shown at the centre 01
. 0 c~ncentnc spherical surfaces with areas A and A The sound
intensity
. per unit area, travelling
. through the two1surfaces.

dim..
1mshe5
as their areas increase.

414 PART THREE I ELASTICITY ANO VIBRATIONS WAVES


' •ANO SOUND

Scanned with CamScanner


ides us with the intensity ratio: Solution
'ch prov
wfl' I
~ I
A
I
,.~I
41Tr2
We have: r 2 .= 720m, J2. = o.10W1m2, and r 1 =180m. ;
/; = A2 = 41Tr~ = ~· FindJ1• • .
Using Eq. [15.24) gives:
te that f 'f. 1/~; that is, intensity diminishes in
we ~~n to the square of the distance from the sound I' p
/ r=.--= - -
propO
IsOU~e... _~~~~~~~~~~~~~~_,, /,
A 41Tr2
Pl41Tr~ r2
..:. = --~ :: _!_
If a source of.sound emits pow.er P unifom1ty in all 11 ·Pl4'7Trr r~
. . ns. as in Fig. 15.9, then, using surface area of a

sphere, . • .
. of' sound at some d.1stance ,. from the
drrecllO the intensity
r1 (r,)2
1, = ...:. 12 = (720m)2(
- -
ll!Om
W) :: I.62·
0.10-:;

W
m
source is cr1vcn
0
as.
...
p
I = 47Tr2' (15.24)

Therefore. the sound intensity drops off as 1;,.2, as shown


CONCEPT QUESTION 15.4
in Fig. 15. I0. You have surely experienced this drop off:
A person listens to a songbird at distance 1.00 m from
ou get further from a sound source, its volume, as you
~~:r it, drops quickly. Thus the ratio of two int~n.sities ~t
I the bird. If the person hears the sound with an intensity of
2.80 x 10··6w1m2, what is the power output of the bird's
diff~rcnt distances from a source of sound cm1ttmg uni- song?
fonnly is given by: (a) 4.42 x 10- 1 W
12 P/47Tr~ ri
(b) 4.42 x 10-6w
I;
= P/41TrT = rf
(I 5.25)
(c) 3.52 x 10- 6w

y
I
l
(d) 3.52 x 10- sw

(e) 3.52 x 10- 4w


J
CONCEPT QUESTION 15.5
A person listens to a songbird at a distance 1.00 ml
from the bird, and hears the sound with an intensity of
2.80 x 10- sw1m2. If the person moves to a distance
4.25m from the bird, what is the intensity of the sound
heard at the new position?
1
I oc - (a) 5.05 x 10- 7 W/m2
r2
(b) 1.55 x 10- 7 W/m2
(c) 5.05 x 10- 6w tm2
(d) 1.55 x 10- 6 w tm2

Distance from source, R


(e) 1.55 x 10- 5 wtm2
J
Figure 15.10 The intensity of sound at some distance r from a source Sound Intensity Level
S!lf'ei.!s out uniformly.
The most commonly used unit system for sound intensity
and pressure variations is based on a logarithmic scale
(base I0). We use a logarithmic scale because both param-
'..EXAMPLE 15.8 eters vary widely-for example, between a whisper and
: ~urne that the sound spreads out uniformly in ail di rec-· a running jct engine- by about 7 orders of magnitude (a
.. ~ frorn the point that a firework explodes. If the sound factor of 107 ). The sound intensity level (fl) is defined
· lllteilsity at 720 m from the firework starting point is relative to a standard threshold of hearing intensity as:
~ o 10Wtrn2, what is the sound intensity at 180 m from the
.
'.; !tarting point? Ignore the reflections from the ground. · ( intensity )
· tcnsit level er. log 10 . . .
m Y threshold intensity

CHAPTER 15 I SOUND 415

Scanned with CamScanner


The threshold intensity 10 is chosen as n~ar the EXAMPLE 15.10
faintest detectable sound for the human ear, and is
If a child and his twin each cry with an intensity lev
10 = I0- 12 J/(m 2s) = 10-12 W/m2. Thus the intensity level so.a dB. what is the intensity level of both together? el Of
(IL) is defined as:
Solution
(15.26) Since the intensity levels in decibels are logarithmic
can't simply add them. So y~u ca~·~ say the total i~:e:
sity level is 160 dB. But the intensities can be added .
The unit of IL is a decibel (dB). So decibel. dB, is a mea-
the normal way: First we calculate the intensity that e Ill
sure of sound intensity level. ·
child makes, and then the 1ntens1 ·t y IeveI of both togeth
acti
can be calculated from the sum of the intensities. Usi er
Eq. [1526] and substituting the given intensity level Val~~
yields:
What is the intensity, / , of O dB?
IL (dB) = 10 log 10 (k)
w
I = 10- 12 ?, and I = I
The pre-factor I 0 is chosen arbitrarily, but it also o nr 1 2
changes the unit of I from bel (B) to decibel (dB).
80 = 101og 10 (i)
~
=> 8 (i)
= log 10
~
=> i
~
= 10s
CONCEPT QUESTION 15.7
(a) What is the difference in intensity level, ML. of two
I
I
= 1081
0
= 108 (10- 1 2~)
~
ww
= 10- 4 ~
sounds with intensities / 1 and / 2? (b) What is the difference
in intensity level, till, of two sounds at different distances
r 1 and r 2from the sound source?

Now, again using Eq. [15.26), the total intensity level


of the crying twin sound is given by:
EXAMPLE 15.9

(/'°( )= (2
A sound has intensity 5.0 x 10-1J/m2s. (a) What is the
intensity level IL of this sound? (b) By how many decibels
does this value increase if the intensity of the sound is IL.-i = 10Jog 10 1
101og 10
x 10- 4 w) = s: ,,
m
2

increased by factors of 100 and 1000?


0 10-12 w
m2

Solution So doubling the intensities changes the intensity level


Solution to part (a): The intensity level IL is defined in 3dB.
Eq. [15.26). Substituting the given intensity value yields:

5.0 x 10- 1_J


2
)
m s
( Sound intensity is related to the pressure variati11 :5
discussed in the previous section with Eq. [ 15.23). 1e
Solution to part (b): We look again at Eq. [15.26] and pressure variation in a sound wave is defined as sou::d
write it for the difference of two sound intensities / 1 and J2: pressure level (SPL):

AIL• IL, -
12) ([')
IL, = 10log (I, - IOlog I, = IOlog
00 00
[(t)]
(f)
SPL(dB) = 20log !'_
to Po'
(15.27)

with re~erence pressure Po = 2 x I o-s Pa. The unit


of SPL is also called decibel (dB). Since the relation
AIL = IOlogio(f.).
! 2
~ tlp leads to 2 log 10(p/p 0) = log 10(Jtl 0), a factor of2
is mcluded to accommodate the difference in the inten·
sity level (IL).
Substituting / 2 = I 00/1 yields a difference in intensity level
. Th~ human ear judges sounds of equal IL values as
=
of All 20dB; that is, when the sound intensity increases
by a factor of 100, it is reported as an increase of 20 dB.
quite different, depending on the frequency of the sound
For 12 = 1000/1, we find All = 30dB. (you do not hear a loud dog whistle at all). Instead. at th~
end of the next section, we will introduce the para111eter
loudness in unit phon.

416 PART THREE I ELASTICITY AND VIBRATIONS, WAVES, AND SOUND

Scanned with CamScanner


---.....""'"'" ....... ,,,, .. _.......... . . - ~ ~

f>llllntSS: The.Ears Response


L ,
. Jlun1•111
l':ir 1s l'Xtrcmdy Sl'nsitivc to s
· 011111.1s ran , · lcvd 0 r 1I
ic threshold of pain changes widely with fre-
111~ £
111 . \ll :thlmt 20 kll7. (dogs he·ir g1ng quency Tl
. irll 1l' · ' up 10 40 kl I· · · tc range of nonnal conversation is shown as the
I!\ . l'l111un111111.:atl' at subsonic frl. \ll'll . · • . z,
11·rh:1111:. . . I 4 c.:1es th:11 c·1r gn:y.~~rea near the ct•111rc of the plot.
c h fut1hl'f in :ur). I l)Wevn. the Sl'llsitivit ' . • ry I .he. car is· musI !>C11s1t1 · · vc al ahout 3 kHz lo 4 kl lz, as
111uc. ' \':1ril'S :Kniss this fre4uency interv 1)I ol .hum~1n ti te n11n1m·1 · I
• rn I 1c curves show. As frequencies . hecornc
hc3flll~ . . • • prrnnriJy
• sc: (lf th\.' n·sl11t:llll'e prnpcrt11:s we will d '. . ' lower or hi••"'l1··r ' · 1c scns111v1ty ( 11nm1s 1cs untr·1 rt. ceases
ti . . . 1· . . I
tii.~·:tll· 'l' I . . " ·11 IS( llSS later
at .the low"r (> r IIlg
· 111.:r 1.rcl1uency l11nit
. . o f. lhc car. The varr-
.
the: outer l':tr. lls Is I ustr.1tcd quant it at ivcl . . '
fl1r • • t' 111 . tc·11 s·1·1)' l•'\"'
, ' I ,.c.•r
: s·11. s· 1·re4 uencv .. 11 •y1 in the atums '" percei ved loudness as a function of frequency
cuf'\C) 0 • J• c.:.i l:l l' «11111I
ncss curHs (or I· ktdtl'r-l\lunson r un •, ·) r: ndcr the phys ical scales of intensity level (//,; sec
1oud Tl • 1. , , .1 . c:s • shown
. F., tS. 11. tc tgurc.: s tllws the lrcllllency . . Eq. f 15.2<i J)and sound pressure level (SPL; sec Eq. [ 15.27 j).
1n 1!:i· . . . . . . r.1ngc from
sbOU
t Io llz to 16 kll1.••md sound intensity 1, 1
, . evc s from
T~c curves arc deemed less useful hecausc we all disagree
with the idea that a JOO d[J sound at 40 llz is as loud as a
0 JB to l .tO dR. Lach . curve
. 111 the plot n:p .
. . n:s1:nts the
. ·nsil)' levels as a I unction ol lrl.'lJUency th· 1 . I00 dl3 sound at 3000 Ilz. For this reason. a new param-
1ntc a ·' p1:rson elcr for loudness is introduced based on fig. 15.11 , and is
. Jcc:s hi: 1:qually lllud. The lowest curve• sitown as the
Jll ~ 10
thicker line at tlu: bollom, corresponds to the threshold of recorded in unit phon. The convention is to set the decibel
scale and the phon scall! equal at a sound frequency of
••rin".,. and 1h1: lop curve shown as the thick··r
b•" ' 1me
· at t11c
I kl lz: I 00 dB = I 00 phon at I kl lz. Loudness values
top. corresponds to the threshold of pain. As F' 15 1
. . I I rg. . I deviale from IL and SPL values at all other frequencies.
itlustr:ites. the intensity e~c of the threshold of pain does
Table 15.2 provides several examples for sounds, with
not ch:inge very much with frequency. but the intensity
loudness reported in unit phon.

dBIL

110
- .... ....._ 120
I

Thershold of pain
l

\
I

I\..
l I

,_ ~V
-- --...... ............__
110 _, ~
...
1...--
.___.-/
~
.....
~
- 1.....-

------
/
-.__/
100

--- - -- -- --
r--. --
100

---
90
----
--
I--

_,,.. /
l/ / 1....--
/ v
/
-/
--
/

--
/

--- ---- ----


?:::: t::::- 80 I
~

::: ~ - - .. --[/ v
/

-?
80 ,.______ 7
:::--:::::: ..........._ ._ ..._
·~
/
!/
............
r--..... ' r-..... 1o~
------ hl
1...--
"-
~~
~
~ _.. I/
!'-.... r-..... I'---..
r-.....
~ II
60 ' ' "" " I'.... ~ I'-.
r--- 60
'
...... r-__ : __. v I/

v
/

~
!'-.... !''-....

~
!'-....
~ ~
·- 1'. r--. 50
~) I/
I/

-
i- '--

"" ~
I'-.... I/
4~ I~
r--__ /
~ t--- II
-
:::;v
~

----
............
40 I/

~~
i---._

" r):: I'--- ..........


I'-.... !'---.
30
r--.
- [/
I
) ',. . i- '-'

~ 20
20 -;
r-...
......
- 1.....- ....-

---
/
r--._
!'---. r--
-- ,_ ..... 10 v _,,
'
I-

Thersho ld of hearing """" ......~ r---


0
,,, 1/ Ii,
0
""" ...._____ ~

-- ~ '
1000 10000
20 100
[(Hz)

Figure . wn as a function of the frequency of the sound (f; abscissa) and as a function of the sound
ilt-.:..~5.1 1 The hearing range of the human ear, sho d that are judged to be equally loud. The lowest curve is the threshold of hearing, and
~·~ 1Y level (IL; ~rdinate). Each line i~ the plot represents soun s
Sclwte
h9iest curve IS the threshold Of patn. . . 3OUnported license: httpsl/crealivecommons.org/licenses/by-saf3.0/deed.en
:Cam. This file is licensed under tile Creative Commons AttribUtion-Share Abke ·

CHAPTER 15 I SOUND 417

Scanned with CamScanner


Ea r canal
TABLE 15.2
Sound perception in unit phon

I Loudness (phon) Example


4 threshold of normaI heanng -
20 rustle of leaves
40 whispering: talking in a low voice
60 normal conversation
80 city traffic
100 industrial plant
110 comfort limit
120 thunder
Ear drum
130 pain threshold
140 jet engine Figure 15.12 Overview of the human e~r. We can_ distinguish thre
main sections of the ear: the outer ear, with the auditory canal en~UI
at the eardrum; the middle ear, with the three ossicles, hammer, air,j
and stirrup (from left); and the inner ear, with the vestibular organ.]h
EXAMPLE 15.11
vestibular organ includes the semicircular canals and the maculae,Im
A 20 dB sound varies from 20 Hz to 20 kHz in frequency.
discussed in earlier chapters.
Which frequency may a person hear?

Solution 15.4.1: Standing Sound Waves


Consider Fig. 15.11 and draw a horizontal line at 20 dB. Harmonic sound waves travelling back and ;;mh ina
The line crosses the equal loudness curves. This line inter- closed tube are a special application of the SU:' ·position
sects with the threshold-of-hearing curve at about 200 Hz
principle. The reflection at the end of the tulv -: tomati·
and 14 kHz: so frequencies between this range, 20 Hz <
cally guarantees that both waves have the sam, rplitude
f < 14000 Hz, can be heard.
(because of the absence of sound absorptio Jngular
frequency, and wave number because the inn Jnd the
reflected waves are caused by the same vibnl' . In this
15.4: Sound Waves in a Confined case, the two waves are written as:
Medium
D 1(-r, t) = Dmsin(wt - lex) 15.28)
The previous sections allow us to understand how sound { D (x,t) = -Dmsin(wt +la). 1
2
travels from a sound source through air. However, the
concepts introduced are not sufficient for us to under- Note that the reflected wave carries an addit1· I nega·
stand how the human voice operates, or how the human tive sign. This is because the wave has a ph.1 .hift of
ear detects sound. The most important outstanding issue ~cp = rr when reflected off a wall. Fig. I ~ ; illu~·
is the spatial confinement ofa sound wave. Sound waves trates how this arrangement can lead to a stan J 1 ., ~ " 3.'e
are confined during the hearing process, as illustrated Fig. I 5. l 3(a) shows a sound source at the right ,·nd 01an
by the anatomic overview of the ear in Fig. 15.12. The enclosed tube filled with a gas. The sound wa' i:' 111011n"=
auditory canal resembles a cylindrical tube, allowing b?ck ~nd forth overlap such that the amplitu~c ot._th',l~l 1
~n·
one-dimensional waves to travel inside. The auditory vibration of a small gas element becomes tinll' ind p
canal, however, is a half-closed tube, and sound cannot dent · A st an d'mg wave is generated if the d.1stanc~, h~twe~n{i
travel further than the eardrum, which separates the outer
the sound source and the wall of the tube at the kft en i.
ear from the middle ear. The current section focuses on
a multiple of half a wavelength. .
waves that are confined in closed- closed, open-open, \lT . 111· 1f 001
or half-closed tubes. As in the previous sections, we vve can also derive this result mathemattca · .
Eq. [ 15 -28]. Superposition of these two waves leads to.
assume that the absorption of sound by the medium is
negligible. D - . ( + ,t,-)].
stan - D, + D2 = Dm[sin(wt - lex) - sin ciJI
The study of a longitudinal wave in a confined space
begins with a simple model system: a cylindrical tube Using the trigonometric relation
filled with an ideal gas and closed at both ends. We use
closed tubes to introduce the most important features of
sin(a - /3) - sin(a + /3) = -2cos O' sin p
t0 . .
confined waves, that is, standing waves, harmonics, and simplify this equation:
the concept of resonance.
Dstan == -(2Dmsin(lex))cos(wt).
418 PART THREE I ELASTICITY AND VIBRATIONS. WAVES, ANO sn11~1f\ ••fl

Scanned with CamScanner


cell
Fi must be . bent for a signal
. to be sent to the brain.
ol' 1S.30. 1I_lustrates a simple physical model of bending
· stercov1Ih causc:·d bYth e v1'bration
. of basilar
. and tecto-
1
na membranes.

SUMMARY
DEFINITIONS
Tectorial membrane
• Displacement Din a one-dimensional hannonic sound wave
(sound wave !Unction):
B
D(x, t) = Dmsin(wt - kx)
Hair cells
where k is the wave number, with k = 21T'IA, and A is the
wavelength

• Pressure difference lip as the pressure wave function of a


harmonic sound wave:

lip(x,t) = lipmcos(wt - kx)


where /ipm = (vpw)Dm, w is the angular frequency, p is
c density, and vis the speed of sound in the medium

• Energy density e of a sound wave:

where A is the amplitude, p is the density, and w is the


angular frequency
• Intensity I of a sound wave, that is, the energy liE pass-
ing an area A per time unit !lt, or the power P passing an
Figure 15.30 Movement of basilar and tectorial membranes and the area A:
deflection 'of the stereovilll. A shows the basilar membrane at rest, B (!lEl!lt) P
slMlWS how the upward movement of the basilar membrane deflects hairs / = -- =-
A A
fll"Nard lhe outside. Cshows the basilar membrane at rest. Cshows how
I
Ille downward movement of the basilar deflects hairs toward the Inside. I = ve = -vpA2w2
total 2
The internal auditory hair cells are connected to the auditory nerve.
S-.1.1te: With kind permission from Springer Sclonce+Buslness Modla: Fund11mcni1ts of
~Plly&io/opy. Springer Study Edition, 'Physiology of Hearing." 1981, pp. 180-204, where vis the speed of sound
kinke, Copyrjght C 1981, Springer. • Intensity level IL (in unit dB):

s~n i~ Fig. I5.28(b). This is a task accomplished by .the IL ::: IOlogio_!_wilh/0 = IX


10
10- 12~
m·s
basilar rnembnmc. A localized vibration of different pomts
on !he basilar membrane resonated by a particular frequency • Sound pressure level SPL (in unit dll):
~kc the complex wave into separated simple waves, shown
inFig. 15.28(c). SPL == 20logroLwithp0 "" 2 X 10-~Pa
Po
How docs the resonance of the basilar membr~ne
~ad to a signal to the hrnin'? The vibratio1! of the basila~ Sound standing wave:
"~brane causes the tectorial membrane in the orgnt~
0 • D1tn11 = -[2D, ,sin(h) Jcos(wt)
"'ITIJ •A 'b · o f c·xc1ta· 1
• · 0\1 vi rate in synchrony. The mcc1 1::1111sm .
~= f!ic organ of Corti is illustrated in Fig. 1s.29• ~lu~h111
111
lhc wa tl_ tc two membranes in their equil ibriu111 position UNITS _1 _1

jK ,~~per ~ketch, and when moved upward by.an angle <1'


• Wuve numher k: m • rnd/m
''"IC Io
II) wcr sketch. The narrowness oft I1c gap Ile•tween . Wnvclcngth A: m
v,~ 1 ~0 lllcmbrancs causes the slercovilti Ill heml sul~· • rrc411cncy/ s- 1 • llz
ay, du ·
1• conn.
· I f'l 1ry h·ur
~11' .ring the vibr:ttion. Only the intcrna am 1< • ' f • Energy density e: JlmJ
~ llt cctcd to a neuron (the 1Jranc Iic<"' extension
· . o

·. l'Vc ~ell); thus, the slcrcovilli of the internal tiuditor)'
~ . CHAPTER 15 I SOUND 429
litl.··,t

Scanned with CamScanner


We compare two sound waves in a·
MC-15.3. . ir at ro
• Intensity of a sound wave/: J/(m2s) = W/s pcrature. Wave 11 ha.s twice t.he frequcnc ~Ill tcllJ.
Which of the following relations holds b: 01wavt l
• Sound intensity level (Jl) and pressure level (SPL): dB ? ctwce .
wavelengths . nthtir
(a) A1 = Au
LAWS (b) Ar> Au
• Speed of waves: (c) A1 <Au
(d) Such a. conclusion cannot be dr
d\\ n w·11h
• in fluids: .
given information. the
v = )% MC-15.4. A sound source I generates sound with t\',J ·q h
quency of sound source 11. Compared tr; 11 .. efre.
sound of source I, the speed of sound o: , ~ ~Ctdor
1
5
where Bis the bulk modulus and pis the density of the
medium (a) twice as fast . eIi 11
• in air, assuming adiabatic pressure variations (Laplace's (b) half as fast.
(c) four times as fast.
equation):
(d) one-fourth as fast.

v =fa; MC-15.5.
(e) the same.
The intensity of a spherical wave 3 ~ r
where K is the adiabatic coefficient, p is the pressure, source is 140 W/m 2 . What is the inte;. .ramthe
·it apoini
and p is the density of the medium 9.0 m away from the source?
(a) 19W/m2
• Standing waves for a reflected wave:
(b) 21 W/m2
Dstan = -[2Dmsin(kx)]cos(wt) (c) 54 W/m2
where Dstan is the displacement in the wave that results from (d) 360 W/m2
the superposition (e) 925 W/m2
MC-15.6. How far must one stand from a 6.1 m \\· ~·nint sound
• Harmonics: source if the intensity at that location i< al the hear-
• for a closed or open tube for n-th harmonic: ing threshold? Assume the sound waws travel to the
- wavelength: An = 2Lln
listener without being disturbed.
(a) 0.6 km
- frequency: f,, = nvl(2l) = nfi (b) 1.0 km
- frequency difference between two consecutive (c) 5.0 km
harmonics: f,, - f,, _ 1 = nfi (d) 14 km
(e) 22 km
• for a half-open tube: MC-15.7. A bell emits sound energy uniformly in all directiolli
- wavelength: An= 4Lln with n = I, 3, 5, 7, at a rate of 2.60 x 10- 2 W. What is the intensity of
the wave 300 m from the bell?
- frequency: f,, = nv/(4L) = nfj with
(a) 2.30 x 10-8 W/m2
n = 1,3,5, 7, ... (b) 2.30 x 10-7 W/m2
(c) 4.60 x 10-2 W/m2
(d) 5.75 x 102 W/m2
MULTIPLE-CHOICE QUESTIONS (e) 6.90 x 107 W/m2
MC-15.8. If you perceive a point-like source of sound as_toJ
MC-15.1. The frequency of a sound wave has which of th loud, you should move away from the source. This~
following units? e because of the following relation between the soun
(a) s intensity and the distance from the source.
(b) l/s (a) Intensity is independent of distance.
(c) mis (b) Intensity increases linearly with distance. ci
(d) s2 (c) Intensity increases non-linearly with distan .
(e) J/s2 (d) Intensity decreases linearly with distance. ce
MC-15.2. We compare two sound waves in air at
room tem- (e ) Intensity decreases non-linearly wt'th d1stan
d1·nuni1
perature. Wave II has twice the frequency f MC-15.9. Th e ·intensity level (/L) of a sound is
· reporte
. . . o wave I . ease a
Which of the following relations holds bet ·
ween their decibel (dB). How does IL change if we incr
speeds of sound?
(a) v1 = v11 sound's intensity by a factor of 1O?
(b) v1 >Vil (a) It remains unchanged
(c) v1 < v11 (b) It increases by 1 dB to 2 dB
(d) Such a conclusion cannot be drawn w'th (c) It increases by 2 dB to 20 dB.
·
given · fiormat1on.
m · I the (d) It increases by 20 dB to 200 dB.
(e) It decreases.

Scanned with CamScanner


CONCEPTUAL QUESTIONS
0-15.1. Is a sound wave transverse or longitudinal?
0-15.2. Having established that a sound wave corresponds
1
to pressure fluctuations in the medium, what can you
conclude about the direction in which the pressure
fluctuations travel?
0-15.3. Describe the motion of the particles that make up the
medium through which a sound wave travels.
0-15.4. Why one cannot hear the sound of an alarm clock if it
is placed in a vacuum?
0-15.5. How can an object generate sound waves?
0-15.6. If the frequency of a sound wave is increased two-fold
(doubled), how much does its wavelength change?
How much does its speed change?
0-15.7. How much does the intensity of sound change if the
distance from the sound source is tripled?
0-15.8. Why is dB defined on logarithmic scale? Is there any
relationship between threshold intensity and 0 dB?
0-15.9. How much should one change the sound intensity to
decrease sound intensity level by 3 dB?
0-15.10. How much should one change the distance from a
point source to decrease the sound intensity level by
3 dB?
0-15.11. The intensity levels.of two sounds differ by 1 dB. What
is the ratio of their intensities?
0-15.12. What is phon?
0-15.13. If a pipe with one end closed has a fundamental fre-
quency equal to the third harmonic of an open-open
pipe, what is the ratio of their lengths?

Scanned with CamScanner


underwater microphone is used to reco d
~,1 5.25· An 1·ttcd by porpoises.
· 1-1ic 1111n1111u
.· .
111 intcnsit
r sounds
P-15.32. Al1
r etl1 , •• d · . Y 1cvc1 the
. trunient c.m m.:or is I 0 dB. Assuming . . YPothcsis says the upper limit in frequency a human
ins·)ts sound at a rate of• 0.05 J/s, what is the "n Porpoise
. ..
ein · . . . iax1n1uin car can hear can be determined by the diameter of the
. tancc at which the an1111al will still be r. d
d1s • . . . ccor ed?
C<~rdrum, which should have approximately the same
Neglect soun.d absorption in water, and treat the por- diameter ns the wavelength at the upper limit. If we
isc as a point sound source. use this hypothesis, what would be the radius of the
PoAt a point. the intensity level is measured as d eardrum for n person able to hear frequencies up to
1 85 13 · 18.5 kllz'!
f' 5.26· ·
15 . . t f I
What the mtens1 y o t 1e sound at this point?·
P-l5.33. lfwe model the human auditory canal as a tube that is
1. Two sound waves ~iavt: intt:nsitics of 1 == I oo J/(in2s)
1 closed at one end, and that resonates at a fundamental
P'15.2 and I = 200 J/(m s). Uy how many decibels do ti
two s~unds differ in intensity kvel? ie frequency of 3000 Jlz, what is the length or the canal?
Use normal body temperature for the air in the canal.
P,1 5·2s.
A standard man loudly shouting produces a 70 d" P-15.34. The longest pipe (one side closed) on an organ is
sound at a d'1stance o 1·5 111. At wI1at rate docs the per- u
5.86 m. (a) What is the lowest frequency of this pipe?
son emit sound energy? Express the result in J/s. (b) How long is an open-open pipe having the same
29 The human ear can distinguish a difference of 0.6 dB frequency?
P'15· . . . I I F' d h
in sound 111tens1ty eve . 111 t e percentage of inten- P-15.35. A bugle has the same fundamental frequency as a
sity increase needed to get a 0.6 dB increase. 1.5-in-long pipe open at both ends. What is the fre-
P-1 5·30· What is the ratio of the pressure amplitudes of two quency of its first two harmonics?
sound waves of equal frequency but differing in inten- P-15.36. An organ has open-ended pipes. It can produce a fre-
sity levels by 25 dB? quency range from 65 Hz to 2090 Hz. Find the length
P..ls.31. At 25 m, the intensity level from a uniformly radiating of the shortest and the longest pipes.
source is 70 dB. (a) At what distance from the source is P-15.37. A tube of length L is placed near a sound source of fre-
it 45 dB? (b) At what distance from the source is it barely quency 759 Hz. Find the possible values of L that cause
perceivable? Assume no absorption in either case. the tube to resonate with the sound source (a) for an
open-ended tube, and (b) for a tube with one end closed.

ANSWERS TO CONCEPT QUESTIONS Concept Question 15.5: (b). Using Eq. [15.25]:
Concept Question 15.1: (c). Using Eq. [15.5] and
Eq. [15.7] yields:
/ r =>
~=
JI r2
2
2
-.!. J =
2
(r-
r2
1) 2
1 1 = ( - I)
4.25
2
( 2.80 X W)
10 _ 6 -
m2

7.0 x I0 10 Ntm2 = l.55 x IO - 7 wr


(-!;,
vI
m
' . ...., = = 2.70 X 103 kg/m3 = 1.
9 2
35
vsound in water 2.1 x 10 N/m Concept Question 15.6: 10. Using Eq. [15.26]:
Pw 1.00 x 103 kg/m3
IL (dB) = JOlogJO(*) => OdB = IO!ogio(:J
Concept Question 15.2: The speed of sound in moist air
·is greater than the speed of soun d m
· d ry at· r at the same
temperature. In moist air, a large num be r of water mole-
. .
cuIes occupy the places of either
. .
nitrogen or oxygen. Smee
.
h
100 = (*). I= l o.
the average molecular mass of moist • atr
· 15
• less than t at
f ·
0 dry atr (molecular m < molecu1ar moxygcn + . molec-
.
.
The mtens1.ty 1·s the same as that at the threshold intensity.
I
uarmnitrogen
· of sound is greater m moist atr.
)' the speedwater . Concep t Question 15·7(a): tl(/L) = 10log(/if11). Using
c00cept Question 15.3: (e). Since t = di v 1·s the time Eq. [15.26]:
taken by sound to reach a distance d:
d m b.(/l) = ll2 - IL, = IO!og(~)- IO!og(~)
5130-
-
1air Vair
= -- = - - =
viron _ _s
_ = 14.9.

[ (~)] = ('2
/. d m
iron
viron
344-
s
= iOlog (f,) )
I, . IOlog
Concept Question 15.4: (d). Using Eq. [ 15· 24]:

"' 3.52 x 10- sw.

Scanned with CamScanner


Geometric
Optics

ne of the three necessary conditions for life is the

O recognition of external stimuli and the ability to


respond. Arguably one of the most astonishing
achievements of the evolutionary process in satisfying this
does it measure? Even though we return with these ques-
tions to the psychophysical boundary between the exact
physical sciences and the subjective psychological per-
ception we discussed in the context of hearing, it is neces-
condition is vision. The human eye's complexity in design sary to investigate the physics of our vision to understand
and versatility in function is unmatched by engineered imi- its limitations.
tations. The anatomy of the human eye, shown as a cross- ln order to understand some of the properties of the
sectional side view in Fig. 21 .1, identifies at least eight eye that allow us to perceive visual stimuli, we need to
individual components required for us to see: light reaches study a diverse range of phenomena related to the propa-
the eye at the cornea (3), then passes through the anterior gation of light. Additionally, we must describe how the
chamber (2). Its intensity is adjusted by the iris (4) and the collection of light can be enhanced through the use of
light rays are focused by the lens ( 1). To accomplish the lenses so that the specific arrangement in Fig. 21. l could
focusing, the lens must be adjusted, which is achieved by be better understood.
the ciliary muscles (6). Before forming an image on the We start the chapter with a discussion of the
retina (7), the light passes through the vitreous body (5). propagation of light as a wave leading to the phenomena
The retina then converts the image into electrical signals, of reflection and refraction. We then introduce some
which are sent to the brain through the optic nerve (9). The simplifications to the wave model, focusing on the
interplay of these components allows us to clearly see struc- geometric aspects of propagation of li ght and employ
tures on an object as far away as the Moon, or to read small
this ray model to study plane and spherical mirrors. The
letters in a book at just 20 centimetres in front of the eye. application of the ray model to the propagation of light
between different mediums will then be used to describe
. But does the complex human eye indeed work as pre-
refraction through flat and spherical interfaces, and the
cisely as an instrument? What does it measure, and how
extension to multiple spherical interfaces naturally
leads to the study of lenses. We conclude the chapter
with applications of the results derived for lenses to the
study of the eye and some of its defects, and two optical
instruments: the magnifying glass and the compound
microscope.

21.1: What Is Optics?


Three different models have been developed to describe

:e 21 ·1 Cross-sectional sketch of a human eye: (1) lens, (2) ante:ior


(8) fo~' (J) co~ea, (4) iris, (5) vitreous body, (6) ciliary muscle, (7) retina,
the physics of light: the ray model, the wave model, and
the corpuscle model. The initial development of the field
came in the 17th century when Rene Descartes, Christiaan
centrahs, (9) optic nerve, and (10) blind spot.
CHAPTER 21 I GEOMETRIC OPTICS 569
lltl

Scanned with CamScanner


Figure 2u Geometric sketch tor the law of reflection on aftat intcrtar..i:
bnsed on Huygens' principle. A planar wavefron~ approaches .the intcrtac.e
=
under nn angle ir,. At time 1 O (top sketch) 1t reaches po1nt A1 at 111!;
Interface. In tho bottom sketch, spherical waves are drawn from points A
and A, forming the reflected wavefront W'. Note that the initial w~vi:fr~
whas-just reached point A3 in the bottom sketch, which appllc, d! bf!IE
1 = Ric, with R a distance defined In the top sketch and c the • "eedrJ
the wave.

The construction in Fig 21.2 allows us to 1 geo-


21.1.~ Wave Reflection metric arguments to derive the law of reftectior 1ich
w1,~ ~::"""ti' n t:- a familiar pht"nl'tllt"non for light and connects ai, the angle of the incident wavefront. i the

~ ~ \\:i'~ ti.)r t".'l.:tmrk light rdk\:tion from tht" sur- interface, and ar' the angle of the reflected '' . ·om.
f.JI..~ ,,i \\ :t'.';!1'. ,,r n:h,, ti.,m13ti0n in mountains. We use with the interface as illustrated in the bottom of f ~ 1.2.
F~. :: 1 ~ w rel.lte the :.mglt"s l-t"tWt"t"n incidc::nt _and For a quantitative result, we compare the two tri:. , .J
1
~:.:~""tN " :l\ a.. Th~ figure sh,)ws a planar wawtront and !!.2 in the figure. We note the following three :res:
~p-,'ll.\.t:~ J tht intafa1.~ :it :m anglt" a,. The:: interface
~ d~ t-y the !'.''tnts .-1 1• .-1 1• and .-1 ,. with A 1 halfway • The sides from A 1 to A 3 are equal in both tn :s.
~~ ..t 1 :l.'>J Ay We:: choost" the:: initial time I = 0 at • The sides from A 1 to C 1 and A 3 to B3 are equ iong
t!:t" icsu:n '' h.:n tht" wavefront has rc::ached point A 1• because sound travels same distances in the s~ 1i:ne.
lli,; .,.-:iwfrl'nt is shown in the top pan of Fig. 21.2 as a
~P:t lint" b t-t"ll.:d \\'. • The angle A 1C 1A3 is 90° because the tangen: . cir-
t:sing Huyg;:ns· principle we construct the location cle is perpendicular to the radius. The angk B_,.~;
of th:! "'a'etro:lt after reflection in the bottom part of is also 90° because the wavefront is, per dl· ;ion.
Fr:;. 21.2. The figure shows the system at the time instant perpendicular to the wave propagation direct.l·~
v.-~ the im:ident '' awfront has reached point A . The
3 These three conditions establish that the two shad~d tri·
tbit" elapsed bt"meen the top frame and the bottom frame
in f ig.. 212 is~= R c. in which R is the distance from B3 angles in Fig. 21.2 are the same; this leads to the b11 of
to A ;· and c is the speed of the wave.
reflection:
The elementary spherical wave emerging from point
A 1 in the bonom part of the figure has the same radius R.
TI:e elementary wave emerging from point A2 has radius
R.'2 because the incident wavefront reached point A afler
. d. 2
21.1.2: Wave Refraction
~"':le ..ll_':!, an 1t travelled a distance R/2 in the remaining
In general, waves are not completely renected at 1m1~1
tune. ~ote that "° e connect the elementary waves in the
surfaces but penetrate the surface and tra\'d in th(.
~of the figure according to Huygens' principle; this
yte!,eh the Wa\efront \V'. new medium. Fig. 21 .3 allows us to relate the :111t!k 01
. .d . . .J
mci ence with the interface to the angle ofthi.: tr:ins1111111

570 PART FIVE I ATOMIC. ELECTROMAGNETIC, ANO OPTICAL PHENOMENA ~ll

Scanned with CamScanner


sulting law is called the law of refr t.
. the re ·~ . ac ion. The
11_,1·c, 1vs two dlllerent media, labelled I and 11 . These
id . two fior mu 1as are combined · · · ·
by ehmmatmg the
-·~ sh0 d , Wllh
ji~ll'· _d of values c1 an c 11 respectively F' entical length A 1A3 and substituting the product of
• spec · 1g. 2 I 3
i1":1''e cs the case cl> ell. . speed of the wave and !:lt for the two radii:
i!lllsttat
sinai R c
- - = -1= -
1
sinal R11 c 11 •
Th' 15
·, .
is called Snell's law for Willebrord Snell, who d1s-
covered ·it ·m 1621. It states that the sine of the angle of a
wavefront with a flat interface is proportional to the speed
of wave in the respective medium.
The corpuscle theory was revived in the early 1900s
~hen Albert Einstein used a corpuscle model (in which
light particles are called photons) to explain the photo-
electric eITect, discussed in Chapter 19, which describes
the ability of light to knock electrons out of solid matter.
11 Our modem view of light is that it has a dual, wave-and-
corpuscle character. This is discussed with the double-slit
experiment in Chapter 19.
t =Ric
In what remains of this chapter, we will discuss optics
with a simpler model, called the ray model. This greatly
simplified model is applicable as long as the objects
involved in an optical study are not smaller than the
wavelength of light, which for the case of human vision
lies in the vicinity of 500 nm. The physical laws that we
derive with the ray model are summarized by the term
geometric optics because we can construct its features
Figure 21.3 Geometric sketch for the Jaw of refraction at a flat interface with geometric methods.
baSed on Huygens' principle. The top sketch shows an incoming wave-
froot wat an angle a;. At t =0 the wave has just reached point A 1 in the mU!llil In the ray model, the assumption is made that
llterface. The bottom sketch shows the refracted wavefront w' at time light moves along straight lines while travelling within a
1= R/c1, with c1the speed of sound in medium I. At that time, the wave- homogeneous medium. It may change its direction when
troot whas just reached point A 3 in the interface. Huygens' spherical reflected by and/or passing through an interface into
waves have been drawn at points A 1 and A 2 to illustrate the formation of another medium according to the laws of reflection and
lhe wavefront W'. refraction derived from Huygens' principle.

The fundamental concepts of geometric optics arc


established in the next two sections as special cases of
The top sketch of Fig. 21.3 is identical to the top the two laws already discussed. These two cases include
sketch in Fig. 21 .2: it shows the instant t = 0 when the reflection oIT a mirror and refraction when light passes
incident wavefront has reached point A 1• The wavefront through a transparent interface.
arrives at an angle a. with the interface. It will reach point
Al a time 61 = Rifc later. This later instant is illustrated in
1
the bottom sketch of Fig. 21.3, which shows two elemen- 21.2: Reflection
tary spherical waves, one emerging from point A 1 and one
from point A2. Both elementary waves are in medium II 21.2.1: Flat Mirror
and therefore travel slower· the wave from point A 1 has Fig. 21.4 shows a 1ight ray that is reflected off a planar
reached a distance of R = ~ 6.t and the elementary wave mirror. The direction of the travelling light ray is shown
from . II II , 1 by arrows. We will frequently use the following two
. point A2 has travelled half as far as it emerged on Y
at time llt/2. The following relations are obtained geo- features:
metrically for the angles of the incident and transmitted
Wavefronts: • The incoming and reflected rays are in the same
plane as the vector directed perpendicular to the
. R, mirror surface. This vector is called the normal
sma. = - - vector, in which the word "normal" is synonymous
1 A1A3 with "perpendicular" (in the same manner as we
used the term when we defin ed a normal force in
. Ru
sma1 =-A. mechanics).
A1 3

llll CHAPTER 21 I GEOMETRIC OPTICS 571

Scanned with CamScanner


eeing it in a flat mirror. We start w· h
used to S . . It a .
. 1 t source (solid dot at top right). The light Point 1
I1g 1 I I t . h .
Ii ht rays that trav~ a on.g s ra1g l lines in alt rceefll1
scu . I~
gh such a straight I me reaches the "' . direq1 ~
W en . h fi ... 1rr Ori
l c igure), the law 0 r s~.rf. 1
(shO\vn for three rays m d fi h of r n ilte
. lied as illustrate or t e centre ray . t11eq
IS app • . In f j lr~I
... After reflection, the rays continue lo travel alon, g. 215
g . until they reach the eye of the observ g Strai,L
I mes . er. Ob ~~
can 1 ·nterpret the light rays rcac111ng the eye.1n t ~el\er1
11
either they are aware o~ the presence o~ the rnir~ lla~i
dra w the rays as shown m front of the rnirro or a11~
. <l r, Orth.
aware of the mirror an extrapolate the r· . ei a:t
not . b h' d h . a1s st .
to the dashed point e m t e mirror at thi.: left ra1gt.:
fi g Ure . At the. point where these lines cross. I he j1n~r.'
Figure 21.4 A light ray reflects off a flat mirror: ii is perpendicular to the
·
f the point-like light source forms. The posit. lllage
mirror surface, ain is the angle of the .rncomrng
. ray, and a out is the angle 0 . d. .
light source (object) an Its Image are both :it d. •1:
Ion Of •L

d firo m the. mirror. This is a direct consequen~,


of the outgoing reflected ray. af istanct
"- o the la.
of reflectton. ~
• The angle between the incoming ray and the nonnal
We call an image a real image when Ii_
vector a . is equal to the angle between the reflected . rays aC!tJ.
' m' ally reach the image. In Fig. 2 I .5, light car;
ray and the normal vector, a out· "hysicalh
reach the image since a mirror contains a ; _;Ilic la ~
These two conditions constitute the Jaw of reflection that prevents light from passing through. Jp .1 '
as derived from Huygens's principle in the previous case. ~ie
image is called a virtual image.
section, and is quantitatively written for the case In Fig. 21.6, we generalize our choi 1f object.
described as: replacing the point-like light source of r 21.5 llil.C
(21.1) an extended object of height h (reaching t· poimP.
to point P 1). The object is still at distan.. from~;
The angle aout is called the specular angle.
mirror. Using the Jaw of reflection for ligh• ·s comir.;
•UUl!lii The law of reflection states that the angle of an separately from points P 2 and P 1, that is. · = a' anc
incoming light ray with the nonnal direction of the mirror a 1 = a'2 , we find that the image in Fig. : · '1 is ~ i i;.
is equal to the angle of the reflected light ray with the tual image, forming at distance d behind th, 11irror. fa
normal direction of the mirror. comparison with later cases, note also that the irnm 1;
upright; that is, the corresponding image p0ints P~-ar.~
We use Fig. 21.5 to illustrate how the law of P; are at positions y = 0 and y = h with a \'Crtical r:a.\i;.
reflection leads to the formation of an image as we are respectively. .

Figure 21.5 Apoint-like light source located a distan d. ~


dbehind the mirror surface. The image of the light source . in front of a flat mirror (vertical bl . 111e ~,s::~
53
indicate real rays; dashed lines represent virtual rays bc~. s constructed using the law of
1
ue .hne) Produces a virtual image seen at the tre 50-'id~·
1
e nd the mirror. 1
re lection, as indicated for the light ray at the cen ·

f:,7') DllDT Cl\/C I l\TfUAlr Cl CrTonu·~· · - -

Scanned with CamScanner


This is illustrated in Fig. 2 1.1 3. with a sketch of a light ray
coNCEPT QUESTION 21.1 approaching a glass surfocc from a vacuum in part (a) and
with an experimental setup in p;irt ( h). The reflected ray
• · 2 1.12(b) shows the cent'e 01 cun<atu'e c, the ;ocal J obeys the law of reflection. The angle between the direc-
f ·. t F. and an ob1ect 0 for a concave mirror. Which of the
p01n d. t . tion of the normal vector of the glass surface (along the
r propose image cons ructions is correct?
fOU - - - - - - ·- ·- - - - --- _ __ --· thin vertical line) and the light ray that has hccn transmitted
through the interface. lahellcd a 1r in Fig. 2 I. I 3(a ). depends
eXAMPLE 21.1 on the material forming the interface with the vacuum. As
dentist uses a mirror to examine a tooth. The tooth
discussed in Section 21. 1.2. the relation of this angle to the
~ . cm in front of the mirror and the
15 1 0
image is formed angle of the incoming ray is given by Snell's law, written
.0 cm behind the mirror. Determine (a) the radius of cur- here for the spccific case of light:
0
~ature of the mirror and (b) the magnification of the image. sin a in
- .- - = fl, (2 1.6)
solution sm a 1r
Solution to part (a): ~he to.cal length of the mirror is found where 11, the index of refraction. is a dimensionless con-
trom the mirror eq~at1on w1~h p == 11 .0 cm and q :: -1 cm. o.o stant dependent on the material. Table 21 .2 lists indices
llle image length 1s negative due to the sign conventions of refraction for a range of materials. Note that gases
of Table 21 .1. Thus: have values for 11 very close to the value for vacuum.
I I I I I
-=-+ - = - - - - - (a)
! p q O.QI m 0. I m'

which yields:
f= + I.It em.
111us, the radius of curvature of the mirror is R == 2/:: 2.22 cm.
Solution to part (b): We obtain the magnification from
the mirror equation in part (a):
11.f = _'!. = - 10.0cm
= + 10.0.
p I .0 cm

The image is upright because the magnification is posi-


tive. This is convenient, since this is the way the dentist
prefers to look at the image.

21.3: Refraction
21.3.1: Flat Interface
Light can pass through transparent media; for example,
visible light passes through window glass. Except in a
vacuum, the intensity ofl ight attenuates as it passes through
any ml!dium. leading to the definition of the optical depth
of a medium. As an example. you have no problem seeing
an object at the bottom of a beaker filled with water, but
you cannot see the bottom surface of a deep lake. Also,
you can see the ground through Earth's atmosphere from
outer space (for example. from the International Space
Station). but you cannot sec the surface of Jupiter during
a fly-by mission. Still. for short distances. the gases in
Jupiter's atmosphere (helium and hydrogen) are trans-
parent to visible light. In the remainder of this chapter
we refer to transparent materials with the assumption that
their thickness is chosen such that light travels through the
material without a noticeable loss in intensity.
When light is incident upon an interface between two
transparent media under a not too steep angle, we observe
that a fraction of the light is reflected and a fraction of the
light passes through the interface into the second medium.

Scanned with CamScanner


· . f . . WC cIiap t c··rs·. the speed of light varies between
. dip·•eren
This is convenknt from a practical point 0 . vie":· . materials. When ligh.t passes through a me_dium, its elcc'.
need not distinguish whether we do th~ c.xpenmcnt m a~ · field interacts with that of the atoms and mole 1
t nc I 1·1 d cu es
or in a vacuum. The wavelength restriction noted bclo .m ti1e medium. This . slows
. t le 1g it own_ to speeds. 1e~~·
Table 21 .2 is due to the effect of dispersion. .
th an the speed of light 111 ,a vacuum at c - 3 x I 0~
. . 1 . S . lll/s
As Fig. 21.13 indicates. refraction. causes light to < l' From Huygens prmc1p e m ection 21 1 '
vlight · · \\·e
travel closer to the normal direction m denser mate-
find:
rials. Since the path or a light ray is reversible, light :an
of course also be sent across an interface approach1~g sin ain c
-=--. (21 .7)
from the denser medium. fig. 21.13(b) shows that, m
this case, the light is refracted away from the ~ormal
direction. This allows us to choose an angle a such Eq. [21.7] shows that the ratio of the two sine terms is
that the angle on the vacuum side becomes 90~. Thi.s is constant, since c and vlight are both constant across a given ,
called the threshold angle for total reflection, smce light interface. Due to Eq. [2 I .6), the index of refraction repre.
approaching the interface from the denser side at a~gles sents the factor by which the speed of light is lowert·~ in
larger than a• cannot leave into the less dense medium. a medium in comparison to a vacuum. This index is r• . ~r
The threshold angle for total reflection is calculated from smaller than I, as light cannot be faster in any m, ,. •n
Eq. [21 .6]. than in a vacuum. It can be slowed down considern . _
Note that Snell's law in Eq. (21.6] is not in the for example in diamond, by more than a factor of ~
simplest possible mathematical form, which would be If we replace the vacuum in Fig. 21.l3(a) , 3
n = a . la . The underlying cause becomes evident when second medium with another index of refraction 11 ..
comp:red to the case of sound waves discussed in earlier Snell's law generalizes to the law of refraction:
n 1sin a 1 = n 1sin a 1

for an incident ray of angle a 1 coming from a med· ,f


TABLE 21.2 refractive index n 1 and passing through an interfac ,1
Index of refraction for various materials a medium of refractive index n1 . Eq. [21 .8] then .s
angle a 2 measuring the direction of the refracted ra) e
Material Index of refraction
that both angles a 1 and a 1 are measured with resp. u
Solids at 20°c the normal to the refracting surface that separates th. 0
Diamond (C} 2.42 different mediums.
Sapphire (Al2 0s) 1.77
IUUl!lii A light ray passing through an interfac ~ is
Fluorite (CaF2) 1.43 refracted. The law of refraction relates the angles or· thl!
Fused quartz (Si02) 1.46 light ray with the nonnal on both sides of the interfan: to
Crown glass 1.52 the two indices of refraction. Snell's law is a special c:i e
where one medium is a vacuum (or air).
Flint glass 1.61
Ice (H2 0, at 0°C) 1.31
Sodium chloride (NaCl) 1.54 EXAMPLE 21.2
Liquids at 20°c . A light ray travelling through air is incident on a flat slab
Benzene (C6 Hs) 1.50 of transparent solid material. The incident beam makes
Carbon tetrachloride (CCIJ an angle of 40° with the normal, and the refracted beam
1.46
makes an angle of 26° with the normal. Find the index of
Ethanol (C2 H50H) 1.36 refraction of the transparent material.
Glycerine 1.47
Water (H2 0)
Solution
1.33
Sugar solution (30%) We use the law of refraction to solve for the unknown
1.38
i~dex of refraction of the transparent material with the
Sugar solution (80%) 1.49 grven data and the index of refraction of air as n 1 =
Gases at 20°c and 1 atm (see Table 21.2):

Air 1.00027 sina .


nslab = nair • air = 1.00 sin 400 = 1.47.
Carbon dioxide (C02 , at 0°C) 1.00045 sin aslab sin 26°
Vacuum 1.0 If we want to identify the material based on this
The data are measured with fight rays of vacuum wavelength result, we might suggest fused quartz, with n 1.46 from =
589 nm. · Table 2 1.2, since glycerine is a liquid.

Scanned with CamScanner


where a 10 and a Ir arc the respective angles of the light ray
PE== 2.5cm with the nonnal
• Refractive power
• for a single interface: ~II = !J.n/R
q0 == l - PE= 12.5cm.
• for a thin lens: ~ll = I/f
The object distance Po is then found from the thin- • Standard near point: s0 = 25 cm
rrnula applied to the objective lens:
iens fo • Angular magnification: m = 0100
1 I I I I I
-+-==-'*-+ - - - = - - where 0 is the angle subtended by the object and 00 the
Po qo fo Po 12.5cm l.Ocrn·
angle subtendcd by the same object placed at the standard
=
wtiich yields Po 1.09 cm.
near point
we now know all the data needed to determine the
UNITS
gnification of the objective lens. Note that we do not
rnalculate • Refractive power ~I: dpt (diopters) = m- 1
the angu Iar magni"f'1cat·ion of the objective lens,
ca the observer is not involved in the process of its image
:rrnation. We find with Eq. [21.11]: LAWS
• Reflection. flat mirror:
qo 12.5cm
M.0 ==--=---=-115
p0 l.09cm · ·
where angles are measured to the normal of the mirror
Thus, the total angular magnification of the microscope is:
surface.
M1ou1 = M0 mE == (-11.5)(10.0) = -115. • Law of refraction at the interface between media I and 2:
Solution to part (b): The problem becomes a simple
n 1sin a 1 = n2sin a 2
substitution problem when applying Eq. [21.20] in its
approximate fonn: where angles are measured to the normal of the interface
• Mirror equation and thin-lens fonnula:
I 1 I
-=- + -
(0.15 m)(0.25 m) ! p q
- -- - - - = -150 where p is the object distance and q is the image distance
(0.01 m)(0.025 m) ·
• Magnification of spherical mirror or thin lens:
The difference between the results parts (a) and (b)
illustrates the extent to which the second approach yields M= _<!._
an approximate result. For most applications, the result in p
part (b) is sufficient. • Law of refraction for a spherical interface between media
I and 2:
,,, 112 112 - ,,,
- + - = -- -
p q R
• Magnification of a spherical interface between media 1and 2:
SUMMARY
h1 nlq
M=-=-
DEFINITIONS ho nip
• Focal length of a spherical mirror: f = R/2 • Angular magnification m of a lens
where R is the radius of curvature of the mirror • for a relaxed eye:
1 Magnification of a spherical mirror or thin lens: So
m= -
f
hi • for an eye focused at the near point:
M=-
ho
So
m= I + -
where h1 is the height of the image and h0 the height of f
the object
• Total angular magnification of a compound microscope:
I
Index of refraction /1 (Snell's law: the incoming ray must L so
travel through a vacuum or air): m1otal = iHo m E = - fo /E
sin ain
--=11 in which index 0 stands for objective lens and index
sin a 1r stands for eyepiece.Lis the distance between the two

Scanned with CamScanner


Con,truct the m1ai;c' lv1 thc thn:c OhJc h ~ho" n ' fl
n-21 ,1. (,l) S 1llllll l11•1t k1w r . u.l vl W ill!•''"'' \VUtl'llllU1111\H r' in
P- 21.3.
I i• , 2 1.4 I.
lull ~ 11t hlill 1•' ht oll 11111 1>1111" Ill k 1I\ t" • tl1tt: hl lh<' ft h,'. ll'l•
111~ 1•tkd 111 11 .11.:r .l111pl.·1 ~ 111 1111, 111h1ll' t l':1 " 111~hk'!
1/1111 I 1.·<11 Ill . II ' ' "'' tl r11p k1 I" II • plil' IC pl iil l'" ltll 11tc (.1)
l,·al°, '" ' h"" 11 111 1 tµ, ~ I ·ltl, 111111ll• l' 1111: 1hi11 kn ~ 1111•
111111.1 (hi O '"" '""' 11111 it'''"'' I )11 )1111 I.now why ti I'
bltll 11111 11~1 1111l11k11 11111 111 cr tl111ll·1s in 1'1111 ~11111i glt1'!
_.1.I_ ._ _
0 r.
I ro111 Jiu I.

(h)

c 1: 0
Fronl l\Jck

(c)

Figure 21.40 0 c
Front 11.ick
0-21 .8. Optl1lllt'trists llSl' the Sndkn trst tu evaluate their
patiL·nts' ,·isi\1n. The Sndkn test consists of letters
of \Ii lkrt·nt sill's that n pl·rsnn with healthy eyes can Figure 21.41
!\'ml nl p;1rtil·ular di~tann·s. The patient is plan•d 6.1 m
t:?O ti.·et) t'ni111 tlll' diart and ash'<.! to rL·ad the lellers. If P-21.4. A light ray enters~' layer of water at an ungle of36° \\ith
thl· pa1ient's l'~ l·s arl' hL·althy. he/she will read the same the vertical. What is the ungle betwt•cn the refraeted
li1w "itl11•11l l'rn>rs tlial the healthy rcli.·rl·nce group was light ray and the vertical'!
ubk 111 rL·ad at 1ha1 di~tance. We thacliire call this 20/20
P-21.5. /\light ray strikes a llat, L .. 2.0·cnHhick bloc!.. of glass
\'isi11n. ,\ ju\·L·nik may ha,·e :?0/1 0 vision. which means
(11 = 1.5) in fig. 21.42 at an angle of fl "' 30° \\ith the
that hdshe can rcad a line that a healthy adult can read
normal. (a) Find the anglcs of incidence nnd refrncti1in
onl\' (\I a distance of 3.05 m (I 0 frl'l). Vision· impaired
at each surface. (b) Calculate the latcrnl shift of the lighl
paticnts may sl'llrl' as low as 20/200. which corresponds
ray d.
to the sin!.!k. lar!.!est lener at the top of 1he Snellen test.
A pl·rsM-with l;L·althy eyes can read that letter as far
away as 6 1 m (:?00 fret). '' hich coincides with the
distancc :it "hich the eye is accommod:ited for vision
11f l'hjL·cts at infinite dist:ince. Many optometrists have
ofliccs in a mall with high rcnt. To kecp the cost down,
thc cxumination ro11111 may ha,·c a lcngth of only 4 m,
with thc pati1.·nt sitting :it the examination instrnments
nt•ur thc centre of the room. Suggest an appropriate L
sctup for the Snellcn test in this room.
Q-21.9. The optic ncrve and the brain invert the image fonned
on the n:tina. Why then do we not see everything upside
down?

ANALYTICAL PROBLEMS
P-21.1. \\'hen you look at your face in a small bathroom mirror Figure 21.42
from a distancc of 40 cm. the upright image is twice as
t:ill as your face. What is the focal length of the mirror?
P-21.2. A C\111Ca\ c phcrical mirror h:is a radius of curvature of P-21.6. In Fig. 21.43, an uhrasonic hc:1m enters an.organ (£r<hy)
,
:?O cm. Loc:ite the im:igcs for object distances as given at 0 • 50'>, then reflects off n tumour (~rccn 1'n I .C1
.
surroundmg organ and !caws the organ \vi ·111 o ta1
, erll'
licll)\\ . In l'ach case. state " hether the image is real or
h'fi · 1 0~ . 1e 1
, irtual JnJ urright \' r im Crtl·d. and find the magnifica. s 1 t L • 12 cm. If the spec(] of the wuvc 1~ ' i)l(
111iri. !a l1• 1n1.·ni: tb 11, 20 c111: tc11> - 40 cm. in the organ than in the medium above, 1k1crniloc
depth of the tumour he low the Ml!""·~ ~urfucc.

598 PART FIVE 1 ATOMI C. ELEC TR OMAGN E TI C. AN O OP TI CAL PHENO MENA

Scanned with CamScanner


I I
:.-1.--.:
I I
P-21.13 A
·
.
c~nvcrging lens has a focal
length/= 20.0 cm. Locate
I I
I I the 1mag1:s for the object distances given below. For
I I
I each case state whether the image is real or virtual
I
I and upright or inverted, and find the magnification.
I p (a) 40 cm; (b) 20 cm; (c) to cm.
I
I -21.14. W~ere must an object be placed to have no magnifi-
Pcgan cation (IMJ = 1.0) for a converging lens of focal length
!= 12.0 cm?
P-2115 ' 2 1.45 shows an object at the left, a lens at the
• · f ig.
centre (vertical dashed line), and a concave mirror at

figure 21.43
- Tumour

P-21.7. A light ray. travels. through air and then strik es the sur-
the right. The respective focal lengths and the distance
between lens and mirror are indicated at the bottom of
the figure. Construct the image that forms after light
from the object has passed through the lens and has
reflected off the mirror.

Lens
face of mmera101 1a~ an angle of 23.1 o with the normal
to the surface. What 1s the angle of refraction ifthe light 0
ray travels at 2.17 X I0 8 mis through the oil?
P-21.8. A light s?urce at the bottom of a 4.0-m-deep water pool
sends a hght ray up at an angle so that the ray strikes the
surfac~ 2.0 m from the point straight above the light source.
What 1s the emerging ray's angle with the normal in air? i.-11~1-.f
P-21.9. The laws for refraction and reflection are the same for i.-111---.i.--i.----- 2(/1 +Ji)
light and sound. If a sound wave in air approaches a Figure 21.45
water surface at an angle of 12° with the normal of the
water surface, what is the angle with the normal of the
refracted wave in water? Use for the speed of sound P-21.16. An object is placed in front of a converging lens with
340 mis in air and 1510 mis in water. f = 2.44 cm. The lens forms an image of the object
P-21.10. A slab of ice with parallel surfaces floats on water. What 12.9 cm from the object. How far is the lens from the
is the angle of refraction of a light ray in water if the object if the image is (a) real or (b) virtual?
light ray is incident on the upper ice surface with an P-21.17. A contact lens is made of plastic with an index of
refraction of n = 1.58. The lens has a focal length of
angle of 30° to the normal?
P-21.11. A light ray is incident from air onto a glass surface with
f = +25.0 cm and its inner surface has a radius of curva-
ture of +22.0 mm. What is the radius of curvature of
index of refraction n = 1.56. Find the angle of incidence
the outer surface?
for which the corresponding angle of refraction is one-
P-21.18. A person can see an object in focus only ifthe object
half the angle of incidence. Both angles are defined is no farther than 30 cm from the right eye and 50 cm
with the normal to the surface. from the left eye. Write a prescription for the refract-
P-21.12. Construct the images for the three lenses shown in ive powers m(in diopters) for the person's corrective
Fig. 21.44. Note that the third case is a diverging lens. lenses.
P-21.19. The near point of an eye is JOO cm. A corrective lens is
(a) to be used to allow this eye to focus clearly on objects

~
25 cm in front ofit. (a) What should be the focal length
! •F •F
of the lens? (b) What is the refractive power !)l of the
lens?
0
P-21.20. A person who can see clearly when objects are between
30 cm and 1.5 m from the eye is to be fitted with bifocals.
(a) The upper portion of the corrective lenses is designed
(b)

~
such that the person can see distant objects clearly. What
refractive power 91 does that part of the lenses have? (b)
•F t
0

F
The lower portion of the lenses has to enable the person
to see objects comfortably at 25 cm. What refractive
power 91 does that part of the lenses have?
(c)
P-21.21. The near point of a patient's eye is 75.0 cm. (a) What
should be the refractive power 9l of a corrective lens
prescribed to enable the patient to clearly see an object
!0 •F •
at 25.0 cm? (b) When using the new corrective glasses,
the patient can see an object clearly at 26.0 cm but not

*
F
at 25.0 cm. By how many diopters did the lens grinder
miss the prescription?
Flv~e 21.44
CHAPTER 21 I GEOMETRIC OPTICS 599
~q

Scanned with CamScanner


·....
- ... ' .
..
'··
ot>·ect is located 20 cm to the left of a com·npng
P-21.22. A magnifying gla!>s is used to examine the structural P- 21.26. An ~ Ad. · • . ...
'"ocal leneth
Iens of J •
25 cm. ivcrgmg 1ens 'II.1th focal
.._,.. .gh
di..-uils of a human hair. The hair i~ held 3..S cm in front
-
eth Io cm is located !5 cm to u n; n t of the C(){)-
of the magnifying gla5s and the image i!I 25.0 cm from
1
,:;ing lens. Find the position of the final .image:
P-21.27. Iflight is incident at an angle(} ·~om a ~d~""r.the (lf inde'-
the lens. (a) What is the focal knb'1h of the magnifying
glas<;'! (b) What angular magnification is achieved'! of refraction n 1 to a medium v.1 n~ sue t ".t angle
P- 21.23. T¥>o converging lenSl"S that have focal length~ of/1 "' between the reflected and refracted beams is /3. show
10.0 cm and[, ~ 20.0 cm are placed l = SO cm apart.
The final image is i.hown in fig. 21 .46. (a) How far to
that:
the left of the fir~t lens is the object placed if I = 3 \ cm?
(b) What i'> the cnmhined magnification (not the total
angular magnifkation in this case!) of the two lenses
using the same data as in part (a)1 Hint: U$C the fonnula for sin (a 1 + a 2) from the Math . ,.
Review on Trigonometry.
I
1!1
I
:1i
I
P- 21.28. Light of waYelength A0 in ,·acuwn has a wa,·elength of
A =438 nm in water and a wavelength of -'i-"" 390 nm
I I
i;benzene. (a) Whal is the wa,·elength Ao in a vacuum?
1 I I
I
I I
I
I (b) Using only the gi\"en in~ormation. determine the
I I
0 I
I l I
I
ratio of the index of refraction of benzene to that of
water.
\.-11--.: I
t •I ' I
I
I
P-21.29. A 400-nm-wavelength light ray is incident at ao angle
of 45° on acrylic glass and is refracted. What wave-
I. length of light. also incident al an angle of 45° but on
fused quartz, refracts at the same angle? Hint: Use
Figure 21.48
Fig. 21.48.

P-21.24. A mkroscopc has an objective lens with/= \6.22 mm


and an eyepiece with/ ""' 9.5 mm. With the length of
the microsco(l\!'i. barrel set at 29.0 cm. the diameter
of an erythrocyte' s image subtends an angle of 1.43
mrad with the eye. If the final image distance is 29.0 cm
from the eyepiece. what is the actual diameter of the
erythrocyte'! I/int: Start with the size of the final image,
then use the thin-lens formula for each lens to find their
combined magnification. Use this magnification to cal-
culate the object siLe in the final step.
P-21.25. Fig. 21.47 shows two converging lenses placed L1 =
20 cm apart. Their focal lengths arc / 1 = I 0.0 cm and 1.50
/ 2 .. 20.0 cm. (a) Where is the final image located for
an object that is l. 2 ,.. 30 cm in front of the first lens'!
(b) What i-; the total magnification of the lens system'?
N<1tc: Oo nut calculate a total angular magnification in ··:~.
~ · -~
' .
this case iiincc we arc not dealing with a microscope.
)i:j
~quartz . ·~~~
·.;1~
.~ \ .~t1
\.45 •. ,J
. . :~~""'.··~~
400 500
'1~i.·l
.,'.... .
600 700 ~·~;

A(nm) .:;:;4
·.. ·.)~
Figure 21.48. Dispersion relation, ll(A), In the visible part of the: ~:~
electromagnetic spectrum for three types of glass: crown glass. acfYlic, ··~.i;,:
Q I I
and fus00 quartz. •, (. · .~

I
I
I
I
. . ..:.~~: ~
I
I
I
·1
. ··.:::.:.!~
P- 21.30. The ind f . · ,.· "\~~
,, .
I
I ex o refraction of red light in wati:r is n "Y ~~
1.3~ 1• and for blue light it is /1 = 1.340. If a ray \)f :.;·~~1
While r I t" · ; ,1;
,, .'g tl enters the water at an angle of incld~nce 0 ~.;~
83
Figure 21.47 • • ~ha~ are the underwater angles of refiacttOl\ f.it,\:~rt~
the: two hght components? .. ;· i, ·::}i}.~
,• · ·:i '"'~

600 PART FIVE I ATOMIC. ELECTROMAGNETIC, ANO OPTICAL PHENOME NA ~·· ·.~~~~)

Scanned with CamScanner


d
21 .31 • ' \ v •
l'S h IS an indl'\

or rd'ral·tion for l>lu
• .
1· I
e ig II at A ,.
P-- .i : l) 0111 ol /1 • 1.650. and tur rl·d !Hu "t , 60 lli111: To Fig. [21.14] add the object and image of
-. •• • • . C:- " " ... •)O nm
it is 11 :11 I .h 15. I I a ltg.ht ray contain in I! tll . . .
. ~ C ~c t\\ o col- heights /i 1 and /,0 , rchpectivcly. Then u~c trigonometric
1,urs i · inl·1d.:nt at an angk or 30° 011 thl· ., 1• . . . con~iderations along with [ q. [2 1.9] to arrive al the
. .1ss. \\ 11a1 ts
~
th~ anl!k

bet\\ ccn the two ltght rnmpoii 'nt · .d
c s tns1 e th1:
desired rc~uh .
clJS ? P-21.34. Show that the maximum angular magnification of a
2. ·u~inl! thl..' l:lw of refraction in E11. (1 J 8] a·nd ti magnifying gla~s is achieved when the image is fonncd
P-213
. · • .. . . .... - · 11..' mall
:ingk appro~ 1m~t1 on . prO\ e the re~ ult of Eq. 1. at the near point Ju instead of at infinity, and is thus
12 9
rcbted to rdract10n :icruss a sphl·ril·a] intcr~a Cc•.I 11 US-I given by the expn:ssion mm.. "' I + .1,jf
1
trJlCd in Fig. [- I. I-I) .
P-21.33· Pro\e the result for magnification in Eq. [21. 10). That
is. how th:it:

ANSWERS TO CONCEPT QUESTIONS Concept Question 21.2: (e). You confirm this result with
Table 21.4 and Eq. [21.11 ].
Concept Question 21.1: (d). In (a) the light ray from
othrough F to the mirror violates the law of reflection; Concept Question 21.3: (d). The same object is required
in (b) the light ray from 0 through C does not reflect to make a relative statement at two distances.
otT the mirror; and in (c) the light ray from 0 to I con- Concept Question 21.4: (d). Note that s 0 is the near point
tributes to the image before it has been reflected ofT the of the standard man, not of any particular person using the
mirror. microscope.

CHAPTER 21 I GEOMETRIC OPTICS 601

Scanned with CamScanner


..... .- , ... .,_,:.! •"'-•- "'er ,, t•..
•~~ ,. · t'fe1cn(..C ,.
v'' 1.... ,..., o ir.d1ccs Solution
. ; -:. ' 4'~ . ,~.JI. ~
• - ....... • u ... d
r! 1 :~·4;(.•;rr , <>.;:~. 1:ie u-.c n " n2 111:. :n:ed m (a). an . the rofrcic ting lntcrt:lco la flat Dnd the
In th•~ prob1em. I h I" t ·
. . th denser medium, wh C "' t 10 m0d1um With
··~ c.\~ n, ... n1 1!:_-·:.i!::d r; 1 ( b1. .....- .sin e Th'
f:i lt'~. ·.rr.1 ,, J,q (21 <fl. v.e i:;uodoce a formula t~t
uu-r--:
the h tghef I
'ndex of refraction. 1s ca~e corre:lpof'd'> to
.
e apply Eq. (2 1.1OJ with " z • 1 (for ail'} llnd
~~~n ~ t--t i,-, ux-..
• ,..~ ~~ rr;;:bfJffJCZltun for the image ~1ze Fig 21.1 5(a)· W d t f' d
,,, ,,,, r.; (f,;~ u1 ~.e-:v.i h,; n, "' 1.33
' (f water) and I' = d In or er o m tho Image,
or
distance q:
M• !:J_ r: - 12_ ~ (2LIO) n
2 p ., - ·-
I
- d ,.. - 0.75J .
ho n1 p 11
,,,
... - - I .33

·n~ f'..!rmn:ori ,,fti1i' formula i~ quite ~traight forward Thus. thO apparent depth of the fish Is three-quarters Of
r;~:.-~ r:ie k 11t,f refrttl.f11m and at.mall i:ngle approximation
its actual depth.
1,61 " u h!1 1f11 t/f'j~ clme 111 the upt scal axis. and is
k! ~~ 11ri e-u-1c1~ YCJ".I can d 1) a ,;mplc experiment at
f ,111t',1; II/ rll~tr;rtc 01111 J-<1 f2 1.11J) indeed a ll u~s fo r a
m:f;J.J•1f 1l111µm r.f t1hj~~1~ ~~ri acr~'{ a ~i ng le t.phcrical
,,,..e-rf;,,-/.! < t..1 11ft Ifie tJt1Wm 1 C1ml t<1p lid<1 <•f a food can. 21.3.3: Thin Lenses
I 1~ t1 vrtet 1r1.e erJd 1>I tk <..t.tn with tr<Jn<;parcnl plastic Lenses are transparent objects with c.ith~r two partially
'11ti.p I'"
'/'"Jf k1td~!:"H "'"k with water and drop a penny spherical refracting surfaces or a ~ombmatton of a flat and
1:1 o~ ~'~''-' h1~li rt~ " " " td•w1 the watc'r surface with a spherical refracting surface. Ltght passes through both
di":' f}lif~ l f{, 'llfiff' 1.hwn . ., he water rrcc;~ ure forces the surfaces before an image is formed. Lenses come in many
µ1)f'O. ' ... 'llf!flJ llJPttHd. v. hid1 <.reale\ a ctphcritally shaped shapes. as illustrated in Fig. 21 . 17. They arc characterized
t1r11: rfit1...t!. If you r11r"11 11~r..e 1 'te the pcm1y tl11ough the can,
by two physical parameters:
tJ'"' will ~~ st ''"';tmlle(I
• Lenses arc grouped as either converging lenses
[Fig. 21.17(a)] or divcrJ:ing lenses [Fig. 21.17(b)],
£¥.AMPLE Zl,:J based on their effect on incoming light rays; and
A t,11."1 f~, to. &1lttflfn1t1-J tt1 nd<JfJfh ti 1,ofaw the surface of
1

II f~(l.Jlll, M &1101111 In f!lg. 21:10. Wltat lg thei af)parent • Lenses a rc distin guished a s thick lenses or thin
<w.J.11v~ ti~ ff'-.J1~~ ¥Jl1MY1 rJ1r&1,fly from t11Juve1 kn ~c ~. based on their th ickness in relation to other
lengths. such as the object and image distances. A thin
lcnc; is modelled hy a single n: fracting plane that com-
bines the contributions both surfaces.or

I 'f
_J

I luoro 21.11 v1 i "" ,Ming


tlfjl111J 11,19 f\ ll!iii b 1A~111vud 1tu111 11 111mlllo!I wi1tlt11llv '"' 11tcait. I Ii "8. Which nt o·': ~1119 IYPO!l ul C:tlin1t11111ly U~(>tl lcnsi\S' \\I) ~·v~.~
~.; th) t11vorulnl} tnu• I lokoi nl tho OIJllcnl tUl!l lhnn townn1 \lit' r1l9eS..~
AIO lh11 101111c ll!J(ltl, Whlt h nro l1lll111n1 nl lhu O~lliCill l\XlS The 11.:isncdlil
' t vu 011lh.nl Ml's

fl/\llj 11\/t l /\ 11Jf11L t trt ffllllAl\(, l~f I ll M~ tl IJli ll t l\l t1llCNOMtNI\


5er 6 • p

Scanned with CamScanner


this textbook. we limit our discussion t ti .
In . . o 11n h:nscs
ore often choose converging lenses f light source (object) at infinite distance. The focal point is
:i:id 111 . •d. . . ~. or examples
, discussion ol iverging lenses is entirely analot.!ou . the ~oint (if it exists) at which all these light rays intersect,
fb• h and therefore duplication of the p . ~ . s, tha~ is. where they form a point image of the light source.
thoug · . resentation
. rni1ted without 1oss of generality. \Ve d T~1s approach is sullicicnt to define a focal point F for a
I) o . . I . o need to
· ider diverging enst!s when discussing ey• d •1• lh111 lens if the light rays travel not too far from the optical
con~ o· . . . ~ e e ects axis, as chosen in Fig. 21.18.
in the chapter. 1scuss1011 ot thick h:nses , Id
!Jter . . vou 1ead
m:ithemat1cally compltcated formulas but d •. Fig. 21.18 illustrates the thin lens simplification: a
co .. I. . h 01.:s not dashed line is drawn perpendicular to the optical axis at
·bute add1ttona ms1g t. Also. applicatioi1s f 1
contrl . o enses the centre of the lens. allowing us to limit the ray refraction
. the Life Setences are based almost exclusivel
1n · Y on t h"111 lo a single step in geometric image constructions. We
ienses. .
~sc three light rays to determine the position and size of
We first reintroduce the concept of a focal po· t ti
. .., m or a images, as we did in the case of mirrors:
thin lens. Fig. _I . 18 shows several parallel incoming light
rays as in Fig. 21.8.(a), where we defined the focal point • a light ray travelling along the optical axis,
for a mirror. Incommg parallel light rays correspond to a
• a ray incident on the lens along a path parallel to the
optical axis, and
• a ray incident on the lens after passing through its
focal point.
We show how this approach allows us to construct
an image for an object at finite distance from the lens in
Fig. 21 .19. The object 0 of height h 0 is placed at distance
p to the left of a thin converging lens. The image is
constructed by placing one end of the object on the optical
axis. Two light rays are followed from the upper end of
the object. The ray parallel to the optical axis is refracted
through the focal point on the right side of the lens (the
side of the lens opposite the object) and passes through
/--~ .., the optical axis at that point with an angle 0. A second ray,
Figure 21.18 Focal point for a converging lens. Parallel incoming light emerging from the upper end of the object, passes through
rays from an object at an infinite distance converge at the focal point F the focal point at an angle <P on the left side, which is the
located at a distance equal to the focal length /from the lens. The lens same side of the lens on which the object is located. This
is athin lens, which is indicated by the dashed vertical line at which the ray becomes a light ray travelling parallel to the optical
refraction is drawn in a single step. axis beyond the lens. The light rays that emerge from the

F"tgure . b. ct 0 at distance p from a converging lens of focal point at F. Note that only three
rays ar 21 ·19 Construction of the image I at distance q tor an. 0 ie is from the bottom end of the object; (2) a ray travelling from the top of the object,
llaraJiete needed to find the image size h1: (1) a ray along the ?p~ica d~ ) a ray travelling from the top of the object, through the focal point, and refracted
Paraii to the optical axis, and refracted through the focal point, an
3
el to the optical axis.

CHAPTER 21 I GEOMETRIC OPTICS 581

Scanned with CamScanner


,_•

dcriYcd in thi~ section for conYcrgil'lg ·•


All fortl1U Itl f'. Icnscs
' • a' s we II. . 'Al St"t of
. <l i vcrguig
lens~s 11pplY, to must tic followed, a~ summarized 111 ,-.
l'i~n convc:nt ions
Tobie 21.4.

TABLE.~.1~4------·'- - ---.-...
si;~~onvcntlons for thin lenses - - ---- .. _,,___ ...::
p Is positive
. 1· Obj; ;·t-ls in front or the lert~~ · ·~.
. ---- - ... . - - -·-·~ -· •.. -·--·
· ..J, ,
·· ··· · ··- • . ... .. · .Object is behind the lens. - ·• -1
p Is negative ......--~-· ... . _ _......... - :-- -- - ........., •.•• ~ _
·- · ... ··,--· ·
q Is positive .
I
Image is behind the lens.
··--- .... - --· . .. - ~ . - -- . • . - · ...,
...
~·-1

-- - -- -- --~· .... - ----- ~--, Image is in front of the lens-. .... ~'

q Is negatlv~--- --·-· --- ~- -- · _ ··--- - - -- --.~--.,_,,...


R. . I
- - ·· ~j ·re positive Centre of curvature for each
- .~
- q t:mo q R, an 2 a ; surface is behind the lens. •'

(21.11) '· >,


I' lUntr = -µ· ·------ ci Ra~·;;tiv;-Tc;~!r; ~f~~rv~i~;;;;-;; -·-· . <,

R1 an 2 surface is in front of the lens.


\\hi.:h is the ma\!niih:-:1til)I\ of the kns und. <.'onvr.:nicntly, ··,· -s·-
is the ~1m~ fonn~1la that we l'bt:1i11cJ for the magnilication 1 p~itiv~ -- - -- --·-- Co~~e-;gi~Q"j~~;--- - - ...- .. ----,...
+---- ·--------·- --- ----
of3 1;."\.)n~a\ e- mim)r in Eq. [21.5]. r l ;;egative Div:'.gin~-l~n~-- - -- -
Further. usith! Fi~. ~ l . l 9. we dcn:lop a rdation
These~;~e~~i~~~~~-:S:ci wh~n the ~hin-len~ form~~~·--
br:twe<n the Yario~ Ji:t~mces rdevant for the rcfraction the lens magnification formula are applied. R 1 1s the radius of
of a kns, p. q. and f Tl) llo this. we start with the two cul'\lature of the front surface of the lens, and R2 is the radius
tri.'U1~les I:t~lkd ~, imd u~ in the figure. 6 1 and 6 2 a~e of cul'\lature of its back surface. These are used when the lens
similar. sin1.~ both contain the angk ti. From the geometric maker's equation is applied as described in the next section.
relations for these two triangles. we find: ,1
..•
·~

. . •';1
21.3.4: The Lens Maker's Equation · · ,·i
We can approach the quantitative treatment of a lens in · ·~!j
a second manner, starting with the results for a single · ·~A
spherical interface. Fig. 21.20 is introduced to quantify +.~
the optical properties of two consecutive refracting sur- . <]
which leads to: faces. The first interface separates material 1, with index · ., 1
,,, q -f of refraction n 1, and material 2, with index of refraction ·~.J
-=-- n2• The two materials are chosen such that this interface '.:>.]
ho J' alone does not allow the formation of an image on the .. ;::
Combining this equation with Eq. [21 .1l] we find: right side. This means specifically that the change in the ~..
index of refraction at the first interface is not sufficient to ,J
ca~se diverging l~ght rays from the object O at distance •c;,~~
f!1 m front of the mt~rface to converge aft~r passing the ·};~
?terface. A second interface, from material 2 to mate- ., -.,;,
which is rewritten as:
~ial 3, with i.ndex of r~fraction n3, is needed so ~hat an '-.~-~1-·.:..•·
q q m_age I at distance q 1s formed. Note that the d1stanee · · ~
-=--1
q ~ defin~d with reference to the same point along the ~ ·~~~·
1
p f
and finally leads to:
~ptical .axis.as the distances p, p', and -q' . This is a g~ ~;:~
pproximat1on for the case of a thin lens where the thick· · .,.d1
I I I ness of material 2 is negligible when compared with P· .:.~~
- + - = -. (21.12) q, R1, and R2• . '.~-.;.~
p q f
We now develop a formula describing the relations . ('i·~
· ·
Eq. [2 l.12] is called the thin-lens formula. Note ti t
h r la :m~ng the relevant paramet~rs in Fig. 2 l.20. For this.we~.·:::~:~
e same 1ormula we found for the spherical ·
11 ts t
.m Eq......
[" 1 J] mirror PP y Eq. [21.9] at the first interface: . · /~~I

folijt@I The fonnulas for the magnificat1'0


. . . .n an d ior
r
the
~ + 112 = n2 -
P q' R1
111
.,,::;.·:z;:tij
• .
:,Jiil
relation between focal. object, and image lengtlis ~ ~~ ~ ~ t~
. . ~ · arc t 1tc .' i..

0.~ -. , ;. I'.
same fior spherical mirrors and thin lenses. where q' is the . . h i:
in Fi . 21 ima?e distance, which is n~gative~s s .rs\ >~~1l~
· · R1 ts the radius of curvature of the . · :..:; ...~,
20 6
g
582
PART FIVE I ATOMIC, ELECTROMAGNETIC, AND OPT
ICAL PH~NOMENA tlf~ ··:~~:.,
.
~~-.?i.~;.:.~

Scanned with CamScanner


-~

21.3.5: Refractive Power (a)


It is interesting to note that the left-hand side of Eq. [21. 12)
connects the object and image distances of a lens in the
same manner as the left-hand side of Eq. (21 .9); the right-
hand side is therefore a measure of the ability of the lens
to refract the light. Using this observation, we define the
refractive power of a lens ~t as:

(21.15) (b)

Eq. (21.15) combines the effect of two refractive


interfaces; for single interfaces the refractive power was
defined earlier as D.n/R, with R the radius of curvature of
a spherical interface. Eq. (21.15) is consistent with that F
definition so that the combined refractive power of two
spherical interfaces, which is the sum of two D.nl R terms,
is equal to the inverse focal length of the lens formed by Figure 21.21 (a) Sketch of an image I formed for an object o located
the two interfaces. farther away than the focal length. A dashed vertical line represents
the thin lens. The three rays shown allow us to construct the image.
13i1Qoi!lii The refractive power of a thin lens, ~l in unit (b) Sketch of an image I formed for an object O located closer than
diopters (dpt), is equal to its inverse focal length. the focal length. Again, three rays are used to construct the image.

This result is illustrated in Fig. 21.21(b). The negative


EXAMPLE 21.5 image distance represents a virtual image, that is, an
A converging lens with f = 1O cm (a lens of refractive image on the same side of the lens as the object. A
power 9t = +10 dpt) forms images of objects at (a) 30 cm, positive magnification M > 1 means that the image is
(b) 10 cm, and (c) 5 cm from the lens. In each case, find enlarged and upright.
the image distance and describe the image.

Solution
Solution to part (a): Substituting the given values in the
thin-lens formula, we fi.nd: CONCEPT QUESTION 21.2
I I I
--=--+- In Fig. 21.21(a) you see an object 0 and its image I that
!Ocm 30cm q'
formed for a converging lens we represented by a vertical
which corresponds to q = +15 cm. Further substitution in dashed line. Which of the following quantities is a negative
the magnification formula yields: number in this case?

M = _ fJ_ = _ 15 cm = _ 0.5. (a) object distance p


p 30cm
(b) image distance q
A positive q value means that a real image is formed (c) focal length f
on the side of the lens opposite the object. The image
is half the height of the object and is inverted due to the (d) refractive power of the lens m
negative sign of M . This case is shown in Fig. 21 .21(a).
(e) magnification M
Solution to part (b): Replacing p = 30 cm with p = 10 cm
in part (a) leads to q = oo. Note that this case is equivalent
but with reverse light rays to Fig. 21 .18, where an object at
the focal length has an image at infinite distance.
Solution to part (c): In the third case, the object lies
inside the focal length. Replacing p = 30 cm with p = 5 cm
21.4: Applications in Optometry
in part (a) leads to: and Ophthalmology
I I I
- - =-+- Two main applications of the ray model of optics exist in
!Ocm 5cm q'
~he ~ife Sciences: vision and microscopy. We discuss both
which yields q = -10 cm. Further substituting the new m this textbook, beginning in this section with the healthy
values in Eq. [21.11) leads to: eye. We then proceed to the most common eye defects
q IOcm an~ their corrections. The discussion of light microscop~s,
M= - -
p
=- - = +2.0.
5cm which allow us to see objects too small to observe with
the naked eye, follows in the next section. \
584 PART FIVE I ATOMIC, ELECTROMAGNETIC, AND OPTICAL PHENOMENA NEL

Scanned with CamScanner


zt.4.1: The Eye
. 21 I is a sketch of the side view c.:r . .
f1J;· . . ~ ." . oss-scct1on of ,
it)'cb.111. The optically active parts - "
hun1:1n o 1 t11c eye .
rnea :ind the lens. The cornea c.:or t' . arc
th!! co I .trns a con
rn :il intcrface (facies extema) and a c , . vex
e;\te . . ,. ' onc,1vc internal
. .-3 ce tfi1c1es mtuna). It has a small r· . 1 · •
intt~n· ' . .1urus of curva-
of about 8 111111 and bridges the biggest .,. 11•• .
rure . . ur crcnce 111
. dices of refraction. lrorn 11 = 1.0 for air t0
in I . II = I 33
hich is close to t 1e value for water. Thus th · •
" b. ti . . • e cornea Clo~e vision
provides th.t: rgl gedst fi r~c_tron ol the refractive power of
the eye. Usrng .
t 1e e 1111t1on of the refract iv,
fl . e power for a
spherical rnterface ollowrng Eq. [21. 9) we have:

, _ lln _ 1.33 - 1.0


18 + 14 dpt
.Hcomca - R - 0.00 8 111 = 41 dpt.

Due to the variability of the curvature of the c


'll - 40 d . ornea,
a value o f · cornea - pt rs generally adopted in the
literature.
The lens is suspended by fibres (suspensory liga-
ment of the lens, or zonu/a ciliaris) that are stretched or
loosened by ciliary muscles. The iris defines the opening
of the lens. allowing light to pass through only the vis-
ible area of the lens, which is called the pupil. The iris
can vary the diameter of the pupil to adjust the total light
intensity reaching the retina. We do not emphasize light-
intensity-related issues in the current context since the
related topic of sound intensity is extensively discussed
in a previous chapter. Figure 21.22 Anatomy of the lens, ciliary muscles, and ligaments of the
The lens is a transparent, pliable, biconvex body human eye. Note the changes of these three components between close
with an index of refraction of n = 1.41. The elastic varia- vision (top half of figure) and far vision (bottom half of figure).
tion of the lens is illustrated in Fig. 21.22, which allows
for a change of refractive power between ~H = 18 dpt and which results in a focal length of less than 1.4 cm. The
~ = 32 dpt. When the ciliary muscle is relaxed (lower process of changing the refractive power of the lens due
part of Fig. 21 .22), the suspension fibres of the lens are to ciliary muscle action is called accommodation.
stretched and the lens is elongated. This leads to a flatter The ability to view objects close up deteriorates with
surface with increased radius of curvature and reduced age due to the sclerosing effects ofthe lens. Physiologically,
refractive power ~H. When the relaxed eye looks at an this is quantified by defining the near point. The near
?bject at infinite distance (approximated by p 2: 60 m), point is the shortest object distance p for which the human
rts focal length is calculated from the two contributions eye produces a sharp image on the retina. Table 21.5 illus-
~o the refractive power of the eye (see the right-hand side trates the change of the near point distance with age.
in Eq. [21.13]):
TABLE 21.5
~!relaxed eye = ~)lcomca +~Hiens= (40 dpt)+ (18 dpt) Near point as a function of age

= 58 dpt, fAge (years) I Near point (cm) I


10 7
Which results inf= I. 7 cm.
20 9
While focusing up close, as illustrated in the upper
Pan of Fig. 2 l.22, the ci liary muscle is contracted, 30 12
relaxing the suspension fibres and allowing· th e lens to 40 22
contract in response to its elasticity The surfaces of the 50 40
~Ontracted lens have smaller radii of.curvature, and thus a 60 100
arger r .. .
Ciractrve power: 70 400
>75 00

mfoc .
U~td eye
= 'H
' cornea
+ •\H Jens = (40 dpt) + (32 dpt)
For the standard man, a near point of s0 = 25 cm is used.
== 72 dpt,
CHAPTER 21 I GEOMETRIC OPTICS 585

Scanned with CamScanner


- ,, Th is is a typical effect of old a..
hen rcad t 0 ~ ·
\lo b I. ~. b-.t
2 1.4.2: Eye De fects and Diseases ..v-cur when the C)C al 1 too ishon . T~ -
m.3) a 1so v .... • • . v: t <
. · done ,, ith prescri ption g 1a ssc ... , a\ 11i,,.... ..
1 .-. .; ..: .r-::-.•. ~ : • •_._._ _ ~-1 ::: .z ~ ~ <l f~"t.5 are- c~ussed in recu on r . --a.~
. . ., '?J(c ). The correct1\e le n\ rs a CO:t\e,
~ ·-. - ~ C.~...!; . i.~ eu:r:;: i ::-,-v l.-b.YJ. (rrescriptron C)<g~ rn f 1g. - 1·- . 'P 1~1
• · • 1· .... e refracti \ e po " er . 1 tu add to th
~·"! :_;.....,: 1:.:- .:• rr~"1.."t • :or. <kfi~•c:T"-~<es. " ith a p-0s 1 • . 1c tQo
small refracU\e power of the e~e . As '.ndrcated in tfit
Hyperopia (Fanigh~ness} he trlasses caus e a n apparent s hift of the ob·
fiaure. t ~ . h. h h d JC..'l
~
H;.per-q:u . ~ c::·.: defect l!SSOC'ia:...-d \\ith an insuffici~t to greater d"..,... '"nce • a distance a t \\ 1c t e · efccti\ c C)c
e~~ ;:r.:- o '.'" tt-= t~.s. 1~0 to an incomplete re5hapmg is a ble to see it clearly.
..
... :n ~ 'it!S:::et'.St{r.l fibres of the r~ are rela..,ed- A5
a resu!l, ~ r:r..a..U.."!fu:n refracti\ e po'-\ er o f the kns of
~ = 3.: d;ot is r.ot re:?ciied.. a:td the eye Clil!lo< form an
i::t2,:e of ne-a..t:- obj~u on th~ m ina. EXAMPLE 21.6
H~~cro;>ia is. illustra ted in Fig. :? 1.23. Part (a) Thenear point of a particular person is at 50 cm. Y.Jhat
st:o-~ s t.b~ o p:rcal p roµeni e-s of a hy~ropic eye \\hen focal length must a corrective lens have to enable the f?ye
obsen ing a n obje1:t a t g reat distance. T he ci liary to clearly see an object 25 cm away?
rn~k is rel:?.:u~d. the s uspens ion fibres and the lens
a.--e stretch~ ari.d an image is formed pro~rly on the Solution
retir..a. Thl.!5. the pers-0n is called farsi g hted. Pa rt ( b) we use the thin-lens formula wit h an object distance of
il!~tra!es the p:obl~ o f the patient whe n focusi ng o n =
P 25 cm. The lens w e want to prescribe m_ust form an
a n~a..--by obj ect. "' h~n the cilial)' musc le contracts a nd image on the same side of the lens as the ObJect but at a
~ Ier~ should relax tO\\ ard its most sp herical shape. distance of 50 cm. The eye of the person looking at that
If u~ object is closer than the near point, the image is image then sees it clearly. Due to the sign in Table 21.4,
form~ behind the retina. If this near point is too far =
we write q -50 cm:
from the eye, the patient has a problem, for exa mple,
1 I I I I
- = -+- = -- + - - .
f p q 0.25m - 0.S m

This leads to a focal length off= +0.5 m. 9t = 1/f = +2.0 dpt


is the refractive power of the prescribed glasses.

\
0 atp = x Myopia (Nearsightedness}
Myopia is an eye defect that is due to an insufficient
stretching of the lens when a person tries to obtain a
lower refractive power of ~)t = 18 dpt. Myopia is illus·
trated in Fig. 21.24. Part (a) shows the eye trying to
observe an object at great distance. The lens is not suffi-
ciently elongated and thus the image is formed in front of
the retina. The same person can see an object at the near
point of the standard man (at 25 cm distance) without
0 atp < Pnc.r
any problem as the elasticity of the lens is sufficient to
reshape the lens to form an image on the retina [illus·
trated in 21.24(b)]. Fig. 21.24(c) shows how myopia is
corrected with prescri ption glasses: parallel light rays
reaching the eye from an object at great distance are
refracted away from the optical axis such that they form
an image I' at a point closer to the eye. This image is
observed ~vith the myopic eye. forming the final image
0
~ the retma. Note that, while hyperopia was corrected
wit~ a convergent lens. myopia must be corrected with
a divergent lens in order to lower the excess refractire
power of the eye.
Typical causes of myopia are elongated eyeball:> N
figure 21.23 Hyperopia, or farsightedness: (a) far vision, (b) close weakened ligaments and muscles, for ex;mple. due to di3•
vision, (c) correction with prescription glasses. betes mcllitus.

;e6 PART FIVE I ATOMIC ELE CTROMAGN ETIC. ANO n o Tt rA• -· · - ·

Scanned with CamScanner


EXAMPLE 21.7
EXAMPLE 21.8
certain person cannot see objects clearly when they are
P. d 3 distance of 50 cm. What focal length should th An artificial lens is implanted in a patient's eye to replace
::::ibed lens have to correct this problem? e a diseased lens. The distance between the artificial lens
and the retina is 2.8 cm. In the absence of the lens, the
image of a very distant object (formed by the refraction of
soiution . . the cornea) is formed 2.53 cm behind the retina. The lens
we chant to enable thedistance
oose the ob1ect as infinite 17 _ s·
' - ' mce
eye to see anything beyond
00 is designed to put the image of the distant object on the
we W
· ob'1ect s very far away. The image of the retina. What is the refractive power !I! of the implanted
50 cm. including lens? Hint: Consider the image formed by the cornea as
prescription lens must be on the same side of the lens a virtual object.
as the object and cannot ~e further than 50 cm; that
rncans q = -5.0 cm (the negative sign results from the sign Solution
con"entions in Table 21.4). 1.f the len~ accomplishes this,
Following the hint, we consider the image formed by
then the eye can look at the intermediate image and see it
the eye without the implanted lens as a virtual object
properly. The thin-lens formula reads:
for the implanted lens. To later use the thin-lens formula
I I I I I to determine the focal length of the implanted lens, we
-= - + - =- + - need to determine the object distance for the implanted
! P q 00 - 0.Sm'
lens from the image distance of the cornea without a
tens. The image distance is 'I = (2.53 cm + 2.8 cm) =
which yields for the focal length, f = -0.5 m. A negative
5.33 cm, as shown in Fig. 21 .25(a). The two lengths
focal tength means a concave lens must be prescribed
are added since the original image is formed behind
(based on the sign conventions in Table 21.4). The tens
1he retina, that is, farther away from the location of the
has a refractive power ~I = -2.0 dpt.
missing lens.

(a) (h)

(3)

0 ar /' = o:: Figure 21.25 (a) Eye with a surgically removed lens. The cornea
(light red crescent) is the only focusing component of this eye with
Insufficient refractive power forming blurred images behind the
retina. (b) The artificial lens Implanted into the eye must correct
the position of the image such that the image is formed on the
(b)
retina.

When the new lens is implanted, the image distance


calculated above becomes the object distance for the
implanted lens. The object distance is p = -5.33 cm,
0 at /1 = /'nr.ir
where the negative sign indicates that this is a virtual
object, that is, an object that appears behind the lens. The
implanted lens must now form a final image on the retina,
as indicated in Fig. 21 .25(b). at a distance If = +2.8 cm
behind the lens. The thin-lens formula yields:
le)
1 I I I I
- = - +- = + --
! fl If -5.33cm 2.Rm'

--- which allows us to calculate the focal length as f =


I' +5.9 cm. Using Eq. (21.15], we find the refractive power
of the implanted lens to be :H = +17.0 dpt. Note that this
is a much larger value than a typical refractive power for
a prescription lens, but close to the value of the natural
lens In Fig. 21.22.
~9ure 21 ~4 .Myopia, or nearsightedness: (a) far vision, (bl close vision,
)COnection With prescription glasses.

CHAPTER 21 I GEOMETRIC OPTICS 587

Scanned with CamScanner


23 . ..

Diagnostic
Ultrasound
Imaging

eer's Jaw describes the absorption of sound inten-

B
Ultrasound applications in medicine differ from sound
sity as an exponential function of the distance of a perception in physiology, as discussed in a previous chapter,
wave from the sound source. The absorption coef- not only in the relevant frequency range. The discussion of
ficient depends on the medium and the frequency of the audiology focused on waves in confined media and their
sound. resonance phenomena. Concepts important in the medical
Waves reflect at interfaces. They travel in a different use of ultrasound focus more on absorption of a travelling
direction after they pass an interface at an angle (Jaw wave, and sound transmission and reflection at interfaces.
of refraction). At most interfaces, both reflection and The current chapter starts therefore with the introduction
transmission occur; however, the transmitted intensity of additional fundamental concepts.
is greatly reduced when the two media adjacent to an
interface differ significantly in their density and/or speed
of sound. The combined factor of density and speed of 23.1: Sound Absorption
sound is therefore defined as the acoustic impedance.
In ultrasound imaging. a transducer generates a sound The sound intensity reduction in Case Study 15.5 is a geo-
that travels through the skin into the tissue along a ray metric effect. The sound travelling through a medium can
that is called a line. It reflects at various interfaces and further diminish due to energy loss to the medium when
is detected as an echo when it returns to the transducer. the gas pocket vibrations are not perfectly harmonic. This
Several parameters have to be considered for ultrasound effect is called sound absorption because sound energy
imaging applications. Sound absorption limits the depth to is absorbed by the medium. For example, if the medium
which tissue can be sampled. The echo delay time plays a is air, the absorption is caused by vibrations of air mol-
role in the total time the acquisition of an image will take. ecules that are not perfectly adiabatic. Thermal energy
It detennines the maximum pulse repetition, which in tum loss occurs, which slightly heats up the air through which
d~terrnines the frame rate of imaging when combined the sound travels.
For a quantitative description of this effect, we study
With the number of lines that constitute the image. The
a planar wave travelling in a one-dimensional gas column
~pt~ resolution (that is, the independent o~servation of that is aligned with the x-axis. This eliminates the geometric
0
interfaces at different depths) depends inversely on
the frequency. sound intensity loss we discussed in Case Study 15.5,
allowing us to focus exclusively on the absorption effect.
The Doppler effect is the change in received frequency
When eu· her the source or the receiver move relative
· to the August Beer observed that the rate of loss of sound
sound . · · intensity along the x-axis is proportional to the intensity
th -carrying medium. Jn Doppler ultrasound d1agnos1s,
itself at every point along the axis. We want to follow
· e.s~ed of blood is measured for the echo received from
Beer's further reasoning carefully as he formulated this
::tvidual erythrocytes. The distribution of speeds allows observation in equation form.
vc health practitioner to determine, for example, blood First. we cannot write for the rate of loss of intensity
th ssei stenosis that results in turbulent blood flow near along the axis Af/ tu. If we were to do this, we would average
c constriction.

CHAPTER 23 I DIAGNOSTIC ULTRASOUND IMAGING 625

Scanned with CamScanner


We ·s·1w · a pr1.'\ 1ous
· QIUIC[I Thrt'C pliy•dr a l principl~·ii ,·u11ti ihui ,
. · 111 chapter that the sound intc:nsity · · (II ti •~ 1· • • 11 011;· t11 lb1:
1s. pro~nni~nal to the square of the amplitude of the gas ultrJ"11un'1 1111ag~. rl·I il t I' \ ~• 'm:c111111 111Cl1ln
· I I
vibrations Ill the gas column; that is. I ex A~. Thus, we- rdkrll'd. und 1 n111~n1ttt\'l ~1111111 w n H ·s . {II) lhi' r . n.
can cxp:u~d on the displac1.·me11t by using Eq. [23.2] for iutcn~itks llf tht' 1h1l'•' waws; and ( I ll) tit" 11 ~~ 11~. \'l~~''.o
1
an absorbltlg medium in the form: atlt'llUUtic111 llf tlH:St' WU\'t'S. l\'i b1·

D = Ac- 'Ytsin(wt - k.r), (23.5) We have ;tcldrl'SSt..'d attem111ti1ln in the 1, . .


. • I~\ l\l \U
S1..'Cti11n; in the c·urrt..•nt Sl'l'llOll \W 1\1\'.ll\ (Ill\\ llVt' llir . .
in '~·hie!' 'Y is the decay coetncient for the sound amplitude, . · · ~( llOft
and then on "aw 111tl'ns111cs. ,,
wh.1ch is ~los~ly related to the absorption coctlicient {3.
This function 1s shown in Fig. 23 .1. 23.2.1: Relative Directions of Waves:
Huygens' Principle and Snell's law
(a) D
The propagati~m of wavt..·s •. inc.lu<~i ng pl~11'.1r w11ws, ,.31\ I\'
constmch.'d w11h lluygcns pnnc1plc. ( hmtiuan lluvg
introducl'd in the late 1600s the rnnct..•p1 c1f 1111 ek· 111 ,,: 1en~
a1v 11
spherical point sourn!. 11.e sl111wcd llu~t \\'\,' c·1111ld lbrn~
any wavefront at a laler t1111e by alluw111g a spht..·ril·al \iavc
to cml..'rge from 1:ad1 point of the wavefront at nn carlic-
instant. We use this principle to establish 11uantituliH· rela~
tions for rctlection and transmission of sound waves at a
-Ao- - - L_\ _____ J; __ ~ _Ji _
flat inkrfacc.

law of Reflect/on
(b) D Wave reflection is a familiar phl'nomcm>n for light nnil
sound waves, for example, light reflection from tht• sur.
Ao ' face of water or echo formation in mo1111t11i11s. The la\\
'' of retlcction for light was introduct'd in tlw chaph.·r 011
'' Geometric Optics, and here we extend th1.~ l'1lncc·p1 to the
' '
case of sound waves. We use Fig. 23.2 to rl'latc the 1111gk-..

,. ,.
,,. ,,.
-Ao /

Figure 23.1 Free and damped planar wave. The free wave has a con-
stant amplitude. In the damped case (bottom), the amplitude decreases
exponentially from its initial value (following the envelope function that is
included as a dashed line). ·

23.2: Reflection and Transmission


of Waves at Flat Interfaces
In ultrasound imaging we obtain images of organs and
tissues within the human body. These images show ana-
tomical features, such as the position of the surface of a
specific organ. Such features can be visualized because
ultrasound waves sent into the human body are reflected
Figure 23.2 Geometric sketch for the Jaw of renection on a flat lntDrlace
at interfaces between different tissues. The reflected wave
based on Huygens' principle. A planar wavefront approaches tile JnterfacO
(called an echo) is detected with the same device that
~nder an angle Q,. At time 1 = O(lop sketch) it reaches point A1 at tllO Inter·
emitted the initial ultrasound signal (called a transducer). ace. In the bottom sketch spherical waves are drawn from points .f i aod
We are interested in both reflection and transmission A2• forming the reflected wavefront w·. Nole that the Initial wavefront W
because the transducer signal has to pass the transducer- h~ just reached point A3 In the bottom sketch, Which applies at time t "' Jil( ,
skin interface for imaging applications. with R a distance defined in the top sketch, and " the speed of soond·

PART SIX I APPLIED CLINICAL PHYSICS NH


628

Scanned with CamScanner


. •·J.·nt and rdkct1.•d sound \\a\•'s Tli, -
11 i 0 ~ 1 ' ' -· 1: 11 i;uf'I!
diff'-·rent medi:t. tabdkd t and II . Fn'm Tnbk 15.1 \H'
tc"'et plJn:ir ,, J \ c:front approaching a tbt intc:rfa·· .
-~ ,,, 5 3
!>"'
. . . d .
. The." interlace 1s eltrwJ b~ the points I
'c; at
, e:-.pect the sp~·cd of S\)llllJ to \ ary i'1l't\\ ~\.'JI lhc l\\ 0 111edi:\
!lft'' 1e (1I b r I • rt •• "ith ' :ilues c 1 :rnJ c11 n:spcl.'.tivdy. Fig. :!3.3 ilJu.;trates the
4-1 ~. " ith A: hall" a~ . et\\ een .-1 1 and A 3' \\'e l·h~'OS-c
4
sr.J: :'. 1time: 1 = O:it the mstant "ht•n the ,, avl'front 1 . c:ise c 1 > ell'
~e 1n1ua ._ •. . . i:is
i;•• c:J roint A 1· Th!:> \\ 3\ c; lrnnt IS sho\\ n Ill the top pan

~3·~ 1 · .., :is :i straight line l:ibl'lkd \\'.


,f fie. --'-- . . .
' Vsing Huygens. prme1pk ." e _constnict the lcx:ation
the " :ndront :titer rct1ect1on m the bottom p:irt of
1
~~i?. 3.~- Th~ figure sl~ows the system at the t_imc instant
- the incident wa\ efront has reached point .i Th ...
~hen • 3· ~
. e!:ipsed between the top frame and the bottom frame
...
-' omf~" ')).:! is. t.1= Ric. in which R is the disl.'.lnce from B3
Ill 1: · -
, 3Jld c is the speed of sound.
· ~":" ro.1y h . I . . II
. ' The elementary sp enca "a\·e emergmg from point
· ~.
' ' I A, in the bottom part of the ~gure has th~ same radius R.
The e!ement:iry ~"a~·e emergmg from pomt A~ has radius
.\.:.• I .. I Is Ric

. R2because the incident wa,·efront reached point A, after


... :·.
time M/2. and it tm·el!ed a distance R/2 in the rem;ining
time. Note that we connect the elementary waves in the
bonom of the figure according to Huygens· principle; this
yields the wa,·efront \V '.
· The construction in Fig 23.2 allows us to use geo-
metric arguments to derive the law of reflection con-
necting a,. which is the angle of the incident wavefront Figure 23.3 Geometric sketch for the law of refraction at a flat
·l~ with the interface. and err which is the angle of the interface based on Huygens· principle. The top sketch shows an Incoming
.. .:. I reflected wavefront with the interface, as illustrated in the wavefront Wat an angle a,. At 1 = Othe wave has just reached point A 1
bonom offig. 23.2. For a quantitative result, we compare in the interface. The bottom sketch shows the refracted wavefront W' at
the two triangles Ll 1 and Lli in the figure. We note the fol- time 1 = RJc., with c 1 the speed of sound in medium I. At that time, the
lowing three features: wavefront Whas just reached point A.i in the interface. Huygens' spherical
waves have been drawn at points A 1 and A 2 to illustrate the formation of
1 The sides from A 1 to A 3 are equal in both triangles. the wavefront \V '.
1 The sides from A 1 to C1 and A 3 to B 3 are equally The top sketch offig. 23.3 is identical to the top sketch
long because sound travels the same distances in the in Fig. 23.2: it shows the instant t = 0 when the incident
same time. wa\'efront has reached point A 1• The wavefront arrives at
y an angle ai with the interface. It will reach point A 3 at time
- • The angle A 1C 1A3 is 90° because the tangent to a cir-
cle is perpendicular to the radius. The angle A 1B3A3
is also 90° because the wavefront is, per definition,
!it = R/ c1 later. This later instant is illustrated in the bottom
sketch of Fig. 23.3. which shows two elementary spherical
waves, one emerging from point A 1 and one from point
perpendicular to the wave propagation direction. Ai· Both elementary waves are in medium II and therefore
These three conditions establish that the two shaded tri- travel slower; the wa,·e from point A 1 has reached a
angles in Fig. 23.2 are the same; this leads to the law of distance of Ru = cu~t, and the elementary wave from point
reftection: Ai has travelled half as far, as it emerged only at time ~t/2.
The following relations are obtained geometrically for the
ai = err. (23 .6) angles of the incident and transmitted wavefronts:
I ~ The !aw of reflection states that the angle of an . R,
I llltrdent wave with a flat surface is congruent to the angle smer. = --
' A 1A3
of reflection.
. Ru
sma1 = - -.
A1A3
I.ow 01Refraction (Snell's Law}
Sound w . f: These two expressions are combined by eliminating the
b aves are not completely reflected at most sur aces,
prut. ~netrate the surface and travel in the new medium as identical length A 1A 3 and substituting the product of
ev1ous1y . . r· 23 3 speed of sound and M for the two radii:
allow seen mthe chapteron Geometric Optics. 1g. ·
to th s us to relate the angle of incidence with the interface sineri R1 c1
is c el a.ng!e of the transmitted wave· the resulting law
a led h •
sina
1
= ~ = c11 ' (23.7)
t e law of refraction. The figure shows two

CHAPTER 23 I DIAGNOSTIC ULTRASOUND IMAGING 629

Scanned with CamScanner


- ~

in ,, hich the index I represents the medium C<lm


Th i ~ 1 r;illcJ Sndl'\ l:rn for \\'tlkbrord Snell. ''ho di - . d -, JL":6
the incident ''a\ e an d t he in ex - represent\ L
t'rcd II in fl<~ I. a~ et"n in C h;ipter 2 1. Geometric
O\ . . d t!.~
medium ca!T) ing the tran mute " a\ e.
Opt1 . h s1..11c:- th;it the sme of the an!!le of a ''a\ efront
"1th 3 fht interface is propon ional to the peed of sound We use Eq. [:!3.8] to eliminate ' 'r in Eq. {23.9]:
in the respecti\ e medium.

23.2.2: Relative Intensities of Waves:


Acoustic Impedance
We consider again a flat interface bet\\ een different media.
for e'\am ple. the interface bet\\een t\\ O difft"rent tissues Note that the first tenn in the las t line cancels " it~ It.~
in ultr.L'Ound im;iging. Fig. 23.4 shO\\S schematically the term on the left-hand side of the equation. \\'e di\ ide eic
acou tic interaction zone (dashed box) at a flat interface. last equation by ,., on both sides to obtain:
At this interface an inc ident '' a\ e ( 1) is panially reflected
(:! )and pmiall~ tr:msmitted (3). To quantify sound reflec-
tion and ound trans mission at the interface. \\e determine
the ratios of the retkcted and transmitted wave intensities
This yields a relation between the ma'\imum transmitt.."'d
to the incident intensity. Two conditions must apply in the
dashed zone: and incident speeds:

This equation illustrates that the ma'\imum gas pocket


,--- - -- ,
l velocities in sound propagation are connected to each
I' I
I
I
other with terms containing the product of the speed of
sound and the density of the medium. Such terms are
I
I called acoustic impedance Z:
I
I
L_ __ _
Z=pc. (23.10)

3
Z has unit kg m- 2 s- 1, which is usually called n~I.
Table 23.2 shows that impedances \'ary widely. with g:ises
at the low end of the range with rnlues between 1 " I0:
F"tgure 23..4 Cooceptual sketch of a wave passing through an interface. and I x 103 rayl, and condensed matter at the up~ end
A1. the interlace section (dashed box). three wave components have to
with values of 1 x 106 to I x 1O7 ray I.Inserting Eq. (23.10]
be constdered: (1) an incoming wave, (2) a renected wave, and (3) a
in the formula for,., yields:
transmrtted wave.

• To satisfy the continuity of physical propenies across


the interface. the sum of the maximum gas pocket
speed of ' ibration in the incident wave. 1·1• and the
m:ucimum gas pocket speed in the reflected wave. ,."'
must match the ma.'\imum gas pocket speed in the
trans mitted wave, v1:
TABLE 23.2
(23.8) Acoustic impedance values for various physiologically
important materials
• The energy passing through the dashed zone in
Fig. 23.4 must be conserved. The energy density trans-
ported in the incident wa\'e is gi,en in Eq. [ 15.22]. In
Material z J
that equatio n \\e rewrite the angular frequency w of Air at 20°C 414 ray!
the gas pocket as its ma.ximum ,·elocity: vnu, = wA Fat at 37°C 1 .33 x 1a6 ray!
with A the amplitude of the wave. Thus: i -- - - -
Water at 2o·c 1.48 x 1a6 rayl
-
Muscle at 37•c 1.66 x 1a6 rayl
(23.9) Bone 6.73 x 1a6 rayl

630 PART SI X I APPLIED crnllCAL PHYSICS Nfl

Scanned with CamScanner


NeXl we substitute this
. equation
. in the equation 1or
r
Answer The alternative anatomical setup we assume
. -itYof a wave, given 111 Eq. [15.22]. The ratio of
oterl!> . • r in this Case Study is obtained by removing the middle
1~e l . d 10 incident 111tcns1ty 1ollows:
u-ans1111tte ear, such that the eardrum and the oval window collapse
into a single membrane. Sound must then couple across
I '
It 2C2P2Vj this membrane from the medium air of the outer ear
T, =1.= I to the denser medium perilymph of the inner ear. The
hypothetical interface is equivalent to the wave reflection
I - cp V2
2 I I I
model we introduced in Fig. 23.4. We are interested in
the transmission across the interface. The ratio of the
transmitted to the incident sound intensity is taken from
Eq. [23.11 ):

4paorPprrilymphcaircp<rilymph
7j = )2
which yields: (p, ;,cair + Ppcrilymphcp<'l'ilymph
z.;,.Zpcr;1ymph

(23 .11) = 4( )2'


zair + z prrilymph

in which p is the density, c is the speed of sound, and Z


The ratio I/Ii is called the intensity transmission r 1• is the acoustic impedance as defined in Eq. [23.10). We
We further determine the intensity reflection R1, identify medium 1, which carries the incident wave, as air
which is defined as R1 = I/Ii. A fast way to quantify R and medium 2 as perilymph. Note that this equation is
startS from substituting the fonnula for vt in Eq. [23.9] and symmetric with respect to the two media adjacent to the
sorting the terms that contain vi and vr: membrane; the same result applies if the wave is incident
from the perilymph side. The last equation can be simplified
4P1P2C1C2 ) in the case of a significant difference between the two media,
v2 = I - v?
r ( (p,c, + P 2C2)2 •. that is, when Ppenlymphcpcril)lnph >> Paircair• which means also
Zpcril)mph >> z air· In this case we get:

The ratio of the reflected intensity to incident intensity is


then calculated as:

Before we apply this result, we confirm the condition


Zpen.1ymph >> Zaor. for the human ear with the impedance
values given in Table 23.2. Then we calculate the ratio of
sound intensities between air and water:

4 (4 14 ray I) _ _
T1 = 6
- I.I x 10 3.
1.48 X 10 rayl

This value confirms a more general observation: sound


transfer between media of significantly different density
is very ineffective. In particular, this means that nature
mDl:D Intensity transmission and intensity reflection could not construct our ear with a single membrane
of sound at an interface are governed by the acoustic between the outer ear and the inner ear, as a sound
impedances of the two adjacent media. The acoustic intensity transfer of only about 0.1 % would leave us
impedance is the product o f the density and the speed of essentially deaf.
sound in a given material.

CASE STUDY 23.3


Why do land-living mammals have a middle ear that sepa-
rates the outer ear's eardrum from the oval window at the EXAMPLE 23.2
entry of the cochlea?
For an ultrasound wave with an incident intensity /incident =
Hypothesis: We answer the question by assuming 5.0 x 10-2 J cm- 2 s- 1 = 50 mW/cm2 , calculate at a flat
~ rru'ddie ear does not exist and propose that t he ear
' hthencan no longer serve its purpose.
' h' interface between fat and muscle tissue, (a) the inten-
Once proven, t ~s sity reflection R1 and (b) the intensity transmission r 1•
1 YPothesis allows us to conclude that the middle ear is
Note: Ultrasound applications in medicine use sound
1 ,~ssary because sound travels through a significant dif-
erenc · · f 10n intensities between 10 mW/cm2 and 100 mW/cm2.
'. of th .e 111 density from air to perilymph (for a descnp
:e ear anatomy see Chapter 15). conttnu«J

CHAPTER 23 I DIAGNOSTIC ULTRASOUND IMAGING 631

Scanned with CamScanner


. • 111Wil·I1:. 1·11 tcrf·rcl'
t:it • ·
If" c 1wglcct 'lour1d oh,orpt run
1
. l" )'<:r tJri -; corrcspornh to on c ho rntcn o
Solution I IlC I n1 •' , • 11t) Of
0.(1 rn\\'k rn-.
Solution to part (a): We use L,-., c 1.33 >< 1r/> r.iyl and Zmu... k • The rurrl·nt ~c.·l·trcin ntl·nd' th11'l' rnulh by includ r.·
1.66 )( 1r/> rayt from Table 23.2. With Eq. (23.12) we find:
. I· .,rptiiut and t·t·ho &!du~· limt'. Bnth cflc.-t l
~OUlllI ·' , ,, . I • • s p1<I)
'

R z2 -
= (- --z I )'• = ( 1.66 - 1.3.l )'• 0 0 I"'
- -
I .,. ,
.. - ·- ' ' ' an rlltJH' n "·•rit rnk 111 . rh rn "111g th1: ' <1 r111u , paramcr·~rs ,•Or
.
I Z1 -;- Z~ 1.66 ., l..l.l the.· imaging applil·a1H1n. and allm~· II\ lo pu11111111) to "'hat
. · • ,,cd S rnn~ '' l' h;l\ c rnlrod urcd the r..r..,
Thus, only 1.2% of the incident intensity is reflected, c::in I11: 1111 •1"' • • ' • ara
2 physic:il ronrcpt' aln:ady. '' c.· pn:~c:nt the dis u~'>ion rn
which in this part1cul;:ir case corresponds to 0.6 mW/cm .
Solution to part (b): We use the same Z values as in the fonn 0f t'' o examples.
part (a). From Eq. (23.11} we find:

r, = .iz,z: = -i(1.66}( l..l.l) = o <JSS = 98.8%.


23.3.1: Sound Absorption in Ultrasound
1 (Z 1 + Z1) : ( 1.66 + l..l3): . Imaging
Note that the sum of intensity reflection and intensity
transmission must be 100% ; thus the last result could
have been obtained without calculation once the result to EXAMPLE 23.3
part (a) was found. 98.8% of the incident intensity passes we use an ultrasound signal with frequency f = 2.0 II.Hz
through the fat-muscle interface; for ultrasound imaging and an incident intensity of ' •nckirnl = 5.0 x 10- 2 J cm-2 s·
this means little contrast can be expected from an inter· =so mW/cm2. (a) How much intensity does the signal lose
face between two similar tissues. before it reaches the fat-muscle interface, ln 1 in Fig. 23.5,
where / 1 = 2 cm below the skin surface? (b) W'hat is~
total intensity loss for the recorded echo?

23.3: The Ultrasound Image Solution


In this section we simulate an actual ultrasound imaging Solution to part (a): Typical sound attenuation coetf!OeM
application as illustratt'd in Fig. 23.5. The ultrasound values for fat are found in Table 23.1. We use a va!ue of
sigml is emitted from a transducer at the left. \\'e assume a r41 = t .O dB cm- 1 MHz- 1• Using Eq. (23.4). we ca'aia~e
that ultrasound of frequency 2.0 ~lllz and intensity of from this value the intensity loss (ML in unrt dB) at 2 err
50 m\\'/cm~ is used. The interface to the skin is adjusted depth of a fat layer:
to the acoustic impedance of fat '' ith a jelly-like fluid
(shaded area). The tissue system consists of a fat layer .1/l = - (1 .0 ~)(2.o
~lllzcm
111)(:'!.0cm) = - -tOdB.
of acoustic impedance Z1 and thickness / 1 = 2.0 cm, fo(.
lowed by muscle tissue of acoustic impedance Z2 . We Thus, the signal loses 4 dB from the skin surface to the
want to explore the various propenies of the ultrasound fat-muscle interface.
echo ''e recei,·e from the fat- muscle interface labelled ln 1 So/utjon to part (b): Two steps follow the process n
(where we use the abbreviation In for interface). part (a) before the echo can be recorded: the ~ t-zs
to be reflected and then it has to tr avel back to the ska!
ln 1
surface, where the transducer w ill act as a delectOC'
The intensity loss in reflection at the 111terface is cal-
culated in Example 23.2 as 1.2 % . The correspcnd.:ng
intensity loss in dB is:

- 19~dB .

The intensity loss for the echo travel;ng bar:k ~


the fat-muscle interlace to the skin surface IS aga..., 4 CS
1--1,- 1 because the pre-., ous equation api)l.es eqJa . · for VlS
step. The convenient property of .'lll va':ies is t:-.at woe
Figure 23.5 Ultrasound measurement with an internal interface rn, at
can ~dd losses in subsequent processes, ma: IS. ttie ro~
a distance / 1 below the skin. The interface separates two med!a With
loss rn the current example is:
acoosbc impedances Z 1 and Z1. A gel is used to provide perfect acoustic
coupling of the transducer at the transducer-skin interface. .l/linu1 = M4,..,<1 ,.. - :i ll,~,..__ .. •HJ...,,,..c1_.
Numerica!ly this COrT'esponcjs to:
Part of this application has been addressed in
E:\amplc 23.2. From that example '' e know that only :ii~ • -4 0 d B - 19.: dB - 4 O <!B • - : .; lB

632
1.2% of the incident sound intensity is reflected at the

PART SIX I APPLIED CLINICAL PHYSICS


---
-c:::'·

Scanned with CamScanner


(J )
,... ,,,,
-
~

u.,.,
f'l~,,~
,.,f~11
...
ttl4
Y<"'4"._,'\
('1

,..'
'-"'~,.:.,,n11
' " •J

,
lU•
1Z IZ Ct1 ..... )

,,,.,
'Jy ..... ,., ...
c... JJ '•

... ,_
$ Ul& ....
H Pl•lfU•4 ,, , ~ 4i;i ,.._
rr1.Js.t"u ~;
~o1'1'1' tl.c:' C-t>,~, ........
~S]J,_,:.l'l;•l

.,,..
>Al.,..
"IUW.Ol IU
r 1.,.. Cirt ~rd V
,.._
..
~

~
.un L ir.t I -
i!
II ~

t
,..
7
=

Figure 23.12 (a) Typical Doppler shift pattern for erythrocytes in an artery. The Doppler shift is converted to a speed of the blood using Eq. [23.17}.
The penoc!ic speed variation of blood cells between limes tA and 41 is due to the rhythmic action of the heart. (b) The same data as recorded With
a dinical ultrasound setup.

(a) (b)

Figure 23.13 (a) Coloured angiogram of the heart with a blood vessel obstruction (stenosis). The stenosis section occurs In the circumflex
COl'onary artery: find the narrowed section immediately above the inverted LI-shaped artery in the centre. (b) Illustration of the origin of a broad-
ening of the Doppler shift for blood cells passing through a blood vessel with a stenosis.

SUMMARY UNITS
• Acoustic impedance Z: kg m-~ s- 1 = rayl

DEFINmONS
• Acoui.tic impedance Z: LAWS
Z = pc • Sound absorption:
.,., ith p the density and c the speed of sound in the medium • for intensity (Deer's law):
lnttmity transmission T1 is the ratio 1/ 1, for sound reaching
• an interface
Intensity reflection R1 is the ratio 1/ 1; for sound reaching · cocf-
I o ·is the source intensity at x =0. f3 is absorpllCln
• an interface ficient. X8"' is the absorption length

638 PART SIX I APPLIED CLINICAL PHYSICS

Scanned with CamScanner


f,>r 1101 plitude: (c) ii~
arnplitmlc i, t(l(J liigh.
• D = A0e- nx sin(wt - J..\) (d) its pn:~sure varia1io1n arc too liigli.
D is the gas pocket di splacen~t·nt and u is the decay (c) its frequency i~ too high.
codlicicnt for the sound amplitude MC-23.4. Fig. 23. 14 ~how~ a hat u~ing echolocation to detc'-t
its prey. ·1he animal u ~c~ 1he rcflcclcd frcquenc..y
(otensity transmission T 1 at an interface between medium to analy£e the stale of motion o f 1he insecl. ·1tiiJ i'
• and rnedium 2: possible becau~e of
1 (a) 1he formation of beats.
(b) the formation of standing waves.
(c) the Ooppler effect.
(d) the adiabatic processes during sound propa~tiun.
Intensity reflection R1 at an interface between medium (e) the formation of a second harmonic.
• and rnedium 2:
R =
I
(zZ12-+ Z21)2 Z

[)oppler effect:
• for moving receiver and stationary source:

. = JOr ( 1 ::!:::
!.recel\er \'rccei"")
c

for moving source and stationary receiver:



f source = fo( 1 ~ ) ::!:::~
c

• for receiver and source moving relative to medium:

l Vrccciver )

h ombined = fo
(
~
::!:::_ source
1 + --
c

MULTIPLE-CHOICE QUESTIONS
When sound is absorbed in a medium, its intensity
MC-23.1.
level IL decreases with distance travelled through the
medium x as (Note: f3 is a constant)
(a) IL :x e- f3x.
figure 23.14 A bat catching prey in the dark.
(b) IL oc -x.
(c) IL :x {3.
{d) IL :x ln(- x). . II A moth flies along a path perpendicular to the fli ght
MC-23.2. A sound travels from medium I into medium · MC-23.5.
Consider the following four conditions: (i) F~r the
path ofa bat. While the moth is within a narrow range
of angles in front of the bat, the bat detects a reflected
speed of sound, c =c applies. (ii) For the density of
1 11 frequency that is
the medium fl = p applies. (iii) For the wavelengths
• I II . ( 1' v) for the (a) less than its emitted frequency.
in the two media, A1 = A11 app11es. . (b) the same as its emitted frequency.
. . h d'a r - r applles. No
f rcquenc1es m t e two me 1 ,; 1 - 1 II
reflection of sound intensity at the interface. ·betwe~n
(c) more than its emitted frequency.
d ( ) 1s/ (d) no longer in the range it can hear.
media I and JI occurs if the following con iuon s (e) in a range that attracts dogs. like a dog " histle.
are fulfilled. Doppler ultrasound is used in medicine to detect the
MC-23.6.
(a) only (i) following physiological feature:
(b) only (ii) (a) bone fracture .
(c) both (i) and (ii) (b) blood flow \'elocity.
(d) both (i) and (iii) (c) blood pressure.
(e) both (ii) and (iv) be e (d) nervous breakdown.
~C-23.3. Ultrasound cannot be heard by humans caus (e) respiration rate under strcs .
(a) its intensity is too tow.
(b) its frequency is too low.
_ ... ~Trn .,~ I n1AGNOSTIC ULTRASOUND IMAGING 639

Scanned with CamScanner


: CHAPTER 24

Diagnostic X-ray
Imaging

The atomic model introduced in Chapter 19 allows

X
-rays im: eketromagnl'tie radiation with energies
exceeding I kc\'. They arc gl'nl.'ratl.'d whl'n fast us now to approach radiology. which is a major field in
dl'r trons hit a 1111.'tal surfo~·._., such as th\.' :modi! in an modern medicine. We will look deep into the atom to
X-ray tube. Both bn:msstrahlung and d1aractl.'ristic X-rJys establish the origin of X-rays. which are the tool of radi·
w ntribute to a broaJ X-ray c:mission that is accompanied ology due to their capability to penetrate living tissue and
by a signifkant loss to thermal enl'rgy. nr. . msstrahlung yet interact with the typical non-unifonnity of human tis-
rl.'sults "hc:n the incident ek ctron interacts strongly with sues to an extent that allows the fonnation of images with
a single atom. causing it to lose a significant fraction of sufficient contrast and spatial resolution.
its i..inctic energy. Characteristic X-rays occur when the At the same time. our understanding of the atomic
incident electron collides with an ekctron in an inner shell model and the abil ity of atoms to form biomolecules
such that it is rc:mo,·ed from the atom. allows us to answer basic questions about safety that
X-rays arc high-energy photons that in tum interact underlie the practice of radiology. What effects do X-rays
with mattc:r through Rayleigh scattering, Compton scat- have on the human body? Do we ha\'e to fear the high-
tering, the: photoelectric effect, or pair production. The energy parts of the electromagnetic spectrum? If so. does
relative contribution of each of these processes depends this radiation reach dangerous intensity le\'els anywhere.
on the photon energy and the: atomic number of the target for example, near an X-ray machine in the hospital?
matter, yielding case-specific linear attenuation coeffi- Based on the simple grounds of experience. the
cients in Bl.'er's law. The mass attenuation coeffici ent is answer is both yes and no. The widespread use of X-ra~s
introduced when we want to separate the density of the for radiography, mammography. and CT scans (computed
medium from the attenuation coefficient. This allows us tomography) implies that safety in handling X-rays is pos-
also to define the half-value layer, a thickness at which sible for both the radiologist and the patient. At the same
half the incident X-rays have been remo\'Cd from the time, we know high doses of X-rays can be lethal.
beam. X-rays cause alterations to li\'in!! omanisms also at
Contrast and image resolution are the two param- lower non-lethal doses, as shown i~ H; rmann Joseph
eters defined to quantify the relative yield of radiation Muller's 1927 experiment that pro\·ed genes are (artifi·
for different parts of a non-unifonn sample. Both param- cially) mutable. Muller bombarded fruit flies with X-rays
eters depend on the physical properties of the incident and linked new defonniti es in the offspring to th~ radia·
X-ray beam and can therefore be optimized for specific lion, proving that he had artificially altered genes in the
applications. insects. In 1940, George Beadle and Edward Tatum used
The interaction of X-rays with the penetrated tissue the same X-ray technique on a species of bread mould
may cause tissue damage. Thus. the amount of radiation (Neurospora) to pro\·e the correlation between genes and
required in a clinical study, called the dose, and the rate enzymes: the mutation of certain genes Jed to a lack of
at which this dose is administered, called the dose rate
certain enzymes in the offspring. .
arc additional parameters that determine the selection and We know from the medical use of X-rays. in ,,hrch
setup of the imaging modality used. energies in the range of 25 keV to 150 keV are used. th 3t

64 2 PART SIX f APPLIED CLINICAL PHYSICS

Scanned with CamScanner


------------------------------ N0 I

Mo!!~ M
I t traction of the energy of the cathode electrons is
Wild~ ted as X-rays in an X-ray tube used for (a) typical Lu Lil L'Y
ra iat X-ray radiograms,
. f h" h
or w 1c we use a tungsten
cheS . . I ' I.
anode with an operat1~g potent1a of 100 kV; (b) typical
1 arnmograms, for which we use a molybdenum anode

I
~:= 42) and an operatin~ potential of 30 kV; and (c) typical Ku K13 K'Y K&
radiation therapy, fo~ which we use a tungsten anode with
an operating potential of 6 MV?
' .. . K

24.1.4: Characteristic X-rays


Note that the spectrum in Fig. 24.3 displays several sharp
X-ray peaks between 50 keV and 70 keV. This radiation
is characteristic for the metal used in the anode, as we
confinn experimentally by changing the material in the
anode: while the bremsstrahlung part of the X-ray spec-
trUm varies little, the sharp peaks shift to new energies.
As a result, we refer to these spikes as characteristic
X-rays.
We saw already in the discussion of Bohr's model
for elements heavier than hydrogen that the same transi- Figure 24.7 Electron transitions in an atom in which an electron has
tions require more energy for heavier elements; the tran- been removed from an energetically lower orbital. Electrons from ener-
sition from n = 2 ton= 1 releases 10.2 eV in hydrogen, getically higher orbitals make transitions to the vacancy, in either one
or several steps. This corresponds to a range of possible transitions in
but 40.8 eV in He+. However, these values apply only
heavier atoms, with several of the transitions in the X-ray part of the
to atoms and ions with a single electron. To remove a
electromagnetic spectrum (characteristic X-rays). The figure indicates
valence electron from heavier atoms requires less energy. the standard notation used for these transitions, with the capital letter
The situation during the bombardment with energetic indicating the principal quantum number of the orbital with the vacancy
electrons is different. The energetic electrons act like (K-shell corresponds to n = 1). The index indicates the difference in the
particles and can eject electrons from one of the inner principal quantum numbers of the initial and final orbital of the electron
orbitals of the anode atoms. Once an inner electron is that makes the transition (a corresponds to /!in =1).
removed, electrons in higher orbitals fill the vacancy.
The energy difference for the electrons involved in such
transitions is large due to the large positive charge in
the nucleus and the large energy difference between the
24.2: Photon Interaction Processes
lower lying orbitals. . with Matter
To judge what consequences X-rays have on the pen-
etrated tissue, the physical effects of the interaction of
Grml!ll Characteristic X-rays originate from atoms the X-ray photon with matter have to be identified. We
in which a collision of a fast incident electron removes an introduce in this section four processes by which X-rays
electron from an inner shell. The photon is released with interact with matter: (1) Rayleigh scattering, (2) Compton
an energy that represents the energy change of an outer scattering, (3) the photoelectric effect, and (4) pair pro-
shell electron as it drops into the vacancy below. duction. Combining these four effects allows us to quan-
tify the loss of X-ray intensity as the radiation passes
through biological tissues.
There are several possible transitions of electrons into
a lower orbital vacancy as illustrated in Fig. 24.7. For an 24.2.1: Rayleigh Scattering
electron removed from 'the innennost orbital
· (the orb'itaIs
w·h
it quantum number n = J are called the K-shell), In Rayleigh scattering (named for J. W. Strutt, Lord
filour Possible transitions are shown, labelled with Greek Rayleigh), the incident photon interacts with the entire
.etter ·indices.
· · ·
Other transitions leading to X-ray emission atom, causing all electrons to oscillate together. The photon
involve the initial removal of an electron from the L-shell is instantaneously re-emitted, causing a slight change in
(th t. h . direction of the photon but no change in its energy. The
or~ is, orbitals with n = 2) or from the M-shell (t at ts,
bitals with n = 3). process is quantified in a classical electromagnetic model

CHAPTER 24 1 DIAGNOSTIC X-RAY IMAGING 647

Scanned with CamScanner


in "hich the photon is described as a wave and the elec-
trons in the atom respond to the electric field vector of the
Wa\ c, '"hich forces the oscillations.
For X-rays, Raykigh scattering occurs mainly for
\'cry low energy diagnostic X-rays, as used in mammog- - 1)"1-
raphy. At 30 keV photon energy. Rayleigh scattering hf
contributes about 12% of the interactions with tissue. In
medical imaging, the Rayleigh-scattered X-rays have a
negative effect on image quality because X-rays of the
same energies as the incidcm beam reach the detector
but not along a straight line from the X-ray source.
However, this type of interaction has a low probability
of occurring at typical diagnostic X-ray energies: it
contributes less than 5% of the X-ray interactions with
tissue for photon energies above 70 keV. Even at low
X-ray energies, Rayleigh scattering contributes less
effectively to photon- tissue interactions than the photo-
electric effect, and is therefore neglected in further
discussions.
Figure 24.8 Principl.e of Compton scattering: the incident Photon Of
energy /if interacts with a valence electron of an atom, removing tile
valence electron. Simultaneously, a Compton scattered photon With a
energy less than the incident energy (A2 > A1) leaves the atom with n
In the visible part of the electromagnetic spectrum, the scattering angle 8. The scattering angle is defined as the angle betwee~
Rayleigh scattering probability is proportional to the fourth the trajectory of the incoming photon and the trajectory of the scattered
power of the frequency of the light, / 4 • Can this type of photon.
scattering explain the generally blue colour of the sky on
a bright, cloudless, sunny day?
The scattering event is also subject to the conserva.
tion of momentum. The two conservation laws allow us to
calculate the energy of the scattered photon as a function
24.2.2: Compton Scattering of the energy of the incident photon and the angle of the
Compton scattering (named for Arthur Compton, who scattered photon with respect to the incident trajectory
observed it for the first time in 1922) is the main interac- (for the geometry see Fig. 24.8):
tion of X-rays in the energy range from about 250 keV
to 30 MeV. Compton scattering is a process that occurs £photon, i
£pholon, f = E . (24.2)
when an X-ray photon interacts with a valence electron, photon, i ( )
as illustrated in Fig. 24.8. The electron is ejected from the + I - cosO
mec2
atom and the photon is scattered, with reduced energy.
The Compton scattering event results in ionization of As the incident photon energy increases. both the
the atom, with a division of the incident photon energy scattered photon and the electron arc more likely scat·
between the scattered photon and the ejected electron. tered in the forward di rection. Thus, in X-ray transmis·
The ejected electron then loses its kinetic energy into sion imaging. the scattered photons are much more likely
the surrounding material, producing further ionizations. to be detected in the image, reducing image contrast, as
The scattered photon can interact further with the tissue discussed later in this chapter.
sample or it can reach the detector, which is detrimental As the energy of the incident photons increases (high
for image quality because it did not travel along a straight kilovoltage radiography), an increasing fraction of the
line from source to detector. incident photon energy is transferred to the scattered electron.
Due to energy conservation, the energy of the inci- This is evident from Eq. 124.2] for a gi\'en angle q: as the
dent photon E horon, i is equal to the sum of the energy of term Ephoton. i increases in the denominator 0n t.he right·
the scattered photon £photon, fand the kinetic energy of the hand side, the second term in the denominator increases
ejected electron £electron: compared to I, and thus decreases F~ph01on. r

Ephoton, i -- r;'photon, f + £'dcctron· WUJ!:il A good agrl!emcnt between theory and experi·
mental measurements in Compton scnt1cring confinntd
In this expression. the binding energy o f the ejected elec- the applicabil ity of the corpuscle model of t11e X·r3_>
tron is neglected because it is small compared to the three photon. Conservation of both eneri;y and mom~ntum
15

energy tenns shown. obeyed in the interaction of radiation with mat1er.

6 40 PART SIX I APPLlfD CLINICAL PHYSICS

Scanned with CamScanner


energy
. ·
increases u .
illustrated ·1n F' p to energies of about 50 keV This is
15 compton scattering an elastic collision? 1g. 24 9 w . . .
free, a mu ch I . . . ere the electrons in the sample
.,, wer We defined elastic collision in
,....5 . . . h'
.
c1ass1ca1 rnech . Compton scatt e~scr dependence of the probabil ity for
a5
a coll1s1on 1n w 1ch both the kiner
1c energy a d
an1cs The probabilit er~:~ below 50 keV would be observed.
..,0 mentum are conserved. Momentum is n the X-ray cnerg· •.y. b Compton events then decreases for
'" · a1ways con
however, conservation of kinetic ene~gy d served; · of th Jes a ove I 00 kc· V· The decrease ts
lion · a rellec-
I . . I oes not alw e cross-sect. fi h
apply (for examp e, 1n rne astic collisions) W ays ti ow the h. h ion or t e photon-electron interac-
subatomic particles that require quantum · hen studying . ig er the energy th d'
mechanical must hit th . • e more irectly the X-ray
siderations. such as the various equations we . con- e e1ectron to interact.
the particle-wave dualism, the term elast' .d.enved for
to be extended. In particular, the classica1lc colhs1on needs
case of an i ·
dent particle of equal mass hitting head-on . nc1-
a particle at re t
cannot be extended to a photon hitting s
.. . f an e1ectron The
relat1v1st1c mass o the photon may match th t ·
h a~h~~
tron. but the p oton ca~~~t be at rest after the collision
If we assume the def1nit1on includes a quant ·
. um mechan-
ically behaving X-ray photon (with zero mass at t)
. . . res • then
Compton seattenng 1s quasi-elastic as only a . .
. . • neg11g1ble
energy contribution due to the binding energy of th _
. t t df . e e1ec
tron 1s no accoun e or in the kinetic energy form 1
. b t' . u a. 0 ne
may st1 11 e cau . .1ous. using . the term elastic • w'ith 1
'ts c1as- 10 102 103
sical mechanics 1mphcat1ons. E(keV)

Figure 24.9 Probability for Compton scattering as a function of incident


photon energy.Thesolid curve 1 assumes interaction with a bound electron.
EXAMPLE 24.3 The incident photon energy must be substantially greater than the binding
energy of the electron before Compton scattering becomes likely, leading to
What fraction of the energy of the incident photon is taken a peak near 50 keV. For free electrons (dashed curve 2), Compton scattering
up by the released electron for a (} = 60° scattering angle depends much less on the incident energy below 50 keV.The probability for
when using (a) 100 keV X-rays in chest radiography, and Compton events decreases for X-ray energies above 100 keV.
(b) 5 MeV X-rays in radiation therapy?
At a given X-ray energy, the probability of Compton
Solution scattering also depends on the number density of elec-
Solution to part (a): We use Eq. [24.2] with mc2 = 511 keV, trons in the tissue. The number density of electrons con-
which represents the energy of an electron at rest. For tains two factors: the number of electrons per ki logram of
100 keV X-rays we find that the constant 1 dominates the tissue, times the density in unit kg/m3. With the excep-
denominator on the right-hand side of the equation, and tion of hydrogen, the number of electrons per kilogram
that the scattered photon picks up 91 keV (that is, 91 % of is fairly constant in tissue. As a result, the probability of
the incident energy). This leaves only 9% of the energy for Compton scattering per unit mass is almost independent
the electron, or 9 keV. of the atomic number of the elements in the tissue, Z, and
Compton scattering at the lower X-ray energies used the probability of Compton scattering per uni t volume is
in diagnostic imaging (18-150 keV) causes the majority of approximately proportional to the density of the material.
the incident X-ray energy to be transferred to the scattered Compared to other elements, the absence of neutrons in
Photon. These photons may still reach the detector and the hydrogen atom results in an approximate doubling of
contribute to image degradation. For example, an 80 keV electron density. Thus, hydrogenous material has a higher
X-ray photon leads to a minimum energy of about ke~ 6? probability of Compton scattering than hydrogen-free
for the scattered photon in a Compton even t • which 1s still
material of equal mass.
high enough to penetrate tissue. . MeV
Solution to part 1'b ': We use again Eq. [24.2). A
5 tUU@J Rayleigh scattering is the interaction between a
X ' 'I 'th the scat-
-ray photon loses most of its energy, WI t lassical wave with an entire atom, and Compton scat-
tered photon carrying only 850 keV. The difference, abou ct ing is the interaction of a light corpuscle (photon) with
' 4.2 MeV, is the energy picked up by the electron (repre- er . h' h
an individual valence electron wtt m t e atom.
senting more than 80% of the incident energy).

24.2.3: Photoelectric Effect


be substantially otoelectric effect, the entire energy of the inci-
The incident photon energy must before ln tI1e Ph hoton is transferred to an e1ectron, wh'1ch m ·
oreat h . . f the electron
" er t an the bmdmg energy 0 b b'lity dent .X-ray P · d · F' 24 JO
Co . Th s the pro a 1 . ted from the atom as illustrate m 1g. . .
tum IS CJeC
lllpton scattering becomes hke 1Y· u : 'd t photon •
oreompton scattering . increases
. as the me• en
CHAPTER 24 I DIAGNOSTIC X· RAY IMAGING 649

Scanned with CamScanner


CASE STUDY 24.2

Agurt 24.1o Pnnc1p10 of pttotoclcctnc obsorption: an inc1dcnt phOton ot


energy hf lntcrncts with en electron of on Inner shell. The released elec-
tron h.1s :i kinetic energy equal to ltle energy ol tM lncid~t photon minus
the binding energy of tho released oton11c electron. In tM case shOWO in
the figure. the binding energy is the K·shell binding energy.

l liU!l.jl R a~ kigh s\·ath:ring is '-' ''-'lusi\ cly a scauain!_!


cOi:ct: Compton scalll.'rini.: is a sca u\· ri n~ dl'c1.·t "ith n
frnctitin of th\· cncr)ly ah:-tirl''"'" in th'"· t i,~uc ~:rnipk b)
an cjcctl'd \ aknc1.· 1:k..:tn1n: and th\' pho1<11.·k,·tric dli:rl i
cxclus i\ d y an :1h\iorptlon proct•,o; in" hidi ::il l th\· c.-m·1~s
is ah orh1.·J hy th\· l'jCCll'd d c..:tron nnd its hi n,ting 1.'ll\'I)!)'. with tho constant ~t cf enerw 300 a~~
To obtain the ratio. we divide the t\\t> forr!U3s:
A second dilli:n:nce with Compton scattering is the
greater probability in the photockctric cff1.·ct that electrons
c.k.:~r in the atomic shell arc im ,11' ed. Thus. the conser-
vation o f .:ncrgy in th<: photoelectric effect is" rittcn ns:
(2·U)

that is. the kinetic en.:rgy of the cj1.'Ctcd eketron is 1.-qual to


the incident photon .:n.:rgy minus the binding l'l1l'l1:!Y of the We substitute the gi\ien vn.'ues:
electron in its orhital. TI1c hinding energy is now a major
I'm.mm.~~ ( 7 150\.c\ '))'
contribution ~cause the photockctric etli..'\:t n.·pn.·s1.•nts an ----
I',a.1i"~~
- l~ (:He\') - ~(,
--.
incla. tic collision betw.:cn the photon nnd the ekctrnn.
Obviously. photock <.·tric absorption of an X-rny phl'ton can Thus. the frnction of incident X--ray ph tons~ i
occur only \\ hen the incident photon carries a gn:;itcr energy in a photoelectric event is much gre3t"&" in m..~~Y 1
than the binding c111.·rgy of the .:kctrun that is 10 be cj1.~ll-d. than in chest radiography. Photoelectric n~"lan to3S a
The r.:ason that clccrrons in deeper shells arc im olwd benefit in X-my transmission in that photons n.~ rernc\'ed l
is that X-ray cncrgil:s far c:\cecd the binding energy of from the beam: that is, no scattered ph tons re,:ich t1ie •
detector to degrade the image. The strong irwe..-se ~
1
outer valence electrons. being more clc,s1.·I>· matched with
the hinding energy of electrons in lower shdls. The proba- dependence of the probability for photoelectnc a~tio.-i I
bility that un electron \\ill be ej1.' l'tcd through a photockc- explains to some extent why image contrast deere~ at \
tric even! incr.:ascs the clllser the X-my plh,ton energy higher X-my energies.
is to the binding energy r1f the elcctrnn. For example..'. if
the photon energy exceeds the K-shell binding energy in
the atom. then the photnekctric intcmction with n K-shell Can the photoelectric cffC"Ct )('3d to X-rn\' emi~:>l\ r.~
electron is most probable. ns well? The answer is yes. alth\ u~h s li...h c~liss.io:l is ~l
The photoclrctric effect itself docs not produce n sec- significantly lower enerey. The 1 n~uction ('If the ~-r:lY i~
ondary X-ray photon, and therdore allcm s only ptimnry n secondary effect nnd occurs for hoth the ekcmm ejf°''\N
photons to reach the detector in medical imaging tech- in the primary photoek ctric cwm and th..• ionircd !ltl'l.'ll
niques. This is an advantage in comparison to Compton with an electron missing in 3 COl'l' shell.
scallering, which results in loss of image contrnst due to Using Eq. l~·t3] we detem\inc: the.• kinetic ene~· .)f
scattered photons reaching the dctL'Ctor. the c.•jectcd electron for two cxampks. A 100 kc\' in.-iJ~'

;so PART SIX 1 APPLIED CLINICAL PHYSICS

Scanned with CamScanner


ton can eject an electron from all shell .
. )' Pho . s ma electrons in carbon). Jn heavier atoms, the newly created
~-r:i tom (BaC03 1s frequently used as a radiogr h.
~
;lfiuJll 3 . •. z B ) _ . . .
material since ( a - 56). the bmdmg enc
ap IC vacancy in the L-shell is then filled by an electron with
(\lll'i;5K-shell ek ctrons is 37.5 keV. An eli:ctron ejec:;~
1 an even lower binding energy. leading to an electron
cascade from outer to inner shells. Jn carbon. the differ-
l:f~ the K-sh~ll has a~ ener.gy of 100 k_eV - 37.5 keV : ence in binding energy of the K- and L-shells is released
!fO k v It ,,111 lose Jts ern.:rgy. most likely in the fi
,5 e · . orm as a photon of energy 280 eV: that is. the emitted photon
of·tht'f1Tlal energy. _but may alternatively generate some
does not lie in the X-ray range. For barium. the highest
o rnsstrahlung. with a peak at an energy well below
ntt , eV (compare" ith Fig. 24.5). energy photon emission is generated by the direct transi-
6,-·5j\ext
" we cons1·der a I'1g hter e I~ment present in tissue. tion of an outer shell electron into the K-shell \ acancy,
yielding a photon with an energy around 37 keV. The
"f}le binding energy for an ekctron '".the K-shell of carbon
more likely transition of an electron from the L-shell
. ·, en as 0.29 keV. The electron ejected in a photoel _
is e1 . .d h ec yields a photon of energy 31 .8 keV. These photons have
tri; e\'ent ,, i~h an mc1 ent p .oton :ner~y of J00 keV gen- much lower energy than the incident beam, and can be
"" therefore.
er.it••• · ek
_ ctrons with a kmet1c. enero)' exceed·mg
e · pre,·ented from reaching the detector using filters.
ke\'. A b~e"'.S!>trahlung spectrum with photon energies
99
cJoser to the incident : nergy s~ould be generated; however, • In Fig. 24.11 (right) the vacancy in the core shell is
the initial phot~electnc e\·ent 1s much less likely due to the filled by an outer electron. but the released energy is
'·'·
. ,~

lln!e energy difference between the incident photon and transmitted in a radiationless process to another elec-
the- binding energy of the K-shell electron. Thus, the elec- tron, which leaves the atom with a characteristic kinetic
iron ejected in the photoelectric effect does not contribute energy. This process is called the Auger process, and
si!!llificantly to the X-ray emission from the sample. the ejected electron is called an Auger electron (named
- ~ext we consider the radiation from the ionized atom for Pierre Victor Auger. who disco,·ered this process
\\ith an inner-shell electron vacancy. This ion undergoes in 1924). The Auger process does not yield X-ray
one of two processes, which are illustrated in Fig. 24.11 emission. and therefore does not cause a reduction
for a carbon atom. in contrast in medical imaging techniques. Note that
Auger electron emission is farnured for samples with
1 In Fig. 2-t I l (left) the ,·acancy is filled by an electron small atomic numbers, such as soft tissues, while char-
from the L-shell (which is the outmost shell carrying acteristic X-ray emission occurs for heavier atoms.

Lm -o---O--CHH:>--

Lu --------t1>-1---

-:

Lu ---------~~--
hf
L1 _ _ _..__ _._ __.1----'

~ 24-11 Two types of radiation from an ionized cart>on atom with a K·shell electron vacancy. At the left the vacancy is filled by an electron from
?le r;;xst;"·~ difference in binding energy of the K- and L-shells is released as a_ photon.of_energy 280 ev. At the right (Auger process!, the vacancy in
Yt!tlt a ct!ell is Mled by an outer electron, but the released energy is transmitted in a radiationless process to another electron, which leaves the atom
acteristic kinetic energy.

CHAPTER 24 I DIAGNOSTIC X·RAY IMAGING 651

Scanned with CamScanner


1oton can eject an electron from all shells in electrons in carbon). In heavier atoms, the newly created
:om
'f_-r.3Y P
uni 3 _ .
a
(OaC03 is frequently used as radiograpll.a
. . . z 1c vacancy in the L-shell is then filled by an electron with
~J0 ·t niatenal smce ( 8 a) - 56 ): the binding energy
ntr.i~ · · 37 5 k an even lower binding energy. leading to an electron
cv the !{-shell electrons is . ·eV. An electron cjectt:d cascade from outer to inner shells. In carbon. the differ-
~f thi! K-shcll has a~ energy of 100 k.cv - 37.5 keV = ence in binding energy of the K-and L-shells is released
01
~ keV. It will lose its energy, mo~t likely in the form as a photon of energy 280 c V; that is, the emitted photon
5
6-· h nnal energy. but may alternatively generate some does not lie in the X-ray range. For barium, the highest
of .t esstrahlung. wit. h a peak. at an energy well below
br~ni . I1 F'1g. 24 .5). energy photon emission is generated by the direct transi-
, . keY (compare wit tion of an outer shell electron into the K-shcll vacancy,
6·-' Next we cons1.der a 11g · hter e Iement present in tissue.
yielding a photon with an energy around 37 kcV. The
The binding energy for an electron in. the K-shell of carbon more likely transition of an electron from the L-shell
. i,en as 0.29 keV. The electron ejected in a photoelec- yields a photon ofenergy 31 .8 keV. These photons have
15 g . 'd h
uic event with an mc1 ent p .oton :ner~y of IOO keV gen- much lower energy than the incident beam, and can be
er.ites. therefore. electrons with a kmet1.c energy exceeding prevented from reaching the detector using filters.
99 keY. A b~e1~sstrahlung spectrum wtth photon energies • In Fig. 24.11 (right) the vacancy in the core shell is
closer to the mc1dent energy should be generated; however
the initial photoelectric event is much less likely due to th; filled by an outer electron, but the released energy is
large energy difference between the incident photon and transmitted in a radiationless process to another elec-
the binding energy of the K-shell electron. Thus, the elec- tron, which leaves the atom with a characteristic kinetic
tron ejected in the photoelectric effect does not contribute energy. This process is called the Auger process, and
significantly to the X-ray emission from the sample. the ejected electron is called an Auger electron (named
Next we consider the radiation from the ionized atom for Pierre Victor Auger, who discovered this process
with an inner-shell electron vacancy. This ion undergoes in 1924). The Auger process does not yield X-ray
one of two processes, which are illustrated in Fig. 24.11 emission, and therefore does not cause a reduction
for a carbon atom. in contrast in medical imaging techniques. Note that
Auger electron emission is favoured for samples with
1 In Fig. 24.11 (left) the vacancy is filled by an electron small atomic numbers, such as soft tissues, while char-
from the L-shell (which is the outmost shell carrying acteristic X-ray emission occurs for heavier atoms.

Lm -0---0----0-+-<~-
Lu---------+---
L1 ___________.._ __

K-e-------<::~--

Lm - - - - 0 - - - 0 - - - 0 - -

lu ----------e~­

L, --------41~-.i----

K K--<.----e-----
~~~· 11 Two types of radiation from an ionized carbon atom with a K-shell electron vacancy. At the left, the vacancy is filled by an electron from
tie Core ~The difference in binding energy of the K· and L·shells is released as a photon of energy 280 eV. At the right (Auger process), the vacancy .
w111i ach~a 11 is .f•l~ed by an outer electron, but the released energy Is transmitted in a radiationless process to another electron, which leaves the ato~
ctenstic kinetic energy.

ljfl
CHAPTER 24 I DIAGNOSTIC X·RAV IMAGING
651

Scanned with CamScanner


CONCEPT QUESTION 24.4
Image contrast Is a major concern for all X-ray imaging Prr (% )
techniques. Let us consider the contrast between bone
and soft tissue in chest radiography using 100 keV X~rays.
50
What is the ratio of X-rays stopped by bone and soft tissue so
based on the photoelectric effect?

At photon energies below 50 keV, the photoelectric


effect plays an important role in imaging soft tissue. ~he
photoelectric absorption process is exploited to amplify
00 IOQ
differences in attenuation between tissues with different Iooo
atomic numbers to improve image contrast. The photoelec- E (keV)
tric process is most important when lower-energy X-ray
Agure 24.12 The relative contributions.of the photoelectric ett
photons interact with high z
materials. as illustrated in
vertical axis) and the Compton effect (right vertical axis) t ec1 ~
Fig. 24.12. The figure shows that, at 50 keV, photoelectric
attenuation process in various materials as a function 01 X-~atlie X-~
absorption is the dominant interaction of diagnostic X-rays
The contributions are expressed as probabilities Pin unit ncrce ene% Yt

with screen phosphors, radiographic contrast materials, and . " n (%J tr·
index CS refers to Compton scattenng, and PE to photoele: t·ic H · t
bone. Jn tum, Compton scattering dominates in tissues. . eec1~
curves shown corre~pond to tissue (1), bone (2), Nal ((3· :'1 'Sl fr~
Although Case Study 24.2 illustrated how the prob· used detector material), and lead ((4); Pb, used for shield1; . tiote thatntri
ability of the photoelectric effect decreases with increasing energy range I (between the left dashed vertical lines) ,~ . ~lied d.tr~
10
X-ray photon energy, there is an additional opposing effect nostic radiology, and the energy range II (between the r· 11051 !las:
we need to take into account. This effect is illustrated for vertical line and the right border of the graph) is used 1; • ~therapy kr
a lead (Pb, Z = 82) sample in Fig. 24.13. The figure shows radiation treatment.
absorption of X-rays of increasing energy. The sharp
absorption edges in the otherwise decreasing probability energy for these electrons. The change in 1 robabilih
of the photoelectric effect occur when the photon energy may be as large as a factor of I 0 across an :i~ tionedge
crosses a value at which electrons in the next deeper shell The absorption edges are labelled :ll ng to ilie
can be ejected; that is, the X-ray energy exceeds the binding atomic shell of the affected electrons, fl: Jmple. in

20 40 60 100
E(keV)
Agure 24.13 X-ray absorption in lead. The vertical axis shows the a . .. isiole.~1tfi~
K-edge at 88 keV and the three L·edges below 20 keV. bsorption coefficient In arbitrary units. Four absorption edges are v

652 PART SIX I APPLIED CLINICAL PHYSICS

Scanned with CamScanner


.
1~.tJ us K. ~1· L2·.
f1S·'sh• ~an·ous orbitals
:m.d L3. The .subl>cripts distin-
of slighll y vary111g 1.·ncri•v Fr 24.3: Photon Attenuation I
I

. ") eJ.
gu1.• u:ition in Sc:c.:111>11 _.t ~ . I we rnnduJc that lhl· X-ray
urn II
11tc ,4 ·n•)' l'l1rrcs1wrHl1t11! tu a partrcuhr ab ·oriJt°
h 100 er1'·e- . -: . . •.
r (1 1'nc~:iscs \\1th the a!Cllllll nurnhcr z ul !he sampkd
ron 24.3.1: The linear Attenuation Coefficient I
The ability tu p1.·nctrate vari\111s materials dl'rl·nds on the
• en!. All the: primary ckmcnts
(Jgc: . rnmprisin!.! 1>ntl tiss •
. ~ Uc actual cn1.·rgy of the X-ra ys anJ the composition of the
e~I 01 ~• and 0) h~l\C t11e1r a11l>Orpt1011 1.•d1•c;.·s b ·I
IC '"· e 1: uw Pl'nt'tratl'd malll'r. Whi:n a hcam of X-rays of energy of
(I · V'th:it is. al cncrgil'S hl'lc1\\ the range us1.·d in nwdical
I kC .' ,, ,, c:H·r. ekmcnts sud1 as iodine and bariun
k ss than I McV enters a sample. ii hises pholons primarily
'(11!1!!10:· 110 . - . 1. by l\\ u of the processes we Jiscusscd aho\'c: ( i) through
1. - \aluc:s nl·ar )0. an: crn11111011ly used 111 radiographic
111th 1 at!ents to pro' idc cnh:m ccd X-ray attcnuatio a.~ sorptiun of photons in the photoekctric efTect, and
ciJO!r3S1 ~ . . . 11. ~11) through a combination o f energy loss ond scattering
fh6f elements have K-absorpt1011 edges between 30 keV 111 the Compton cllc1.·t.
and ~O keV. Thl'sc two l'lli:cts combine to yield the tolal altl'nU·
ation of the i111.· idcnt beam. The total allcnuation depends
24.2.4: Pair Production on the material and the energy o f the incident beam. At a
\\'ith increasing photon energy. the probability for the giwn photon energy, the number of photons in the trans-
hotoelcctric ell~cl decreases rapidly and proportionally mitted beam. N. depends un the depth in the sample in the
~o £-3. The prohability of Compton scattering decreases following way:
as well. Still. X-ray absorption of mailer increases in
the MeV photon energy range. This absorption is due (24.5)
10 a new, energy-requiring process: the generation of an
electron-positron pair. where N 0 is the number of incident X-ray photons. N(x) is
Apositron is the antimatter particle of an electron: it the number of photons transmilled to a depth x. and µ is
is identical to the electron but carries a positive charge. the linear a ttenuation coefficient in unit m- 1. Eq. [24.5]
Positrons are stahk in isolation but they are annihila~ed states that the number of X-ray photons decreases with an
when they encounter electrons. Thus. a newly gener- exponential law (Beer's law). The same law applies to
at~d positron slows down in the material in which it sound and light absorption in mailer.
is formed. When caught electrically by an electron,
the annihilation occurs in less than 1.5 x I 0-1 s. The I ju 4,jj/jl Beer's law states that the probability of scattering
annihilation results in two X-ray photons of 511 keV or absorbing a photon at each depth in a sample is pro-
energy each. a process which is the physical basis for a portional to the number of incident photons reaching that
powerful technique in nuclear medicine called positron depth.
emission tomography, o r PET, which is discussed in the
ne\t chapter. lfwe label with dNthe numberofphotons scallered or
An electron- positron pair is generated when a photon absorbed (that is. removed) at depth x, and N the number
of sufficient energy interacts with matter. Conservation of photons reaching that depth. then the basic principle
of energy requires that the photon carries enough energy expressed by Beer's law is written in the fonn:
lo create the two particles that is E
. •
> 2m.:c2 ' where
• photon - dN(x) cc N(x).
"',is the mass of the electron. Thus E h must exceed
I 02 . .. • p oton
· .~eV. If the 11111ial photon energy is higher, the
which is rewrillen in equation form as:
remammg energy is distributed as kinetic energy to the
electron and positron:
- dN = µNdr,

E•.nV) + Elm
· (e- ) = Eincident photon - 1·02 MeV· (24.4) whereµ is a constant and dr is the depth interval in which
the scallering/absorption takes place. We rewrite this
pair~onse~ation of momentum further requires that the equation by dividing by de
Th·1 onnation occurs in the vicinity of another particle.
· Ic is
ab 1s. other panic · usually an atom, causing the prob- dN
111y for . - - = µ N.
matter. pair production to depend on the surrounding <fr
Pair prod . This can be integrated to find the depth dependence of N
tional . uct1on does not play a role in conven-
1Mev.ract1ooraph
We~ y, a~ X-ray energies are well ~e ow
I that is, the function N(x). For the integration, we need t~
11onof X-ra e~lcct. th1s effect, therefore, in the d~scus- find a function for which both itself and its change with
11gnificanu ~ ima~ing. However, the efTect contributes depth have the same mathematical form. The only math-
medicine. Yin radiotherapy, and is exploited in nuclear ematical function of this type is the exponential function .
Integration leads then to Eq. [24.5).

CHAPTER 24 I DIAGNOSTIC X·RAY IMAGING 653

Scanned with CamScanner


2 6 10 14 18
Dose (Gy)

Figure 24.18 The surviving fraction of three types of human cells as a


function of energy dose in unit Gy. The dose is the energy deposited by
the radiation per kilogram of tissue. Note the tower steepness at doses
below 1 Gy, which is due to self-repair mechanisms in living cells. Various
cells respond with different sensitivity to radiation: (1) thyroid cells,
(2) mammary cells, and (3) bone marrow.

of exposure to X-rays is significantly higher. This applies


particularly to the van Allen radiation belt between l 000
and 24000 km above Earth, where intensive streams of
energetic particles, mostly electrons, are coupled with an
extremely low concentration of gas molecules.

24.6: Clinical Applications


We now examine some examples of clinical applica-
tions of X-ray imaging in humans. The first example,
in Fig. 24.19, corresponds to an anterior (that is, the
patient is facing the X-ray tube when the image is taken)
chest X-ray of a healthy middle-aged female volunteer.
The gender should be fairly obvious due to the outline of
the mammary glands in the lower chest and just above the
diaphragm. By convention, the patient is assumed to be
looking at the observer, thus the patient's left side is on
the right side of the image. Note that the pulmonary fields
appear dark as expected, since most of the X-rays are able
to penetrate through the lungs with ease and impress the
film, thus darkening it upon chemical processing. Bony
structures such as the rib cages, clavicles, and even cervical
vertebral bodies can clearly be identified. These structures
appear white on the film as they are effective in stopping
the X-rays by photoelectric absorption, due to the rela-
tively high effective atomic number of bone compared to
soft-tissue ( 13 versus 7) leaving the film relatively intact,
which upon processing appears as white in colour. The
cardiac silhouette is clearly visible on the patient's lower
chest slightly to the left of the midplane. Cardiac muscle
is relatively denser than lung, and thus appears whitish in

660 PART SIX I APPLIED CLINICAL PHYSICS

Scanned with CamScanner


. •ctcd to increase
·. pn'Ji: . dueI to. higher
. smoking rates
rJte I~ rol!ressin!ly ag11~g popu ation .Ill many countries. contains a heavy metal such as iodine, which. due to its
3nd l P ldwidi;: economic costs associated with this dis- high atomic number (Z = 53). has a much higher photo-
" \\Ur
l1•e . direct tn:atment costs an d .u1srupted

economic electric absorption probability than the surrounding soft
e Iro111 . b $ I ·11 · tissues. Thus the technique allows the iodine-containing
els. . is estimated at a out tn ton/year. In sum-
act1"1trhc best way _10 ~~otect yourself against this dis- arteries to stand out in exquisite contrast to soft tissu~s
n1:ifY: stop smok111g ti you already do. or avoid taking upon X-ray imaging. One application of angiography ~s
ea habit in the first p1ace.' In r·
se IS to · 1g. 24.20 we present the detection and diagnosis of aneurysms. An aneurysm IS
up the 1 X-ray of a 62-year-old male with a 30 year his- a localized, blood-filled, balloon-like bulge in the wall of
·"e ches k . I' . a blood vessel that can occur anywhere in the body. As an
u• of smoking two pac ' S o cigarettes per day, who on
10
~sultation presents shortness of breath. coughing with aneurysm increases in size, the risk of rnpture increases.
co A ruptured aneurysm can lead to bleeding and subsequent
sputum. and fatigue. Laboratory
. tests revealed a severe
hypovolemic shock. leading to death. Aneurysms result
·on in lung capacity. and a follow-up chest X-ray
reductt . . . . from weakened blood vessel walls, and can be associated
was m · dicated. The . X-ray reveals. s1gn1ficant
. changes 111
with hereditary factors or an acquired disease. Jn Fig. 24.21
the inferior and middle lobes ot the right lung, compatible
we present a cerebral angiogram of a patient who presented
with diffuse emphysematous changes. These areas appear
acute speech difficulties, accompanied by left hemipleg!a
white on the X-ray, due to healthy lung tissue, which does
(paralysis on one side of the body). These are classic
tabsorb X-rays, being replaced by denser inflammatory
~onssue, which has a greater ability to absorb X-rays via the symptoms of the Middle Cerebral Artery Syndrome, which
can be caused by an aneurysm of the artery, so the patient
hotoelectric process. The changes become obvious when
~omparing the inflamed .tissue in the m.iddle.and lo:-ver was referred for an immediate contrast angiography as part
of the normal clinical work-up. The angiogram in fact con-
lobes of the right lung with the healthy tissue 111 the nght firmed the existence of a significant aneurysm, which was
upper lobe, or in the left lung. then successfully treated surgically.

figure 24.21 Cerebral angiogram revealing an aneurysm in middle


fg cerebral artery.
u

Figure 24.20 Chest X-ray of a patient with emphysema. Note the SUMMARY
change ·
s in the middle and lower right lung.
DEFINITIONS
• (Photon) Yidd )':
X The last clinical example w1: consider involves an
·ray
. t ccI1n1que
· known as contr:ist ang1ograp · I1y us1.:d
. ,\'(l )
to..V1sua1·
· ' lie t11c tnstdc, or lumen, ot loo vc.: ss -· with
. . · l l d •"'els r (L) = T
Part1cul· · o
th . ar interest in the arteries. The teeIin1qm: · • i1l\'olves
rac· intravcnous administr:ition of a metallic. and. thus N(/.) is the numhl·r ofphntnns passing a sampk of thickness
d10-op·1qu "' / •• N0 is. the numher of incident photons.
•••.
' C, Contrast agent. .f Ile contras ·t •·1••e11t
e ty111cally

CHAPTER 24 I DIAGNOSTIC X-RAY IMAGING

Scanned with CamScanner


.. lo; :
.

• ll.1lf-,a lu ...• hl)cr (Ill '/.) is the dt•pth at \1hi h 50'}. of the • l•h,,tnn cm:rny
o
uml atomic . .
numhl·r 1.-dl.'p...·nJ··r
muss ulll·nuati1111 ern:tl1e1c·11t µ/p :
' icc or
t1rc
numn.:r of incident X-rJ) photons urc stoppt•J. It relates lo
the linl·ar :ittcnuati1lll rn...·llleicnt µ : ( 1i J11) 1 1 -Y c1111~t for /:'rt"''"" • I 00 le V

<1 11)1 L <J. (I //:'"""""') for fr"'''"" ... Joo l cV


l/l 'L c: 1112
µ
CE stands li1r Compton cffrct. The rncllicicnt i .
~ lndc.
pemknt of Z.
• Contrast C bl:1,1ccn any t\\ o points A and B of an image is:
• Photoelectric clfrct
• Enc·rgy balance:

F.~ l.m. rlcl·trun .. £"rhoton - Eh111dmg


Points A anJ B arc chosen such that 1\ 'A ~ -"w r is the yield
and,\' is the number of photons passing the sample of giwn
thid.n..:ss. t:.r-.m. c1
{'1.' lhl ll
is the kinetic energy of the electron• Eph
is the incident photon energy, Eh•ndm~ is the hind'i~~
energy of the c Iectron m . Its
. orh'Ila I. g
UNITS
• Linear attenu:ition codlicient µ: m • Photon energy and atomic number Z-dependcnce of the
mass attenuation coefficient:
• l\13ss att..:nu:ition coefficient µ/p: m~/kg
• Exposure R (roentgen)" ith: z )3
( £rho1
{µ /p)PE oc - -
011
Q ion pairs
IR = 2.08 x 10 - - -
cm3
• Pair production

I R = 2.58 X 10-~ ~ • Energy balance: we require Erlwton ~ 2nrcc2; that is,


kg 1.02 MeV. nrc is the mass of the electron
£photon?'.:

LAWS
• Ratio of energy loss due to X-ray radiation to thermal
energy in the fonn of heat Q:
• Attenuation
• Total attenuation with the linear attenuation coefficient
(Beer's law):

• Z is the atomic number. £km is the kinetic energy of the


incident electron in unit eV. The pre-factor in the bracket N0 is the number of incident X-ray photons. N(x) is the
leads to the energy fraction in percent(%). number of photons transmitted to a depth x, andµ is
the linear attenuation coefficient.
• Compton scattering
• Energy balance: • Total attenuation with the mass attenuation coefficient

N(x) = N0 e-<µlrlr.•
£rhoton. i = £photon. f + £electron
µ./p is the mass attenuation coefficient, pis the density
£ phOhlfl.I. is the energy of the incident photon. EphOlon.f of the tissue.
is the energy of the scattered photon, £cl<ctron is the
energy of the ejected electron.
• Scattered photon energy:
CONCEPTUAL QUESTIONS
0-24.1 . Explain the difTercnce between Rayleigh scattering
£phoron. f = and Compton scattering.
Erhnron. i(
+ - -- I - cosO) 0-24.2. Describe with the help of a diagram the generation of
mcc2
X-rays by the bremsstrahlung mechanism.
Ef 11Uh..>n, I. is the energy of the incident photon. mr is the 0-24.3. Describe briefly the di!Tercncc l>etween the photoelec-
electron mass. c is the vacuum sp...·cd of light. and O tric effect and Compton scattering.
is the angle of the scattered photon with respect to the 0-24.4. I low arc electron- positron pairs produced'.'
incident trajectory. 0-24.5. Explain what is meant by contrast of an X-ray image.

662 PART SIX I APPLIED CLINICAL PHYSICS

Scanned with CamScanner


Diagnostic
Nuclear Medicine
Imaging

uppose you ,~·ish to understand why a sophisticat~d


25.1: Historical Introduction
S piece of equipment such as a modern automobile
e\periences a mechanical malfunction. In general,
there are two approaches: you could visually inspect the
Nuclear medicine is a branch of medicine that employs
radioactive isotopes, also referred to as radioisotor>cs or
components you suspect are the cause of the problem and radionuclidcs, for the diagnosis and treatment of dis-
th.:n perhaps repair or change them. or you could inter- eas·e. In this chapter. we will concentrate exclusively on
rogate the automobile·s electronic computer to search for the diagnostic aspects of nuclear medicine; we leave the
clues to the cause of the malfunction. Both approaches are consideration of therapeutic applications aside. In nuclear
ralid, as they complement one another. and technicians medicine, radionuclides are chemically bound to physio-
11ill often employ both methods to properly diagnose a logically important molecules to form radioph:trmaccu-
malfunction. The first method relies on visually detecting ticals. These radiophannaceuticals arc then administered
structural abnonnalities such as a broken belt. a detached to the patient via intra\'enous injection or oral ingestion.
gear. etc. The second method relies on the analysis of after which they will localize in specific organs or ana-
functional information, such as whether the engine is tomical structures. External detectors can then capture the
operating at the proper temperature. or whether the fuel radiation emitted by the radiopharmaccuticals. and this
meets certain requirements of purity or octane content. information can be used to mathematically rcconstnict an
Why do we bring this example up? Because the practice image. The physiological nature of radiopharmaccuticals
of modem radiology. the branch of medicine that concerns allows nuclear medicine to diagnose disease based on the
itself with the diagnosis of disease through images, uses status cellular function or physiology. rather than relying
the same two approaches. On the one hand we have such on anatomical changes, as do other well-established
techniques as com·entional X-rays. computed tomography, imaging modalities such as X-rays and ultrasound
ultrasound imaging. and mammography, which look at imaging, which were discussed in previous chapters.
structural or anatomical abnormalities in the body. On Typically in disease. physiological changes precede ana-
the other hand. we have nuclear medicine, which focuses tomical changes by weeks or months. Hence. in many
specifically on functional or physiological abnormalities. instances an earlier diagnosis is possible with the use of
fl'.agnetic resonance imaging is a special in-between tech- nuclear medicine. allowing treatment at an earlier stage.
~ique as it can be used for either anatomical or functional which can translate into a higher cure probability. This
unaging. depending on the specific application. Of course advantage is especially significant in the management of
the optimal clinical strategy is to combine the strengths malignant tumours, or cancers.
of _both anatomical and physiological imaging methods The discovery of artificially produced radioisotopes
to tncre
This. ase ~he probability ··
of makmg"':'"
a corre_ct d.1.agnos1~.
· in February 1934 by Frederic Joliot and Irene Joliot-
15 precisely what the modern practice ot radiology JS Curic can be considered a most significant milestone in
11
a about. In this chapter we focus on some of the math- the history of nuclear medicine. An equally important
ematical
. • Ph)'Steal,
· and clinical principles of funct10na· I or turning point was reached in December 1946 when an
Ph·Ys1olog11·ca I imaging.
· · d'
that is, nuclear me 1cme.· article describing the successful treatment of a patient

CHAPTER 25 I DIAGNOSTIC NUCLEAR MEDICINE IMAGING 665

Scanned with CamScanner


'•

photon~ 11> u chaructrn-;ur (lf the crv~tul u."ed A tilllf:k


m l•mtnf' f.Umrnu ru~ 11> capahk of pmdur:mr th u!.nnd.\
of the-.c: '11>1hlc: photon,1,. hut the 1mponn111 concrf11 1~ thal
the numher of phl•tc•m pmdw:ed 1;. ulway;. prctjl~111 1 onul
to the enerh: of the: on~mal mrommf> p .1 mma ni~ Thi;.
1 a d1rc"C"t con;.c:que11CT of enc: ~-' con;.en ut1or. Crys:ab.
are typ ical!~ con<;t:uc-tc-d of ' ul. w wh1c mmui: umoun~
of tMll 1um ure add::d for 1rnpn" ed !>cmtil!m1on perfor-
m:i.nce. There arr sn era! rea;.on;. wh~ 'ul 1;. cho. en
25.4: The Gamma (At ger) Ca::i~:-3
for tht!> appltcut1on. amling them 1L., relut1 , d~ ht!-'.h den-
it) (3 .67 g c-m1 ) and the htf!h au in11c numher of mrline
CZ =- 53) Bcith fanors mcrem.e tht:' probahiln~ (lf ca~
-
ln the U.n: J lh. (H-.. 1n::- ~11mmn er. ti.. fl!!t"J" . c.nn 1!!':"ll \",.
turing the incoming g.amma ray!.. which f'.l\!"S 1\a] a htf!h d,.,,# J ·m~d tr\. Hu] ".nr=:
- - ·1- -
or ~ l.:;cr.1::a . e11::rrc:::-
-
WL". '·r-
...._
al the l •m1 e:-1:>10 o: [ ui!f:1:111L l : H Ile m:·•· v·au:: 1 tt::~
gamma-I~' 1sihle sc:mttllatH•n c<in' en.ion efftc:ienC') .
These c:ryst.als dci. howe'er. hil\ e the d1sadvantuges of in.o;rrurru:nt 1 nu.~ l::u· m!::k:111!' 1 : 1uc~ um. h...-n:::. 1. : i-.it
being ' el) !>en. 1ti' e 10 mechanical and thermal loam Cthal cl.-scrih!:' IL'- h:!Sll 07'~..l!Jnf plll.'.:ljl ~ h._..,-_ ll!. ~
is. rapid temperature changes) that can result m fracture . cmnern can s 1multane'.lu!.l~ r:::::rr nm'. rrt.:::!.\trr:: lit. r.l.i:i-
Their performan e is ab0 degraded by humidity. thus they i:;0tojl!:' cnn::ent::-a~1 ~n m ITU.It'~ er.pm.'- IT! iu. fi:.:1t o' \ t::•
need to be sealed hermettca ll~ . as a funcrnm of trme Th= ;inys 1~ln~1::a~ r::1=-rnn.:::. n: ~­
The !>ec:ond st.er in the process. the conversion of is th: ahihi' w ex.a...T!llne 1h: :-at::~ n.: v·n1:t r: :-.ui1 ~::.~
visible ph0tom mto elec:tncal pulses. tales place further mac::utical ;!- accumulat.erl er :::l 1mma~ fr:Jrr L D:mJ.:Ur:r
downstream m t.he photomultiplit>r tube (PMT L which organ.
is a glass-encased structure under vacuum. The ph01ons TI1!:' first com?nnen: rr. E pimmr: .:a~~ L' t tr.:'..!:.'
strike the front end. or photocathode. of the P.MT and. 4:i:n:ular ~al !.:::m:illati:m cr:-·s-.a' o: tn:.' typ:.> a.:~ih=r. r::--
through the photoelectric: effecL electrons are ejected. The \·iousl~ . ·wnh a dmrn:.:l!:T of amunc 4 ~ 1 ~ nn t tn~"l'.!i·
ratio of electrons ejected per incoming photon is referred nf l O mm Dir~J~ h:::hm: th:.' C0'Siul 1< m: !L.!l~ cf !<
to as the quantum efficienc-~ of the PMT. and il 1s always 37. or 61 PMT!.. d~endmf m:: th~ d:.:~t:r. s :..z.:: :mr: run:-
less than umt) . Some dtstance awa~ is a metallic swface.. consid!:rat1 ns. thm d=e::t s::m:iTla:1:lr fizsn~ 1~ tr
called a d~ilode. t.hat is kept at a positive 'oltzge such the crystal ~ mci.:l::n: pumns rn~'!- Cn ~ nu~ si.ci:1.=
that the electrons are attracted tCI iL A foe-using electrode of the cnme:-a an: drre.c:!~ he:v.·e-"T. th!- f:amtr.:. ..nut::.
may be placed berv. een the phot.ocat.hode and the d~nnde organ and tht" crySUll. ar. e xte:nal r:.:m:>1·nhl= .:nlhm:.1:....
to limit elenron diH-rgenc:e due to electros:tatic repul- is mount::d. Th1!- col11Tru1rn: 1s man!' r{ c h.::...,,~ tn!1
sion. When e:tch incomin~ elecrron stril..es thjs d)ilO!k.. sucb m, lead. nnd n contntT'~ mnr~ lh:11!.'!lna..;. n~ Ut'~ r-n..
several second~ electrons are ejected. The ratio of sec- alleJ hol~ wh:l;;;!:' fun:.:t1:l. n 1!- t :' nl1:--v. :m·, tn:. rr:m~.!­
ondary to incoming electron is called the multiplication sion to the crys:al of f:Rmnu. rcy. cnl.!:.mf n::r.:i.lL::. tr !:'.:
factor. \.\ hich is constant for all dynodes and it 1s always b oles.. whii!:' ahso:tiinf :2y !- ::nt=JTlf :.i.: ~lan: m1~ 1:s 7r.?-
greater than unity. The sec:ondaJ: ele-ctrons are ai.L:ract.e!l results in a rt"du.:7ion nf JTrul.L'e hl:.i.; n;:du-~.; h s::i.tl!r:t
to a second dynode. and t.he process 1s repeaterl thwugh ra.Ji11ti0n assn.:uned "'·1:. cn-mma ~"· 1n.::i~n,· m s!:mt:t
some 9 to 12 d)ilodes such that t.he O\ erall electron mul- angle!.. m illustratd m ft~. 2: .:; -
tiplication factor is exponential. The final electron bundle \\ nen a gam:na re~ ;!- m.::1.: km 11' n v ::7lo: m r...:r
ejected from the last dynode is colleded at the PMls ·v mical d1rec.1~n. n will Tll!.'-~ tn::•uc , c !to!:- n th: ·~
anode and sent downstream for fort.her processing. The 1imaior nnd p:-,•du~ a s~:m:1l1n:1or. 1 th:.- .::~:~. ~
total charge collected at t.he anode is proponional to the positioning c 1:-cuiu, th~ det.c:mrn:· wh~~ 1r. .tl\t .:;....:.i
number of photons striking the photocathode. "hich is the ~intilln::1o;i e\·em to~I.. p ln:.-.c 'U. '~n c F-:m1iJ ..1n.ir

\ I
f.~-u•;ini: f>r no.:le
rlr-nr dC'

658 PART SIX I APPLIED CLIN ICAL PHYSICS

Scanned with CamScanner


] - - - Collimator

} - - Detector cryMal

] - Pho<om"h;pJ;,. '"Ix ' " "

::J-- Position logic circuits

Detector cover

COMPUTER

figure 25.3 Major components of a gamma camera.

takes place, light will be emitted isotropically such that the crystal, a two-dimensional map of the radioisotope
all PMTs will be excited to some extent. The intensity activity distribution may be obtained. This map is referred
of light reaching a given PMT will depend on the dis- to clinically as a nuclear medicine scan. An example of
tance between it and the scintillation event. Since the such a scan is presented later in the chapter.
signal collected at the anode of each PMT is proportional
to the intensity of the light reaching it, by the clever use I liUlll1i The gamma camera is an instrument that can
of electronic circuits the distance between each PMT and simultaneously record and measure the radioisotope con-
the scintillation event can be determined. With this infor- centration in many organs as a function of time.
mation. the exact location of the scintillation event can
~:~:: i always be computed; in the early gamma cameras this
process was accomplished with analog electronics, in
;~u.·::
~e more modem units it is done digitally. By adding the
25.5: Single-Photon Emission
intensities of all the signals of all the PMTs, the energy Computed Tomography (SPECT)
of the original gamma ray can be computed. A special Imaging
energy filter called a pulse height analyzer is then used
to selectively allow only gamma rays of a specific energy A fundamental limitation of the gamma camera is that it
to be recorded by the camera, while rejecting all other produces flat, or planar. images in which different organs
gamma energies. This is particularly important because if are superimposed. or "fused:· on each other. It may there-
an organ is being imaged with Tc-99m with 140 keV pho- fore be difficult at times to establish which anatomical
~ons for example. then all other incoming photons of dif- structures are actually contributing counts to an image. One
1erem
b energ1es,
· such as cosmic rays and other terrestna · 1 method of oYercoming this limitation is to combine use of
b~ckground sources. must be eliminated as they create the gamma camera with the principles of computed tomo<>-
u~ ~nd degrade image quality. Finally. once the proper raphy. In this technique. called single-photon emissi:n
Pos1t1on
and 0 r an ·incoming gamma ray has been determine · d computed tomo!!raphy (SPECT) imaging. the camera is
the pulse height analyzer detennines it is of the proper rotated around the patient so that many images. or projec-
energy h . tions. are obtained. each at a different angh:. A special math-
' t e gamma ray can he recorded as a valid count
on a Ph ematical procedure called hack-projection is then used to
be otographic film. or a computer monitor. or it can
13 Stored •n · a sunable
. electronic format for later anaIys1s.
· reconstmct two-dimensional images. or slices. of selected
Y Properl
\\·i·th Y P acing counts on an x1·-plane to correspon d
1 · planes inside the anatomy. compktely eliminating compli-
the ·· ~ · cating contributions from all anterior and posterior planes.
pos1t1ons where the scintillations took place 10

CHAPTER 25 I DIAGNOSTIC NUCLEAR MEDICINE IMAGING 669

Scanned with CamScanner


..

the patient ha. thr~e dist in t region' of act1vit~. with hlac~


l~IK· rr1X't'S:. can ht> comp:m·d IOO!>d~ to studying the inte-
rior of a l11::if of brt·ad "' slicinc it into man\' thin slices and
reprc. enting the h1gh~sl a~d wh1.•c the lowc~L. n1 carrier-a
then record:. a radiation intcn II). or pro_1ce11on Profil
~tudyin:! t':td1 !'lice ca~dully ~1 hile disreg;rding all other
which is simply the ~um of all the radiation pa,~Jn~;
slices momt·m:iril~ . \\"e prest>nt here a ,·ery qu::ilit::irive
thrClugh the cc1llim::itM at each g1\'cn IC1Cat1on !\.ote lha~
approoch to SPECT ima!.!irn!. as the m:11hcmatical dct::iils
of tht' im::ige acquisition 'and reconstruction pr0ccsscs arc where there JS mor.e rndmacll\.11) (hlad. ). the projection
profi le is highest. JI ta ~ e a dip in the region '4.Jth lo11.
much beyond the aims of thi te:\t. and arc encountered
typically in a graduate course in mt•dical physics. radioacti\ it} (white) and vanishe:- at th edge of the
\\'e begin our approach b) considering a gamma patient. .
camera with a collimator th:it allows only perpendicular Now. if the camera 1s allowed to rotate around the
photons to pass through. as seen in Fig. 25.4. Note that patient. as seen in Fig. 25.5. the projection profile \4 ill
change as a function 0f the angle of rotation. TypicalJ
Sum of the camera acquires a prnjccti0n prnfile at angular inte;:
aclrvily vals of between 5° and I0°, such that between .~ 6 and ;~
along hne
ol response projection profiles will he c0llcctcd for a complete ~tiO'
orbit. The data from all profiles arc then pmc.cssed h) an
onboard computer using the bad:-projN·tion al~orithrn
to produce a reconstructed image of a s ingle plane. or
slice. through the patient.
In order to appreciate at least intu iti ve!~ ho~ the
back-projection algorithm works. suppos~ we hc:iin
with a simple point source of radiation. as illu,trat:.-d~rn
1
Ij I
I I Fig. 25.6(A). ~·ote that we folio" the camera over a IRO"
I degree arc and acquire eight projection profiles at 2~S
11 / 11 Ii intervals. With respect to the fixed camera cnord ina~
Uneof
res;>onse
system. the profi le shifts from the right edge 0n the up~:
acquisition. to the left edge on the lo"·er acqui;,it1on If an
image matrix is defi ned uch that it corre"\p<"ind~ rnughl~
to the dimension of the patient slice it5-elf. a- depicted J
Fig. 25.6(8). then the profi le for each angle can he '1tack-
projected:· so to spcaJ.:. into this matrix. Typical maim.
sizes used in clinical practice arc 64 " 64 or 12 " 12~
rtgUre 25.4 Generation of a projection profile in a SPECT system. pixels. It is important to rememher this had-proJC~tion

670 p;,QT SIX I APPUEO CWil(Al PHYSICS

Scanned with CamScanner


u_
y~ I
~~
Backprojection
of profile at O"
across Image
matrix

Backprojection after
Object 2 angles
. . . ..
Bangles 256angles

.... . . ' "


1 • ,, ... .
• • • 4•

.. "...... ..-........"'·1·:.:

:~

"' ...
I
.:·; .1: · ~ .. " '":
I
,. ,,
t I
I
.
, ..

~
~ to
~ ...
• '-

A B

figure 25.6 The steps in bac~·projection. {A) Generation of projection profiles at different angles. {B) Back-projection of a single-projection profile (top),
and generation of the image using multiple back·projections {bottom).

happens in a virtual or mathematical space rather than referred to as positron emission and gives the technique
in any physical dimension. We see the first such back- its name. Positron emission was discussed in detail in
projection corresponding to the upper acquisition angle Section 22.3 of Chapter 22, and the reada is referred to
in the top illustration of Fig. 25.6(8). Also, in the same that discussion for further information. Once the posi-
figure but below, we see back-projections ·of 2, 8, and tron is ejected within an organ, it tra\·els a fraction of
256 angles, respectively. Note that, as the number of a centimetre before it interacts with a nearby electron
angles-or profiles-increases, the quality of the recon- by a process called positron annihilation. In this pro-
struction is improved. A fundamental limitation of the cess, both the positron and the electron disappear and,
back-projection algorithm is that there will always be in order to conserve charge, energy, and momentum, two
some image blur present due to some counts being incor- 511 ke V photons are emitted at nearly 180° to each other,
rectly assigned to locations outside the true location of that is, along a nearly straight line to satisfy momentum
the object. In practice, very sophisticated filtering algo- conser\'ation. Furthermore, the energy of each photon is
rithms are used to eliminate these undesirable blurring exactly equal to the rest mass energy equivalent of the
effects. but we will not address such advanced topics electron or positron (both are identical). as required by
here. In order to save time in the acquisition of images, conservation of energy and relativity theory. Because
modem SPECT systems can have two or three cameras the photons are emitted opposite each other, in order
attached to the same rotating gantry, such that the data are to detect the photon pairs a PET camera consists of a
gathered two or three times faster than in a single-camera ring of stationary crystal- PMT assemblies surrounding
system. the patient. Based on which two crystal- PMT assem-
blies are activated, the diameter (or secant) line along
which a given annihilation event took place can always
25.6: Positron Emission be detennined. As opposed to SPECT, no collimation is
necessary since only events that simultaneously activate
Tu_mography (PET) Imaging diametrically opposite pairs of crystal- PMT assemblies
Positron emission tomography (PET) is another form are taken into consideration. The lack of collimators
of nuclear medicine imaging that has similarities, as makes PET much more sensitive than SPECT; in other
~ell as differences with SPECT imaging. Like SPECT, words, less radioactivity is needed to record counts.
It r I' ' · Such events are referred to as coincidences, and all
e ies on back-projection for the mathematical recon-
shtruction of tomographic slices of the anatomy, but both other signals are electronically rejected. PET also has a
t e ph 5 . d. higher spatial resolution than SPECT, meaning smaller
Y ical and the biochemical principles employe m
anatomical structures can be resolved. Finally. the data
~~neratin_g the data differ significantly from SPECT.. 1n
set consisting of all coincidences is mathematically
T, radioisotopes with half-lives of the order of mm-
lites v.h· h . reconstructed using back-projection, much like it is in
.' ic decay by emitting positive electrons, or
Positrons . I t in SPECT. The data acquisition process is illustrated in
th · arc used. This process in wluc 1 a pro on .
Fig. 25.7.
e nucleus ejects a positron and turns into a neutron ts

~l CHAPTER 25 I DIAGNOSTIC NUCLEAR MEDICINE IMAGING 671

Scanned with CamScanner


. the hi<o1 h cost of the camera
1s .
and cyclotron--thc
. . dC\t1ce
.
t produces the F-18- typ1ca 11y running into the sevc
tha . · · h' rat
millions of dollars. thus I '?11tmg t is tee1inol.ogy to larger
institutions with appropriate . budgets. A picture of the
author about to undergo a PEl scan for rcscarc~ pu~s
is shown in Fig. 25.8.

Agure 25.7 The principle behind PET imaging. Note the multiple oppo-
site pairs of photons generated that are detected in coincidence by dia-
metrically opposing pairs of detectors. Figure 25.8 The author about to undergo a PET scan for research
purposes.

One significant clinical advantage of PET with Contrary to Tc-99m and other radioisotopes used in
respect to SPECT and conventional nuclear medicine SPECT and conventional nuclear medicine that are pro-
(that is. the Anger camera) is that radioisotopes of bio- duced in a nuclear reactor as described previously. the
logically important elements such as carbon, nitrogen, positron-emitting isotopes used in PET are produced in
and oxygen are used. These radioisotopes are used to a special circular accelerator called a cyclotron. Due to
label biologically occurring molecules that then serve as the extremely short half-lives of these isotopes, cyclo-
exquisite indicators of physiology, since the body cannot trons are typically located either on-site or within a few
differentiate between a radioactive and a non-radioactive kilometres from the PET imaging centre. Because the
carbon, nitrogen, or oxygen atom. Jn SPECT and con- method of production of PET isotopes differs fundamen-
ventional nuclear medicine, non-biologically occurring tally from that of isotopes used in conventional nuclear
radioisotopes such as Tc-99m are employed, which may medicine and SPECT, we now turn our attention to some
not be fully representative of natural physiology, as the specific physical and mathematical aspects of their pro-
body considers them foreign substances. The single most duction. The typical production scheme involves high-
often used radiopharmaceutical in PET is fluorodeoxy- energy protons accelerated in a cyclotron to tens of
glucose (F-18 FOG), in which one oxygen atom in the mega-electron volts (MeV) of kinetic energy and col·
glucose molecule is exchanged for a positron-emitting liding them against neutrons in a suitable target element.
F-18 atom. The resulting glucose is structurally very The neutrons are knocked off the nuclei while the pro- l

similar to natural glucose and has a physiological behav-


iour that is essentially indistinguishable from it. Since
tons are captured, thus turning into "proton rich" posi- ~!.
tron emitters. We define the number of protons/second
•'>
..
glucose is the primary fuel used by cells to power bio-
logical processes, its rate of consumption is an excellent
striking the target as the proton current /, the number of
neutrons/cm 2 of area in the target element as the radio-
j
/~.·
indicator of the state of health of a given tissue or organ. logical thickness n, and the area of each neutron in cm2
A practical example of this is the use of F-18 FOG in the as the cross section u. Then the number of radioactive
, ..
diagnosis of cancer. In cancer, the rate at which malig- nuclei
. or atoms Nrad created in a time interval /J.t will be ; .
nant cells divide is increased, thus increasing the power given by: · -~~
.,,.~
requirements since cell division requires energy and thus
glucose. By quantifying the rate at which F-18 FOG is (25.8) . . .:~
consumed in tissues, and comparing these numbers with
but they are lost due to radioactive decay at the rate of . ~
baseline rates for healthy tissues-along with other clin-
ical considerations-a probable diagnosis of cancer can >..Nrad· So in a time interval lit, the net change in the :,:]
be made. Other clinical applications of PET include car- nuodmber of radioactive atoms D.Nrad is given by the number · ·Al
diology and neurology. A distinct disadvantage of PET pr uced minus the number lost, or: :..;.,
;'~
with respect to SPECT and conventional nuclear medicine · ll.Nrad = Inu!l.t - >.NrJdAI. (25.9) '.'~J
. 4 •
,::f.l
•.i:U
672 PART SIX I APPLIED CLINICAL PHYSICS NEL ·:~i}

j}.f:~
Scanned with CamScanner
. differential equation and it can be solved b
15 8
fhiS th Jimit as /lt approaches zero and using calculu y 25.7: Clinical Applications
s)ll·o"t: e • s.
I fution is:
fhtSO Nuclear medicine in any of its three modalities is fun-
lnu(l - e-A9 damentally a functional or physiological, rather than
N{t)rad = ,\ (25.10)
anatomic imaging method. One of the most frequently
performed procedures in nuclear medicine is the bone
, e what we are really interested in is the activity scan, with the intent of assessing the state of health of
.<ol SJllC f .
';' function o time: bony structures. Because of its clinical importance, we will
A(t) as a
A(t) = AN(t)rad = lnu(I - e-"0. (25.11) examine some basic aspects of this procedure. The proce-
dure starts with a molecule called methylene diphospho-

:eJ , quantity /nu is referred to as the saturation activity


since, after a sufficiently long period of time, this
be the maximum value the activit~ will approach.
nate (MOP), which has a biochemical behaviour similar
to the Ca.. 2 ion. The ca+2 ion is collected and metabo-
lized by the internal structure of healthy bone-playing
()!her physical phenomen~ ~hat are desert bed by the same an important role in bone physiology-particularly in
equation-and hence ex~11b1t the same sat_uration charac- bone growth. Therefore, regions with active bone growth
fdjstics-are the saturation or final velocity of an object will have increased uptake ofca+2, and hence MOP. The
. free fall through the atmosphere, and the charge of a MDP molecule is labelled with Tc-99m and adminis-
1
~argiog capacitor in an RC circuit. Fig. 25.9 shows a plot tered to the patient intravenously. It takes about 3 hours
:f A(t) for the production of a typical PET isotope. after administration for the Tc-99m-MDP molecule to be
properly deposited in the skeleton, and imaging is per-
-g formed typically at this time. The usual injected activity is
JOO ------ - ~ around 700 MBq. Certain types of cancers, such as those
~ of the breast and prostate, have an unfortunate tendency
~ ~ to spread quickly to distant organs, predominantly the
i j bones. Such spread is called metastasis and is typically
~:- -:: ~ associated with terminal disease, which can be treated,
j ~ but not cured. Metastatic deposits in bone are associated
~ ~ with rapid but abnormal bone growth that can be detected
~: easily with a bone scan weeks or months before they can
·~ ~ be detected with conventional radiological methods such
~ ~ as radiographs or computed tomography. This early detec-
·:: ~ ~ lion capability occurs because, in disease, physiological
'f· i ·~ changes precede structural anatomic changes by weeks or
7F f ~months. When an early diagnosis of metastasis is made,
0 2 3 4 5
Time of irradiation in units of the half-life 276
< - appropriate treatment can be initiated earlier, which may
l· r111ure 25.9 Plot of activity A(f) as a function of time during cyclotron be associated with longer and better quality survival. It
iradialioo fix' the production of typical PET isotope. Time is expressed in also avoids the unnecessary burden of surgical proce-
lllils of halt-rrves, and activity as a percent of~· dures when it is clear the disease is terminal. Because of
its inexpensive, safe, effective, and non-invasive nature,
combined with its early diagnostic ability, the bone scan
.... •EXAHPlE 25.2 has played a historic role in establishing the clinical impor-
Suppose a cyclotron with / = 1.25 x 1O14 proVs is used to tance of nuclear medicine. Fig. 25.10 shows a bone scan
irradiate a suitable target with n = 102° neutrons/cm for 2 obtained at the author's institution of a patient with pri-
! the PrDduction of F-18 (T112 =2 h). Furthermore, suppose
11 = 3 x 10-26 cm2 for the reaction. Compute (a) A..1 for
mary breast cancer that has metastasized to the skeleton.
It is a posterior view of the thoracic region. The multiple
U.s reaction, (b) the activity after 6 hours of irradiation, yellow areas in the ribs and vertebral bodies correspond to
• and (c} the number of F-18 nuclei (or atoms) after 6 hours metastatic sites, indicative of advanced disease.
, Of irTadiation. We next focus our attention on the case ofa 60-year-old
female patient who presents with a known malignant lump
' Solution in her breast, and was referred to the PET clinic for initial
. Solution to part (a): A = /nu = (1.25 x 1014 prot/s) staging of her disease. In oncology, there are three well-
1 51
MQ20 neutrons/cm2){3 x 0-2a cm2) = 3.7 x 108 Bq. established applications of PET scanning: the initial diag-
· · Solution to part (b): From Eq. (25.11 J: A(t) = nosis of disease, the initial staging of the disease-which
, 11111(1 - e- A? = 3.7 x 10s Bq(l ~ e-(.693X3~ =3.23 x 108 Bq. means the determination of the anatomical extent of the
• Solution to part (c): Also from Eq. (25.11): N(r),ad = disease--and the assessment of the efficacy of treatment.
"'4(t)I~ = (3.23 x 10s Bq)(7200 ·sJ0.693) = 3.35 x 1012 Jn the case of this patient, her initial disease was diagnosed
. ~oms. .
-.. through alternate means, that is, a mammographic exam

CHAPTER 25 I DIAGNOSTIC NUCLEAR MEDICINE IMAGING 673 ' f j

Scanned with CamScanner


Figure 25.10 Oon'
• , c scnn o1 p:itu::nt 1\1111 p111mry bre:ist ~ncer thJt
1
t.1:> metnst:is1:eo to 1110 skrlr tQl1. The multiple yellow nrr:ls m the 11bs
nnd rrrletmil bodirs corresrond to mrtnstntrc s.itt's. Technk:n1scan data
rre3r on the n9ht sldo or the lm..1gc.
l~l ll ll\\ l'. d b\'
. •
tiior'=-.'\'• 1.11 " I11~
[I

. ·I1 :i sm:t II sampk· llf. trssul:"
.
~s su~ r~:i lly "-'lll(l\l'd !C.r l.1lx1r.ll11ry nnnl~ sis. I knl'l'. th.:
lll!l'll~ !'I the pnx·l·dun.· in ltl'f C:tSl' " as 111 lktl·m1irw h1n\ far
thl· d r~l':tSl' h.1d sp1\·ad in ha ~xi~. or. in (lthl·r " onls. th.:
lkll'l'lll' ll l1 t' p111l·111i.1l nwt:ist:llil· sitl'S. St:it:in!.! thl· diSl':lSI:"
is. nn imp1 111:111t nmtpllncnt 111' thl· initial l:linirnl " ork-up
nl n n~ l':lll(l'r patil'llt. :is dilli.·n.·nt st:1nd:trdi1l·d. d inil·:tll\'
Pl\1\l'll tn.·at111l·11t pmHx·11ls l'Xist fi.1r l':ld t sp..·l·itll· SHl!.!l' 1;f
d isl·~tSl'. In pn.·p.1r.1t il•11 !(,r her pnx·l·lhrn.-. tlw p:ll il·n; ,, as
nskl·d 111 fo st for a p..·ri0..I o f 6 hour,; 1t1 lk l'l\':tSl' d uwsl.'
uptake in gl·twral. and to amid till' inl:tl-l· o f anY st ir~rnbnts Figure 25.11 Coroml F•1S FOG image of tne ttnax and at>oo.inei a1 1
60-ye:u-old patient 1\1111 a "110\\TI pnmary ba•3st cancer. tikl'.l' tlr ~­
s11d1 :is 1t1bal'rl\ calfrirw. or akoh11I. as "di a~ an\' ml:"di-
ence of multiple ft>sJOns in the liver and tile s.i~le ,·e.rt?!iraf bi.l..~. 11~0
cations that cciuld confound the n.·suhs. Shl· was ; skl-d to appear as purple spots. ·
fl'lllll\'l' all llll'tallit· ob.kets s11d1 ns .il'" dry. gbsS('S. keys.
l' IC.. "hid1 l'llt1ld lk gr.Hk irn:tgl' quality. Till· p.1til· nt " ns
administl'rl·d :t dtlSl' llf 7-tO ~ IBq ( ~ O mCi) of F-I S FOG than cur:lti' c in intl:"nt. In fnct. this p;iticnt wns refm\"J fi'r
\'i:t in tr.I\ l'lll'US i11j t'l'til111. :ind instnKtl·d to rd:tx for :ihout furtlta tn.·:itmt•nt to a clinical onrol0gis1 " ho S(X'\.'iilizo in
.to 111im1fl•s to :rllnw fi.'r till· distribution :ind uptake of th.: thl:" application of chcmother:ipy.
radinph:mn:tl'l'Util·:tl thrnughout lxxly organs. In prcp:ir:l-
1io11 li.lr artual imaging. till' p:Hil·nt was :ilso instnrck·d to
drink pknty llf fluids and niid till' blackkr lh.'qlll'ntly. The
SUMMARY
patient " as tht•n plal'l'cl Clll thl· imaging tank :ind image
acquisition was initi:tll'll. :t proct•ss \\hich took npproxi- DEFINITIONS
mately I:' mimllt'S. during "hid1 thl' p:ttil·nt was asked to • l'\ur k nr nwd icinc is :i t>r:rnch C\f ml'\lirint' thlt e1:1r?,,ys
fl.'lll:lin still and n.·bxl·d. A conm:il S('Ction of thl' thor:icic r:tdiC':iC'liw isot<'pl·s- :il,;o rt"ti.· m'll "' :i~ rJdit~i;«'l<'!'P N
and alxlllllt in:il t:a\'ities was nbt:1inl·d. whid1 is (ll'\.'Sl' ntcd in r:idionurlidl:"s- for 1h.· di:ignC',;is :md w :itmt'nt N- <li~-.,~.
Fi!!. 25. 11 . In that imagl'. \\l' St't· mult ipk :11\': IS ofincrt•:tscd • The physiological n:iturt• of nudt•:ir m1.-dicinl" :ill," \' t~~
F-l 8 FDG uptake in thl' hcp:ttit· organ (liH·r) that appear as diagnC'sis Pf disc:tsl· 1'~,; <'<l 0n tht· stJtus C'f ,·dlul.tr fai :ti....,
dilfoSl' purpk splits. as \\di as :t ck:ir spot on :t \'t't1l'br:tl rnther than relying l'll :rn:itomir:il rh:rns:~. 11,; \'tht•r in\J!-"-
body around T-12 or L- 1 ( 12th th0r.wit· 0r Ist lumbar lxxly). ing m0\blities Sll('h ns :\-ra~ s. CT. and ~ltr.1>1'1111.I im.1~tl'..;'.
ThcSl' PET tindings art' consistent with ml.'t:tstatic disease. • Thl· g:imma rnmt•r.1 i,; :m in>tmnwnt th.II nm s1111ul1:1:w·
which ml·:tns that thl• carK·cr has spread !Cl distant o~a ns l'llsly fl'l'flrU nod 111 ..·nsun.· th..• r:idi<'iSi'll"pt' •<'lh."t'ntr.llll'11 1n
and is rims at an ad\':tnt·ed. potentially tem1inal stage. The many l'rt::ms as :1 fun•1 i1'll l'f tinw. " hi•·h :ilkm s Siii\~~ ,,f
onl v tn:atrnenl option a \':tibbk nt this point is chcmo- tht• r:tll·s 111 "hidt n r:t.li0ph:m11a,·,·uti•:1I is ;1,"l.·u111ub1,.J '"'
thc~ipy. which is systemic in nature. nnd pnllinti,·e r:lther eliminntl'd fr1,m n particul!\r nJ1!.m.

~-
PART SIX I APPLIED CLINI CAL Plt YS ICS
6 74

Scanned with CamScanner


'l'I CT ja1111i;111~ i'i 1111 C\ 11l11tiu11 Of the gn11111111 camera Ulld
• : 1, (~ _.111i.-\·11ni; ii.Hu II\ a a 360° ungk, und rernri 1 , 0-25.8. lktl·rmine tire minimum total angle that a SPECT
""'' . . . ~ flllt· camera lras to rotate through in order to acquire suffi·
.. (IUlh)l:\lflJ'lll\' 1mugc u'mg a 111athl·111aticul kch 1114uc
·
11 1~.. . . cicnt d<tta to rcconwuct a full image of a slice through
,alll'.I !llld:· p111J1'1't11111.
a patient. In working out this problem. a series of
rrr i11u1giug i!> u ,~,~~rl'llt lll~l~lrty tl~at. im.ulvcs dckcting sketches may prove u~cful.
• 'hil ,111 phul1llls fi1nn u (lll~llrorh'llllllrng 1so11,1..... Its . 0-25.9. In PET, the use of collimators is not required. v. hi ch
onnr •111 . . ""· marn
ad,•ani.1sc is that 1s..1h•1x·sof naturally c>1:nrrring clements such results in a greater ~ensitivity compared to SPECT.
l~ ('wNll• 1>X)'gc11. an~l nrtrc1gc~1 can be imaged. It is the only Explain with a diagram why collimators arc not required
1ncth<lll cum·ntly U\arlahlt· to rm:ige glucose metabolism. in PET.
0-25.10. In PET, is it possible to have a coincidence from an
annihilation event that does not occur along the straight
UNITS r d' . . line joining the two activated crystal- PMT assemblies?
• lllc d1'(·uy.ratc ~ aSlra ''.''Sot,~(~~ is referrc:d to as the activity, If so, explain with a diagram.
and it is given 111 units o s or becqucrcls (Oq).
The dt-cny constant A of e:ich isotope is unique and is given
I
in units of s- 1. ANALYTICAL PROBLEMS
P-25.1. Show that AT112 = In 2.
LAWS P-25.2. Show that. in a generator at t = Im• A13(t) = AA(t).
1
The activity of a ra.dioisotopc as a function of time, A(t), is P-25.3. Show that, in a generator. the maximum activity of
gi,cn by the equation A(t) = A0e-A', where A0 is the initial Tc-99m is reached at t :::: 23 hours.
activity. P-25.4. For a 50 mg Ra-226 sample (T112 = 1600 years) with an
If parent radioisotope A dec:iys to daughter radioisotope o, activity of 1.83 x I09 13q. compute (a) A; (b) the number
1
then the activity of D as a function of time is given by of atoms in the sample, based on mass considerations;
and (c) the number of atoms in the sample based on its
A8(1) = AA(t)[ ~1 ][1 - e- <Au - AA)'], P-25.5.
activity.
In a special generator, Te-131 (T112 = 30 hours) decays
An AA
to 1-131 (T112 = 8 days). Initially, the activities of
\\here AA(I) is the activity of the parent at timer.
Te-131 and 1-131 are 1.85 x 108 13q and zero respec·
1 After a sufficiently long time, the ratio of parent-to-daughter lively. (a) Compute rmfor the 1-131 activity. (b) What
activities is given by the approximate expression are the Te-131 and 1-131 activities attm? (c) How many
A 13 (t) A11 atoms of each exist at Im?
- - =: P-25.6. Suppose a given crystal- PMT assembly has a quantum
AA(t) A13 - AA. efficiency of 0.3 and a multiplication factor of 3.
1 The activity of a cyclotron-produced isotope as a Furthermore, the crystal is capable of generating a
function of irradiation time is given by the expression visible photon for each 30 eV of energy deposited on
A(t) = Ina( I - e-AI), where Ina is the saturation activity. it. The PMT has 12 dynodes in it. Compute the charge
collected at the anode when a single Tc-99m 140 keV
photon strikes the crystal.
CONCEPTUAL QUESTIONS P-25.7. A cyclotron with I = 1.00 x 10 14 prot/sec is used to
irradiate a I g 0-1 8 target with 11 = 5 x 1ow neutrons/
0-25.1. Explain how nuclear medicine differs from other cm2 for the production off-18 ( T112 = 2 h). Furthem1ore.
medical imaging techniques, discussing relative suppose a = 3 x 10-2<> cm2 for the reaction. Compute
advantages and disadvantages. (a) A sai for this reaction, (b) the activity after 6 hours
0-25.2. Diffrrcntiate by structure and function, a radioisotope of irradiation, and (c) the ratio of F-18 to 0-18 atoms
from a radiophannaceutical. after 6 hours of irradiation.
0-25.3. Explain the advantage of employing a Mo-99ffc-99m P-25.8. For the situation described in Problem P- 25.7, compute
generator for clinical use in nuclear medicine. (a) the irradiation time required for the F-18 activity to
0-25.4. In voking energy conservation, explain why the equal 109 Aq. and (b) the additional activity generated
number of monoenergetic visible photons produced by irradiating for another 6 hours. that is. for a total of
in a scintillation crystal must be proportional to the 12 hours. (c) Comment on whether this constitutes a
number of incoming gamma rays. Can the constant of reasonable method of generating more F-18 activity.
proportionality be greater than unity? Explain. P-25.9. The human body nonnally has some amount of radio-
0-25.s. With the aid of a diagram. explain how a collimator activity due to a variety of factors. most notably the
works to reduce the scattered radiation reaching the uptake of fallout from atomic tests in the atmosphere.
detector, hence improving image quality. Assume 32 million atoms undergo radioactive decay in
11-25.s. S_uggest a method by which the distance between a scin· your body every day; compute your activity.
trllation event and a particular PMT can be detennined P-25.10. Based on your knowledge of radiological sciences
'." an Anger-type camera. and any additional information you can find, explain
11-25.7, why. prior to a PET scan, the patient is usually asked
Suggest an explanation for how the use of multiple
~etectors in a SPECT system can save time in the to drink plenty of fluids and void the bladder
frequently?
image-acquisition process.

CHAPTER 25 I DIAGNOSTIC NUCLEAR MEDICINE IMAGING 675

Scanned with CamScanner


• Velocity v: mis
bject is
• Acceleration a: m/s2

1roduce
MULTIPLE-CHOICE QUESTIONS
MC-2.1. A student walks from home to school and back again.
Which of the following is most correct?
(a) The student has zero displacement and zero aver-
duce a age speed.
(b) The student has zero distance travelled and posi-
tive average velocity.
tstant:
(c) The student has zero average velocity and zero
. distance travelled.
(d) The student has zero displacement and positive
Llrve at average speed.
MC-2.2. An object has zero acceleration in the x-direction. In
the y-direction, its acceleration must be which of the
veloc- following?
(a) positive
would (b) negative
(c) zero
(d) Any of the previous answ~rs is possible.
MC-2.3. A father grabs his daughter by the hands and spins her
around him in a nearly horizontal circle. The daugh-
ter's motion is clockwise around the father when
ticular viewed from above~ The daughter's acceleration points
in a direction that is pointed in approximately a direc-
tion that goes from the centre of her body to her
(a) ·right.
(b) left.
, by an (c) father.
can be (d) feet.

Scanned with CamScanner


ANSWERS TO CONCEPT QUESTIONS Concept Question 2.4: (b). If the maximum is equal
to the average, the minimum must also be equal to the
Concept Question 2.1: (c). Motion to the right is shown average. For example, if your highest test mark in this
by increasing x-values. Motion to the left is shown by class is 80%, and you have an average of 80% in this
decreasing x-values. Stopping produces constant x-values. class, you must not have had any marks higher or lower
than· 80% in this class.
Concept Question 2.2: (d). Start and end points can give
infonnation on displacement, but not on the distance trav- Concept Question 2.5: (c). The slope of A is larger than
elled between those points. the slope of B, but the slope ofB is increasingly positive,
Concept Question 2.3: (b). The magnitude of the average whereas the slope of A is constant.
velocity is the average speed for the straight-line path Concept Question 2.6: (d). According to Eq. [2.27],
between the initial and final points. Since no path from the a .L = 0 when r = 0 at both the North and the South Pole.
initial to the final point can be shorter than a straight line,
any deviation from that path makes the average velocity
smaller and the average speed larger.

'. ·-·
' . ., .

..· .. . ',,

':

di .. .

..• .


.

,,

..
· ; J ..... •• • • • • • • •
• t
·.
'. "·.· ' .
".
!- ... ·

.;

·I
.... ~· • • ....... "I'

..
.
,~ .

.... .
• . . .·. ,.
.. • I "" •
.
·,
... .
~ ., .
;
~ r.

,,..
...

;
. ~

;,:: ... . 1
:>
'

. ·, ... '· :·_. .. ~ \ I" ....,

. ..
...· "i .
. "':: -;-:. ·", .

' .~ ' .. .
.... • .: .·.... \.
., ,, .
· ·: ._..
I •

Scanned with CamScanner


Mc-9.6. Fig. 9.32 shows ap-V diagram with the initial and
MULTIPLE-CHOICE QUESTIONS states of the system indicated. The sysie111 fi~
. undt:r
a process that follows the I me s1iown in lht· ,, . g\\-1
Mc-9.1. Consider a generic system, mechanical or n~n­
u1a''r
During this process <> <l.'ll
mechanical, that receives work. Which of the following (a) the system received a net transft•r of work
mmt take place in this case? (b) the system released a net amount of\\ ork ·
(a) Its 'olume decreases. (c) no net transfer of work occurred. ·
(b) Its temixrature increases. (d) we cannot draw such conclusions from thl' fi
(c) Its pressure increases. l~'llrt.

(d) none of the above p


Mc-9.2. Air. initially at 100 kPa. is se~led in a container bl
a mobile piston of cross-sectional area 10.0 cm ·
Final
Now we push the piston with an additional force of
magnitude F = I 00 N to compress the air. What is
the final pressure p of the sealed air when the piston
reaches mechanical equilibrium?
(a) p = 2 x JO" Pa
(b) p = 1.0001 x 105 Pa
(c) p = 2 " 105 Pa
(d) p = l x 106 Pa
Mc-9.3. A gas sealc:d in a container by a mobile piston expands.
Which of the following statements is correct? Initial
(a) The piston does work on the gas.
(h) No \\Ork is done in the: process.
(c) The gas does work on the piston.
(d) The gas and the piston exchange heat only.
Agure 9.32
Mc-9.4. A gas is compressed from a \'Olume of5 L to a volume
of I L with its pressure held constant at 4000 Pa. To
achieve this compression, the following work is done
on the gas. Mc-9.7. Fig. 9.28 shows a cyclic process in a p-1' d. ·m. In
(a) IV = 20 kJ the process shown
(b) 11' = 16kJ (a) the system receives work per cycle.
(c) II' = 20 J (b) the system releases work per cycle.
(d) IV = 16J (c) no net transfer of work occurs during :i l ., lc.
Mc-9.5. Fig. 9.31 shows a p-1' diagram for a gas. Three (d) we cannot draw such conclusions from th-: ,ho11n
processes (labelled a. b, and c) are investigated that figure.
bring the gas from an initial volume to a final volume. Mc-9.8. Fig. 9.33 shows three steps, labelled I, II. and Ill. that
Which path yields the largest value for work? Hint: fonn a cyclic process in ap- V diagram. Ran ~ th~ 11orl.
5 J > 2 J. but -5 J < -2 J. from largest to smallest for the steps (Note: - 5 J < - ~ J.
(a) path a but +2 J < +5 J):
(b) path b
(c) pathc
(d) At least two paths yield the same work. p (arm)

/>
1.0 ----
c Initial

c b 0.5

FinJI b

_ _ _ _ _ _ _ _ _ _ _ ___....\!
l

-------+---------~ \' (111 1


Figure 9.31 Three paths for the compr ·
ilitlal to a sman final ess100 of a gas system from an 0.01
er volume. The paths are labelled a, b, and c.
Agure 9.33
zse PART TWO I ENERGY. THERMODYNAMICS AN
• 0 TRANSPORT ouc ...... - .. _ ..n

Scanned with CamScanner


(0 ) I • ti. Ill
(hl t. Ill. II MC-9.12. Thl· specific hl•;tt for material/\ is greater than that for
le) II. I. Ill lllJtcria l B. If equal a mounts o f heat nre added to both
(dl II, Ill, I lll3ll-rials. the one rcachinc a higher temperature will
(el Ill. I. II be (assume thnt no phase~t r.111 sition occurs in either
(f) Ill. II. I " . . material )
-· , ~4
"-' -:hows thn.:1: steps lonnm!.!
~
a cvc(i ' c pro Cess (a) material A.
I' I!..· "· • ,
. -;1 p- 1' diagram: step I from stall' A to st·ite 13 ·t (b) material 13 .
Ill• ' , Sep
11 from state D to state C. and step Ill from state C (e) neither. they reach the same tt·mperatun.:.
back to state A. Rank the work from largest amount to (d) impossible to determine rrom the information
srn•·illest amount for the steps. 1\ 'o te: - 5 J < - -J J• b lit
provided.
+2J <. +5J. MC-9.13. We double the amount oflu:at that is added to a c losed
(a) I, 11. Ill system. If no phase transitions occ ur in the system.
(b) 1. Ill. II and the system absorbs the heat full y into its themrnl
(c) 11. I. Ill energy. we expect
(d) II, Ill, I (a) the temperature of the systt:m to double.
(e) Ill. I. II (b) the temperaturl· of the system to increase fourfold.
(f) 111. II, I (c) the temperature of the system to be halved.
(d) the temperature of the system to decrease to one
quarter.
(c) None of the above is correct.
p (kPa) MC-9.14. We increase the gas pressure in a cylinder o f fixed
volume by heating the gas. The gas is the system. In
B
this process.
s {a) work is done on the gas.
(b) work is done by the gas .
(c) no work is done on the gas.
6 (d) whether work is done on the gas o r by the gas
depends on the temperature.
MC-9.15. The specific heat of ethyl alcohol is about 50% that
4 of water. If equal amounts of alcohol and water in
separate beakers are supplied with the same a mount
of heat, which liquid will show the larger increase in
2 temperature?
(a) water
(b) ethyl alcohol
L----+----.----r----- \! (m .l) (c) Both will show the same temperature increase.
6 8 10 MC-9.16. Fig. 9.35 shows the p - V relationship in the left
ventricle o f the human heart (heart cycle). The curve
Figure 9.34 is traversed counter-clockwise with inc reasing time.
The stroke volume is 100 ml - 35 ml = 65 L.
The systolic pressure is 11 8 torr (equal to 15.7 kPa).
and the diastolic pressure is 70 torr (equal to 9.3 kPa).
MC-9.10. An object of mass m = 5 kg is dropped from a height of The ventricular pressure drops to below the diastolic
I 0 metres. Just before reaching the ground, the thermal pressure, while the pressure in the arteries remains
energy of the object is (use g = I 0 m/s2) about 70 torr because the aortic valve has closed.
preventing backflow. To simplify calculations, the
(a) unchanged.
dashed straight lines in the diagram allow us to
(b) Elhcmial = 50 J.
replace the curved segments. Which of the following
{c) £thermal= 500 J.
statements about the p -V diagram in Fig. 9.35 is
(d) £thermal = 5 kJ. · correct?
MC-9.11. Which of the following things cannot happen m a
(a) The diagram shows the data in standard units.
closed system?
(b) The maximum pressure variation during the heart
(a) Heat is transferred to the environment.
cycle exceeds the atmospheric pressure value.
{b) Work is done on the system by an object in the
(c) The work done in a single cycle can be determined
environment. from the area enclosed by the solid line in
(c) Matter flows into the system. Fig. 9.35.
{d) The temperature of the system increases. . (d) The system is the blood in the ventricle. This
(e) The internal energy of the system remams system is an isolated system.
unchanged.

CHAPTER 9 I THERMAL PHYSICS 259

Scanned with CamScanner


anu ..... - -· ~
' cytop Iasm, d to behave like a
'his allows bloo J1"qt1ids) When p + I pv 2 + pgz -- canst,
·
n (mixture
of tWO
.
.
f the red blood 2
•orecration o is the pressure, z is the height, and v .
ve\'er. age e . c·illed mmmwla- where P
·urs in a process <
. r. ·1 stnicture resemb mg a
r of the fluid.
is the
sPeeQ
lls 1onn • .
arion creates a highly viscous sus- . ·tie's law for a Newtonian fluid in a cylind r1.cai
d in liquid). • Po1seUJ
tube.
i:i V 1r i:ip .
- - r4 -
At - 8 'T'/ tube l '

where rtube is the radius of the tube, and Ap is lh:


difference along the length I of the tube. :ressure
Jes (around flow lines) are not cre- Ohm's law for a Newtonian fluid:
o not intersect or vanish in the flow. •
uid whose flow is laminar. The fluid i:iV
ilp = R -
through elastic collisions with con- ilt'
: motion). where R is the flow resistance. The flow resis
--e in a
· whose flow is laminar. The fluid cylindrical tube is:
stically with each other and with 8/
ing a velocity gradient in the fluid. R = 4 TJ.
'TT"rtube
~resistance to flow in a fluid. It is
!eded to move a plate at constant
plate at rest at distance ily, both
MULTIPLE-CHOICE QUESTIONS
MC-12.1. The diameter of a tapering tube decreases from d 10
/Av/ I
d2 = d 1/2. As a result, the volume flow rate oflaminar
71A~.
L.ly flow changes to
(a) AVifAt = AV/At.
: plates, 7J is the viscosity coef- (b) AVifilt = (112)/lVifilt.
locity gradient. (c) AVifAt = 2AV/At.
-ansition from laminar to turbu- (d) AV/At= (1/4)LlV1/Llt.
be (Re < 2000 is laminar; Re > (e) AV/At= 4AV1/Llt.
MC-12.2. Fig. 12.25 shows a cylindrical tube of changing
diameter with an ideal dynamic fluid (blue) flowing
toward the right with initial speed v. The vertical col-
- ,
umns are connected to the main tube. Which of the
1]

:ed of the fluid, d is the diam- four choices shows the proper elevations of the fluid
::osity coefficient, and p is the in each of the three vertical columns?
MC-12.3. The volume flow rate and the mass flow rate in Iami·
nar flow are
(a) the same.
5 (b) proportional to each other.
(c) inversely proportional to each other.
(d) unrelated.
(e) related in a non-linear manner.

IODYNAMICS, AND TRANSPO~T our· •. - . - -

Scanned with CamScanner


(Al
(B)
4
3 4 4 MC-12.a. Bemoulh' Llw i- an
3 4 3 4 ~ t'\pr s1on of
2. (a) the c~n atiC'!l of rtuss.
2 3 l 3 4
2. 2 (b) thecon~r.ationof ' .
3 a.met:.: en~v.
l (c) thl." Cl'n n au n ui toul ~-
(d) the conser. at1on oi \e! ~:~·
kl the con:.er.:i:ion of mom;.:-•·:n.
MC-1 2.9. Th ......
I." conserY:ition of m.:iss lc-Jtl.5 to t'ie follov. i:?,, bv.
that \\C- ux to dncribe I ,~ · fi · . "'
...._.. 1rur C\\ in a thud.
(C) (a) Ckmoulli's la\\
(0)
(b) Pa.seal's la\\
4 4
3 4 3 4 (c) equation of rominuitv
3 4 (d ) Poiseuille's bw ,
2 2 3 4
l 3 (e) Ohm's law
2 3
l MC-12.10. 0 0 the folio\\ ing e'l:perimait as shown in fi.:. l ~..26:
Push a pin thr0ug.h thc- cc-mre of a thin sh~~cf c:?...-d·
bOJrd. Lcxate thc- tip of the pin in thc- ccri~nl hok
of a thrc-:id sprol fron ~IO\\ . H IJ tfie C1Jilll\.-u.'1i
from below and st:ll1 to b!ow throu!;:h th~ h0!~. Thc-
cardboord 1\ ill not drop to the fl1.'<lr 1\ h~n \ ou rel~
Figure 12.25 An ideal dynamic fluid (blue) flo th . . it. Which IJw expbins this eft-...Ct? ·
·th · ·ti I d- ws rough a cyfmdncaJ
tube wt m1 a sp~e \". Vertical columns are connected to th . (a) Poi~ilk's law
to measure the fluid pressure. e ma.m tube (b) Ohrn·s law
(c) equ:ition of continuitv
(d) Bc-moulli's bw
(e) Pascal's law
MC-12.4. An artery has ballooned at one location outward (to
a larger cross-section) due to an aneurysm. Which of
the following statements is correct for blood flowino
through this broadened section? Treat blood as a~
ideal dynamic fluid.
(a) Blood will rush faster through the broadened
section due to the equation of continuity.
(b) The equation of continuity predicts a decrease
of the blood pressure in the broadened section.
(c) Bernoulli's law predicts that the blood pressure
in the broadened section is lower than in an
adjacent section of the blood vessel, causing the
blood Yessel to temporarily collapse. _--~ 1-----..
+ _ ...
(d) Bernoulli's law predicts that the blood pressure
in the broadened section is higher than in an II
adjacent section of the blood vessel. figure 12.26 A pin is placed at the centre of a thin cardboard sheet
MC-12.5. The equation of continuity is an expression of The pin's tip is then placed in the central hole of a thread spool. Hold tile
(a) the conserYation of mass. cardboard then start blowing through the hole. Release the cardboard.
(b) the conservation of t~tal ~nergy.
(c) the conservation of kmet1.c energy.
(d) the conserYation of velocity. . MC-12.11. Do the following experiment: Hold m o sheets of
MC-12.6. Bernoulli's law contains a term that wa~ den~ed from paper p:irallel to e:ich other at a distance of atx>ut
the kinetic energy o f t he 1·dea I dynamic fluid. What I cm to 3 cm. Blow gently betwt"en the two sht't'tS
.
unit does this term carry·
']
from their t'dgt"s. The two sheets 11 ill be pulled
(a) J
- .
together. Which law t'Xplains this obsen·auon.
~

(b) N (a) Poiseuille's law


(c) m/s (b) Ohm"s law
(d) m/s 2 (c) equation of continuity
(d) Bernoulli's law
(e) Pa the speed of an ideal dynamic
MC-12.7. Which law connects (e) Pascal's law
fluid to its pressure?
(a) Pascal's law
(b) Newton's second law
(c) Bernoulli's law
(d) Laplace's law

339
CHAPTER 12 I FLUID FLOW

Scanned with CamScanner


'
• ,t
'
...
UNITS
• Frequencyf s
. -• ==· Hz .·
Solution · . deter.:· •
· frequencies eed ,, .. • Wave n~b.el." k: rad/n1 .
; Sinco the string resonates at certain
·· mined by the length l of the stnng
. and the wave sp .. . , •,. · • Angular frequen~Y w: ~dis ,
using EQ. (14.24): , •·: .~ ...
r. "" -..\,. ... -2l"ff
."
· -µ. for n = I,2,3•"· ·· · .>\;] LAWS . . . - (Tl
. Speed of wave on a s!1'1ng. vWllve - • µ) ·,Where 7" T
·in -. .<
" 'I ,.,. • ••• ~· ' r..1 . ,
• . and µ is the lmear mass denstty (mass Pe is~ ·..:
\."'
,... :: tension . . r len&th). ';
As Fag. 14.38 shows, T-= mg, so: ~ : )
..:,'"
. . '<

" 21.'/f! µ1_~ c~i n = .:~" ~ . '•


. : !. = !!... t,2,3: MULTIPLE-CHOICE QUESTIONS
Waves are typically characterized by frequen . . \
=4: MC-141
.. · d l h cy, angu. •·
'. Fot the fourth harmonic n
. .. f
lar frequency, pe~10 wave engt • and amplitude '!
I
\

Which of these pat.rs o parameters are related .' •
10
other in a linear fashion? Nole: More than one eat!t _ ;
may apply. · ansv.·er '·.
(a) period and frequency
(b) period and angular frequency
(c) frequency and angular frequency
. :.·
(d) wavelength and period
., .
(e) amplitude and frequency
MC-14.2. The shorter the frequency of the wave
=
. Substituting L 1.6 m,h =fviorat.or 180 Hz, and µ = =2A g/m · (a) the smaller its amplitude.
in above equation for m yields: (b) the shorter its period.
(c) the smaller its speed.
(d) the longer its wavelength
(e) the greater its amplitude.
MC-14.3. The particles of the medium in a transverse wave. .";
~ ,_ · m = 5.08kg.
move ' .
(a) with waves.
' .. .~ (b) in ellipses.
(c) in circles.
SUMMARY (d) parallel to the direction of the wave travel.
(e) perpendicular to the direction of the wave ~vet\:
MC-14.4. An object hit by a wave makes 20 complete vibratlcms -~/ '
DEFINITIONS in 10 seconds. What is its period? ' ·' '
• Speed of wave:
frequency
vwave,,,. ..\/,where A is wavelength and/is (a) 2 Hz
(b) 10 s
•• Wave function: J{t, t) ""f(x ± vt)
(c) 0.50 Hz
(d) 2 s
• Hannonic wave function: ;(x, t) = Asin(kx ±wt), where k = (e) 0.50 s . , ,.
2TTIA is the wave number and w = 2TrlT= 2TT/is the angular MC-14.5. The distance betw~en a crest of a sinusoidal w~~T
frequency . .
wave and the next trough.is 2 m. If the frequency~~/
• Superposition principle: For two overlapping waves the the water wave is 2 Hz, what is its speed? ·' ·. '
(a) 8 mis
>..:
· ·· ''"i~'.,
resultant. displacement at any point is the algebraic 'sum
of the displacement of the ~ndividual waves. yR(x, t) = (b) 4 mis · , ',>~J 'i l:.
~· ,. ~/~ '.»;·*;)~,
ya(.t, t) ± .>"i(x, t), where YR 1s the resultant displacement (
c) 2 mis . -<· ·.~, · '!;~:·:-'
(d) I mls . , ...,• :·q·.~
. (wave function).

Standing waves for iwo similar waves moving in op '.


. . , ) . . pos1te
(e) Not enough infonnation is 'given to de(~e.-~.:~,
. wave speed.
h I . "'.'>

.. . ·: ·. •.· . >. ·'.-::~; ~~,,~~


~ ( \ ,:-..f;f!;

d1recllons: }\X, ~ =2A sm(kx)cos(wt) ·


MC-14.6. If the time interval between two succe·~sive'.~~~ 1l
• ' Position of nodes: xnodes = n A/2 for n = o, 1, ~. 3, . . . · .· of a travelling wave past an· observ~rJs~,,o~~~-e(Br
• Position of antinodes: x u· od = (n + •1 ),\/2 ti then · · ' . .· :: . " ·:~·i.~~:~)fa#~t ·"
. an n es · · n or n =O 1 (a) its wavelength 1. 2 5 . . ' . . :·. ·;}: . :~>&:::!it~
2,3,... . . . . , '
.(b) its frequency is ~ m:·. ,. , .·":·> ', .;,;. )~i;9;j1;:
Hz. ; .
·• . .Harm~nics for a string 'o nength ·l ·fix~d at end . . _ () • ~ · .~ ..~<::.· :}> 'i~:.~~·r,;,:~~
n(vl2L) =nfi :. . . . . . . s. !,, - c ~ts wavelength is 5 m. · · : •..:: :.;/' ''.1~·1.:(
(d) its frequency \s 2.5..Hz .~·; ,.·: ·; ~,; ,,~'.~t2,.
, (e) its veloCity of propagatio~ 1~'2s ~s~~~·;;t
. . . . .; ',··~\.-~<~:~:;:ri~t:~~1~:~~§

Scanned with CamScanner


Scanned with CamScanner

You might also like